Menu Top
Latest Maths NCERT Books Solution
6th 7th 8th 9th 10th 11th 12th

Class 12th Chapters
1. Relations and Functions 2. Inverse Trigonometric Functions 3. Matrices
4. Determinants 5. Continuity and Differentiability 6. Application of Derivatives
7. Integrals 8. Application of Integrals 9. Differential Equations
10. Vector Algebra 11. Three Dimensional Geometry 12. Linear Programming
13. Probability

Content On This Page
Example 1 to 4 (Before Exercise 7.1) Exercise 7.1 Example 5 & 6 (Before Exercise 7.2)
Exercise 7.2 Example 7 (Before Exercise 7.3) Exercise 7.3
Example 8 to 10 (Before Exercise 7.4) Exercise 7.4 Example 11 to 16 (Before Exercise 7.5)
Exercise 7.5 Example 17 to 22 (Before Exercise 7.6) Exercise 7.6
Example 23 & 24 (Before Exercise 7.7) Exercise 7.7 Example 25 (Before Exercise 7.8)
Exercise 7.8 Example 26 & 27 (Before Exercise 7.9) Exercise 7.9
Example 28 to 34 (Before Exercise 7.10) Exercise 7.10 Example 35 to 42 - Miscellaneous Examples
Miscellaneous Exercise on Chapter 7


Chapter 7 Integrals

Welcome to the comprehensive solutions page for Chapter 7: Integrals, arguably the most extensive and fundamentally crucial chapter in the Class 12 Mathematics syllabus, as presented in the Latest NCERT (2024-25) textbook. This chapter delves into the world of integration, encompassing both Indefinite Integration (finding anti-derivatives) and Definite Integration (calculating the net accumulation or area under curves). Mastery of integration is paramount for advanced calculus, differential equations, physics, engineering, and numerous other quantitative fields. These solutions provide exhaustive, step-by-step guidance, fully adhering to the current rationalized syllabus.

The chapter commences by formally introducing integration as the inverse process of differentiation. If $\frac{d}{dx} F(x) = f(x)$, then $\int f(x) dx = F(x) + C$, where $C$ is the constant of integration. The solutions reinforce the necessity of mastering a core set of Standard Indefinite Integrals. These include formulas for integrating basic functions like $x^n$, trigonometric functions ($\sin x, \cos x, \sec^2 x,$ etc.), exponential functions ($e^x, a^x$), $1/x$, and those leading to inverse trigonometric functions ($\int \frac{1}{\sqrt{a^2-x^2}} dx$, $\int \frac{1}{a^2+x^2} dx$, etc.). Direct application of these formulas is the first step towards proficiency.

The true depth of indefinite integration lies in the various Methods of Integration developed to tackle more complex integrands. These solutions meticulously demonstrate:

Solutions also cover techniques for integrating special types of functions, often involving completing the square in quadratic expressions or using specific substitutions, leading to standard results involving $\log|\dots|$ or inverse trigonometric functions.

The second major part transitions to Definite Integrals. While the concept of the definite integral as the limit of a sum ($\int\limits_{a}^{b} f(x) dx = \lim\limits_{h \to 0} h \sum\limits_{r=1}^{n} f(a+rh)$, where $nh=b-a$) provides the theoretical foundation, its evaluation primarily relies on the Fundamental Theorem of Calculus (Part II). This theorem elegantly connects differentiation and integration: If $F(x)$ is an antiderivative of $f(x)$ (i.e., $F'(x) = f(x)$), then $\qquad \mathbf{\int\limits_{a}^{b} f(x) dx = [F(x)]_{a}^{b} = F(b) - F(a)}$. Solutions demonstrate evaluating definite integrals by first finding the indefinite integral and then applying this theorem.

Crucially, the power of definite integration is often unlocked by using its various Properties. These properties can significantly simplify calculations, sometimes avoiding the need to find the antiderivative altogether. The solutions extensively explain and apply key properties, including:

By diligently studying these exhaustive solutions, students can gain mastery over standard integration formulas, become highly proficient in applying all major integration techniques (substitution, parts, partial fractions, etc.), confidently evaluate definite integrals using the Fundamental Theorem, and learn to strategically utilize the properties of definite integrals to solve complex problems efficiently.



Example 1 to 4 (Before Exercise 7.1)

Example 1: Write an anti derivative for each of the following functions using the method of inspection:

(i) cos 2x

(ii) 3x2 + 4x3

(iii) $\frac{1}{x}$, x ≠ 0

Answer:

(i) For the function $f(x) = \cos 2x$, we need to find a function whose derivative is $\cos 2x$.

We know that $\frac{d}{dx}(\sin x) = \cos x$.

Using the chain rule, $\frac{d}{dx}(\sin(ax)) = a \cos(ax)$.

We want the derivative to be $\cos 2x$. Let's consider the derivative of $\sin 2x$.

$\frac{d}{dx}(\sin 2x) = 2 \cos 2x$.

This is twice the function we want. So, we should consider $\frac{1}{2} \sin 2x$.

$\frac{d}{dx}\left(\frac{1}{2} \sin 2x\right) = \frac{1}{2} \cdot (2 \cos 2x) = \cos 2x$.

Thus, an anti derivative of $\cos 2x$ is $\mathbf{\frac{1}{2} \sin 2x}$.


(ii) For the function $f(x) = 3x^2 + 4x^3$, we need to find a function whose derivative is $3x^2 + 4x^3$. We can find the anti derivative of each term separately.

For the term $3x^2$, we know that $\frac{d}{dx}(x^n) = nx^{n-1}$. We want the derivative to be $3x^2$, which suggests the original function involved $x^3$.

$\frac{d}{dx}(x^3) = 3x^2$.

So, an anti derivative of $3x^2$ is $x^3$.

For the term $4x^3$, we want the derivative to be $4x^3$. This suggests the original function involved $x^4$.

$\frac{d}{dx}(x^4) = 4x^3$.

So, an anti derivative of $4x^3$ is $x^4$.

By the linearity of differentiation, the derivative of a sum is the sum of the derivatives. Therefore, the anti derivative of the sum is the sum of the anti derivatives.

An anti derivative of $3x^2 + 4x^3$ is $x^3 + x^4$.

Check: $\frac{d}{dx}(x^3 + x^4) = \frac{d}{dx}(x^3) + \frac{d}{dx}(x^4) = 3x^2 + 4x^3$.

Thus, an anti derivative of $3x^2 + 4x^3$ is $\mathbf{x^3 + x^4}$.


(iii) For the function $f(x) = \frac{1}{x}$, $x \neq 0$, we need to find a function whose derivative is $\frac{1}{x}$.

We know that $\frac{d}{dx}(\log|x|) = \frac{1}{x}$ for $x \neq 0$. The condition $x \neq 0$ is given, which means we should use the absolute value in the logarithm.

Thus, an anti derivative of $\frac{1}{x}$ is $\mathbf{\log|x|}$.

Example 2: Find the following integrals:

(i) $\int \frac{x^3 − 1}{x^2} \;dx$

(ii) $\int (x^{\frac{2}{3}} + 1) \;dx$

(iii) $\int (x^{\frac{3}{2}} + 2e^x − \frac{1}{x}) \;dx$

Answer:

Solution:


(i) We need to find $\int \frac{x^3 − 1}{x^2} \;dx$.

We can rewrite the integrand by splitting the terms in the numerator:

$\frac{x^3 − 1}{x^2} = \frac{x^3}{x^2} - \frac{1}{x^2} = x - x^{-2}$

Now, we integrate term by term using the power rule for integration, $\int x^n dx = \frac{x^{n+1}}{n+1} + C$ (for $n \neq -1$), and the rule $\int dx = x + C$:

$\int \frac{x^3 − 1}{x^2} \;dx = \int (x - x^{-2}) \;dx$

$= \int x \;dx - \int x^{-2} \;dx$

$= \frac{x^{1+1}}{1+1} - \frac{x^{-2+1}}{-2+1} + C$

$= \frac{x^2}{2} - \frac{x^{-1}}{-1} + C$

$= \frac{x^2}{2} - (- \frac{1}{x}) + C$

$= \frac{x^2}{2} + \frac{1}{x} + C$


(ii) We need to find $\int (x^{\frac{2}{3}} + 1) \;dx$.

We integrate term by term using the power rule, $\int x^n dx = \frac{x^{n+1}}{n+1} + C$, and $\int dx = x + C$:

$\int (x^{\frac{2}{3}} + 1) \;dx = \int x^{\frac{2}{3}} \;dx + \int 1 \;dx$

$= \frac{x^{\frac{2}{3}+1}}{\frac{2}{3}+1} + x + C$

$= \frac{x^{\frac{5}{3}}}{\frac{5}{3}} + x + C$

$= \frac{3}{5} x^{\frac{5}{3}} + x + C$


(iii) We need to find $\int (x^{\frac{3}{2}} + 2e^x − \frac{1}{x}) \;dx$.

We integrate term by term using the power rule, $\int x^n dx = \frac{x^{n+1}}{n+1} + C$, the rule for exponential functions, $\int e^x dx = e^x + C$, and the rule for the reciprocal function, $\int \frac{1}{x} dx = \log |x| + C$. We also use the linearity of the integral, $\int c f(x) dx = c \int f(x) dx$:

$\int (x^{\frac{3}{2}} + 2e^x − \frac{1}{x}) \;dx = \int x^{\frac{3}{2}} \;dx + \int 2e^x \;dx - \int \frac{1}{x} \;dx$

$= \int x^{\frac{3}{2}} \;dx + 2 \int e^x \;dx - \int \frac{1}{x} \;dx$

$= \frac{x^{\frac{3}{2}+1}}{\frac{3}{2}+1} + 2e^x - \log |x| + C$

$= \frac{x^{\frac{5}{2}}}{\frac{5}{2}} + 2e^x - \log |x| + C$

$= \frac{2}{5} x^{\frac{5}{2}} + 2e^x - \log |x| + C$

Example 3: Find the following integrals:

(i) $\int (\sin x + \cos x ) \;dx$

(ii) $\int cosec\; x (cosec\; x + \cot x) \;dx$

(iii) $\int \frac{1 − \sin x}{\cos^2 x} \;dx$

Answer:

Solution:


(i) We need to find $\int (\sin x + \cos x ) \;dx$.

Using the linearity of the integral, we can integrate each term separately:

$\int (\sin x + \cos x ) \;dx = \int \sin x \;dx + \int \cos x \;dx$

We know the standard integrals $\int \sin x \;dx = -\cos x + C_1$ and $\int \cos x \;dx = \sin x + C_2$.

So, $\int (\sin x + \cos x ) \;dx = -\cos x + \sin x + C$, where $C = C_1 + C_2$ is the constant of integration.

$= \sin x - \cos x + C$


(ii) We need to find $\int \text{cosec}\; x (\text{cosec}\; x + \cot x) \;dx$.

First, expand the integrand:

$\text{cosec}\; x (\text{cosec}\; x + \cot x) = \text{cosec}^2 x + \text{cosec}\; x \cot x$

Now, integrate the expanded form term by term:

$\int (\text{cosec}^2 x + \text{cosec}\; x \cot x) \;dx = \int \text{cosec}^2 x \;dx + \int \text{cosec}\; x \cot x \;dx$

We know the standard integrals $\int \text{cosec}^2 x \;dx = -\cot x + C_1$ and $\int \text{cosec}\; x \cot x \;dx = -\text{cosec}\; x + C_2$.

So, $\int \text{cosec}\; x (\text{cosec}\; x + \cot x) \;dx = -\cot x - \text{cosec}\; x + C$, where $C = C_1 + C_2$ is the constant of integration.

$= -\cot x - \text{cosec}\; x + C$


(iii) We need to find $\int \frac{1 − \sin x}{\cos^2 x} \;dx$.

We can rewrite the integrand by splitting the fraction:

$\frac{1 − \sin x}{\cos^2 x} = \frac{1}{\cos^2 x} - \frac{\sin x}{\cos^2 x}$

Using trigonometric identities, $\frac{1}{\cos^2 x} = \sec^2 x$ and $\frac{\sin x}{\cos^2 x} = \frac{\sin x}{\cos x} \cdot \frac{1}{\cos x} = \tan x \sec x$.

So the integral becomes:

$\int (\sec^2 x - \sec x \tan x) \;dx$

Using the linearity of the integral, integrate each term separately:

$= \int \sec^2 x \;dx - \int \sec x \tan x \;dx$

We know the standard integrals $\int \sec^2 x \;dx = \tan x + C_1$ and $\int \sec x \tan x \;dx = \sec x + C_2$.

So, $\int \frac{1 − \sin x}{\cos^2 x} \;dx = \tan x - \sec x + C$, where $C = C_1 - C_2$ is the constant of integration.

$= \tan x - \sec x + C$

Example 4: Find the anti derivative F of f defined by f (x) = 4x3 – 6, where F(0) = 3

Answer:

Solution:


We are asked to find the antiderivative F(x) of the function $f(x) = 4x^3 - 6$, such that $F(0) = 3$.

By definition, the antiderivative F(x) is the indefinite integral of f(x). So, we have:

$F(x) = \int f(x) \;dx$

$F(x) = \int (4x^3 - 6) \;dx$

Using the properties of integration (linearity) and the power rule for integration, $\int x^n \;dx = \frac{x^{n+1}}{n+1} + C$ (for $n \neq -1$) and $\int c \;dx = cx + C$:

$F(x) = \int 4x^3 \;dx - \int 6 \;dx$

$F(x) = 4 \int x^3 \;dx - 6 \int dx$

$F(x) = 4 \cdot \frac{x^{3+1}}{3+1} - 6x + C$

$F(x) = 4 \cdot \frac{x^4}{4} - 6x + C$

$F(x) = x^4 - 6x + C$

Here, C is the constant of integration.

We are given the condition that $F(0) = 3$. We use this to find the value of C.

Substitute $x=0$ into the expression for F(x):

$F(0) = (0)^4 - 6(0) + C$

Since $F(0) = 3$, we have:

$3 = 0 - 0 + C$

$3 = C$

Thus, the constant of integration is $C = 3$.

Substituting the value of C back into the expression for F(x), we get the specific antiderivative:

$F(x) = x^4 - 6x + 3$



Exercise 7.1

Find an anti derivative (or integral) of the following functions by the method of inspection.

Question 1. sin 2x

Answer:

Solution:


We need to find an antiderivative of $f(x) = \sin(2x)$ by the method of inspection.

The method of inspection involves finding a function whose derivative is the given function.

We know that the derivative of $\cos x$ is $-\sin x$. Therefore, we can guess that the antiderivative of $\sin(2x)$ might involve $\cos(2x)$.

Let's find the derivative of $\cos(2x)$:

$\frac{d}{dx}(\cos(2x)) = -\sin(2x) \cdot \frac{d}{dx}(2x) = -\sin(2x) \cdot 2 = -2\sin(2x)$

We want the derivative to be $\sin(2x)$, but we got $-2\sin(2x)$. To adjust this, we need to multiply our initial guess, $\cos(2x)$, by a constant such that its derivative is $\sin(2x)$.

Consider the function $-\frac{1}{2}\cos(2x)$. Let's find its derivative:

$\frac{d}{dx}\left(-\frac{1}{2}\cos(2x)\right) = -\frac{1}{2} \cdot \frac{d}{dx}(\cos(2x))$

$= -\frac{1}{2} \cdot (-2\sin(2x))$

$= \sin(2x)$

The derivative of $-\frac{1}{2}\cos(2x)$ is indeed $\sin(2x)$.

Therefore, an antiderivative of $\sin(2x)$ is $-\frac{1}{2}\cos(2x)$.

The general antiderivative (or integral) is obtained by adding the constant of integration C.

So, the antiderivative of $\sin(2x)$ is $-\frac{1}{2}\cos(2x) + C$.

Question 2. cos 3x

Answer:

Solution:


We need to find an antiderivative of $f(x) = \cos(3x)$ by the method of inspection.

The method of inspection involves finding a function whose derivative is the given function.

We know that the derivative of $\sin x$ is $\cos x$. Therefore, we can guess that the antiderivative of $\cos(3x)$ might involve $\sin(3x)$.

Let's find the derivative of $\sin(3x)$:

$\frac{d}{dx}(\sin(3x)) = \cos(3x) \cdot \frac{d}{dx}(3x)$

$= \cos(3x) \cdot 3$

$= 3\cos(3x)$

We want the derivative to be $\cos(3x)$, but we got $3\cos(3x)$. To adjust this, we need to multiply our initial guess, $\sin(3x)$, by a constant such that its derivative is $\cos(3x)$. The constant should be $\frac{1}{3}$.

Consider the function $\frac{1}{3}\sin(3x)$. Let's find its derivative:

$\frac{d}{dx}\left(\frac{1}{3}\sin(3x)\right) = \frac{1}{3} \cdot \frac{d}{dx}(\sin(3x))$

$= \frac{1}{3} \cdot (3\cos(3x))$

$= \cos(3x)$

The derivative of $\frac{1}{3}\sin(3x)$ is indeed $\cos(3x)$.

Therefore, an antiderivative of $\cos(3x)$ is $\frac{1}{3}\sin(3x)$.

The general antiderivative (or integral) is obtained by adding the constant of integration C.

So, the antiderivative of $\cos(3x)$ is $\frac{1}{3}\sin(3x) + C$.

Question 3. e2x

Answer:

Solution:


We need to find an antiderivative of $f(x) = e^{2x}$ by the method of inspection.

The method of inspection involves finding a function whose derivative is the given function.

We know that the derivative of $e^x$ is $e^x$. Therefore, we can guess that the antiderivative of $e^{2x}$ might involve $e^{2x}$.

Let's find the derivative of $e^{2x}$:

$\frac{d}{dx}(e^{2x}) = e^{2x} \cdot \frac{d}{dx}(2x)$

$= e^{2x} \cdot 2$

$= 2e^{2x}$

We want the derivative to be $e^{2x}$, but we got $2e^{2x}$. To adjust this, we need to multiply our initial guess, $e^{2x}$, by a constant such that its derivative is $e^{2x}$. The constant should be $\frac{1}{2}$.

Consider the function $\frac{1}{2}e^{2x}$. Let's find its derivative:

$\frac{d}{dx}\left(\frac{1}{2}e^{2x}\right) = \frac{1}{2} \cdot \frac{d}{dx}(e^{2x})$

$= \frac{1}{2} \cdot (2e^{2x})$

$= e^{2x}$

The derivative of $\frac{1}{2}e^{2x}$ is indeed $e^{2x}$.

Therefore, an antiderivative of $e^{2x}$ is $\frac{1}{2}e^{2x}$.

The general antiderivative (or integral) is obtained by adding the constant of integration C.

So, the antiderivative of $e^{2x}$ is $\frac{1}{2}e^{2x} + C$.

Question 4. (ax + b)2

Answer:

Solution:


We need to find an antiderivative of $f(x) = (ax + b)^2$ by the method of inspection.

The method of inspection involves finding a function whose derivative is the given function.

We know that the derivative of a function of the form $(u(x))^n$ is $n(u(x))^{n-1} \cdot u'(x)$. In our case, $u(x) = ax + b$ and the power is 2. This suggests that the original function might be of the form $(ax+b)^3$.

Let's find the derivative of $(ax + b)^3$ using the chain rule:

$\frac{d}{dx}((ax + b)^3) = 3(ax + b)^{3-1} \cdot \frac{d}{dx}(ax + b)$

$= 3(ax + b)^2 \cdot (a \cdot 1 + 0)$

$= 3(ax + b)^2 \cdot a$

$= 3a(ax + b)^2$

We want the derivative to be $(ax + b)^2$, but we obtained $3a(ax + b)^2$. To get rid of the factor $3a$, we need to multiply our initial guess, $(ax + b)^3$, by $\frac{1}{3a}$ (assuming $a \neq 0$).

Consider the function $\frac{1}{3a}(ax + b)^3$. Let's find its derivative:

$\frac{d}{dx}\left(\frac{1}{3a}(ax + b)^3\right) = \frac{1}{3a} \cdot \frac{d}{dx}((ax + b)^3)$

$= \frac{1}{3a} \cdot (3a(ax + b)^2)$

$= (ax + b)^2$

The derivative of $\frac{1}{3a}(ax + b)^3$ is indeed $(ax + b)^2$.

Therefore, an antiderivative of $(ax + b)^2$ is $\frac{1}{3a}(ax + b)^3$.

The general antiderivative (or integral) is obtained by adding the constant of integration C.

So, the antiderivative of $(ax + b)^2$ is $\frac{1}{3a}(ax + b)^3 + C$.

(Note: If $a=0$, the function is $f(x) = b^2$, which is a constant. The antiderivative of a constant $k$ is $kx + C$. So if $a=0$, $\int b^2 dx = b^2 x + C$. The formula $\frac{1}{3a}(ax + b)^3 + C$ is for $a \neq 0$.)

Question 5. sin 2x – 4e3x

Answer:

Solution:


We need to find an antiderivative of $f(x) = \sin(2x) - 4e^{3x}$ by the method of inspection.

The method of inspection involves finding a function whose derivative is the given function. We can find the antiderivative of each term separately.

First, consider the term $\sin(2x)$. We know that the derivative of $\cos(ax)$ is $-a\sin(ax)$.

So, $\frac{d}{dx}(\cos(2x)) = -2\sin(2x)$.

To get $\sin(2x)$, we need to multiply by $-\frac{1}{2}$.

$\frac{d}{dx}\left(-\frac{1}{2}\cos(2x)\right) = -\frac{1}{2} \cdot (-2\sin(2x)) = \sin(2x)$.

So, an antiderivative of $\sin(2x)$ is $-\frac{1}{2}\cos(2x)$.

Next, consider the term $4e^{3x}$. We know that the derivative of $e^{ax}$ is $ae^{ax}$.

So, $\frac{d}{dx}(e^{3x}) = 3e^{3x}$.

We have $4e^{3x}$. We can guess a function of the form $A e^{3x}$ such that its derivative is $4e^{3x}$.

$\frac{d}{dx}(A e^{3x}) = A \cdot 3e^{3x} = 3A e^{3x}$.

We want $3A e^{3x} = 4e^{3x}$, which means $3A = 4$, so $A = \frac{4}{3}$.

Thus, $\frac{d}{dx}\left(\frac{4}{3}e^{3x}\right) = \frac{4}{3} \cdot 3e^{3x} = 4e^{3x}$.

So, an antiderivative of $4e^{3x}$ is $\frac{4}{3}e^{3x}$.

Combining the antiderivatives, an antiderivative of $\sin(2x) - 4e^{3x}$ is the difference of the individual antiderivatives:

$-\frac{1}{2}\cos(2x) - \frac{4}{3}e^{3x}$.

The general antiderivative includes the constant of integration C.

Therefore, the antiderivative of $\sin(2x) - 4e^{3x}$ is $-\frac{1}{2}\cos(2x) - \frac{4}{3}e^{3x} + C$.

Find the following integrals in Exercises 6 to 20:

Question 6. $\int (4e^{3x} + 1)\;dx$

Answer:

Solution:


We need to find the integral $\int (4e^{3x} + 1)\;dx$.

Using the linearity property of integration, we can split the integral into two parts:

$\int (4e^{3x} + 1)\;dx = \int 4e^{3x} \;dx + \int 1 \;dx$

We can take the constant factor out of the integral:

$= 4 \int e^{3x} \;dx + \int dx$

Now, we use the standard integral formulas: $\int e^{ax} \;dx = \frac{1}{a}e^{ax} + C_1$ and $\int dx = x + C_2$.

For the first integral, $a=3$. So, $\int e^{3x} \;dx = \frac{1}{3}e^{3x} + C_1$.

For the second integral, $\int dx = x + C_2$.

Substituting these back into the expression:

$= 4 \left(\frac{1}{3}e^{3x}\right) + x + C$

where $C = 4C_1 + C_2$ is the combined constant of integration.

Simplifying the expression:

$= \frac{4}{3}e^{3x} + x + C$

Question 7. $\int x^2 (1 − \frac{1}{x^2} ) \;dx $

Answer:

Solution:


We need to find the integral $\int x^2 (1 − \frac{1}{x^2} ) \;dx$.

First, simplify the integrand by expanding the expression:

$x^2 \left(1 − \frac{1}{x^2}\right) = x^2 \cdot 1 - x^2 \cdot \frac{1}{x^2}$

$= x^2 - \frac{x^2}{x^2}$

$= x^2 - 1$

Now, we integrate the simplified expression:

$\int (x^2 - 1) \;dx$

Using the linearity property of integration, we can integrate each term separately:

$= \int x^2 \;dx - \int 1 \;dx$

We use the power rule for integration, $\int x^n \;dx = \frac{x^{n+1}}{n+1} + C$, and the rule $\int dx = x + C$.

For the first term, $n=2$:

$\int x^2 \;dx = \frac{x^{2+1}}{2+1} + C_1 = \frac{x^3}{3} + C_1$

For the second term:

$\int 1 \;dx = x + C_2$

Combining the results, we get:

$\int (x^2 - 1) \;dx = \left(\frac{x^3}{3} + C_1\right) - (x + C_2)$

$= \frac{x^3}{3} - x + (C_1 - C_2)$

Let $C = C_1 - C_2$ be the constant of integration.

So, the integral is:

$\int x^2 (1 − \frac{1}{x^2} ) \;dx = \frac{x^3}{3} - x + C$

Question 8. $\int (ax^2 + bx + c) \;dx$

Answer:

Solution:


We need to find the integral $\int (ax^2 + bx + c) \;dx$.

Using the linearity property of integration, we can integrate each term separately:

$\int (ax^2 + bx + c) \;dx = \int ax^2 \;dx + \int bx \;dx + \int c \;dx$

We can take the constant factors out of the integrals:

$= a \int x^2 \;dx + b \int x \;dx + c \int dx$

Now, we use the power rule for integration, $\int x^n \;dx = \frac{x^{n+1}}{n+1} + C$ (for $n \neq -1$), and the rule $\int dx = x + C$.

For the first term, $n=2$:

$\int x^2 \;dx = \frac{x^{2+1}}{2+1} = \frac{x^3}{3}$ (plus constant)

For the second term, $n=1$:

$\int x \;dx = \frac{x^{1+1}}{1+1} = \frac{x^2}{2}$ (plus constant)

For the third term, which is a constant integral:

$\int c \;dx = cx$ (plus constant)

Combining the results and adding a single constant of integration C:

$= a \left(\frac{x^3}{3}\right) + b \left(\frac{x^2}{2}\right) + c(x) + C$

So, the integral is:

$\int (ax^2 + bx + c) \;dx = \frac{a}{3}x^3 + \frac{b}{2}x^2 + cx + C$

Question 9. $\int (2x^2 + e^x) \; dx$

Answer:

Solution:


We need to find the integral $\int (2x^2 + e^x) \; dx$.

Using the linearity property of integration, we can split the integral into two parts:

$\int (2x^2 + e^x) \; dx = \int 2x^2 \; dx + \int e^x \; dx$

We can take the constant factor out of the first integral:

$= 2 \int x^2 \; dx + \int e^x \; dx$

Now, we use the standard integral formulas: the power rule $\int x^n \;dx = \frac{x^{n+1}}{n+1} + C$ (for $n \neq -1$), and $\int e^x \; dx = e^x + C$.

For the first integral, $n=2$. So, $\int x^2 \; dx = \frac{x^{2+1}}{2+1} = \frac{x^3}{3}$ (plus constant).

For the second integral, $\int e^x \; dx = e^x$ (plus constant).

Combining the results and adding a single constant of integration C:

$= 2 \left(\frac{x^3}{3}\right) + e^x + C$

So, the integral is:

$\int (2x^2 + e^x) \; dx = \frac{2}{3}x^3 + e^x + C$

Question 10. $\int \left( \sqrt{x} − \frac{1}{\sqrt{x}} \right)^2\;dx$

Answer:

Solution:


We need to find the integral $\int \left( \sqrt{x} − \frac{1}{\sqrt{x}} \right)^2\;dx$.

First, we expand the expression inside the integral:

$\left( \sqrt{x} − \frac{1}{\sqrt{x}} \right)^2 = (\sqrt{x})^2 - 2 \cdot \sqrt{x} \cdot \frac{1}{\sqrt{x}} + \left(\frac{1}{\sqrt{x}}\right)^2$

$= x - 2 \cdot 1 + \frac{1}{x}$

$= x - 2 + \frac{1}{x}$

Now, we integrate the expanded expression:

$\int \left( x - 2 + \frac{1}{x} \right) \;dx$

Using the linearity property of integration, we integrate each term separately:

$= \int x \;dx - \int 2 \;dx + \int \frac{1}{x} \;dx$

We use the power rule for integration, $\int x^n \;dx = \frac{x^{n+1}}{n+1} + C$ (for $n \neq -1$), the integral of a constant, $\int c \;dx = cx + C$, and the integral of $\frac{1}{x}$, $\int \frac{1}{x} \;dx = \log|x| + C$.

For the first term ($\int x \;dx$), $n=1$: $\frac{x^{1+1}}{1+1} = \frac{x^2}{2}$.

For the second term ($\int 2 \;dx$): $2x$.

For the third term ($\int \frac{1}{x} \;dx$): $\log|x|$.

Combining these results and adding the constant of integration C:

$\int \left( \sqrt{x} − \frac{1}{\sqrt{x}} \right)^2\;dx = \frac{x^2}{2} - 2x + \log|x| + C$

Question 11. $\int \frac{x^3 + 5x^2 − 4}{x^2} \; dx$

Answer:

Solution:


We need to find the integral $\int \frac{x^3 + 5x^2 − 4}{x^2} \; dx$.

First, we simplify the integrand by dividing each term in the numerator by the denominator:

$\frac{x^3 + 5x^2 − 4}{x^2} = \frac{x^3}{x^2} + \frac{5x^2}{x^2} - \frac{4}{x^2}$

$= x + 5 - 4x^{-2}$

Now, we integrate the simplified expression:

$\int (x + 5 - 4x^{-2}) \; dx$

Using the linearity property of integration, we integrate each term separately:

$= \int x \; dx + \int 5 \; dx - \int 4x^{-2} \; dx$

$= \int x^1 \; dx + \int 5 \; dx - 4 \int x^{-2} \; dx$

We use the power rule for integration, $\int x^n \;dx = \frac{x^{n+1}}{n+1} + C$ (for $n \neq -1$), and the integral of a constant, $\int c \;dx = cx + C$.

For $\int x^1 \; dx$ ($n=1$): $\frac{x^{1+1}}{1+1} = \frac{x^2}{2}$.

For $\int 5 \; dx$: $5x$.

For $\int x^{-2} \; dx$ ($n=-2$): $\frac{x^{-2+1}}{-2+1} = \frac{x^{-1}}{-1} = -x^{-1} = -\frac{1}{x}$.

Combining these results with the constant factors and adding the constant of integration C:

$= \frac{x^2}{2} + 5x - 4\left(-\frac{1}{x}\right) + C$

$= \frac{x^2}{2} + 5x + \frac{4}{x} + C$

So, the integral is:

$\int \frac{x^3 + 5x^2 − 4}{x^2} \; dx = \frac{x^2}{2} + 5x + \frac{4}{x} + C$

Question 12. $\int \frac{x^3 + 3x + 4}{\sqrt{x}}\; dx$

Answer:

Solution:


We need to find the integral $\int \frac{x^3 + 3x + 4}{\sqrt{x}}\; dx$.

First, we simplify the integrand by dividing each term in the numerator by $\sqrt{x} = x^{1/2}$:

$\frac{x^3 + 3x + 4}{\sqrt{x}} = \frac{x^3}{x^{1/2}} + \frac{3x}{x^{1/2}} + \frac{4}{x^{1/2}}$

Using the rule $\frac{x^a}{x^b} = x^{a-b}$:

$= x^{3 - 1/2} + 3x^{1 - 1/2} + 4x^{0 - 1/2}$

$= x^{6/2 - 1/2} + 3x^{2/2 - 1/2} + 4x^{-1/2}$

$= x^{5/2} + 3x^{1/2} + 4x^{-1/2}$

Now, we integrate the simplified expression term by term:

$\int \left( x^{5/2} + 3x^{1/2} + 4x^{-1/2} \right) \; dx$

Using the linearity property and the power rule for integration, $\int x^n \;dx = \frac{x^{n+1}}{n+1} + C$ (for $n \neq -1$):

$= \int x^{5/2} \; dx + \int 3x^{1/2} \; dx + \int 4x^{-1/2} \; dx$

$= \int x^{5/2} \; dx + 3 \int x^{1/2} \; dx + 4 \int x^{-1/2} \; dx$

For $\int x^{5/2} \; dx$ ($n=5/2$): $\frac{x^{5/2+1}}{5/2+1} = \frac{x^{7/2}}{7/2} = \frac{2}{7}x^{7/2}$.

For $\int x^{1/2} \; dx$ ($n=1/2$): $\frac{x^{1/2+1}}{1/2+1} = \frac{x^{3/2}}{3/2} = \frac{2}{3}x^{3/2}$.

For $\int x^{-1/2} \; dx$ ($n=-1/2$): $\frac{x^{-1/2+1}}{-1/2+1} = \frac{x^{1/2}}{1/2} = 2x^{1/2}$.

Combining these results with the constant factors and adding the constant of integration C:

$= \frac{2}{7}x^{7/2} + 3 \left(\frac{2}{3}x^{3/2}\right) + 4 \left(2x^{1/2}\right) + C$

$= \frac{2}{7}x^{7/2} + 2x^{3/2} + 8x^{1/2} + C$

So, the integral is:

$\int \frac{x^3 + 3x + 4}{\sqrt{x}}\; dx = \frac{2}{7}x^{7/2} + 2x^{3/2} + 8\sqrt{x} + C$

Question 13. $\int \frac{x^3 − x^2 + x − 1}{x − 1}\; dx$

Answer:

Solution:


We need to find the integral $\int \frac{x^3 − x^2 + x − 1}{x − 1}\; dx$.

First, we simplify the integrand by factoring the numerator. We can group terms in the numerator:

$x^3 − x^2 + x − 1 = (x^3 − x^2) + (x − 1)$

$= x^2(x − 1) + 1(x − 1)$

$= (x^2 + 1)(x − 1)$

So the integrand becomes:

$\frac{x^3 − x^2 + x − 1}{x − 1} = \frac{(x^2 + 1)(x − 1)}{x − 1}$

For $x \neq 1$, we can cancel the $(x-1)$ term:

$= x^2 + 1$

Now, we integrate the simplified expression:

$\int (x^2 + 1) \; dx$

Using the linearity property of integration, we integrate each term separately:

$= \int x^2 \; dx + \int 1 \; dx$

We use the power rule for integration, $\int x^n \;dx = \frac{x^{n+1}}{n+1} + C$ (for $n \neq -1$), and the rule for the integral of a constant, $\int c \;dx = cx + C$.

For $\int x^2 \; dx$ ($n=2$): $\frac{x^{2+1}}{2+1} = \frac{x^3}{3}$ (plus constant).

For $\int 1 \; dx$: $x$ (plus constant).

Combining these results and adding a single constant of integration C:

$= \frac{x^3}{3} + x + C$

So, the integral is:

$\int \frac{x^3 − x^2 + x − 1}{x − 1}\; dx = \frac{x^3}{3} + x + C$

Question 14. $\int (1 − x) \sqrt{x}\; dx$

Answer:

Solution:


We need to find the integral $\int (1 − x) \sqrt{x}\; dx$.

First, we simplify the integrand by multiplying $(1-x)$ by $\sqrt{x} = x^{1/2}$:

$(1 − x) \sqrt{x} = 1 \cdot x^{1/2} - x \cdot x^{1/2}$

Using the rule $x^a \cdot x^b = x^{a+b}$:

$= x^{1/2} - x^{1 + 1/2}$

$= x^{1/2} - x^{3/2}$

Now, we integrate the simplified expression term by term:

$\int \left( x^{1/2} - x^{3/2} \right) \; dx$

Using the linearity property and the power rule for integration, $\int x^n \;dx = \frac{x^{n+1}}{n+1} + C$ (for $n \neq -1$):

$= \int x^{1/2} \; dx - \int x^{3/2} \; dx$

For $\int x^{1/2} \; dx$ ($n=1/2$): $\frac{x^{1/2+1}}{1/2+1} = \frac{x^{3/2}}{3/2} = \frac{2}{3}x^{3/2}$.

For $\int x^{3/2} \; dx$ ($n=3/2$): $\frac{x^{3/2+1}}{3/2+1} = \frac{x^{5/2}}{5/2} = \frac{2}{5}x^{5/2}$.

Combining these results and adding the constant of integration C:

$= \frac{2}{3}x^{3/2} - \frac{2}{5}x^{5/2} + C$

So, the integral is:

$\int (1 − x) \sqrt{x}\; dx = \frac{2}{3}x^{3/2} - \frac{2}{5}x^{5/2} + C$

Question 15. $\int \sqrt{x}(3x^2 + 2x + 3)\; dx$

Answer:

Solution:


We need to find the integral $\int \sqrt{x}(3x^2 + 2x + 3)\; dx$.

First, we simplify the integrand by multiplying $(3x^2 + 2x + 3)$ by $\sqrt{x} = x^{1/2}$:

$\sqrt{x}(3x^2 + 2x + 3) = x^{1/2}(3x^2) + x^{1/2}(2x) + x^{1/2}(3)$

Using the rule $x^a \cdot x^b = x^{a+b}$:

$= 3x^{1/2 + 2} + 2x^{1/2 + 1} + 3x^{1/2}$

$= 3x^{1/2 + 4/2} + 2x^{1/2 + 2/2} + 3x^{1/2}$

$= 3x^{5/2} + 2x^{3/2} + 3x^{1/2}$

Now, we integrate the simplified expression term by term:

$\int \left( 3x^{5/2} + 2x^{3/2} + 3x^{1/2} \right) \; dx$

Using the linearity property and the power rule for integration, $\int x^n \;dx = \frac{x^{n+1}}{n+1} + C$ (for $n \neq -1$):

$= \int 3x^{5/2} \; dx + \int 2x^{3/2} \; dx + \int 3x^{1/2} \; dx$

$= 3 \int x^{5/2} \; dx + 2 \int x^{3/2} \; dx + 3 \int x^{1/2} \; dx$

For $\int x^{5/2} \; dx$ ($n=5/2$): $\frac{x^{5/2+1}}{5/2+1} = \frac{x^{7/2}}{7/2} = \frac{2}{7}x^{7/2}$.

For $\int x^{3/2} \; dx$ ($n=3/2$): $\frac{x^{3/2+1}}{3/2+1} = \frac{x^{5/2}}{5/2} = \frac{2}{5}x^{5/2}$.

For $\int x^{1/2} \; dx$ ($n=1/2$): $\frac{x^{1/2+1}}{1/2+1} = \frac{x^{3/2}}{3/2} = \frac{2}{3}x^{3/2}$.

Combining these results with the constant factors and adding the constant of integration C:

$= 3 \left(\frac{2}{7}x^{7/2}\right) + 2 \left(\frac{2}{5}x^{5/2}\right) + 3 \left(\frac{2}{3}x^{3/2}\right) + C$

$= \frac{6}{7}x^{7/2} + \frac{4}{5}x^{5/2} + 2x^{3/2} + C$

So, the integral is:

$\int \sqrt{x}(3x^2 + 2x + 3)\; dx = \frac{6}{7}x^{7/2} + \frac{4}{5}x^{5/2} + 2x^{3/2} + C$

Question 16. $\int (2x − 3 \cos x + e^x)\; dx$

Answer:

Solution:


We need to find the integral $\int (2x − 3 \cos x + e^x)\; dx$.

Using the linearity property of integration, we can split the integral into separate integrals for each term:

$\int (2x − 3 \cos x + e^x)\; dx = \int 2x \; dx - \int 3 \cos x \; dx + \int e^x \; dx$

Take the constant factors out of the first two integrals:

$= 2 \int x \; dx - 3 \int \cos x \; dx + \int e^x \; dx$

Now, use the standard integral formulas: $\int x^n \;dx = \frac{x^{n+1}}{n+1} + C$ (for $n \neq -1$), $\int \cos x \; dx = \sin x + C$, and $\int e^x \; dx = e^x + C$.

For $\int x \; dx$ ($n=1$): $\frac{x^{1+1}}{1+1} = \frac{x^2}{2}$ (plus constant).

For $\int \cos x \; dx$: $\sin x$ (plus constant).

For $\int e^x \; dx$: $e^x$ (plus constant).

Combine these results with the constant factors and add a single constant of integration C:

$= 2 \left(\frac{x^2}{2}\right) - 3 (\sin x) + e^x + C$

Simplify the expression:

$= x^2 - 3 \sin x + e^x + C$

So, the integral is:

$\int (2x − 3 \cos x + e^x)\; dx = x^2 - 3 \sin x + e^x + C$

Question 17. $\int (2x^2 − 3 \sin x + 5 \sqrt{x}) \;dx$

Answer:

Solution:


We need to find the integral $\int (2x^2 − 3 \sin x + 5 \sqrt{x}) \;dx$.

First, rewrite $\sqrt{x}$ as $x^{1/2}$:

$\int (2x^2 − 3 \sin x + 5 x^{1/2}) \;dx$

Using the linearity property of integration, we split the integral into separate integrals for each term:

$= \int 2x^2 \; dx - \int 3 \sin x \; dx + \int 5 x^{1/2} \; dx$

Take the constant factors out of each integral:

$= 2 \int x^2 \; dx - 3 \int \sin x \; dx + 5 \int x^{1/2} \; dx$

Now, use the standard integral formulas: $\int x^n \;dx = \frac{x^{n+1}}{n+1} + C$ (for $n \neq -1$) and $\int \sin x \; dx = -\cos x + C$.

For $\int x^2 \; dx$ ($n=2$): $\frac{x^{2+1}}{2+1} = \frac{x^3}{3}$ (plus constant).

For $\int \sin x \; dx$: $-\cos x$ (plus constant).

For $\int x^{1/2} \; dx$ ($n=1/2$): $\frac{x^{1/2+1}}{1/2+1} = \frac{x^{3/2}}{3/2} = \frac{2}{3}x^{3/2}$ (plus constant).

Combine these results with the constant factors and add a single constant of integration C:

$= 2 \left(\frac{x^3}{3}\right) - 3 (-\cos x) + 5 \left(\frac{2}{3}x^{3/2}\right) + C$

Simplify the expression:

$= \frac{2}{3}x^3 + 3 \cos x + \frac{10}{3}x^{3/2} + C$

So, the integral is:

$\int (2x^2 − 3 \sin x + 5 \sqrt{x}) \;dx = \frac{2}{3}x^3 + 3 \cos x + \frac{10}{3}x^{3/2} + C$

Question 18. $\int \sec x (\sec x + \tan x)\; dx$

Answer:

Solution:


We need to find the integral $\int \sec x (\sec x + \tan x)\; dx$.

First, we simplify the integrand by multiplying $\sec x$ through the parenthesis:

$\sec x (\sec x + \tan x) = \sec^2 x + \sec x \tan x$

Now, we integrate the simplified expression:

$\int (\sec^2 x + \sec x \tan x)\; dx$

Using the linearity property of integration, we can split the integral into separate integrals for each term:

$= \int \sec^2 x \; dx + \int \sec x \tan x \; dx$

Now, we use the standard integral formulas: $\int \sec^2 x \; dx = \tan x + C_1$ and $\int \sec x \tan x \; dx = \sec x + C_2$.

Combine these results and add a single constant of integration C ($C = C_1 + C_2$):

$= \tan x + \sec x + C$

So, the integral is:

$\int \sec x (\sec x + \tan x)\; dx = \tan x + \sec x + C$

Question 19. $\int \frac{\sec^2 x}{cosec^2 \;x}\; dx$

Answer:

Solution:


We need to find the integral $\int \frac{\sec^2 x}{\text{cosec}^2 \;x}\; dx$.

First, we simplify the integrand using the identities $\sec x = \frac{1}{\cos x}$ and $\text{cosec}\; x = \frac{1}{\sin x}$:

$\frac{\sec^2 x}{\text{cosec}^2 \;x} = \frac{\frac{1}{\cos^2 x}}{\frac{1}{\sin^2 x}}$

$= \frac{1}{\cos^2 x} \cdot \frac{\sin^2 x}{1}$

$= \frac{\sin^2 x}{\cos^2 x}$

Using the identity $\tan x = \frac{\sin x}{\cos x}$:

$= \tan^2 x$

Now, we need to integrate $\tan^2 x$. We use the trigonometric identity $\tan^2 x + 1 = \sec^2 x$, which implies $\tan^2 x = \sec^2 x - 1$.

So the integral becomes:

$\int (\sec^2 x - 1) \; dx$

Using the linearity property of integration, we integrate each term separately:

$= \int \sec^2 x \; dx - \int 1 \; dx$

Now, use the standard integral formulas: $\int \sec^2 x \; dx = \tan x + C_1$ and $\int 1 \; dx = x + C_2$.

Combine these results and add a single constant of integration C ($C = C_1 - C_2$):

$= \tan x - x + C$

So, the integral is:

$\int \frac{\sec^2 x}{\text{cosec}^2 \;x}\; dx = \tan x - x + C$

Question 20. $\int \frac{2 − 3 \sin x}{\cos^2 x}\; dx$

Answer:

Solution:


We need to find the integral $\int \frac{2 − 3 \sin x}{\cos^2 x}\; dx$.

First, we simplify the integrand by splitting the fraction:

$\frac{2 − 3 \sin x}{\cos^2 x} = \frac{2}{\cos^2 x} - \frac{3 \sin x}{\cos^2 x}$

Using trigonometric identities, $\frac{1}{\cos^2 x} = \sec^2 x$ and $\frac{\sin x}{\cos^2 x} = \frac{\sin x}{\cos x} \cdot \frac{1}{\cos x} = \tan x \sec x$:

$= 2 \sec^2 x - 3 \frac{\sin x}{\cos x} \frac{1}{\cos x}$

$= 2 \sec^2 x - 3 \tan x \sec x$

Now, we integrate the simplified expression:

$\int (2 \sec^2 x - 3 \sec x \tan x)\; dx$

Using the linearity property of integration, we split the integral into separate integrals for each term:

$= \int 2 \sec^2 x \; dx - \int 3 \sec x \tan x \; dx$

Take the constant factors out of each integral:

$= 2 \int \sec^2 x \; dx - 3 \int \sec x \tan x \; dx$

Now, use the standard integral formulas: $\int \sec^2 x \; dx = \tan x + C_1$ and $\int \sec x \tan x \; dx = \sec x + C_2$.

Combine these results with the constant factors and add a single constant of integration C ($C = 2C_1 - 3C_2$):

$= 2 (\tan x) - 3 (\sec x) + C$

$= 2 \tan x - 3 \sec x + C$

So, the integral is:

$\int \frac{2 − 3 \sin x}{\cos^2 x}\; dx = 2 \tan x - 3 \sec x + C$

Choose the correct answer in Exercises 21 and 22.

Question 21. The anti derivative of $\left( \sqrt{x} + \frac{1}{\sqrt{x}} \right)$ equals

(A) $\frac{1}{3} x^{\frac{1}{3}} + 2x^{\frac{1}{2}} + C$

(B) $\frac{2}{3} x^{\frac{2}{3}} + \frac{1}{2} x^{2} + C$

(C) $\frac{2}{3} x^{\frac{3}{2}} + 2x^{\frac{1}{2}} + C$

(D) $\frac{3}{2} x^{\frac{3}{2}} + \frac{1}{2} x^{\frac{1}{2}} + C$

Answer:

Solution:


We need to find the antiderivative of the function $f(x) = \sqrt{x} + \frac{1}{\sqrt{x}}$.

The antiderivative is given by the indefinite integral $\int f(x) \; dx$.

Rewrite the function using exponents: $f(x) = x^{1/2} + x^{-1/2}$.

Now, integrate the function:

$\int \left( x^{1/2} + x^{-1/2} \right) \; dx$

Using the linearity of the integral, we integrate term by term:

$= \int x^{1/2} \; dx + \int x^{-1/2} \; dx$

Apply the power rule for integration, $\int x^n \; dx = \frac{x^{n+1}}{n+1} + C$, for $n \neq -1$:

For the first term, $n = 1/2$:

$\int x^{1/2} \; dx = \frac{x^{1/2 + 1}}{1/2 + 1} + C_1 = \frac{x^{3/2}}{3/2} + C_1 = \frac{2}{3}x^{3/2} + C_1$

For the second term, $n = -1/2$:

$\int x^{-1/2} \; dx = \frac{x^{-1/2 + 1}}{-1/2 + 1} + C_2 = \frac{x^{1/2}}{1/2} + C_2 = 2x^{1/2} + C_2$

Combining the results and letting $C = C_1 + C_2$ be the constant of integration:

$\int \left( x^{1/2} + x^{-1/2} \right) \; dx = \frac{2}{3}x^{3/2} + 2x^{1/2} + C$

Comparing this result with the given options, we see that it matches option (C).


The correct answer is (C) $\frac{2}{3} x^{\frac{3}{2}} + 2x^{\frac{1}{2}} + C$.

Question 22. If $\frac{d}{dx} f(x) = 4x^3 - \frac{3}{x^4}$ such that f(2) = 0. then f(x) is

(A) $x^4 + \frac{1}{x^3} - \frac{129}{8}$

(B) $x^3 + \frac{1}{x^4} + \frac{129}{8}$

(C) $x^4 + \frac{1}{x^3} + \frac{129}{8}$

(D) $x^3 + \frac{1}{x^4} - \frac{129}{8}$

Answer:

Solution:


We are given that the derivative of a function $f(x)$ is $\frac{d}{dx} f(x) = 4x^3 - \frac{3}{x^4}$.

To find the function $f(x)$, we need to find the antiderivative (integral) of the given derivative:

$f(x) = \int \left( 4x^3 - \frac{3}{x^4} \right) \; dx$

Rewrite the term $\frac{3}{x^4}$ as $3x^{-4}$:

$f(x) = \int (4x^3 - 3x^{-4}) \; dx$

Using the linearity property of integration, we integrate each term separately:

$f(x) = \int 4x^3 \; dx - \int 3x^{-4} \; dx$

Take the constant factors out of the integrals:

$f(x) = 4 \int x^3 \; dx - 3 \int x^{-4} \; dx$

Apply the power rule for integration, $\int x^n \; dx = \frac{x^{n+1}}{n+1} + C$ (for $n \neq -1$):

For $\int x^3 \; dx$ ($n=3$): $\frac{x^{3+1}}{3+1} = \frac{x^4}{4}$ (plus constant).

For $\int x^{-4} \; dx$ ($n=-4$): $\frac{x^{-4+1}}{-4+1} = \frac{x^{-3}}{-3}$ (plus constant).

Combine these results with the constant factors and add a single constant of integration C:

$f(x) = 4 \left(\frac{x^4}{4}\right) - 3 \left(\frac{x^{-3}}{-3}\right) + C$

Simplify the expression:

$f(x) = x^4 - (-\frac{3}{3}x^{-3}) + C$

$f(x) = x^4 - (-x^{-3}) + C$

$f(x) = x^4 + x^{-3} + C$

We can write $x^{-3}$ as $\frac{1}{x^3}$:

$f(x) = x^4 + \frac{1}{x^3} + C$

We are given the condition that $f(2) = 0$. Use this to find the value of C.

Substitute $x=2$ into the expression for $f(x)$:

$f(2) = (2)^4 + \frac{1}{(2)^3} + C$

Since $f(2) = 0$, we have:

$0 = 16 + \frac{1}{8} + C$

To find C, isolate C:

$C = -16 - \frac{1}{8}$

Find a common denominator:

$C = -\frac{16 \cdot 8}{8} - \frac{1}{8}$

$C = -\frac{128}{8} - \frac{1}{8}$

$C = -\frac{128 + 1}{8}$

$C = -\frac{129}{8}$

Substitute the value of C back into the expression for $f(x)$:

$f(x) = x^4 + \frac{1}{x^3} - \frac{129}{8}$

Compare this result with the given options. It matches option (A).


The correct answer is (A) $x^4 + \frac{1}{x^3} - \frac{129}{8}$.



Example 5 & 6 (Before Exercise 7.2)

Example 5: Integrate the following functions w.r.t. x:

(i) sin mx

(ii) 2x sin (x2 + 1)

(iii) $\frac{\tan^4 \sqrt{x} \sec^2 \sqrt{x}}{\sqrt{x}}$

(iv) $\frac{\sin (\tan^{−1} x)}{1 + x^2}$

Answer:

Solution:


(i) We need to integrate $\sin(mx)$ with respect to x, i.e., find $\int \sin(mx) \; dx$.

We use the method of substitution. Let $u = mx$.

Differentiating both sides with respect to x, we get $\frac{du}{dx} = m$.

This implies $du = m \; dx$, or $dx = \frac{1}{m} du$.

Substitute $u = mx$ and $dx = \frac{1}{m} du$ into the integral:

$\int \sin(mx) \; dx = \int \sin(u) \cdot \frac{1}{m} \; du$

We can take the constant $\frac{1}{m}$ out of the integral:

$= \frac{1}{m} \int \sin(u) \; du$

We know the standard integral $\int \sin(u) \; du = -\cos(u) + C'$, where $C'$ is the constant of integration.

$= \frac{1}{m} (-\cos(u)) + C$

$= -\frac{1}{m} \cos(u) + C$

Now, substitute back $u = mx$:

$= -\frac{1}{m} \cos(mx) + C$

Thus, $\int \sin(mx) \; dx = -\frac{1}{m} \cos(mx) + C$.


(ii) We need to integrate $2x \sin(x^2 + 1)$ with respect to x, i.e., find $\int 2x \sin(x^2 + 1) \; dx$.

We use the method of substitution. Let $u = x^2 + 1$.

Differentiating both sides with respect to x, we get $\frac{du}{dx} = \frac{d}{dx}(x^2 + 1) = 2x$.

This implies $du = 2x \; dx$.

Substitute $u = x^2 + 1$ and $du = 2x \; dx$ into the integral:

$\int \sin(x^2 + 1) \cdot (2x \; dx) = \int \sin(u) \; du$

We know the standard integral $\int \sin(u) \; du = -\cos(u) + C$, where $C$ is the constant of integration.

$= -\cos(u) + C$

Now, substitute back $u = x^2 + 1$:

$= -\cos(x^2 + 1) + C$

Thus, $\int 2x \sin(x^2 + 1) \; dx = -\cos(x^2 + 1) + C$.


(iii) We need to integrate $\frac{\tan^4 \sqrt{x} \sec^2 \sqrt{x}}{\sqrt{x}}$ with respect to x, i.e., find $\int \frac{\tan^4 \sqrt{x} \sec^2 \sqrt{x}}{\sqrt{x}} \; dx$.

We use the method of substitution. Let $v = \tan(\sqrt{x})$.

Differentiating both sides with respect to x, we use the chain rule:

$\frac{dv}{dx} = \frac{d}{dx}(\tan(\sqrt{x})) = \sec^2(\sqrt{x}) \cdot \frac{d}{dx}(\sqrt{x})$

We know that $\frac{d}{dx}(\sqrt{x}) = \frac{d}{dx}(x^{1/2}) = \frac{1}{2}x^{1/2 - 1} = \frac{1}{2}x^{-1/2} = \frac{1}{2\sqrt{x}}$.

So, $\frac{dv}{dx} = \sec^2(\sqrt{x}) \cdot \frac{1}{2\sqrt{x}} = \frac{\sec^2 \sqrt{x}}{2\sqrt{x}}$.

This implies $dv = \frac{\sec^2 \sqrt{x}}{2\sqrt{x}} \; dx$, or $2 \; dv = \frac{\sec^2 \sqrt{x}}{\sqrt{x}} \; dx$.

Rewrite the integral as $\int (\tan(\sqrt{x}))^4 \cdot \frac{\sec^2 \sqrt{x}}{\sqrt{x}} \; dx$.

Substitute $v = \tan(\sqrt{x})$ and $2 \; dv = \frac{\sec^2 \sqrt{x}}{\sqrt{x}} \; dx$ into the integral:

$\int v^4 \cdot (2 \; dv) = 2 \int v^4 \; dv$

Apply the power rule for integration, $\int v^n \; dv = \frac{v^{n+1}}{n+1} + C'$:

$= 2 \cdot \frac{v^{4+1}}{4+1} + C$

$= 2 \cdot \frac{v^5}{5} + C$

$= \frac{2}{5} v^5 + C$

Now, substitute back $v = \tan(\sqrt{x})$:

$= \frac{2}{5} (\tan(\sqrt{x}))^5 + C$

$= \frac{2}{5} \tan^5 \sqrt{x} + C$

Thus, $\int \frac{\tan^4 \sqrt{x} \sec^2 \sqrt{x}}{\sqrt{x}} \; dx = \frac{2}{5} \tan^5 \sqrt{x} + C$.


(iv) We need to integrate $\frac{\sin (\tan^{−1} x)}{1 + x^2}$ with respect to x, i.e., find $\int \frac{\sin (\tan^{−1} x)}{1 + x^2} \; dx$.

We use the method of substitution. Let $u = \tan^{-1} x$.

Differentiating both sides with respect to x, we get $\frac{du}{dx} = \frac{d}{dx}(\tan^{-1} x) = \frac{1}{1 + x^2}$.

This implies $du = \frac{1}{1 + x^2} \; dx$.

Rewrite the integral as $\int \sin(\tan^{-1} x) \cdot \frac{1}{1 + x^2} \; dx$.

Substitute $u = \tan^{-1} x$ and $du = \frac{1}{1 + x^2} \; dx$ into the integral:

$\int \sin(u) \; du$

We know the standard integral $\int \sin(u) \; du = -\cos(u) + C$, where $C$ is the constant of integration.

$= -\cos(u) + C$

Now, substitute back $u = \tan^{-1} x$:

$= -\cos(\tan^{-1} x) + C$

Thus, $\int \frac{\sin (\tan^{−1} x)}{1 + x^2} \; dx = -\cos(\tan^{-1} x) + C$.

Example 6: Find the following integrals:

(i) $\int \sin^3 x \cos^2 x \;dx$

(ii) $\int \frac{\sin x}{\sin (x + a)} \;dx$

(iii) $\int \frac{1}{1 + \tan x} \;dx$

Answer:

Solution:


(i) We need to find $\int \sin^3 x \cos^2 x \;dx$.

We can rewrite $\sin^3 x$ as $\sin^2 x \sin x$. Using the identity $\sin^2 x = 1 - \cos^2 x$, the integral becomes:

$\int (1 - \cos^2 x) \cos^2 x \sin x \;dx$

Let $u = \cos x$. Then $du = -\sin x \; dx$. So, $\sin x \; dx = -du$.

Substitute $u$ and $du$ into the integral:

$\int (1 - u^2) u^2 (-du)$

$= -\int (u^2 - u^4) \;du$

$= \int (u^4 - u^2) \;du$

Now, integrate with respect to $u$ using the power rule $\int u^n du = \frac{u^{n+1}}{n+1} + C'$:

$= \frac{u^{4+1}}{4+1} - \frac{u^{2+1}}{2+1} + C$

$= \frac{u^5}{5} - \frac{u^3}{3} + C$

Substitute back $u = \cos x$:

$= \frac{\cos^5 x}{5} - \frac{\cos^3 x}{3} + C$

Thus, $\int \sin^3 x \cos^2 x \;dx = \frac{\cos^5 x}{5} - \frac{\cos^3 x}{3} + C$.


(ii) We need to find $\int \frac{\sin x}{\sin (x + a)} \;dx$.

Let $u = x + a$. Then $du = dx$. Also, $x = u - a$, so $\sin x = \sin(u - a)$.

Substitute $u$ and $dx = du$ into the integral:

$\int \frac{\sin (u - a)}{\sin u} \;du$

Use the trigonometric identity for $\sin(A - B) = \sin A \cos B - \cos A \sin B$:

$\sin(u - a) = \sin u \cos a - \cos u \sin a$

So the integral becomes:

$\int \frac{\sin u \cos a - \cos u \sin a}{\sin u} \;du$

Split the fraction into two terms:

$= \int \left( \frac{\sin u \cos a}{\sin u} - \frac{\cos u \sin a}{\sin u} \right) \;du$

For $u$ where $\sin u \neq 0$, this simplifies to:

$= \int (\cos a - \cot u \sin a) \;du$

Since $a$ is a constant, $\cos a$ and $\sin a$ are constants. Use linearity of integration:

$= \int \cos a \;du - \int \cot u \sin a \;du$

$= \cos a \int du - \sin a \int \cot u \;du$

We know $\int du = u + C_1$ and $\int \cot u \;du = \log |\sin u| + C_2$.

$= \cos a (u) - \sin a (\log |\sin u|) + C$

Substitute back $u = x + a$:

$= (x + a) \cos a - \sin a \log |\sin (x + a)| + C$

Thus, $\int \frac{\sin x}{\sin (x + a)} \;dx = (x + a) \cos a - \sin a \log |\sin (x + a)| + C$.


(iii) We need to find $\int \frac{1}{1 + \tan x} \;dx$.

Rewrite $\tan x$ in terms of $\sin x$ and $\cos x$:

$\int \frac{1}{1 + \frac{\sin x}{\cos x}} \;dx = \int \frac{1}{\frac{\cos x + \sin x}{\cos x}} \;dx = \int \frac{\cos x}{\cos x + \sin x} \;dx$

We can write the numerator $\cos x$ as a linear combination of the denominator $(\cos x + \sin x)$ and its derivative $(\cos x - \sin x)$.

Let $\cos x = A(\cos x + \sin x) + B(\cos x - \sin x)$.

$\cos x = (A+B)\cos x + (A-B)\sin x$

Comparing coefficients of $\cos x$ and $\sin x$ on both sides:

$A + B = 1$

$A - B = 0$

Adding the two equations, $2A = 1 \implies A = \frac{1}{2}$.

Substituting $A = \frac{1}{2}$ into $A - B = 0$, we get $\frac{1}{2} - B = 0 \implies B = \frac{1}{2}$.

So, $\cos x = \frac{1}{2}(\cos x + \sin x) + \frac{1}{2}(\cos x - \sin x)$.

Substitute this back into the integrand:

$\frac{\cos x}{\cos x + \sin x} = \frac{\frac{1}{2}(\cos x + \sin x) + \frac{1}{2}(\cos x - \sin x)}{\cos x + \sin x}$

$= \frac{1}{2} \frac{\cos x + \sin x}{\cos x + \sin x} + \frac{1}{2} \frac{\cos x - \sin x}{\cos x + \sin x}$

$= \frac{1}{2} + \frac{1}{2} \frac{\cos x - \sin x}{\cos x + \sin x}$

Now integrate term by term:

$\int \left( \frac{1}{2} + \frac{1}{2} \frac{\cos x - \sin x}{\cos x + \sin x} \right) \;dx$

$= \int \frac{1}{2} \;dx + \int \frac{1}{2} \frac{\cos x - \sin x}{\cos x + \sin x} \;dx$

$= \frac{1}{2} \int dx + \frac{1}{2} \int \frac{\cos x - \sin x}{\cos x + \sin x} \;dx$

The first integral is $\frac{1}{2}x$. For the second integral, let $v = \cos x + \sin x$. Then $dv = (-\sin x + \cos x) \;dx = (\cos x - \sin x) \;dx$.

The second integral becomes $\int \frac{1}{2} \frac{1}{v} \;dv = \frac{1}{2} \int \frac{1}{v} \;dv = \frac{1}{2} \log |v| + C_1$.

Substitute back $v = \cos x + \sin x$:

$= \frac{1}{2} \log |\cos x + \sin x| + C_1$

Combining the integrals and adding the constant of integration C:

$= \frac{1}{2}x + \frac{1}{2} \log |\cos x + \sin x| + C$

Thus, $\int \frac{1}{1 + \tan x} \;dx = \frac{1}{2}x + \frac{1}{2} \log |\cos x + \sin x| + C$.



Exercise 7.2

Integrate the functions in Exercises 1 to 37

Question 1. $\frac{2x}{1 + x^2}$

Answer:

Let the given integral be $I$.

$I = \int \frac{2x}{1 + x^2} dx$


We use the method of substitution.

Let $u = 1 + x^2$.


Differentiating both sides with respect to $x$, we get:

$\frac{du}{dx} = \frac{d}{dx}(1 + x^2)$

$\frac{du}{dx} = 0 + 2x$

$\frac{du}{dx} = 2x$


Rearranging the terms, we get:

$du = 2x \, dx$


Now, substitute $u = 1 + x^2$ and $du = 2x \, dx$ into the integral $I$:

$I = \int \frac{1}{u} du$


The integral of $\frac{1}{u}$ with respect to $u$ is $\log_e |u| + C$, where $C$ is the constant of integration.

$I = \log_e |u| + C$


Substitute back $u = 1 + x^2$:

$I = \log_e |1 + x^2| + C$


Since $x^2 \geq 0$ for any real number $x$, $1 + x^2 \geq 1$. Therefore, $1 + x^2$ is always positive, and the absolute value is not necessary.

$I = \log_e (1 + x^2) + C$


Thus, the integral of the given function is $\log_e (1 + x^2) + C$.

Question 2. $\frac{(\log x)^2}{x}$

Answer:

To evaluate the integral $\int \frac{(\log x)^2}{x} dx$, we can use the method of substitution.


Let $u = \log |x|$.


Differentiating $u$ with respect to $x$, we get:

$\frac{du}{dx} = \frac{d}{dx}(\log |x|)$

$\frac{du}{dx} = \frac{1}{|x|} \times \text{sign}(x)$

$\frac{du}{dx} = \frac{1}{x}$ (for $x \neq 0$)


This means $du = \frac{1}{x} dx$.


Now, substitute $u = \log |x|$ and $du = \frac{1}{x} dx$ into the integral $\int \frac{(\log x)^2}{x} dx$. Note that $(\log x)^2 = (\log |x|)^2$ for $x \neq 0$. So the integral becomes:

$\int u^2 du$


Now, we integrate this expression with respect to $u$ using the power rule for integration, which states that $\int y^n dy = \frac{y^{n+1}}{n+1} + C$ (for $n \neq -1$).

$\int u^2 du = \frac{u^{2+1}}{2+1} + C$

$\int u^2 du = \frac{u^3}{3} + C$


Finally, substitute back $u = \log |x|$ to express the result in terms of $x$.

The integral is equal to:

$\frac{(\log |x|)^3}{3} + C$

Where $C$ is the constant of integration.


So, the solution is:

$\int \frac{(\log x)^2}{x} dx = \frac{1}{3} (\log |x|)^3 + C$

Question 3. $\frac{1}{x + x \log x}$

Answer:

Let the given integral be $I$.

$I = \int \frac{1}{x + x \log x} dx$


First, simplify the denominator by factoring out $x$:

$x + x \log x = x(1 + \log x)$


So the integral becomes:

$I = \int \frac{1}{x(1 + \log x)} dx$

$I = \int \frac{1}{(1 + \log x)} \cdot \frac{1}{x} dx$


We use the method of substitution.

Let $u = 1 + \log x$.


Differentiating both sides with respect to $x$, we get:

$\frac{du}{dx} = \frac{d}{dx}(1 + \log x)$

$\frac{du}{dx} = 0 + \frac{1}{x}$

$\frac{du}{dx} = \frac{1}{x}$


Rearranging the terms, we get:

$du = \frac{1}{x} dx$


Now, substitute $u = 1 + \log x$ and $du = \frac{1}{x} dx$ into the integral $I$:

$I = \int \frac{1}{u} du$


The integral of $\frac{1}{u}$ with respect to $u$ is $\log_e |u| + C$, where $C$ is the constant of integration.

$I = \log_e |u| + C$


Substitute back $u = 1 + \log x$:

$I = \log_e |1 + \log x| + C$


Thus, the integral of the given function is $\log_e |1 + \log x| + C$, where $C$ is the constant of integration.

Question 4. $\sin x \;\sin \;(\cos x)$

Answer:

Let the given integral be $I$.

$I = \int \sin x \;\sin \;(\cos x) \;dx$


We use the method of substitution.

Let $u = \cos x$.


Differentiating both sides with respect to $x$, we get:

$\frac{du}{dx} = \frac{d}{dx}(\cos x)$

$\frac{du}{dx} = -\sin x$


Rearranging the terms, we get:

$du = -\sin x \;dx$

So, $\sin x \;dx = -du$.


Now, substitute $u = \cos x$ and $\sin x \;dx = -du$ into the integral $I$:

$I = \int \sin(u) (-du)$

$I = - \int \sin(u) \;du$


The integral of $\sin(u)$ with respect to $u$ is $-\cos(u) + C$, where $C$ is the constant of integration.

$I = - (-\cos(u)) + C$

$I = \cos(u) + C$


Substitute back $u = \cos x$:

$I = \cos(\cos x) + C$


Thus, the integral of the given function is $\cos(\cos x) + C$, where $C$ is the constant of integration.

Question 5. $\sin(ax + b) \; \cos(ax + b)$

Answer:

Let the given integral be $I$.

$I = \int \sin(ax + b) \; \cos(ax + b) \;dx$


We can use the trigonometric identity $\sin(2\theta) = 2 \sin\theta \cos\theta$.

Let $\theta = ax + b$. Then the integrand is $\frac{1}{2} \sin(2(ax + b)) = \frac{1}{2} \sin(2ax + 2b)$.


The integral becomes:

$I = \int \frac{1}{2} \sin(2ax + 2b) \;dx$

$I = \frac{1}{2} \int \sin(2ax + 2b) \;dx$


Now, we integrate $\sin(2ax + 2b)$ with respect to $x$. We can use substitution for the argument of sine, let $u = 2ax + 2b$. Then $du = \frac{d}{dx}(2ax + 2b) \;dx = 2a \;dx$. This implies $dx = \frac{du}{2a}$.

So the integral $\int \sin(2ax + 2b) \;dx$ becomes $\int \sin(u) \frac{du}{2a} = \frac{1}{2a} \int \sin(u) \;du$.

The integral of $\sin(u)$ is $-\cos(u)$.

$\frac{1}{2a} \int \sin(u) \;du = \frac{1}{2a} (-\cos(u)) + C' = -\frac{1}{2a} \cos(u) + C'$.

Substitute back $u = 2ax + 2b$:

$-\frac{1}{2a} \cos(2ax + 2b) + C'$.


Now, substitute this back into the expression for $I$:

$I = \frac{1}{2} \left( -\frac{1}{2a} \cos(2ax + 2b) \right) + C$

$I = -\frac{1}{4a} \cos(2ax + 2b) + C$, where $C = \frac{C'}{2}$ is the constant of integration.


Thus, the integral of the given function is $\mathbf{-\frac{1}{4a} \cos(2ax + 2b) + C}$.


Alternate Solution (using substitution):

Let the given integral be $I$.

$I = \int \sin(ax + b) \; \cos(ax + b) \;dx$


We use the method of substitution.

Let $u = \sin(ax + b)$.


Differentiating both sides with respect to $x$, we get:

$\frac{du}{dx} = \frac{d}{dx}(\sin(ax + b))$

Using the chain rule, $\frac{du}{dx} = \cos(ax + b) \cdot \frac{d}{dx}(ax + b) = \cos(ax + b) \cdot a$

$\frac{du}{dx} = a \cos(ax + b)$


Rearranging the terms, we get:

$du = a \cos(ax + b) \;dx$

So, $\cos(ax + b) \;dx = \frac{du}{a}$.


Now, substitute $u = \sin(ax + b)$ and $\cos(ax + b) \;dx = \frac{du}{a}$ into the integral $I$:

$I = \int u \cdot \frac{du}{a}$

$I = \frac{1}{a} \int u \;du$


Integrate $u$ with respect to $u$ using the power rule for integration ($\int u^n du = \frac{u^{n+1}}{n+1} + C''$):

$I = \frac{1}{a} \left( \frac{u^{1+1}}{1+1} \right) + C$

$I = \frac{1}{a} \left( \frac{u^2}{2} \right) + C$

$I = \frac{u^2}{2a} + C$


Substitute back $u = \sin(ax + b)$:

$I = \frac{(\sin(ax + b))^2}{2a} + C$

$I = \frac{\sin^2(ax + b)}{2a} + C$, where $C$ is the constant of integration.


Both results obtained by Method 1 and Method 2 are equivalent, differing only by a constant.

Question 6. $\sqrt{ax + b}$

Answer:

Let the given integral be $I$.

$I = \int \sqrt{ax + b} \;dx$

$I = \int (ax + b)^{1/2} \;dx$


We use the method of substitution.

Let $u = ax + b$.


Differentiating both sides with respect to $x$, we get:

$\frac{du}{dx} = \frac{d}{dx}(ax + b)$

$\frac{du}{dx} = a + 0$

$\frac{du}{dx} = a$


Rearranging the terms, we get:

$du = a \;dx$

So, $dx = \frac{du}{a}$.


Now, substitute $u = ax + b$ and $dx = \frac{du}{a}$ into the integral $I$:

$I = \int u^{1/2} \cdot \frac{du}{a}$

$I = \frac{1}{a} \int u^{1/2} \;du$


Integrate $u^{1/2}$ with respect to $u$ using the power rule for integration ($\int u^n du = \frac{u^{n+1}}{n+1} + C'$):

$\int u^{1/2} \;du = \frac{u^{1/2 + 1}}{1/2 + 1} + C'$

$\int u^{1/2} \;du = \frac{u^{3/2}}{3/2} + C'$

$\int u^{1/2} \;du = \frac{2}{3} u^{3/2} + C'$


Now, substitute this back into the expression for $I$:

$I = \frac{1}{a} \left( \frac{2}{3} u^{3/2} \right) + C$

$I = \frac{2}{3a} u^{3/2} + C$, where $C = \frac{C'}{a}$ is the constant of integration.


Substitute back $u = ax + b$:

$I = \frac{2}{3a} (ax + b)^{3/2} + C$


Thus, the integral of the given function is $\mathbf{\frac{2}{3a} (ax + b)^{3/2} + C}$.

Question 7. $x \sqrt{x + 2}$

Answer:

Let the given integral be $I$.

$I = \int x \sqrt{x + 2} \;dx$


We use the method of substitution.

Let $u = x + 2$.


From the substitution, we can express $x$ in terms of $u$:

$x = u - 2$


Differentiating the substitution $u = x + 2$ with respect to $x$, we get:

$\frac{du}{dx} = \frac{d}{dx}(x + 2)$

$\frac{du}{dx} = 1 + 0$

$\frac{du}{dx} = 1$


Rearranging the terms, we get:

$du = dx$


Now, substitute $u = x + 2$, $x = u - 2$, and $dx = du$ into the integral $I$:

$I = \int (u - 2) \sqrt{u} \;du$

$I = \int (u - 2) u^{1/2} \;du$


Distribute $u^{1/2}$ inside the parenthesis:

$I = \int (u \cdot u^{1/2} - 2 \cdot u^{1/2}) \;du$

$I = \int (u^{1 + 1/2} - 2u^{1/2}) \;du$

$I = \int (u^{3/2} - 2u^{1/2}) \;du$


Integrate term by term using the power rule for integration ($\int u^n du = \frac{u^{n+1}}{n+1} + C'$):

$I = \int u^{3/2} \;du - 2 \int u^{1/2} \;du$

$\int u^{3/2} \;du = \frac{u^{3/2 + 1}}{3/2 + 1} = \frac{u^{5/2}}{5/2} = \frac{2}{5} u^{5/2}$

$\int u^{1/2} \;du = \frac{u^{1/2 + 1}}{1/2 + 1} = \frac{u^{3/2}}{3/2} = \frac{2}{3} u^{3/2}$


Substitute the results back into the expression for $I$:

$I = \frac{2}{5} u^{5/2} - 2 \left( \frac{2}{3} u^{3/2} \right) + C$

$I = \frac{2}{5} u^{5/2} - \frac{4}{3} u^{3/2} + C$, where $C$ is the constant of integration.


Substitute back $u = x + 2$:

$I = \frac{2}{5} (x + 2)^{5/2} - \frac{4}{3} (x + 2)^{3/2} + C$


Thus, the integral of the given function is $\mathbf{\frac{2}{5} (x + 2)^{5/2} - \frac{4}{3} (x + 2)^{3/2} + C}$.

Question 8. $x \sqrt{1 + 2x^2}$

Answer:

Let the given integral be $I$.

$I = \int x \sqrt{1 + 2x^2} \;dx$

$I = \int (1 + 2x^2)^{1/2} \cdot x \;dx$


We use the method of substitution.

Let $u = 1 + 2x^2$.


Differentiating both sides with respect to $x$, we get:

$\frac{du}{dx} = \frac{d}{dx}(1 + 2x^2)$

$\frac{du}{dx} = 0 + 2(2x)$

$\frac{du}{dx} = 4x$


Rearranging the terms, we get:

$du = 4x \;dx$

We have $x \;dx$ in the integrand, so we can write:

$x \;dx = \frac{du}{4}$


Now, substitute $u = 1 + 2x^2$ and $x \;dx = \frac{du}{4}$ into the integral $I$:

$I = \int u^{1/2} \cdot \frac{du}{4}$

$I = \frac{1}{4} \int u^{1/2} \;du$


Integrate $u^{1/2}$ with respect to $u$ using the power rule for integration ($\int u^n du = \frac{u^{n+1}}{n+1} + C'$):

$\int u^{1/2} \;du = \frac{u^{1/2 + 1}}{1/2 + 1} + C'$

$\int u^{1/2} \;du = \frac{u^{3/2}}{3/2} + C'$

$\int u^{1/2} \;du = \frac{2}{3} u^{3/2} + C'$


Now, substitute this back into the expression for $I$:

$I = \frac{1}{4} \left( \frac{2}{3} u^{3/2} \right) + C$

$I = \frac{2}{12} u^{3/2} + C$

$I = \frac{1}{6} u^{3/2} + C$, where $C = \frac{C'}{4}$ is the constant of integration.


Substitute back $u = 1 + 2x^2$:

$I = \frac{1}{6} (1 + 2x^2)^{3/2} + C$


Thus, the integral of the given function is $\mathbf{\frac{1}{6} (1 + 2x^2)^{3/2} + C}$.

Question 9. $(4x + 2) \sqrt{x^2 + x + 1}$

Answer:

Let the given integral be $I$.

$I = \int (4x + 2) \sqrt{x^2 + x + 1} \;dx$


We can factor out a 2 from the term $(4x + 2)$: $4x + 2 = 2(2x + 1)$.

$I = \int 2(2x + 1) \sqrt{x^2 + x + 1} \;dx$

$I = 2 \int (2x + 1) \sqrt{x^2 + x + 1} \;dx$


We use the method of substitution.

Let $u = x^2 + x + 1$.


Differentiating both sides with respect to $x$, we get:

$\frac{du}{dx} = \frac{d}{dx}(x^2 + x + 1)$

$\frac{du}{dx} = 2x + 1 + 0$

$\frac{du}{dx} = 2x + 1$


Rearranging the terms, we get:

$du = (2x + 1) \;dx$


Now, substitute $u = x^2 + x + 1$ and $(2x + 1) \;dx = du$ into the integral $I$:

$I = 2 \int \sqrt{u} \;du$

$I = 2 \int u^{1/2} \;du$


Integrate $u^{1/2}$ with respect to $u$ using the power rule for integration ($\int u^n du = \frac{u^{n+1}}{n+1} + C'$):

$\int u^{1/2} \;du = \frac{u^{1/2 + 1}}{1/2 + 1} + C'$

$\int u^{1/2} \;du = \frac{u^{3/2}}{3/2} + C'$

$\int u^{1/2} \;du = \frac{2}{3} u^{3/2} + C'$


Now, substitute this back into the expression for $I$:

$I = 2 \left( \frac{2}{3} u^{3/2} \right) + C$

$I = \frac{4}{3} u^{3/2} + C$, where $C = 2C'$ is the constant of integration.


Substitute back $u = x^2 + x + 1$:

$I = \frac{4}{3} (x^2 + x + 1)^{3/2} + C$


Thus, the integral of the given function is $\mathbf{\frac{4}{3} (x^2 + x + 1)^{3/2} + C}$.

Question 10. $\frac{1}{x − \sqrt{x}}$

Answer:

Let the given integral be $I$.

$I = \int \frac{1}{x - \sqrt{x}} dx$


Factor out $\sqrt{x}$ from the denominator:

$x - \sqrt{x} = \sqrt{x} \cdot \sqrt{x} - \sqrt{x} = \sqrt{x}(\sqrt{x} - 1)$


The integral becomes:

$I = \int \frac{1}{\sqrt{x}(\sqrt{x} - 1)} dx$

$I = \int \frac{1}{(\sqrt{x} - 1)} \cdot \frac{1}{\sqrt{x}} dx$


We use the method of substitution.

Let $u = \sqrt{x} - 1$.


Differentiating both sides with respect to $x$, we get:

$\frac{du}{dx} = \frac{d}{dx}(\sqrt{x} - 1)$

$\frac{du}{dx} = \frac{d}{dx}(x^{1/2} - 1)$

$\frac{du}{dx} = \frac{1}{2} x^{1/2 - 1} - 0$

$\frac{du}{dx} = \frac{1}{2} x^{-1/2}$

$\frac{du}{dx} = \frac{1}{2\sqrt{x}}$


Rearranging the terms, we get:

$du = \frac{1}{2\sqrt{x}} dx$

$2 \, du = \frac{1}{\sqrt{x}} dx$


Now, substitute $u = \sqrt{x} - 1$ and $\frac{1}{\sqrt{x}} dx = 2 \, du$ into the integral $I$:

$I = \int \frac{1}{u} \cdot 2 \, du$

$I = 2 \int \frac{1}{u} \, du$


The integral of $\frac{1}{u}$ with respect to $u$ is $\log_e |u| + C'$, where $C'$ is the constant of integration.

$I = 2 (\log_e |u| + C')$

$I = 2 \log_e |u| + 2C'$

Let $C = 2C'$ be the constant of integration.

$I = 2 \log_e |u| + C$


Substitute back $u = \sqrt{x} - 1$:

$I = 2 \log_e |\sqrt{x} - 1| + C$


Thus, the integral of the given function is $\mathbf{2 \log_e |\sqrt{x} - 1| + C}$.

Question 11. $\frac{x}{\sqrt{x + 4}}$ , x > 0

Answer:

Let the given integral be $I$.

$I = \int \frac{x}{\sqrt{x + 4}} \;dx$


We use the method of substitution.

Let $u = x + 4$.


From the substitution, we can express $x$ in terms of $u$:

$x = u - 4$


Differentiating the substitution $u = x + 4$ with respect to $x$, we get:

$\frac{du}{dx} = \frac{d}{dx}(x + 4)$

$\frac{du}{dx} = 1 + 0$

$\frac{du}{dx} = 1$


Rearranging the terms, we get:

$du = dx$


Now, substitute $u = x + 4$, $x = u - 4$, and $dx = du$ into the integral $I$:

$I = \int \frac{u - 4}{\sqrt{u}} \;du$

$I = \int \frac{u - 4}{u^{1/2}} \;du$


Simplify the integrand by dividing each term in the numerator by $u^{1/2}$:

$I = \int \left( \frac{u}{u^{1/2}} - \frac{4}{u^{1/2}} \right) \;du$

$I = \int \left( u^{1 - 1/2} - 4u^{-1/2} \right) \;du$

$I = \int \left( u^{1/2} - 4u^{-1/2} \right) \;du$


Integrate term by term using the power rule for integration ($\int u^n du = \frac{u^{n+1}}{n+1} + C'$):

$I = \int u^{1/2} \;du - 4 \int u^{-1/2} \;du$

$\int u^{1/2} \;du = \frac{u^{1/2 + 1}}{1/2 + 1} = \frac{u^{3/2}}{3/2} = \frac{2}{3} u^{3/2}$

$\int u^{-1/2} \;du = \frac{u^{-1/2 + 1}}{-1/2 + 1} = \frac{u^{1/2}}{1/2} = 2 u^{1/2}$


Substitute the results back into the expression for $I$:

$I = \frac{2}{3} u^{3/2} - 4 (2 u^{1/2}) + C$

$I = \frac{2}{3} u^{3/2} - 8 u^{1/2} + C$, where $C$ is the constant of integration.


Substitute back $u = x + 4$:

$I = \frac{2}{3} (x + 4)^{3/2} - 8 (x + 4)^{1/2} + C$

$I = \frac{2}{3} (x + 4)\sqrt{x + 4} - 8 \sqrt{x + 4} + C$


This can also be written by factoring $\sqrt{x+4}$:

$I = \sqrt{x + 4} \left( \frac{2}{3}(x + 4) - 8 \right) + C$

$I = \sqrt{x + 4} \left( \frac{2x}{3} + \frac{8}{3} - \frac{24}{3} \right) + C$

$I = \sqrt{x + 4} \left( \frac{2x - 16}{3} \right) + C$

$I = \frac{2}{3} (x - 8) \sqrt{x + 4} + C$


Thus, the integral of the given function is $\mathbf{\frac{2}{3} (x + 4)^{3/2} - 8 (x + 4)^{1/2} + C}$ or equivalently $\mathbf{\frac{2}{3} (x - 8) \sqrt{x + 4} + C}$, where $C$ is the constant of integration.

Question 12. $(x^3 − 1)^{\frac{1}{3}} x^5$

Answer:

Let the given integral be $I$.

$I = \int (x^3 - 1)^{1/3} x^5 \;dx$


We can rewrite $x^5$ as $x^3 \cdot x^2$.

$I = \int (x^3 - 1)^{1/3} x^3 \cdot x^2 \;dx$


We use the method of substitution.

Let $u = x^3 - 1$.


From the substitution, we can express $x^3$ in terms of $u$:

$x^3 = u + 1$


Differentiating the substitution $u = x^3 - 1$ with respect to $x$, we get:

$\frac{du}{dx} = \frac{d}{dx}(x^3 - 1)$

$\frac{du}{dx} = 3x^2 - 0$

$\frac{du}{dx} = 3x^2$


Rearranging the terms, we get:

$du = 3x^2 \;dx$

So, $x^2 \;dx = \frac{du}{3}$.


Now, substitute $u = x^3 - 1$, $x^3 = u + 1$, and $x^2 \;dx = \frac{du}{3}$ into the integral $I$:

$I = \int u^{1/3} \cdot (u + 1) \cdot \frac{du}{3}$

$I = \frac{1}{3} \int u^{1/3} (u + 1) \;du$


Expand the integrand:

$u^{1/3} (u + 1) = u^{1/3} \cdot u^1 + u^{1/3} \cdot 1$

$u^{1/3} (u + 1) = u^{1/3 + 1} + u^{1/3}$

$u^{1/3} (u + 1) = u^{4/3} + u^{1/3}$


So the integral becomes:

$I = \frac{1}{3} \int (u^{4/3} + u^{1/3}) \;du$


Integrate term by term using the power rule $\int u^n du = \frac{u^{n+1}}{n+1} + C'$:

$I = \frac{1}{3} \left( \int u^{4/3} \;du + \int u^{1/3} \;du \right)$

$\int u^{4/3} \;du = \frac{u^{4/3 + 1}}{4/3 + 1} = \frac{u^{7/3}}{7/3} = \frac{3}{7} u^{7/3}$

$\int u^{1/3} \;du = \frac{u^{1/3 + 1}}{1/3 + 1} = \frac{u^{4/3}}{4/3} = \frac{3}{4} u^{4/3}$


Substitute the integrated terms back into the expression for $I$:

$I = \frac{1}{3} \left( \frac{3}{7} u^{7/3} + \frac{3}{4} u^{4/3} \right) + C$

$I = \frac{1}{3} \cdot \frac{3}{7} u^{7/3} + \frac{1}{3} \cdot \frac{3}{4} u^{4/3} + C$

$I = \frac{1}{7} u^{7/3} + \frac{1}{4} u^{4/3} + C$, where $C$ is the constant of integration.


Substitute back $u = x^3 - 1$:

$I = \frac{1}{7} (x^3 - 1)^{7/3} + \frac{1}{4} (x^3 - 1)^{4/3} + C$


Thus, the integral of the given function is $\mathbf{\frac{1}{7} (x^3 - 1)^{7/3} + \frac{1}{4} (x^3 - 1)^{4/3} + C}$.

Question 13. $\frac{x^2}{(2 + 3x^3)^3}$

Answer:

Let the given integral be $I$.

$I = \int \frac{x^2}{(2 + 3x^3)^3} dx$


We use the method of substitution.

Let $u = 2 + 3x^3$.


Differentiating both sides with respect to $x$, we get:

$\frac{du}{dx} = \frac{d}{dx}(2 + 3x^3)$

$\frac{du}{dx} = 0 + 3(3x^2)$

$\frac{du}{dx} = 9x^2$


Rearranging the terms, we get:

$du = 9x^2 \;dx$

We have $x^2 \;dx$ in the numerator of the integrand, so we can write:

$x^2 \;dx = \frac{du}{9}$


Now, substitute $u = 2 + 3x^3$ and $x^2 \;dx = \frac{du}{9}$ into the integral $I$:

$I = \int \frac{1}{u^3} \cdot \frac{du}{9}$

$I = \frac{1}{9} \int \frac{1}{u^3} du$

$I = \frac{1}{9} \int u^{-3} du$


Integrate $u^{-3}$ with respect to $u$ using the power rule for integration $\int u^n du = \frac{u^{n+1}}{n+1} + C'$, where $n \neq -1$:

$\int u^{-3} du = \frac{u^{-3 + 1}}{-3 + 1} + C'$

$\int u^{-3} du = \frac{u^{-2}}{-2} + C'$

$\int u^{-3} du = -\frac{1}{2} u^{-2} + C'$

$\int u^{-3} du = -\frac{1}{2u^2} + C'$


Now, substitute this back into the expression for $I$:

$I = \frac{1}{9} \left( -\frac{1}{2u^2} \right) + C$

$I = -\frac{1}{18u^2} + C$, where $C = \frac{C'}{9}$ is the constant of integration.


Substitute back $u = 2 + 3x^3$:

$I = -\frac{1}{18(2 + 3x^3)^2} + C$


Thus, the integral of the given function is $\mathbf{-\frac{1}{18(2 + 3x^3)^2} + C}$.

Question 14. $\frac{1}{x(\log x)^m}$, x > 0. m ≠ 1

Answer:

Let the given integral be $I$.

$I = \int \frac{1}{x(\log x)^m} dx$


We can rewrite the integrand as:

$\frac{1}{x(\log x)^m} = \frac{1}{(\log x)^m} \cdot \frac{1}{x}$


We use the method of substitution.

Let $u = \log x$.


Differentiating both sides with respect to $x$, we get:

$\frac{du}{dx} = \frac{d}{dx}(\log x)$

$\frac{du}{dx} = \frac{1}{x}$


Rearranging the terms, we get:

$du = \frac{1}{x} dx$


Now, substitute $u = \log x$ and $\frac{1}{x} dx = du$ into the integral $I$:

$I = \int \frac{1}{u^m} du$

$I = \int u^{-m} du$


Integrate $u^{-m}$ with respect to $u$ using the power rule for integration $\int u^n du = \frac{u^{n+1}}{n+1} + C'$, where $n = -m$. Since $m \neq 1$, $-m \neq -1$, so the power rule is applicable.

$I = \frac{u^{-m + 1}}{-m + 1} + C'$

$I = \frac{u^{1 - m}}{1 - m} + C'$


Substitute back $u = \log x$:

$I = \frac{(\log x)^{1 - m}}{1 - m} + C$, where $C$ is the constant of integration.


Thus, the integral of the given function is $\mathbf{\frac{(\log x)^{1 - m}}{1 - m} + C}$.

Question 15. $\frac{x}{9 − 4x^2}$

Answer:

Let the given integral be $I$.

$I = \int \frac{x}{9 - 4x^2} dx$


We use the method of substitution.

Let $u = 9 - 4x^2$.


Differentiating both sides with respect to $x$, we get:

$\frac{du}{dx} = \frac{d}{dx}(9 - 4x^2)$

$\frac{du}{dx} = 0 - 4(2x)$

$\frac{du}{dx} = -8x$


Rearranging the terms, we get:

$du = -8x \;dx$

We have $x \;dx$ in the numerator of the integrand, so we can write:

$x \;dx = -\frac{du}{8}$


Now, substitute $u = 9 - 4x^2$ and $x \;dx = -\frac{du}{8}$ into the integral $I$:

$I = \int \frac{1}{u} \cdot \left(-\frac{du}{8}\right)$

$I = -\frac{1}{8} \int \frac{1}{u} du$


The integral of $\frac{1}{u}$ with respect to $u$ is $\log_e |u| + C'$, where $C'$ is the constant of integration.

$I = -\frac{1}{8} (\log_e |u| + C')$

$I = -\frac{1}{8} \log_e |u| + C$, where $C = -\frac{1}{8} C'$ is the constant of integration.


Substitute back $u = 9 - 4x^2$:

$I = -\frac{1}{8} \log_e |9 - 4x^2| + C$


Thus, the integral of the given function is $\mathbf{-\frac{1}{8} \log_e |9 - 4x^2| + C}$.

Question 16. $e^{2x+3}$

Answer:

Let the given integral be $I$.

$I = \int e^{2x+3} \;dx$


We use the method of substitution.

Let $u = 2x+3$.


Differentiating both sides with respect to $x$, we get:

$\frac{du}{dx} = \frac{d}{dx}(2x+3)$

$\frac{du}{dx} = 2 + 0$

$\frac{du}{dx} = 2$


Rearranging the terms, we get:

$du = 2 \;dx$

So, $dx = \frac{du}{2}$.


Now, substitute $u = 2x+3$ and $dx = \frac{du}{2}$ into the integral $I$:

$I = \int e^{u} \cdot \frac{du}{2}$

$I = \frac{1}{2} \int e^{u} \;du$


The integral of $e^u$ with respect to $u$ is $e^u + C'$, where $C'$ is the constant of integration.

$I = \frac{1}{2} (e^{u} + C')$

$I = \frac{1}{2} e^{u} + C$, where $C = \frac{C'}{2}$ is the constant of integration.


Substitute back $u = 2x+3$:

$I = \frac{1}{2} e^{2x+3} + C$


Thus, the integral of the given function is $\mathbf{\frac{1}{2} e^{2x+3} + C}$.

Question 17. $\frac{x}{e^{x^{2}}}$

Answer:

Let the given integral be $I$.

$I = \int \frac{x}{e^{x^2}} dx$


We can rewrite the integrand as:

$I = \int x e^{-x^2} dx$


We use the method of substitution.

Let $u = -x^2$.


Differentiating both sides with respect to $x$, we get:

$\frac{du}{dx} = \frac{d}{dx}(-x^2)$

$\frac{du}{dx} = -2x$


Rearranging the terms, we get:

$du = -2x \;dx$

We have $x \;dx$ in the integrand, so we can write:

$x \;dx = -\frac{du}{2}$


Now, substitute $u = -x^2$ and $x \;dx = -\frac{du}{2}$ into the integral $I$:

$I = \int e^{u} \cdot \left(-\frac{du}{2}\right)$

$I = -\frac{1}{2} \int e^{u} \;du$


The integral of $e^u$ with respect to $u$ is $e^u + C'$, where $C'$ is the constant of integration.

$I = -\frac{1}{2} (e^{u} + C')$

$I = -\frac{1}{2} e^{u} + C$, where $C = -\frac{1}{2} C'$ is the constant of integration.


Substitute back $u = -x^2$:

$I = -\frac{1}{2} e^{-x^2} + C$


Thus, the integral of the given function is $\mathbf{-\frac{1}{2} e^{-x^2} + C}$ or $\mathbf{-\frac{1}{2e^{x^2}} + C}$.

Question 18. $\frac{e^{\tan^{−1} x}}{1 + x^2}$

Answer:

Let the given integral be $I$.

$I = \int \frac{e^{\tan^{-1} x}}{1 + x^2} dx$


We use the method of substitution.

Let $u = \tan^{-1} x$.


Differentiating both sides with respect to $x$, we get:

$\frac{du}{dx} = \frac{d}{dx}(\tan^{-1} x)$

$\frac{du}{dx} = \frac{1}{1 + x^2}$


Rearranging the terms, we get:

$du = \frac{1}{1 + x^2} dx$


Now, substitute $u = \tan^{-1} x$ and $\frac{1}{1 + x^2} dx = du$ into the integral $I$:

$I = \int e^{u} du$


The integral of $e^u$ with respect to $u$ is $e^u + C$, where $C$ is the constant of integration.

$I = e^{u} + C$


Substitute back $u = \tan^{-1} x$:

$I = e^{\tan^{-1} x} + C$


Thus, the integral of the given function is $\mathbf{e^{\tan^{-1} x} + C}$, where $C$ is the constant of integration.

Question 19. $\frac{e^{2x} − 1}{e^{2x} + 1}$

Answer:

Let the given integral be $I$.

$I = \int \frac{e^{2x} - 1}{e^{2x} + 1} dx$


We can manipulate the integrand by dividing the numerator and the denominator by $e^x$:

$\frac{e^{2x} - 1}{e^{2x} + 1} = \frac{(e^{2x} - 1)/e^x}{(e^{2x} + 1)/e^x} = \frac{e^{2x}/e^x - 1/e^x}{e^{2x}/e^x + 1/e^x} = \frac{e^x - e^{-x}}{e^x + e^{-x}}$


So the integral becomes:

$I = \int \frac{e^x - e^{-x}}{e^x + e^{-x}} dx$


We use the method of substitution.

Let $u = e^x + e^{-x}$.


Differentiating both sides with respect to $x$, we get:

$\frac{du}{dx} = \frac{d}{dx}(e^x + e^{-x})$

$\frac{du}{dx} = e^x + (-e^{-x})$

$\frac{du}{dx} = e^x - e^{-x}$


Rearranging the terms, we get:

$du = (e^x - e^{-x}) dx$


Now, substitute $u = e^x + e^{-x}$ and $(e^x - e^{-x}) dx = du$ into the integral $I$:

$I = \int \frac{1}{u} du$


The integral of $\frac{1}{u}$ with respect to $u$ is $\log_e |u| + C'$, where $C'$ is the constant of integration.

$I = \log_e |u| + C'$


Substitute back $u = e^x + e^{-x}$:

$I = \log_e |e^x + e^{-x}| + C'$


Since $e^x > 0$ and $e^{-x} > 0$ for all real $x$, their sum $e^x + e^{-x}$ is always positive. Thus, the absolute value is not necessary.

$I = \log_e (e^x + e^{-x}) + C$, where $C = C'$ is the constant of integration.


Thus, the integral of the given function is $\mathbf{\log_e (e^x + e^{-x}) + C}$.


Alternate Solution:

Let the given integral be $I$.

$I = \int \frac{e^{2x} - 1}{e^{2x} + 1} dx$


We can rewrite the numerator as $(e^{2x} + 1) - 2$:

$\frac{e^{2x} - 1}{e^{2x} + 1} = \frac{(e^{2x} + 1) - 2}{e^{2x} + 1} = \frac{e^{2x} + 1}{e^{2x} + 1} - \frac{2}{e^{2x} + 1} = 1 - \frac{2}{e^{2x} + 1}$


So the integral becomes:

$I = \int \left(1 - \frac{2}{e^{2x} + 1}\right) dx$

$I = \int 1 \, dx - 2 \int \frac{1}{e^{2x} + 1} dx$


The first integral is $\int 1 \, dx = x + C_1$.

For the second integral, $\int \frac{1}{e^{2x} + 1} dx$, we can multiply the numerator and denominator by $e^{-2x}$:

$\int \frac{e^{-2x}}{e^{-2x}(e^{2x} + 1)} dx = \int \frac{e^{-2x}}{1 + e^{-2x}} dx$


Now, use substitution for this integral.

Let $w = 1 + e^{-2x}$.


Differentiating both sides with respect to $x$, we get:

$\frac{dw}{dx} = \frac{d}{dx}(1 + e^{-2x})$

$\frac{dw}{dx} = 0 + e^{-2x} \cdot \frac{d}{dx}(-2x) = e^{-2x} \cdot (-2)$

$\frac{dw}{dx} = -2e^{-2x}$


Rearranging the terms, we get:

$dw = -2e^{-2x} dx$

So, $e^{-2x} dx = -\frac{dw}{2}$.


Substitute $w = 1 + e^{-2x}$ and $e^{-2x} dx = -\frac{dw}{2}$ into the integral:

$\int \frac{e^{-2x}}{1 + e^{-2x}} dx = \int \frac{1}{w} \left(-\frac{dw}{2}\right) = -\frac{1}{2} \int \frac{1}{w} dw$


The integral of $\frac{1}{w}$ with respect to $w$ is $\log_e |w| + C_2$.

$-\frac{1}{2} \int \frac{1}{w} dw = -\frac{1}{2} (\log_e |w| + C_2) = -\frac{1}{2} \log_e |w| - \frac{C_2}{2}$


Substitute back $w = 1 + e^{-2x}$:

$-\frac{1}{2} \log_e |1 + e^{-2x}| + C_3$, where $C_3 = -\frac{C_2}{2}$.

Since $1 + e^{-2x} > 0$, the absolute value is not needed:

$-\frac{1}{2} \log_e (1 + e^{-2x}) + C_3$


Now substitute this back into the expression for $I$:

$I = (x + C_1) - 2 \left( -\frac{1}{2} \log_e (1 + e^{-2x}) + C_3 \right) + C_4$

$I = x + C_1 + \log_e (1 + e^{-2x}) - 2C_3 + C_4$

$I = x + \log_e (1 + e^{-2x}) + C$, where $C = C_1 - 2C_3 + C_4$ is the constant of integration.


Thus, the integral of the given function is $\mathbf{x + \log_e (1 + e^{-2x}) + C}$.

Note that $\log_e (e^x + e^{-x}) = \log_e (e^x(1 + e^{-2x})) = \log_e (e^x) + \log_e (1 + e^{-2x}) = x + \log_e (1 + e^{-2x})$. The two results are equivalent.

Question 20. $\frac{e^{2x} − e^{−2x}}{e^{2x} + e^{−2x}}$

Answer:

Let the given integral be $I$.

$I = \int \frac{e^{2x} - e^{-2x}}{e^{2x} + e^{-2x}} dx$


We use the method of substitution.

Let $u = e^{2x} + e^{-2x}$.


Differentiating both sides with respect to $x$, we get:

$\frac{du}{dx} = \frac{d}{dx}(e^{2x} + e^{-2x})$

Using the chain rule, $\frac{du}{dx} = \frac{d}{dx}(e^{2x}) + \frac{d}{dx}(e^{-2x})$

$\frac{du}{dx} = e^{2x} \cdot \frac{d}{dx}(2x) + e^{-2x} \cdot \frac{d}{dx}(-2x)$

$\frac{du}{dx} = e^{2x} \cdot 2 + e^{-2x} \cdot (-2)$

$\frac{du}{dx} = 2e^{2x} - 2e^{-2x}$

$\frac{du}{dx} = 2(e^{2x} - e^{-2x})$


Rearranging the terms, we get:

$du = 2(e^{2x} - e^{-2x}) dx$

We have $(e^{2x} - e^{-2x}) dx$ in the numerator of the integrand, so we can write:

$(e^{2x} - e^{-2x}) dx = \frac{du}{2}$


Now, substitute $u = e^{2x} + e^{-2x}$ and $(e^{2x} - e^{-2x}) dx = \frac{du}{2}$ into the integral $I$:

$I = \int \frac{1}{u} \cdot \frac{du}{2}$

$I = \frac{1}{2} \int \frac{1}{u} du$


The integral of $\frac{1}{u}$ with respect to $u$ is $\log_e |u| + C'$, where $C'$ is the constant of integration.

$I = \frac{1}{2} (\log_e |u| + C')$

$I = \frac{1}{2} \log_e |u| + C$, where $C = \frac{C'}{2}$ is the constant of integration.


Substitute back $u = e^{2x} + e^{-2x}$:

$I = \frac{1}{2} \log_e |e^{2x} + e^{-2x}| + C$


Since $e^{2x} > 0$ and $e^{-2x} > 0$ for all real values of $x$, their sum $e^{2x} + e^{-2x}$ is always positive. Thus, the absolute value is not required.

$I = \frac{1}{2} \log_e (e^{2x} + e^{-2x}) + C$


Thus, the integral of the given function is $\mathbf{\frac{1}{2} \log_e (e^{2x} + e^{-2x}) + C}$, where $C$ is the constant of integration.

Question 21. tan2 (2x - 3)

Answer:

Let the given integral be $I$.

$I = \int \tan^2(2x - 3) \;dx$


We use the trigonometric identity $\tan^2 \theta = \sec^2 \theta - 1$.

Here, $\theta = 2x - 3$.

So, $\tan^2(2x - 3) = \sec^2(2x - 3) - 1$.


The integral becomes:

$I = \int (\sec^2(2x - 3) - 1) \;dx$

$I = \int \sec^2(2x - 3) \;dx - \int 1 \;dx$


Let's evaluate the first integral $\int \sec^2(2x - 3) \;dx$ using substitution.

Let $u = 2x - 3$.


Differentiating both sides with respect to $x$, we get:

$\frac{du}{dx} = \frac{d}{dx}(2x - 3)$

$\frac{du}{dx} = 2 - 0$

$\frac{du}{dx} = 2$


Rearranging the terms, we get:

$du = 2 \;dx$

So, $dx = \frac{du}{2}$.


Substitute $u = 2x - 3$ and $dx = \frac{du}{2}$ into the first integral:

$\int \sec^2(2x - 3) \;dx = \int \sec^2(u) \frac{du}{2}$

$= \frac{1}{2} \int \sec^2(u) \;du$


The integral of $\sec^2(u)$ with respect to $u$ is $\tan(u)$.

$= \frac{1}{2} \tan(u) + C_1$


Substitute back $u = 2x - 3$:

$= \frac{1}{2} \tan(2x - 3) + C_1$


Now, evaluate the second integral:

$\int 1 \;dx = x + C_2$


Combine the results of both integrals:

$I = \left(\frac{1}{2} \tan(2x - 3) + C_1\right) - (x + C_2)$

$I = \frac{1}{2} \tan(2x - 3) - x + C_1 - C_2$

Let $C = C_1 - C_2$ be the constant of integration.

$I = \frac{1}{2} \tan(2x - 3) - x + C$


Thus, the integral of the given function is $\mathbf{\frac{1}{2} \tan(2x - 3) - x + C}$.

Question 22. sec2 (7 - 4x)

Answer:

Let the given integral be $I$.

$I = \int \sec^2(7 - 4x) \;dx$


We use the method of substitution.

Let $u = 7 - 4x$.


Differentiating both sides with respect to $x$, we get:

$\frac{du}{dx} = \frac{d}{dx}(7 - 4x)$

$\frac{du}{dx} = 0 - 4$

$\frac{du}{dx} = -4$


Rearranging the terms, we get:

$du = -4 \;dx$

So, $dx = -\frac{du}{4}$.


Now, substitute $u = 7 - 4x$ and $dx = -\frac{du}{4}$ into the integral $I$:

$I = \int \sec^2(u) \cdot \left(-\frac{du}{4}\right)$

$I = -\frac{1}{4} \int \sec^2(u) \;du$


The integral of $\sec^2(u)$ with respect to $u$ is $\tan(u) + C'$, where $C'$ is the constant of integration.

$I = -\frac{1}{4} (\tan(u) + C')$

$I = -\frac{1}{4} \tan(u) + C$, where $C = -\frac{C'}{4}$ is the constant of integration.


Substitute back $u = 7 - 4x$:

$I = -\frac{1}{4} \tan(7 - 4x) + C$


Thus, the integral of the given function is $\mathbf{-\frac{1}{4} \tan(7 - 4x) + C}$.

Question 23. $\frac{\sin^{−1} x}{\sqrt{1 - x^2}}$

Answer:

Let the given integral be $I$.

$I = \int \frac{\sin^{-1} x}{\sqrt{1 - x^2}} dx$


We can rewrite the integrand as:

$I = \int \sin^{-1} x \cdot \frac{1}{\sqrt{1 - x^2}} dx$


We use the method of substitution.

Let $u = \sin^{-1} x$.


Differentiating both sides with respect to $x$, we get:

$\frac{du}{dx} = \frac{d}{dx}(\sin^{-1} x)$

$\frac{du}{dx} = \frac{1}{\sqrt{1 - x^2}}$


Rearranging the terms, we get:

$du = \frac{1}{\sqrt{1 - x^2}} dx$


Now, substitute $u = \sin^{-1} x$ and $\frac{1}{\sqrt{1 - x^2}} dx = du$ into the integral $I$:

$I = \int u \, du$


Integrate $u$ with respect to $u$ using the power rule for integration ($\int u^n du = \frac{u^{n+1}}{n+1} + C$, for $n \neq -1$):

$I = \frac{u^{1+1}}{1+1} + C$

$I = \frac{u^2}{2} + C$


Substitute back $u = \sin^{-1} x$:

$I = \frac{(\sin^{-1} x)^2}{2} + C$


Thus, the integral of the given function is $\mathbf{\frac{(\sin^{-1} x)^2}{2} + C}$, where $C$ is the constant of integration.

Question 24. $\frac{2\cos x − 3 \sin x}{6\cos x + 4\sin x}$

Answer:

To evaluate the integral $\int \frac{2\cos x − 3 \sin x}{6\cos x + 4\sin x} dx$, we can use the method of substitution.


Let the denominator be $u$.

$u = 6\cos x + 4\sin x$


Now, we find the derivative of $u$ with respect to $x$.

$\frac{du}{dx} = \frac{d}{dx}(6\cos x + 4\sin x)$

$\frac{du}{dx} = 6 \frac{d}{dx}(\cos x) + 4 \frac{d}{dx}(\sin x)$

$\frac{du}{dx} = 6 (-\sin x) + 4 (\cos x)$

$\frac{du}{dx} = -6\sin x + 4\cos x$

$\frac{du}{dx} = 4\cos x - 6\sin x$


From this, we have $du = (4\cos x - 6\sin x) dx$.

Notice that the numerator of the integrand is $2\cos x - 3\sin x$. This is exactly half of the derivative we found ($4\cos x - 6\sin x$).

So, $2(2\cos x - 3\sin x) dx = (4\cos x - 6\sin x) dx = du$.

This implies $(2\cos x - 3\sin x) dx = \frac{1}{2} du$.


Now, substitute $u = 6\cos x + 4\sin x$ and $(2\cos x - 3\sin x) dx = \frac{1}{2} du$ into the integral:

$\int \frac{2\cos x − 3 \sin x}{6\cos x + 4\sin x} dx = \int \frac{1}{u} \left(\frac{1}{2} du\right)$

= $\frac{1}{2} \int \frac{1}{u} du$


The integral of $\frac{1}{u}$ with respect to $u$ is $\log |u|$.

= $\frac{1}{2} \log |u| + C$

where $C$ is the constant of integration.


Finally, substitute back $u = 6\cos x + 4\sin x$.

= $\frac{1}{2} \log |6\cos x + 4\sin x| + C$


We can factor out 2 from the argument of the logarithm: $6\cos x + 4\sin x = 2(3\cos x + 2\sin x) = 2(2\sin x + 3\cos x)$.

So the result can be written as:

= $\frac{1}{2} \log |2(2\sin x + 3\cos x)| + C$

Using the property $\log |ab| = \log |a| + \log |b|$, we get:

= $\frac{1}{2} (\log |2| + \log |2\sin x + 3\cos x|) + C$

= $\frac{1}{2}\log |2| + \frac{1}{2}\log |2\sin x + 3\cos x| + C$

Since $\frac{1}{2}\log |2|$ is a constant, we can combine it with the constant of integration $C$ to get a new constant $C'$.

The integral is equal to:

$\frac{1}{2}\log |2\sin x + 3\cos x| + C'$

Thus, the final answer is $\frac{1}{2} \log |2 \sin x + 3 \cos x| + C$, where $C$ represents the constant of integration.

Question 25. $\frac{1}{\cos^2 x (1 − \tan x)^2}$

Answer:

Let the given integral be $I$.

$I = \int \frac{1}{\cos^2 x (1 - \tan x)^2} dx$


We can rewrite $\frac{1}{\cos^2 x}$ as $\sec^2 x$.

$I = \int \frac{\sec^2 x}{(1 - \tan x)^2} dx$


We use the method of substitution.

Let $u = 1 - \tan x$.


Differentiating both sides with respect to $x$, we get:

$\frac{du}{dx} = \frac{d}{dx}(1 - \tan x)$

$\frac{du}{dx} = 0 - \sec^2 x$

$\frac{du}{dx} = -\sec^2 x$


Rearranging the terms, we get:

$du = -\sec^2 x \;dx$

So, $\sec^2 x \;dx = -du$.


Now, substitute $u = 1 - \tan x$ and $\sec^2 x \;dx = -du$ into the integral $I$:

$I = \int \frac{1}{u^2} (-du)$

$I = - \int u^{-2} du$


Integrate $u^{-2}$ with respect to $u$ using the power rule for integration $\int u^n du = \frac{u^{n+1}}{n+1} + C'$, where $n \neq -1$:

$\int u^{-2} du = \frac{u^{-2 + 1}}{-2 + 1} + C'$

$\int u^{-2} du = \frac{u^{-1}}{-1} + C'$

$\int u^{-2} du = -\frac{1}{u} + C'$


Now, substitute this back into the expression for $I$:

$I = - \left(-\frac{1}{u} + C'\right)$

$I = \frac{1}{u} - C'$

Let $C = -C'$ be the constant of integration.

$I = \frac{1}{u} + C$


Substitute back $u = 1 - \tan x$:

$I = \frac{1}{1 - \tan x} + C$


Thus, the integral of the given function is $\mathbf{\frac{1}{1 - \tan x} + C}$, where $C$ is the constant of integration.

Question 26. $\frac{\cos \sqrt{x}}{\sqrt{x}}$

Answer:

Let the given integral be $I$.

$I = \int \frac{\cos \sqrt{x}}{\sqrt{x}} dx$


We can rewrite the integrand as:

$I = \int \cos \sqrt{x} \cdot \frac{1}{\sqrt{x}} dx$


We use the method of substitution.

Let $u = \sqrt{x}$.


Differentiating both sides with respect to $x$, we get:

$\frac{du}{dx} = \frac{d}{dx}(\sqrt{x})$

$\frac{du}{dx} = \frac{d}{dx}(x^{1/2})$

$\frac{du}{dx} = \frac{1}{2} x^{1/2 - 1}$

$\frac{du}{dx} = \frac{1}{2} x^{-1/2}$

$\frac{du}{dx} = \frac{1}{2\sqrt{x}}$


Rearranging the terms, we get:

$du = \frac{1}{2\sqrt{x}} dx$

So, $\frac{1}{\sqrt{x}} dx = 2 \, du$.


Now, substitute $u = \sqrt{x}$ and $\frac{1}{\sqrt{x}} dx = 2 \, du$ into the integral $I$:

$I = \int \cos(u) \cdot 2 \, du$

$I = 2 \int \cos(u) \, du$


The integral of $\cos(u)$ with respect to $u$ is $\sin(u) + C'$, where $C'$ is the constant of integration.

$I = 2 (\sin(u) + C')$

$I = 2 \sin(u) + C$, where $C = 2C'$ is the constant of integration.


Substitute back $u = \sqrt{x}$:

$I = 2 \sin(\sqrt{x}) + C$


Thus, the integral of the given function is $\mathbf{2 \sin(\sqrt{x}) + C}$, where $C$ is the constant of integration.

Question 27. $\sqrt{\sin 2x} \cos 2x$

Answer:

Let the given integral be $I$.

$I = \int \sqrt{\sin 2x} \cos 2x \;dx$

$I = \int (\sin 2x)^{1/2} \cos 2x \;dx$


We use the method of substitution.

Let $u = \sin 2x$.


Differentiating both sides with respect to $x$, we get:

$\frac{du}{dx} = \frac{d}{dx}(\sin 2x)$

Using the chain rule, $\frac{du}{dx} = \cos 2x \cdot \frac{d}{dx}(2x) = \cos 2x \cdot 2$

$\frac{du}{dx} = 2 \cos 2x$


Rearranging the terms, we get:

$du = 2 \cos 2x \;dx$

We have $\cos 2x \;dx$ in the integrand, so we can write:

$\cos 2x \;dx = \frac{du}{2}$


Now, substitute $u = \sin 2x$ and $\cos 2x \;dx = \frac{du}{2}$ into the integral $I$:

$I = \int u^{1/2} \cdot \frac{du}{2}$

$I = \frac{1}{2} \int u^{1/2} \;du$


Integrate $u^{1/2}$ with respect to $u$ using the power rule for integration ($\int u^n du = \frac{u^{n+1}}{n+1} + C'$):

$\int u^{1/2} \;du = \frac{u^{1/2 + 1}}{1/2 + 1} + C'$

$\int u^{1/2} \;du = \frac{u^{3/2}}{3/2} + C'$

$\int u^{1/2} \;du = \frac{2}{3} u^{3/2} + C'$


Now, substitute this back into the expression for $I$:

$I = \frac{1}{2} \left( \frac{2}{3} u^{3/2} \right) + C$

$I = \frac{1}{3} u^{3/2} + C$, where $C = \frac{C'}{2}$ is the constant of integration.


Substitute back $u = \sin 2x$:

$I = \frac{1}{3} (\sin 2x)^{3/2} + C$

$I = \frac{1}{3} (\sin 2x)\sqrt{\sin 2x} + C$


Thus, the integral of the given function is $\mathbf{\frac{1}{3} (\sin 2x)^{3/2} + C}$, where $C$ is the constant of integration.

Question 28. $\frac{\cos x}{\sqrt{1 + \sin x}}$

Answer:

Let the given integral be $I$.

$I = \int \frac{\cos x}{\sqrt{1 + \sin x}} dx$


We can rewrite the integrand as:

$I = \int (1 + \sin x)^{-1/2} \cos x \;dx$


We use the method of substitution.

Let $u = 1 + \sin x$.


Differentiating both sides with respect to $x$, we get:

$\frac{du}{dx} = \frac{d}{dx}(1 + \sin x)$

$\frac{du}{dx} = 0 + \cos x$

$\frac{du}{dx} = \cos x$


Rearranging the terms, we get:

$du = \cos x \;dx$


Now, substitute $u = 1 + \sin x$ and $\cos x \;dx = du$ into the integral $I$:

$I = \int u^{-1/2} du$


Integrate $u^{-1/2}$ with respect to $u$ using the power rule for integration $\int u^n du = \frac{u^{n+1}}{n+1} + C'$, where $n \neq -1$:

$\int u^{-1/2} du = \frac{u^{-1/2 + 1}}{-1/2 + 1} + C'$

$\int u^{-1/2} du = \frac{u^{1/2}}{1/2} + C'$

$\int u^{-1/2} du = 2u^{1/2} + C'$

$\int u^{-1/2} du = 2\sqrt{u} + C'$


Now, substitute this back into the expression for $I$:

$I = 2\sqrt{u} + C$, where $C = C'$ is the constant of integration.


Substitute back $u = 1 + \sin x$:

$I = 2\sqrt{1 + \sin x} + C$


Thus, the integral of the given function is $\mathbf{2\sqrt{1 + \sin x} + C}$, where $C$ is the constant of integration.

Question 29. $\cot x \log \;\sin x$

Answer:

Let the given integral be $I$.

$I = \int \cot x \log (\sin x) \;dx$


We use the method of substitution.

Let $u = \log (\sin x)$.


Differentiating both sides with respect to $x$, we get:

$\frac{du}{dx} = \frac{d}{dx}(\log (\sin x))$

Using the chain rule, $\frac{du}{dx} = \frac{1}{\sin x} \cdot \frac{d}{dx}(\sin x)$

$\frac{du}{dx} = \frac{1}{\sin x} \cdot \cos x$

$\frac{du}{dx} = \cot x$


Rearranging the terms, we get:

$du = \cot x \;dx$


Now, substitute $u = \log (\sin x)$ and $\cot x \;dx = du$ into the integral $I$:

$I = \int u \, du$


Integrate $u$ with respect to $u$ using the power rule for integration ($\int u^n du = \frac{u^{n+1}}{n+1} + C'$):

$I = \frac{u^{1+1}}{1+1} + C'$

$I = \frac{u^2}{2} + C'$


Substitute back $u = \log (\sin x)$:

$I = \frac{(\log (\sin x))^2}{2} + C$


Thus, the integral of the given function is $\mathbf{\frac{(\log (\sin x))^2}{2} + C}$, where $C$ is the constant of integration.

Question 30. $\frac{\sin x}{1 + \cos x}$

Answer:

Let the given integral be $I$.

$I = \int \frac{\sin x}{1 + \cos x} dx$


We use the method of substitution.

Let $u = 1 + \cos x$.


Differentiating both sides with respect to $x$, we get:

$\frac{du}{dx} = \frac{d}{dx}(1 + \cos x)$

$\frac{du}{dx} = 0 - \sin x$

$\frac{du}{dx} = -\sin x$


Rearranging the terms, we get:

$du = -\sin x \;dx$

So, $\sin x \;dx = -du$.


Now, substitute $u = 1 + \cos x$ and $\sin x \;dx = -du$ into the integral $I$:

$I = \int \frac{1}{u} (-du)$

$I = - \int \frac{1}{u} du$


The integral of $\frac{1}{u}$ with respect to $u$ is $\log_e |u| + C'$, where $C'$ is the constant of integration.

$I = - (\log_e |u| + C')$

$I = -\log_e |u| - C'$

Let $C = -C'$ be the constant of integration.

$I = -\log_e |u| + C$


Substitute back $u = 1 + \cos x$:

$I = -\log_e |1 + \cos x| + C$


Thus, the integral of the given function is $\mathbf{-\log_e |1 + \cos x| + C}$, where $C$ is the constant of integration.

Question 31. $\frac{\sin x}{(1 + \cos x)^2}$

Answer:

Let the given integral be $I$.

$I = \int \frac{\sin x}{(1 + \cos x)^2} dx$


We can rewrite the integrand as:

$I = \int (1 + \cos x)^{-2} \sin x \;dx$


We use the method of substitution.

Let $u = 1 + \cos x$.


Differentiating both sides with respect to $x$, we get:

$\frac{du}{dx} = \frac{d}{dx}(1 + \cos x)$

$\frac{du}{dx} = 0 - \sin x$

$\frac{du}{dx} = -\sin x$


Rearranging the terms, we get:

$du = -\sin x \;dx$

So, $\sin x \;dx = -du$.


Now, substitute $u = 1 + \cos x$ and $\sin x \;dx = -du$ into the integral $I$:

$I = \int u^{-2} (-du)$

$I = - \int u^{-2} du$


Integrate $u^{-2}$ with respect to $u$ using the power rule for integration $\int u^n du = \frac{u^{n+1}}{n+1} + C'$, where $n \neq -1$:

$\int u^{-2} du = \frac{u^{-2 + 1}}{-2 + 1} + C'$

$\int u^{-2} du = \frac{u^{-1}}{-1} + C'$

$\int u^{-2} du = -\frac{1}{u} + C'$


Now, substitute this back into the expression for $I$:

$I = - \left(-\frac{1}{u} + C'\right)$

$I = \frac{1}{u} - C'$

Let $C = -C'$ be the constant of integration.

$I = \frac{1}{u} + C$


Substitute back $u = 1 + \cos x$:

$I = \frac{1}{1 + \cos x} + C$


Thus, the integral of the given function is $\mathbf{\frac{1}{1 + \cos x} + C}$, where $C$ is the constant of integration.

Question 32. $\frac{1}{1 + \cot x}$

Answer:

Let the given integral be $I$.

$I = \int \frac{1}{1 + \cot x} dx$


We can rewrite the integrand by expressing $\cot x$ in terms of $\sin x$ and $\cos x$:

$\cot x = \frac{\cos x}{\sin x}$

So, the integrand is $\frac{1}{1 + \frac{\cos x}{\sin x}} = \frac{1}{\frac{\sin x + \cos x}{\sin x}} = \frac{\sin x}{\sin x + \cos x}$.


The integral becomes:

$I = \int \frac{\sin x}{\sin x + \cos x} dx$


We can write the numerator as a linear combination of the denominator and its derivative.

Let $D = \sin x + \cos x$. Then $D' = \cos x - \sin x$.

We want to find constants $A$ and $B$ such that $\sin x = A(\sin x + \cos x) + B(\cos x - \sin x)$.

$\sin x = A\sin x + A\cos x + B\cos x - B\sin x$

$\sin x = (A - B)\sin x + (A + B)\cos x$


Comparing the coefficients of $\sin x$ and $\cos x$ on both sides:

$1 = A - B$

$0 = A + B$


Solving these two equations, we get $A = \frac{1}{2}$ and $B = -\frac{1}{2}$.


Now, rewrite the integrand using these values of $A$ and $B$:

$\frac{\sin x}{\sin x + \cos x} = \frac{\frac{1}{2}(\sin x + \cos x) - \frac{1}{2}(\cos x - \sin x)}{\sin x + \cos x}$

$= \frac{1}{2} \frac{\sin x + \cos x}{\sin x + \cos x} - \frac{1}{2} \frac{\cos x - \sin x}{\sin x + \cos x}$

$= \frac{1}{2} - \frac{1}{2} \frac{\cos x - \sin x}{\sin x + \cos x}$


So the integral becomes:

$I = \int \left(\frac{1}{2} - \frac{1}{2} \frac{\cos x - \sin x}{\sin x + \cos x}\right) dx$

$I = \int \frac{1}{2} dx - \frac{1}{2} \int \frac{\cos x - \sin x}{\sin x + \cos x} dx$


The first integral is $\int \frac{1}{2} dx = \frac{1}{2} x + C_1$.


For the second integral, $\int \frac{\cos x - \sin x}{\sin x + \cos x} dx$, let $u = \sin x + \cos x$.

Then, $du = (\cos x - (-\sin x)) dx = (\cos x - \sin x) dx$.

The integral becomes $\int \frac{1}{u} du = \log_e |u| + C_2 = \log_e |\sin x + \cos x| + C_2$.


Substitute these results back into the expression for $I$:

$I = \frac{1}{2} x - \frac{1}{2} (\log_e |\sin x + \cos x|) + C$

Where $C$ is the constant of integration.


Thus, the integral of the given function is $\mathbf{\frac{1}{2} x - \frac{1}{2} \log_e |\sin x + \cos x| + C}$.

Question 33. $\frac{1}{1 − \tan x}$

Answer:

Let the given integral be $I$.

$I = \int \frac{1}{1 - \tan x} dx$


We can rewrite the integrand by expressing $\tan x$ in terms of $\sin x$ and $\cos x$:

$\tan x = \frac{\sin x}{\cos x}$

So, the integrand is $\frac{1}{1 - \frac{\sin x}{\cos x}} = \frac{1}{\frac{\cos x - \sin x}{\cos x}} = \frac{\cos x}{\cos x - \sin x}$.


The integral becomes:

$I = \int \frac{\cos x}{\cos x - \sin x} dx$


We can rewrite the numerator as a combination of the denominator and its derivative.

Let the numerator be $N = \cos x$ and the denominator be $D = \cos x - \sin x$.

The derivative of the denominator is $D' = \frac{d}{dx}(\cos x - \sin x) = -\sin x - \cos x = -(\sin x + \cos x)$.


We can write $N$ as $N = A \cdot D + B \cdot D'$ for some constants $A$ and $B$.

$\cos x = A(\cos x - \sin x) + B(-(\sin x + \cos x))$

$\cos x = A\cos x - A\sin x - B\sin x - B\cos x$

$\cos x = (A - B)\cos x + (-A - B)\sin x$


Comparing the coefficients of $\cos x$ and $\sin x$ on both sides:

$1 = A - B$

$0 = -A - B$


Adding these two equations:

$(A - B) + (-A - B) = 1 + 0$

$-2B = 1$

$B = -\frac{1}{2}$


Substitute $B = -\frac{1}{2}$ into the first equation $A - B = 1$:

$A - \left(-\frac{1}{2}\right) = 1$

$A + \frac{1}{2} = 1$

$A = 1 - \frac{1}{2} = \frac{1}{2}$


So, $A = \frac{1}{2}$ and $B = -\frac{1}{2}$.


Now, rewrite the integrand using these values of $A$ and $B$:

$\frac{\cos x}{\cos x - \sin x} = \frac{\frac{1}{2}(\cos x - \sin x) - \frac{1}{2}(-\sin x - \cos x)}{\cos x - \sin x}$

$= \frac{1}{2} \frac{\cos x - \sin x}{\cos x - \sin x} - \frac{1}{2} \frac{-(\sin x + \cos x)}{\cos x - \sin x}$

$= \frac{1}{2} + \frac{1}{2} \frac{\sin x + \cos x}{\cos x - \sin x}$


So the integral becomes:

$I = \int \left(\frac{1}{2} + \frac{1}{2} \frac{\sin x + \cos x}{\cos x - \sin x}\right) dx$

$I = \int \frac{1}{2} dx + \frac{1}{2} \int \frac{\sin x + \cos x}{\cos x - \sin x} dx$


Evaluate the first integral:

$\int \frac{1}{2} dx = \frac{1}{2} x + C_1$


Evaluate the second integral $\int \frac{\sin x + \cos x}{\cos x - \sin x} dx$ using substitution.

Let $u = \cos x - \sin x$.


Differentiating both sides with respect to $x$:

$du = (-\sin x - \cos x) dx$

$du = -(\sin x + \cos x) dx$

So, $(\sin x + \cos x) dx = -du$.


Substitute into the second integral:

$\int \frac{1}{u} (-du) = - \int \frac{1}{u} du$

$= -\log_e |u| + C_2$


Substitute back $u = \cos x - \sin x$:

$= -\log_e |\cos x - \sin x| + C_2$


Combine the results of both integrals:

$I = \left(\frac{1}{2} x + C_1\right) + \frac{1}{2} \left(-\log_e |\cos x - \sin x| + C_2\right)$

$I = \frac{1}{2} x - \frac{1}{2} \log_e |\cos x - \sin x| + C_1 + \frac{C_2}{2}$

Let $C = C_1 + \frac{C_2}{2}$ be the constant of integration.

$I = \frac{1}{2} x - \frac{1}{2} \log_e |\cos x - \sin x| + C$


Thus, the integral of the given function is $\mathbf{\frac{1}{2} x - \frac{1}{2} \log_e |\cos x - \sin x| + C}$.

Question 34. $\frac{\sqrt{\tan x}}{\sin x \cos x}$

Answer:

Let the given integral be $I$.

$I = \int \frac{\sqrt{\tan x}}{\sin x \cos x} dx$


We can manipulate the integrand to prepare for substitution.

Divide the numerator and the denominator by $\cos^2 x$:

$\frac{\sqrt{\tan x}}{\sin x \cos x} = \frac{\frac{\sqrt{\tan x}}{\cos^2 x}}{\frac{\sin x \cos x}{\cos^2 x}}$

$= \frac{\sqrt{\tan x} \cdot \frac{1}{\cos^2 x}}{\frac{\sin x}{\cos x}}$

$= \frac{\sqrt{\tan x} \sec^2 x}{\tan x}$

$= \frac{\sqrt{\tan x}}{\tan x} \sec^2 x$

Using the property $\frac{\sqrt{a}}{a} = \frac{1}{\sqrt{a}}$ (for $a>0$), we get:

$= \frac{1}{\sqrt{\tan x}} \sec^2 x$


So the integral becomes:

$I = \int \frac{\sec^2 x}{\sqrt{\tan x}} dx$


We use the method of substitution.

Let $u = \tan x$.


Differentiating both sides with respect to $x$, we get:

$\frac{du}{dx} = \frac{d}{dx}(\tan x)$

$\frac{du}{dx} = \sec^2 x$


Rearranging the terms, we get:

$du = \sec^2 x \;dx$


Now, substitute $u = \tan x$ and $\sec^2 x \;dx = du$ into the integral $I$:

$I = \int \frac{1}{\sqrt{u}} du$

$I = \int u^{-1/2} du$


Integrate $u^{-1/2}$ with respect to $u$ using the power rule for integration $\int u^n du = \frac{u^{n+1}}{n+1} + C'$:

$\int u^{-1/2} du = \frac{u^{-1/2 + 1}}{-1/2 + 1} + C'$

$\int u^{-1/2} du = \frac{u^{1/2}}{1/2} + C'$

$\int u^{-1/2} du = 2u^{1/2} + C'$

$\int u^{-1/2} du = 2\sqrt{u} + C'$


Now, substitute this back into the expression for $I$:

$I = 2\sqrt{u} + C$, where $C = C'$ is the constant of integration.


Substitute back $u = \tan x$:

$I = 2\sqrt{\tan x} + C$


Thus, the integral of the given function is $\mathbf{2\sqrt{\tan x} + C}$, where $C$ is the constant of integration.

Question 35. $\frac{(1 + \log x)^2}{x}$

Answer:

Let the given integral be $I$.

$I = \int \frac{(1 + \log x)^2}{x} dx$


We can rewrite the integrand as:

$I = \int (1 + \log x)^2 \cdot \frac{1}{x} dx$


We use the method of substitution.

Let $u = 1 + \log x$.


Differentiating both sides with respect to $x$, we get:

$\frac{du}{dx} = \frac{d}{dx}(1 + \log x)$

$\frac{du}{dx} = 0 + \frac{1}{x}$

$\frac{du}{dx} = \frac{1}{x}$


Rearranging the terms, we get:

$du = \frac{1}{x} dx$


Now, substitute $u = 1 + \log x$ and $\frac{1}{x} dx = du$ into the integral $I$:

$I = \int u^2 du$


Integrate $u^2$ with respect to $u$ using the power rule for integration ($\int u^n du = \frac{u^{n+1}}{n+1} + C'$):

$I = \frac{u^{2+1}}{2+1} + C'$

$I = \frac{u^3}{3} + C'$


Substitute back $u = 1 + \log x$:

$I = \frac{(1 + \log x)^3}{3} + C$, where $C = C'$ is the constant of integration.


Thus, the integral of the given function is $\mathbf{\frac{(1 + \log x)^3}{3} + C}$, where $C$ is the constant of integration.

Question 36. $\frac{(x + 1) (x + \log x)^2}{x}$

Answer:

Let the given integral be $I$.

$I = \int \frac{(x + 1) (x + \log x)^2}{x} dx$


Rewrite the integrand by dividing the term $(x+1)$ by $x$:

$\frac{(x + 1)}{x} (x + \log x)^2 = \left(\frac{x}{x} + \frac{1}{x}\right) (x + \log x)^2 = \left(1 + \frac{1}{x}\right) (x + \log x)^2$


So the integral becomes:

$I = \int \left(1 + \frac{1}{x}\right) (x + \log x)^2 dx$


We use the method of substitution.

Let $u = x + \log x$.


Differentiating both sides with respect to $x$, we get:

$\frac{du}{dx} = \frac{d}{dx}(x + \log x)$

$\frac{du}{dx} = \frac{d}{dx}(x) + \frac{d}{dx}(\log x)$

$\frac{du}{dx} = 1 + \frac{1}{x}$


Rearranging the terms, we get:

$du = \left(1 + \frac{1}{x}\right) dx$


Now, substitute $u = x + \log x$ and $\left(1 + \frac{1}{x}\right) dx = du$ into the integral $I$:

$I = \int u^2 du$


Integrate $u^2$ with respect to $u$ using the power rule for integration ($\int u^n du = \frac{u^{n+1}}{n+1} + C'$, for $n \neq -1$):

$I = \frac{u^{2+1}}{2+1} + C'$

$I = \frac{u^3}{3} + C'$


Substitute back $u = x + \log x$:

$I = \frac{(x + \log x)^3}{3} + C$, where $C = C'$ is the constant of integration.


Thus, the integral of the given function is $\frac{(x + \log x)^3}{3} + C$.

Question 37. $\frac{x^3 \sin (\tan^{−1} x^4)}{1 + x^8}$

Answer:

Let the given integral be $I$.

$I = \int \frac{x^3 \sin (\tan^{−1} x^4)}{1 + x^8} dx$


We use the method of substitution.

Let $u = \tan^{−1} x^4$.


Differentiating both sides with respect to $x$, we get:

$\frac{du}{dx} = \frac{d}{dx}(\tan^{−1} x^4)$

Using the chain rule, $\frac{du}{dx} = \frac{1}{1 + (x^4)^2} \cdot \frac{d}{dx}(x^4)$

$\frac{du}{dx} = \frac{1}{1 + x^8} \cdot 4x^3$

$\frac{du}{dx} = \frac{4x^3}{1 + x^8}$


Rearranging the terms, we get:

$du = \frac{4x^3}{1 + x^8} dx$

We have $\frac{x^3}{1 + x^8} dx$ in the integrand, so we can write:

$\frac{x^3}{1 + x^8} dx = \frac{du}{4}$


Now, substitute $u = \tan^{−1} x^4$ and $\frac{x^3}{1 + x^8} dx = \frac{du}{4}$ into the integral $I$:

$I = \int \sin(u) \cdot \frac{du}{4}$

$I = \frac{1}{4} \int \sin(u) du$


The integral of $\sin(u)$ with respect to $u$ is $-\cos(u) + C'$, where $C'$ is the constant of integration.

$I = \frac{1}{4} (-\cos(u)) + C'$

$I = -\frac{1}{4} \cos(u) + C$, where $C = \frac{C'}{4}$ is the constant of integration.


Substitute back $u = \tan^{−1} x^4$:

$I = -\frac{1}{4} \cos(\tan^{−1} x^4) + C$


Thus, the integral of the given function is $-\frac{1}{4} \cos(\tan^{−1} x^4) + C$.

Choose the correct answer in Exercises 38 and 39.

Question 38. $\int \frac{10x^9 + 10^x \log_e 10 \;dx}{x^{10} + 10^x}$ equals

(A) 10x – x10 + C

(B) 10x + x10 + C

(C) (10x – x10)–1 + C

(D) log (10x + x10) + C

Answer:

Let the given integral be $I$.

$I = \int \frac{10x^9 + 10^x \log_e 10}{x^{10} + 10^x} dx$


We use the method of substitution.

Let $u = x^{10} + 10^x$.


Differentiating both sides with respect to $x$, we get:

$\frac{du}{dx} = \frac{d}{dx}(x^{10} + 10^x)$

Using the power rule and the derivative of exponential function $\frac{d}{dx}(a^x) = a^x \log_e a$, we have:

$\frac{du}{dx} = 10x^{10-1} + 10^x \log_e 10$

$\frac{du}{dx} = 10x^9 + 10^x \log_e 10$


Rearranging the terms, we get:

$du = (10x^9 + 10^x \log_e 10) dx$


Observe that the expression $(10x^9 + 10^x \log_e 10) dx$ is exactly the numerator of the integrand multiplied by $dx$.

Now, substitute $u = x^{10} + 10^x$ and $(10x^9 + 10^x \log_e 10) dx = du$ into the integral $I$:

$I = \int \frac{1}{u} du$


The integral of $\frac{1}{u}$ with respect to $u$ is $\log_e |u| + C'$, where $C'$ is the constant of integration.

$I = \log_e |u| + C'$


Substitute back $u = x^{10} + 10^x$:

$I = \log_e |x^{10} + 10^x| + C'$, where $C'$ is the constant of integration.


Since $x^{10} \geq 0$ (for real $x$) and $10^x > 0$ for all real $x$, the term $x^{10} + 10^x$ is always positive for real $x$. Therefore, the absolute value is not necessary.

$I = \log_e (x^{10} + 10^x) + C$, where $C = C'$ is the constant of integration.


Comparing this result with the given options, we see that it matches option (D).

The correct answer is (D) log (10x + x10) + C.

Question 39. $\int \frac{dx}{\sin^2 x \cos^2 x}$ equals

(A) tan x + cot x + C

(B) tan x – cot x + C

(C) tan x cot x + C

(D) tan x – cot 2x + C

Answer:

Let the given integral be $I$.

$I = \int \frac{1}{\sin^2 x \cos^2 x} dx$


We can rewrite the integrand using the identity $1 = \sin^2 x + \cos^2 x$ in the numerator:

$I = \int \frac{\sin^2 x + \cos^2 x}{\sin^2 x \cos^2 x} dx$


Now, split the fraction into two parts:

$I = \int \left( \frac{\sin^2 x}{\sin^2 x \cos^2 x} + \frac{\cos^2 x}{\sin^2 x \cos^2 x} \right) dx$


Simplify each term:

$I = \int \left( \frac{1}{\cos^2 x} + \frac{1}{\sin^2 x} \right) dx$


Using the identities $\frac{1}{\cos^2 x} = \sec^2 x$ and $\frac{1}{\sin^2 x} = \text{cosec}^2 x$, the integral becomes:

$I = \int (\sec^2 x + \text{cosec}^2 x) dx$


Integrate term by term:

$I = \int \sec^2 x \;dx + \int \text{cosec}^2 x \;dx$


The integral of $\sec^2 x$ is $\tan x$, and the integral of $\text{cosec}^2 x$ is $-\cot x$.

$I = \tan x + (-\cot x) + C$

$I = \tan x - \cot x + C$


Thus, the integral of the given function is $\mathbf{\tan x - \cot x + C}$, where $C$ is the constant of integration.


Comparing this result with the given options, we see that it matches option (B).

The correct answer is (B) tan x – cot x + C.



Example 7 (Before Exercise 7.3)

Example 7: Find

(i) $\int \cos^2 x \;dx$

(ii) $\int \sin 2x \cos 3x \;dx$

(iii) $\int \sin^3 x \;dx$

Answer:

(i) $\int \cos^2 x \;dx$

Let the given integral be $I$.

$I = \int \cos^2 x \;dx$

We use the trigonometric identity $\cos^2 x = \frac{1 + \cos(2x)}{2}$.

Substitute the identity into the integral:

$I = \int \frac{1 + \cos(2x)}{2} dx$

$I = \frac{1}{2} \int (1 + \cos(2x)) dx$

$I = \frac{1}{2} \left( \int 1 \;dx + \int \cos(2x) \;dx \right)$

Integrating term by term, we get:

$\int 1 \;dx = x$

$\int \cos(2x) \;dx = \frac{1}{2} \sin(2x)$ (using substitution $u=2x$)

Combine the results:

$I = \frac{1}{2} \left( x + \frac{1}{2} \sin(2x) \right) + C$

$I = \frac{x}{2} + \frac{1}{4} \sin(2x) + C$, where $C$ is the constant of integration.

Thus, $\int \cos^2 x \;dx = \mathbf{\frac{x}{2} + \frac{1}{4} \sin(2x) + C}$.


(ii) $\int \sin 2x \cos 3x \;dx$

Let the given integral be $I$.

$I = \int \sin 2x \cos 3x \;dx$

We use the product-to-sum trigonometric identity: $\sin A \cos B = \frac{1}{2}[\sin(A+B) + \sin(A-B)]$.

Here $A = 2x$ and $B = 3x$.

$\sin 2x \cos 3x = \frac{1}{2}[\sin(2x+3x) + \sin(2x-3x)]$

$= \frac{1}{2}[\sin(5x) + \sin(-x)]$

Using the identity $\sin(-x) = -\sin x$, we get:

$= \frac{1}{2}[\sin(5x) - \sin x]$

Substitute the identity into the integral:

$I = \int \frac{1}{2}[\sin(5x) - \sin x] dx$

$I = \frac{1}{2} \int (\sin(5x) - \sin x) dx$

$I = \frac{1}{2} \left( \int \sin(5x) \;dx - \int \sin x \;dx \right)$

Integrate term by term.

$\int \sin x \;dx = -\cos x$

$\int \sin(5x) \;dx = -\frac{1}{5} \cos(5x)$ (using substitution $u=5x$)

Combine the results:

$I = \frac{1}{2} \left( -\frac{1}{5} \cos(5x) - (-\cos x) \right) + C$

$I = \frac{1}{2} \left( -\frac{1}{5} \cos(5x) + \cos x \right) + C$

$I = \frac{1}{2} \cos x - \frac{1}{10} \cos(5x) + C$, where $C$ is the constant of integration.

Thus, $\int \sin 2x \cos 3x \;dx = \mathbf{\frac{1}{2} \cos x - \frac{1}{10} \cos(5x) + C}$.


(iii) $\int \sin^3 x \;dx$

Let the given integral be $I$.

$I = \int \sin^3 x \;dx$

We can rewrite $\sin^3 x$ as $\sin^2 x \cdot \sin x$.

$I = \int \sin^2 x \sin x \;dx$

We use the identity $\sin^2 x = 1 - \cos^2 x$.

$I = \int (1 - \cos^2 x) \sin x \;dx$

We use the method of substitution.

Let $u = \cos x$.

Differentiating both sides with respect to $x$, we get:

$\frac{du}{dx} = -\sin x$

So, $\sin x \;dx = -du$.

Now, substitute $u = \cos x$ and $\sin x \;dx = -du$ into the integral $I$:

$I = \int (1 - u^2) (-du)$

$I = - \int (1 - u^2) du$

$I = - \int 1 \;du + \int u^2 \;du$

Integrate term by term using the power rule for integration:

$I = -u + \frac{u^3}{3} + C$, where $C$ is the constant of integration.

Substitute back $u = \cos x$:

$I = -\cos x + \frac{(\cos x)^3}{3} + C$

$I = -\cos x + \frac{\cos^3 x}{3} + C$

Thus, $\int \sin^3 x \;dx = \mathbf{-\cos x + \frac{\cos^3 x}{3} + C}$.



Exercise 7.3

Find the integrals of the functions in Exercises 1 to 22:

Question 1. sin2 (2x + 5)

Answer:

Solution:

We need to find the integral of $\sin^2(2x+5)$ with respect to $x$.

The integral is given by:

$\int \sin^2(2x+5) \, dx$

We use the trigonometric identity:

$\sin^2 \theta = \frac{1 - \cos(2\theta)}{2}$

Here, $\theta = 2x+5$. So, $2\theta = 2(2x+5) = 4x+10$.

Substituting this into the identity, we get:

$\sin^2(2x+5) = \frac{1 - \cos(4x+10)}{2}$

Now, we can rewrite the integral:

$\int \sin^2(2x+5) \, dx = \int \frac{1 - \cos(4x+10)}{2} \, dx$

= $\frac{1}{2} \int (1 - \cos(4x+10)) \, dx$

We can split this into two separate integrals:

= $\frac{1}{2} \left( \int 1 \, dx - \int \cos(4x+10) \, dx \right)$

First integral is straightforward:

$\int 1 \, dx = x$

For the second integral, $\int \cos(4x+10) \, dx$, we use a substitution method.

Let $u = 4x+10$.

Differentiating both sides with respect to $x$:

$\frac{du}{dx} = \frac{d}{dx}(4x+10) = 4$

So, $du = 4 \, dx$, which means $dx = \frac{1}{4} \, du$.

Substituting $u$ and $dx$ into the second integral:

$\int \cos(4x+10) \, dx = \int \cos(u) \frac{1}{4} \, du$

= $\frac{1}{4} \int \cos(u) \, du$

The integral of $\cos(u)$ is $\sin(u)$.

= $\frac{1}{4} \sin(u) + C'$

Substitute back $u = 4x+10$:

= $\frac{1}{4} \sin(4x+10) + C'$

Now, combine the results of both integrals:

$\int \sin^2(2x+5) \, dx = \frac{1}{2} \left( x - \left( \frac{1}{4} \sin(4x+10) \right) \right) + C$

= $\frac{1}{2} \left( x - \frac{1}{4} \sin(4x+10) \right) + C$

Distribute the $\frac{1}{2}$:

= $\frac{x}{2} - \frac{1}{8} \sin(4x+10) + C$


The final answer is:

$\boxed{\frac{x}{2} - \frac{1}{8} \sin(4x+10) + C}$

Question 2. sin 3x cos 4x

Answer:

Solution:

We need to find the integral of $\sin(3x) \cos(4x)$ with respect to $x$.

The integral is given by:

$\int \sin(3x) \cos(4x) \, dx$

We use the product-to-sum trigonometric identity:

$2 \sin A \cos B = \sin(A+B) + \sin(A-B)$

Dividing by 2, we get:

$\sin A \cos B = \frac{1}{2} [\sin(A+B) + \sin(A-B)]$

Let $A = 3x$ and $B = 4x$. Then $A+B = 3x+4x = 7x$ and $A-B = 3x-4x = -x$.

So, $\sin(3x) \cos(4x) = \frac{1}{2} [\sin(7x) + \sin(-x)]$.

Since $\sin(-x) = -\sin(x)$, the expression becomes:

$\sin(3x) \cos(4x) = \frac{1}{2} [\sin(7x) - \sin(x)]$

Now, we can rewrite the integral:

$\int \sin(3x) \cos(4x) \, dx = \int \frac{1}{2} [\sin(7x) - \sin(x)] \, dx$

= $\frac{1}{2} \int [\sin(7x) - \sin(x)] \, dx$

We can integrate each term separately:

= $\frac{1}{2} \left( \int \sin(7x) \, dx - \int \sin(x) \, dx \right)$

For $\int \sin(7x) \, dx$, let $u = 7x$, so $du = 7 \, dx$, or $dx = \frac{1}{7} \, du$.

$\int \sin(7x) \, dx = \int \sin(u) \frac{1}{7} \, du = \frac{1}{7} \int \sin(u) \, du = \frac{1}{7} (-\cos(u)) + C_1 = -\frac{1}{7} \cos(7x) + C_1$

For $\int \sin(x) \, dx$, the integral is $-\cos(x) + C_2$.

Substituting these back into the main integral:

= $\frac{1}{2} \left( -\frac{1}{7} \cos(7x) - (-\cos(x)) \right) + C$

= $\frac{1}{2} \left( -\frac{1}{7} \cos(7x) + \cos(x) \right) + C$

= $\frac{1}{2} \cos(x) - \frac{1}{14} \cos(7x) + C$


The final answer is:

$\boxed{\frac{1}{2} \cos(x) - \frac{1}{14} \cos(7x) + C}$

Question 3. cos 2x cos 4x cos 6x

Answer:

Solution:

We need to find the integral of $\cos(2x) \cos(4x) \cos(6x)$ with respect to $x$.

The integral is given by:

$\int \cos(2x) \cos(4x) \cos(6x) \, dx$

We use the product-to-sum trigonometric identity: $2 \cos A \cos B = \cos(A+B) + \cos(A-B)$.

This gives $\cos A \cos B = \frac{1}{2} [\cos(A+B) + \cos(A-B)]$.

First, consider the product of two terms, say $\cos(4x) \cos(6x)$:

$\cos(4x) \cos(6x) = \frac{1}{2} [\cos(4x+6x) + \cos(4x-6x)]$

= $\frac{1}{2} [\cos(10x) + \cos(-2x)]$

Since $\cos(-\theta) = \cos(\theta)$, we have:

= $\frac{1}{2} [\cos(10x) + \cos(2x)]$

Now, multiply this result by the remaining term $\cos(2x)$:

$\cos(2x) \cos(4x) \cos(6x) = \cos(2x) \cdot \frac{1}{2} [\cos(10x) + \cos(2x)]$

= $\frac{1}{2} [\cos(2x)\cos(10x) + \cos(2x)\cos(2x)]$

= $\frac{1}{2} [\cos(2x)\cos(10x) + \cos^2(2x)]$

Apply the product-to-sum identity again to $\cos(2x)\cos(10x)$ (with $A=2x, B=10x$):

$\cos(2x)\cos(10x) = \frac{1}{2} [\cos(2x+10x) + \cos(2x-10x)]$

= $\frac{1}{2} [\cos(12x) + \cos(-8x)]$

= $\frac{1}{2} [\cos(12x) + \cos(8x)]$

Also, use the identity $\cos^2 \theta = \frac{1+\cos(2\theta)}{2}$ for $\cos^2(2x)$:

$\cos^2(2x) = \frac{1+\cos(2 \cdot 2x)}{2} = \frac{1+\cos(4x)}{2}$

Substitute these back into the expression for $\cos(2x) \cos(4x) \cos(6x)$:

= $\frac{1}{2} \left[ \frac{1}{2}(\cos(12x) + \cos(8x)) + \frac{1+\cos(4x)}{2} \right]$

= $\frac{1}{2} \cdot \frac{1}{2} [(\cos(12x) + \cos(8x)) + (1+\cos(4x))]$

= $\frac{1}{4} [1 + \cos(4x) + \cos(8x) + \cos(12x)]$

Now, integrate this simplified expression:

$\int \cos(2x) \cos(4x) \cos(6x) \, dx = \int \frac{1}{4} [1 + \cos(4x) + \cos(8x) + \cos(12x)] \, dx$

= $\frac{1}{4} \int [1 + \cos(4x) + \cos(8x) + \cos(12x)] \, dx$

Integrate term by term, recalling that $\int \cos(ax) \, dx = \frac{1}{a}\sin(ax) + C'$:

= $\frac{1}{4} \left[ \int 1 \, dx + \int \cos(4x) \, dx + \int \cos(8x) \, dx + \int \cos(12x) \, dx \right]$

= $\frac{1}{4} \left[ x + \frac{1}{4}\sin(4x) + \frac{1}{8}\sin(8x) + \frac{1}{12}\sin(12x) \right] + C$

= $\frac{x}{4} + \frac{1}{16}\sin(4x) + \frac{1}{32}\sin(8x) + \frac{1}{48}\sin(12x) + C$


The final answer is:

$\boxed{\frac{x}{4} + \frac{1}{16}\sin(4x) + \frac{1}{32}\sin(8x) + \frac{1}{48}\sin(12x) + C}$

Question 4. sin3 (2x + 1)

Answer:

Solution:

We need to find the integral of $\sin^3(2x+1)$ with respect to $x$.

The integral is given by:

$\int \sin^3(2x+1) \, dx$

We can rewrite $\sin^3(2x+1)$ as $\sin^2(2x+1) \cdot \sin(2x+1)$.

Using the identity $\sin^2 \theta = 1 - \cos^2 \theta$, we get:

$\sin^3(2x+1) = (1 - \cos^2(2x+1)) \sin(2x+1)$

Now, the integral becomes:

$\int (1 - \cos^2(2x+1)) \sin(2x+1) \, dx$

We use the substitution method.

Let $u = \cos(2x+1)$.

Differentiating $u$ with respect to $x$ using the chain rule:

$\frac{du}{dx} = \frac{d}{dx}(\cos(2x+1)) = -\sin(2x+1) \cdot \frac{d}{dx}(2x+1)$

= $-\sin(2x+1) \cdot 2$

= $-2 \sin(2x+1)$

Rearranging to find $\sin(2x+1) \, dx$:

$du = -2 \sin(2x+1) \, dx$

$\sin(2x+1) \, dx = -\frac{1}{2} \, du$

Substitute $u = \cos(2x+1)$ and $\sin(2x+1) \, dx = -\frac{1}{2} \, du$ into the integral:

$\int (1 - \cos^2(2x+1)) \sin(2x+1) \, dx = \int (1 - u^2) \left(-\frac{1}{2}\right) \, du$

= $-\frac{1}{2} \int (1 - u^2) \, du$

Now, integrate with respect to $u$:

= $-\frac{1}{2} \left( \int 1 \, du - \int u^2 \, du \right)$

= $-\frac{1}{2} \left( u - \frac{u^3}{3} \right) + C'$

Distribute the $-\frac{1}{2}$:

= $-\frac{1}{2} u + \frac{1}{6} u^3 + C'$

Substitute back $u = \cos(2x+1)$:

= $-\frac{1}{2} \cos(2x+1) + \frac{1}{6} \cos^3(2x+1) + C$ (where $C$ is the constant of integration)

We can rearrange the terms:

= $\frac{1}{6} \cos^3(2x+1) - \frac{1}{2} \cos(2x+1) + C$


The final answer is:

$\boxed{\frac{1}{6} \cos^3(2x+1) - \frac{1}{2} \cos(2x+1) + C}$

Question 5. sin3 x cos3 x

Answer:

To evaluate the integral $\int \sin^3 x \cos^3 x \;dx$.

We can rewrite the integrand $\sin^3 x \cos^3 x$ as follows:

$\sin^3 x \cos^3 x = \sin^2 x \cos^3 x \sin x$

Using the trigonometric identity $\sin^2 x = 1 - \cos^2 x$, we substitute it into the expression:

$= (1 - \cos^2 x) \cos^3 x \sin x$

Now, we use the method of substitution to evaluate the integral.

Let $u = \cos x$.

Differentiating $u$ with respect to $x$ gives:

$\frac{du}{dx} = \frac{d}{dx}(\cos x) = -\sin x$

Rearranging this differential, we get $du = -\sin x \;dx$, which means $\sin x \;dx = -du$.

Substitute $u = \cos x$ and $\sin x \;dx = -du$ into the integral:

$\int (1 - \cos^2 x) \cos^3 x \sin x \;dx = \int (1 - u^2) u^3 (-du)$

$= - \int (u^3 \times 1 - u^3 \times u^2) du$

$= - \int (u^3 - u^{3+2}) du$

$= - \int (u^3 - u^5) du$

We can distribute the negative sign:

$= \int -(u^3 - u^5) du$

$= \int (-u^3 + u^5) du$

$= \int (u^5 - u^3) du$

Now, we integrate term by term using the power rule for integration, $\int x^n dx = \frac{x^{n+1}}{n+1} + C'$ (where $n \neq -1$):

$= \frac{u^{5+1}}{5+1} - \frac{u^{3+1}}{3+1} + C$

$= \frac{u^6}{6} - \frac{u^4}{4} + C$

Substitute back $u = \cos x$ to get the result in terms of the original variable $x$:

$= \frac{(\cos x)^6}{6} - \frac{(\cos x)^4}{4} + C$

$= \frac{1}{6} \cos^6 x - \frac{1}{4} \cos^4 x + C$

Where $C$ is the constant of integration.

Thus, the integral of $\sin^3 x \cos^3 x$ is $\mathbf{\frac{1}{6} \cos^6 x - \frac{1}{4} \cos^4 x + C}$.


Alternate Solution:

We can also solve this integral by using a different substitution.

Start with the integral $\int \sin^3 x \cos^3 x \;dx$.

We can rewrite the integrand as:

$\sin^3 x \cos^3 x = \sin^3 x \cos^2 x \cos x$

Using the trigonometric identity $\cos^2 x = 1 - \sin^2 x$, we substitute it into the expression:

$= \sin^3 x (1 - \sin^2 x) \cos x$

Now, we use the method of substitution.

Let $v = \sin x$.

Differentiating $v$ with respect to $x$ gives:

$\frac{dv}{dx} = \frac{d}{dx}(\sin x) = \cos x$

Rearranging, we get $dv = \cos x \;dx$.

Substitute $v = \sin x$ and $\cos x \;dx = dv$ into the integral:

$\int \sin^3 x (1 - \sin^2 x) \cos x \;dx = \int v^3 (1 - v^2) dv$

$= \int (v^3 \times 1 - v^3 \times v^2) dv$

$= \int (v^3 - v^5) dv$

Integrate term by term using the power rule:

$= \frac{v^{3+1}}{3+1} - \frac{v^{5+1}}{5+1} + C'$

$= \frac{v^4}{4} - \frac{v^6}{6} + C'$

Substitute back $v = \sin x$ to get the result in terms of $x$:

$= \frac{(\sin x)^4}{4} - \frac{(\sin x)^6}{6} + C'$

$= \frac{1}{4} \sin^4 x - \frac{1}{6} \sin^6 x + C'$

Where $C'$ is the constant of integration.

This form of the solution, $\frac{1}{4} \sin^4 x - \frac{1}{6} \sin^6 x + C'$, is equivalent to the first form $\frac{1}{6} \cos^6 x - \frac{1}{4} \cos^4 x + C$, as they differ only by a constant value.

Question 6. sin x sin 2x sin 3x

Answer:

Solution:

We need to find the integral of $\sin x \sin 2x \sin 3x$ with respect to $x$.

The integral is given by:

$\int \sin x \sin 2x \sin 3x \, dx$

We use the product-to-sum trigonometric identity:

$2 \sin A \sin B = \cos(A-B) - \cos(A+B)$

Let's group $\sin x \sin 2x$ first. Using the identity with $A=2x$ and $B=x$:

$2 \sin 2x \sin x = \cos(2x-x) - \cos(2x+x) = \cos x - \cos 3x$

So, $\sin x \sin 2x = \frac{1}{2}(\cos x - \cos 3x)$.

Now multiply by $\sin 3x$:

$\sin x \sin 2x \sin 3x = \frac{1}{2}(\cos x - \cos 3x) \sin 3x$

= $\frac{1}{2}(\cos x \sin 3x - \cos 3x \sin 3x)$

Now, we use another product-to-sum identity for the first term in the bracket:

$2 \cos A \sin B = \sin(A+B) - \sin(A-B)$

For $\cos x \sin 3x$, let $A=x$ and $B=3x$:

$2 \cos x \sin 3x = \sin(x+3x) - \sin(x-3x) = \sin 4x - \sin(-2x)$

Since $\sin(-\theta) = -\sin \theta$, this becomes:

= $\sin 4x - (-\sin 2x) = \sin 4x + \sin 2x$

So, $\cos x \sin 3x = \frac{1}{2}(\sin 4x + \sin 2x)$.

For the second term in the bracket, $\cos 3x \sin 3x$, we use the double angle identity:

$\sin(2\theta) = 2 \sin \theta \cos \theta$

Let $\theta = 3x$:

$\sin(2 \cdot 3x) = 2 \sin 3x \cos 3x$

So, $\cos 3x \sin 3x = \frac{1}{2}\sin 6x$.

Substitute these results back into the expression for $\sin x \sin 2x \sin 3x$:

= $\frac{1}{2} \left[ \frac{1}{2}(\sin 4x + \sin 2x) - \frac{1}{2}\sin 6x \right]$

= $\frac{1}{2} \cdot \frac{1}{2} [\sin 4x + \sin 2x - \sin 6x]$

= $\frac{1}{4} [\sin 4x + \sin 2x - \sin 6x]$

Now, we integrate this expression:

$\int \sin x \sin 2x \sin 3x \, dx = \int \frac{1}{4} [\sin 4x + \sin 2x - \sin 6x] \, dx$

= $\frac{1}{4} \int (\sin 4x + \sin 2x - \sin 6x) \, dx$

We integrate each term using the standard integral $\int \sin(ax) \, dx = -\frac{1}{a}\cos(ax) + C'$:

= $\frac{1}{4} \left[ \int \sin 4x \, dx + \int \sin 2x \, dx - \int \sin 6x \, dx \right]$

= $\frac{1}{4} \left[ -\frac{1}{4}\cos 4x - \frac{1}{2}\cos 2x - \left(-\frac{1}{6}\cos 6x\right) \right] + C$

= $\frac{1}{4} \left[ -\frac{1}{4}\cos 4x - \frac{1}{2}\cos 2x + \frac{1}{6}\cos 6x \right] + C$

Distribute the $\frac{1}{4}$:

= $-\frac{1}{16}\cos 4x - \frac{1}{8}\cos 2x + \frac{1}{24}\cos 6x + C$

Rearranging the terms:

= $\frac{1}{24}\cos 6x - \frac{1}{8}\cos 2x - \frac{1}{16}\cos 4x + C$


The final answer is:

$\boxed{\frac{1}{24}\cos 6x - \frac{1}{8}\cos 2x - \frac{1}{16}\cos 4x + C}$

Question 7. sin 4x sin 8x

Answer:

Solution:

We need to find the integral of $\sin(4x) \sin(8x)$ with respect to $x$.

The integral is given by:

$\int \sin(4x) \sin(8x) \, dx$

We use the product-to-sum trigonometric identity:

$2 \sin A \sin B = \cos(A-B) - \cos(A+B)$

Let $A = 8x$ and $B = 4x$. Then $A-B = 8x-4x = 4x$ and $A+B = 8x+4x = 12x$.

So, $2 \sin(8x) \sin(4x) = \cos(4x) - \cos(12x)$.

Dividing by 2, we get:

$\sin(4x) \sin(8x) = \frac{1}{2} [\cos(4x) - \cos(12x)]$

Now, we can rewrite the integral:

$\int \sin(4x) \sin(8x) \, dx = \int \frac{1}{2} [\cos(4x) - \cos(12x)] \, dx$

= $\frac{1}{2} \int [\cos(4x) - \cos(12x)] \, dx$

We can integrate each term separately:

= $\frac{1}{2} \left( \int \cos(4x) \, dx - \int \cos(12x) \, dx \right)$

Using the standard integral $\int \cos(ax) \, dx = \frac{1}{a}\sin(ax) + C'$:

= $\frac{1}{2} \left( \frac{1}{4}\sin(4x) - \frac{1}{12}\sin(12x) \right) + C$

Distribute the $\frac{1}{2}$:

= $\frac{1}{8}\sin(4x) - \frac{1}{24}\sin(12x) + C$


The final answer is:

$\boxed{\frac{1}{8}\sin(4x) - \frac{1}{24}\sin(12x) + C}$

Question 8. $\frac{1 − \cos x}{1 + \cos x}$

Answer:

Solution:

We need to find the integral of $\frac{1 - \cos x}{1 + \cos x}$ with respect to $x$.

The integral is given by:

$\int \frac{1 - \cos x}{1 + \cos x} \, dx$

We use the half-angle identities:

$1 - \cos x = 2 \sin^2 \left(\frac{x}{2}\right)$

$1 + \cos x = 2 \cos^2 \left(\frac{x}{2}\right)$

Substitute these into the integrand:

$\frac{1 - \cos x}{1 + \cos x} = \frac{2 \sin^2 \left(\frac{x}{2}\right)}{2 \cos^2 \left(\frac{x}{2}\right)}$

Cancel out the factor of 2:

= $\frac{\sin^2 \left(\frac{x}{2}\right)}{\cos^2 \left(\frac{x}{2}\right)}$

Using the identity $\tan \theta = \frac{\sin \theta}{\cos \theta}$, we get:

= $\tan^2 \left(\frac{x}{2}\right)$

Now, we use the identity $\tan^2 \theta = \sec^2 \theta - 1$. Let $\theta = \frac{x}{2}$:

= $\sec^2 \left(\frac{x}{2}\right) - 1$

Now, the integral becomes:

$\int \left(\sec^2 \left(\frac{x}{2}\right) - 1\right) \, dx$

We can integrate term by term:

= $\int \sec^2 \left(\frac{x}{2}\right) \, dx - \int 1 \, dx$

For the first integral, $\int \sec^2 \left(\frac{x}{2}\right) \, dx$, we use substitution.

Let $u = \frac{x}{2}$.

Differentiating with respect to $x$, $\frac{du}{dx} = \frac{1}{2}$. So, $dx = 2 \, du$.

Substitute $u$ and $dx$ into the first integral:

$\int \sec^2(u) \, (2 \, du) = 2 \int \sec^2(u) \, du$

The integral of $\sec^2(u)$ is $\tan(u)$.

= $2 \tan(u) + C_1$

Substitute back $u = \frac{x}{2}$:

= $2 \tan \left(\frac{x}{2}\right) + C_1$

The second integral is straightforward:

$\int 1 \, dx = x + C_2$

Combining the results and adding the constant of integration $C$ ($C = C_1 + C_2$):

= $2 \tan \left(\frac{x}{2}\right) - x + C$


The final answer is:

$\boxed{2 \tan \left(\frac{x}{2}\right) - x + C}$

Question 9. $\frac{\cos x}{1 + \cos x}$

Answer:

Solution:

We need to find the integral of $\frac{\cos x}{1 + \cos x}$ with respect to $x$.

The integral is given by:

$\int \frac{\cos x}{1 + \cos x} \, dx$

We can simplify the integrand by adding and subtracting 1 in the numerator:

$\frac{\cos x}{1 + \cos x} = \frac{1 + \cos x - 1}{1 + \cos x}$

= $\frac{1 + \cos x}{1 + \cos x} - \frac{1}{1 + \cos x}$

= $1 - \frac{1}{1 + \cos x}$

Now, we integrate this simplified expression:

$\int \left(1 - \frac{1}{1 + \cos x}\right) \, dx$

= $\int 1 \, dx - \int \frac{1}{1 + \cos x} \, dx$

The first integral is:

$\int 1 \, dx = x$

For the second integral, we use the half-angle identity $1 + \cos x = 2 \cos^2 \left(\frac{x}{2}\right)$:

$\int \frac{1}{1 + \cos x} \, dx = \int \frac{1}{2 \cos^2 \left(\frac{x}{2}\right)} \, dx$

= $\int \frac{1}{2} \sec^2 \left(\frac{x}{2}\right) \, dx$

Let $u = \frac{x}{2}$. Then differentiating with respect to $x$, $\frac{du}{dx} = \frac{1}{2}$. This gives $dx = 2 \, du$.

Substituting into the integral:

$\int \frac{1}{2} \sec^2(u) \, (2 \, du) = \int \sec^2(u) \, du$

The integral of $\sec^2(u)$ is $\tan(u)$.

= $\tan u + C'$

Substitute back $u = \frac{x}{2}$:

= $\tan \left(\frac{x}{2}\right) + C'$

Combining the results of the two integrals:

$\int \frac{\cos x}{1 + \cos x} \, dx = x - \tan \left(\frac{x}{2}\right) + C$ (where $C$ is the constant of integration)


The final answer is:

$\boxed{x - \tan \left(\frac{x}{2}\right) + C}$

Question 10. sin4 x

Answer:

Solution:

We need to find the integral of $\sin^4 x$ with respect to $x$.

The integral is given by:

$\int \sin^4 x \, dx$

We can rewrite $\sin^4 x$ using trigonometric identities:

$\sin^4 x = (\sin^2 x)^2$

Using the identity $\sin^2 x = \frac{1 - \cos(2x)}{2}$:

$\sin^4 x = \left(\frac{1 - \cos(2x)}{2}\right)^2$

= $\frac{(1 - \cos(2x))^2}{4}$

= $\frac{1 - 2\cos(2x) + \cos^2(2x)}{4}$

Now, use the identity $\cos^2 \theta = \frac{1 + \cos(2\theta)}{2}$ for $\cos^2(2x)$, where $\theta = 2x$:

$\cos^2(2x) = \frac{1 + \cos(2 \cdot 2x)}{2} = \frac{1 + \cos(4x)}{2}$

Substitute this back into the expression for $\sin^4 x$:

= $\frac{1 - 2\cos(2x) + \frac{1 + \cos(4x)}{2}}{4}$

Combine terms in the numerator by finding a common denominator:

= $\frac{\frac{2}{2} - \frac{4\cos(2x)}{2} + \frac{1 + \cos(4x)}{2}}{4}$

= $\frac{\frac{2 - 4\cos(2x) + 1 + \cos(4x)}{2}}{4}$

= $\frac{3 - 4\cos(2x) + \cos(4x)}{8}$

= $\frac{1}{8} (3 - 4\cos(2x) + \cos(4x))$

Now, integrate this simplified expression:

$\int \sin^4 x \, dx = \int \frac{1}{8} (3 - 4\cos(2x) + \cos(4x)) \, dx$

= $\frac{1}{8} \int (3 - 4\cos(2x) + \cos(4x)) \, dx$

Integrate term by term, using the standard integral $\int \cos(ax) \, dx = \frac{1}{a}\sin(ax) + C'$:

= $\frac{1}{8} \left( \int 3 \, dx - \int 4\cos(2x) \, dx + \int \cos(4x) \, dx \right)$

= $\frac{1}{8} \left( 3x - 4 \left(\frac{\sin(2x)}{2}\right) + \frac{\sin(4x)}{4} \right) + C$

= $\frac{1}{8} \left( 3x - 2\sin(2x) + \frac{1}{4}\sin(4x) \right) + C$

Distribute the $\frac{1}{8}$:

= $\frac{3}{8}x - \frac{2}{8}\sin(2x) + \frac{1}{32}\sin(4x) + C$

= $\frac{3}{8}x - \frac{1}{4}\sin(2x) + \frac{1}{32}\sin(4x) + C$


The final answer is:

$\boxed{\frac{3}{8}x - \frac{1}{4}\sin(2x) + \frac{1}{32}\sin(4x) + C}$

Question 11. cos4 2x

Answer:

Solution:

We need to find the integral of $\cos^4(2x)$ with respect to $x$.

The integral is given by:

$\int \cos^4(2x) \, dx$

We can rewrite $\cos^4(2x)$ using trigonometric identities:

$\cos^4(2x) = (\cos^2(2x))^2$

Using the identity $\cos^2 \theta = \frac{1 + \cos(2\theta)}{2}$, with $\theta = 2x$, we get:

$\cos^2(2x) = \frac{1 + \cos(2 \cdot 2x)}{2} = \frac{1 + \cos(4x)}{2}$

Substitute this back into the expression for $\cos^4(2x)$:

$\cos^4(2x) = \left(\frac{1 + \cos(4x)}{2}\right)^2$

= $\frac{(1 + \cos(4x))^2}{4}$

= $\frac{1 + 2\cos(4x) + \cos^2(4x)}{4}$

Now, use the identity $\cos^2 \phi = \frac{1 + \cos(2\phi)}{2}$ for $\cos^2(4x)$, where $\phi = 4x$:

$\cos^2(4x) = \frac{1 + \cos(2 \cdot 4x)}{2} = \frac{1 + \cos(8x)}{2}$

Substitute this back into the expression for $\cos^4(2x)$:

= $\frac{1 + 2\cos(4x) + \frac{1 + \cos(8x)}{2}}{4}$

Combine terms in the numerator by finding a common denominator:

= $\frac{\frac{2}{2} + \frac{4\cos(4x)}{2} + \frac{1 + \cos(8x)}{2}}{4}$

= $\frac{\frac{2 + 4\cos(4x) + 1 + \cos(8x)}{2}}{4}$

= $\frac{3 + 4\cos(4x) + \cos(8x)}{8}$

= $\frac{1}{8} (3 + 4\cos(4x) + \cos(8x))$

Now, integrate this simplified expression:

$\int \cos^4(2x) \, dx = \int \frac{1}{8} (3 + 4\cos(4x) + \cos(8x)) \, dx$

= $\frac{1}{8} \int (3 + 4\cos(4x) + \cos(8x)) \, dx$

Integrate term by term, using the standard integral $\int \cos(ax) \, dx = \frac{1}{a}\sin(ax) + C'$:

= $\frac{1}{8} \left( \int 3 \, dx + \int 4\cos(4x) \, dx + \int \cos(8x) \, dx \right)$

= $\frac{1}{8} \left( 3x + 4 \left(\frac{\sin(4x)}{4}\right) + \left(\frac{\sin(8x)}{8}\right) \right) + C$

= $\frac{1}{8} \left( 3x + \sin(4x) + \frac{1}{8}\sin(8x) \right) + C$

Distribute the $\frac{1}{8}$:

= $\frac{3}{8}x + \frac{1}{8}\sin(4x) + \frac{1}{64}\sin(8x) + C$


The final answer is:

$\boxed{\frac{3}{8}x + \frac{1}{8}\sin(4x) + \frac{1}{64}\sin(8x) + C}$

Question 12. $\frac{\sin^2 x}{1 + \cos x}$

Answer:

Solution:

We need to find the integral of $\frac{\sin^2 x}{1 + \cos x}$ with respect to $x$.

The integral is given by:

$\int \frac{\sin^2 x}{1 + \cos x} \, dx$

We use the identity $\sin^2 x = 1 - \cos^2 x$ in the numerator:

$\int \frac{1 - \cos^2 x}{1 + \cos x} \, dx$

The numerator is a difference of squares ($a^2 - b^2 = (a-b)(a+b)$), where $a=1$ and $b=\cos x$:

$\int \frac{(1 - \cos x)(1 + \cos x)}{1 + \cos x} \, dx$

Assuming $1 + \cos x \neq 0$, we can cancel out the $(1 + \cos x)$ term:

$\int (1 - \cos x) \, dx$

Now, integrate term by term:

= $\int 1 \, dx - \int \cos x \, dx$

The integral of 1 with respect to $x$ is $x$.

The integral of $\cos x$ with respect to $x$ is $\sin x$.

= $x - \sin x + C$

where $C$ is the constant of integration.


The final answer is:

$\boxed{x - \sin x + C}$

Question 13. $\frac{\cos 2x − \cos 2α}{\cos x − \cos α}$

Answer:

Solution:

We need to find the integral of $\frac{\cos 2x − \cos 2α}{\cos x − \cos α}$ with respect to $x$. Note that $\alpha$ is a constant.

The integral is given by:

$\int \frac{\cos 2x − \cos 2α}{\cos x − \cos α} \, dx$

We use the double angle identity $\cos 2\theta = 2\cos^2 \theta - 1$ in the numerator:

Numerator = $\cos 2x - \cos 2\alpha = (2\cos^2 x - 1) - (2\cos^2 \alpha - 1)$

= $2\cos^2 x - 1 - 2\cos^2 \alpha + 1$

= $2\cos^2 x - 2\cos^2 \alpha$

= $2(\cos^2 x - \cos^2 \alpha)$

Now substitute this back into the integrand:

$\frac{\cos 2x − \cos 2α}{\cos x − \cos α} = \frac{2(\cos^2 x - \cos^2 \alpha)}{\cos x - \cos α}$

We use the difference of squares formula $a^2 - b^2 = (a-b)(a+b)$ on the numerator, with $a = \cos x$ and $b = \cos \alpha$:

= $\frac{2(\cos x - \cos \alpha)(\cos x + \cos \alpha)}{\cos x - \cos \alpha}$

Assuming $\cos x - \cos \alpha \neq 0$, we can cancel the term $(\cos x - \cos \alpha)$:

= $2(\cos x + \cos \alpha)$

Now, we integrate the simplified expression:

$\int 2(\cos x + \cos \alpha) \, dx = \int (2\cos x + 2\cos \alpha) \, dx$

= $\int 2\cos x \, dx + \int 2\cos \alpha \, dx$

Since $\alpha$ is a constant, $2\cos \alpha$ is also a constant. We can take constants out of the integral:

= $2 \int \cos x \, dx + 2\cos \alpha \int 1 \, dx$

The integral of $\cos x$ is $\sin x$, and the integral of 1 is $x$.

= $2(\sin x) + 2\cos \alpha (x) + C$

= $2\sin x + 2x\cos \alpha + C$

where $C$ is the constant of integration.


The final answer is:

$\boxed{2\sin x + 2x\cos \alpha + C}$

Question 14. $\frac{\cos x − \sin x}{1 + \sin 2x}$

Answer:

Solution:

We need to find the integral of $\frac{\cos x − \sin x}{1 + \sin 2x}$ with respect to $x$.

The integral is given by:

$\int \frac{\cos x − \sin x}{1 + \sin 2x} \, dx$

We use the identity $1 = \sin^2 x + \cos^2 x$ and $\sin 2x = 2 \sin x \cos x$ in the denominator:

$1 + \sin 2x = \sin^2 x + \cos^2 x + 2 \sin x \cos x$

= $(\sin x + \cos x)^2$

Substitute this into the integrand:

$\int \frac{\cos x − \sin x}{(\sin x + \cos x)^2} \, dx$

Now, we use the substitution method.

Let $u = \sin x + \cos x$.

Differentiating $u$ with respect to $x$:

$\frac{du}{dx} = \frac{d}{dx}(\sin x + \cos x) = \cos x - \sin x$

So, $du = (\cos x - \sin x) \, dx$.

Substitute $u = \sin x + \cos x$ and $du = (\cos x - \sin x) \, dx$ into the integral:

$\int \frac{1}{u^2} \, du$

Rewrite $\frac{1}{u^2}$ as $u^{-2}$:

= $\int u^{-2} \, du$

Now, integrate with respect to $u$ using the power rule $\int u^n \, du = \frac{u^{n+1}}{n+1} + C'$:

= $\frac{u^{-2+1}}{-2+1} + C'$

= $\frac{u^{-1}}{-1} + C'$

= $-\frac{1}{u} + C'$

Substitute back $u = \sin x + \cos x$:

= $-\frac{1}{\sin x + \cos x} + C$ (where $C$ is the constant of integration)


The final answer is:

$\boxed{-\frac{1}{\sin x + \cos x} + C}$

Question 15. tan3 2x sec 2x

Answer:

Solution:

We need to find the integral of $\tan^3(2x) \sec(2x)$ with respect to $x$.

The integral is given by:

$\int \tan^3(2x) \sec(2x) \, dx$

We can rewrite the integrand as follows:

$\int \tan^2(2x) \cdot (\tan(2x) \sec(2x)) \, dx$

Using the identity $\tan^2 \theta = \sec^2 \theta - 1$, with $\theta = 2x$, we get:

$\tan^2(2x) = \sec^2(2x) - 1$

Substitute this into the integral:

$\int (\sec^2(2x) - 1) (\tan(2x) \sec(2x)) \, dx$

Now, we use the substitution method.

Let $u = \sec(2x)$.

Differentiating $u$ with respect to $x$ using the chain rule:

$\frac{du}{dx} = \frac{d}{dx}(\sec(2x)) = \sec(2x) \tan(2x) \cdot \frac{d}{dx}(2x)$

= $\sec(2x) \tan(2x) \cdot 2$

= $2 \sec(2x) \tan(2x)$

Rearranging to find $\sec(2x) \tan(2x) \, dx$:

$du = 2 \sec(2x) \tan(2x) \, dx$

$\sec(2x) \tan(2x) \, dx = \frac{1}{2} \, du$

Substitute $u = \sec(2x)$ and $\sec(2x) \tan(2x) \, dx = \frac{1}{2} \, du$ into the integral:

$\int (u^2 - 1) \left(\frac{1}{2}\right) \, du$

= $\frac{1}{2} \int (u^2 - 1) \, du$

Now, integrate with respect to $u$:

= $\frac{1}{2} \left( \int u^2 \, du - \int 1 \, du \right)$

= $\frac{1}{2} \left( \frac{u^3}{3} - u \right) + C'$

Distribute the $\frac{1}{2}$:

= $\frac{u^3}{6} - \frac{u}{2} + C'$

Substitute back $u = \sec(2x)$:

= $\frac{\sec^3(2x)}{6} - \frac{\sec(2x)}{2} + C$ (where $C$ is the constant of integration)


The final answer is:

$\boxed{\frac{\sec^3(2x)}{6} - \frac{\sec(2x)}{2} + C}$

Question 16. tan4 x

Answer:

Solution:

We need to find the integral of $\tan^4 x$ with respect to $x$.

The integral is given by:

$\int \tan^4 x \, dx$

We can rewrite $\tan^4 x$ using trigonometric identities:

$\tan^4 x = \tan^2 x \cdot \tan^2 x$

Using the identity $\tan^2 x = \sec^2 x - 1$:

$\tan^4 x = \tan^2 x (\sec^2 x - 1)$

Distribute $\tan^2 x$:

= $\tan^2 x \sec^2 x - \tan^2 x$

Use the identity $\tan^2 x = \sec^2 x - 1$ again for the second term:

= $\tan^2 x \sec^2 x - (\sec^2 x - 1)$

= $\tan^2 x \sec^2 x - \sec^2 x + 1$

Now, the integral becomes:

$\int (\tan^2 x \sec^2 x - \sec^2 x + 1) \, dx$

We can integrate term by term:

= $\int \tan^2 x \sec^2 x \, dx - \int \sec^2 x \, dx + \int 1 \, dx$

For the first integral, $\int \tan^2 x \sec^2 x \, dx$, we use substitution.

Let $u = \tan x$.

Differentiating with respect to $x$:

$\frac{du}{dx} = \frac{d}{dx}(\tan x) = \sec^2 x$

So, $du = \sec^2 x \, dx$.

Substitute $u = \tan x$ and $du = \sec^2 x \, dx$ into the first integral:

$\int u^2 \, du$

Integrate with respect to $u$ using the power rule $\int u^n \, du = \frac{u^{n+1}}{n+1} + C_1$:

= $\frac{u^3}{3} + C_1$

Substitute back $u = \tan x$:

= $\frac{\tan^3 x}{3} + C_1$

The second integral is the standard integral of $\sec^2 x$:

$\int \sec^2 x \, dx = \tan x + C_2$

The third integral is the standard integral of 1:

$\int 1 \, dx = x + C_3$

Combine the results of the three integrals and add the constant of integration $C$ ($C = C_1 + C_2 + C_3$):

= $\frac{\tan^3 x}{3} - (\tan x) + x + C$

= $\frac{1}{3}\tan^3 x - \tan x + x + C$


The final answer is:

$\boxed{\frac{1}{3}\tan^3 x - \tan x + x + C}$

Question 17. $\frac{\sin^3 x + \cos^3 x}{\sin^2 x \;\cos^2 x}$

Answer:

Solution:

We need to find the integral of $\frac{\sin^3 x + \cos^3 x}{\sin^2 x \cos^2 x}$ with respect to $x$.

The integral is given by:

$\int \frac{\sin^3 x + \cos^3 x}{\sin^2 x \cos^2 x} \, dx$

We can split the fraction into two terms by dividing each term in the numerator by the denominator:

$\frac{\sin^3 x + \cos^3 x}{\sin^2 x \cos^2 x} = \frac{\sin^3 x}{\sin^2 x \cos^2 x} + \frac{\cos^3 x}{\sin^2 x \cos^2 x}$

Simplify each term by cancelling common factors:

= $\frac{\sin x}{\cos^2 x} + \frac{\cos x}{\sin^2 x}$

We can rewrite these terms using reciprocal and quotient identities:

$\frac{\sin x}{\cos^2 x} = \frac{\sin x}{\cos x} \cdot \frac{1}{\cos x} = \tan x \sec x$

$\frac{\cos x}{\sin^2 x} = \frac{\cos x}{\sin x} \cdot \frac{1}{\sin x} = \cot x \text{ cosec } x$

So, the integrand becomes:

= $\tan x \sec x + \cot x \text{ cosec } x$

Now, integrate this simplified expression:

$\int (\tan x \sec x + \cot x \text{ cosec } x) \, dx$

We can integrate term by term:

= $\int \tan x \sec x \, dx + \int \cot x \text{ cosec } x \, dx$

These are standard integrals:

$\int \tan x \sec x \, dx = \sec x + C_1$

$\int \cot x \text{ cosec } x \, dx = -\text{ cosec } x + C_2$

Combining the results and adding the constant of integration $C$ ($C = C_1 + C_2$):

= $\sec x - \text{ cosec } x + C$


The final answer is:

$\boxed{\sec x - \text{ cosec } x + C}$

Question 18. $\frac{\cos 2x + 2\sin^2 x}{\cos^2 x}$

Answer:

Solution:

We need to find the integral of $\frac{\cos 2x + 2\sin^2 x}{\cos^2 x}$ with respect to $x$.

The integral is given by:

$\int \frac{\cos 2x + 2\sin^2 x}{\cos^2 x} \, dx$

We can simplify the numerator using the double angle identity $\cos 2x = 1 - 2\sin^2 x$:

Numerator = $\cos 2x + 2\sin^2 x = (1 - 2\sin^2 x) + 2\sin^2 x$

= $1 - 2\sin^2 x + 2\sin^2 x$

= $1$

Substitute this back into the integrand:

$\frac{\cos 2x + 2\sin^2 x}{\cos^2 x} = \frac{1}{\cos^2 x}$

Using the reciprocal identity $\sec x = \frac{1}{\cos x}$, we get:

= $\sec^2 x$

Now, we integrate this simplified expression:

$\int \sec^2 x \, dx$

This is a standard integral:

= $\tan x + C$

where $C$ is the constant of integration.


The final answer is:

$\boxed{\tan x + C}$

Question 19. $\frac{1}{\sin x \;\cos^3 x}$

Answer:

Solution:

We need to find the integral of $\frac{1}{\sin x \cos^3 x}$ with respect to $x$.

The integral is given by:

$\int \frac{1}{\sin x \cos^3 x} \, dx$

We can rewrite the integrand by multiplying the numerator and the denominator by $\sec^4 x$:

$\frac{1}{\sin x \cos^3 x} = \frac{1}{\sin x \cos^3 x} \cdot \frac{\sec^4 x}{\sec^4 x}$

= $\frac{\sec^4 x}{\sin x \cos^3 x \cdot \frac{1}{\cos^4 x}}$

= $\frac{\sec^4 x}{\frac{\sin x}{\cos x}}$

= $\frac{\sec^4 x}{\tan x}$

We can rewrite $\sec^4 x$ as $\sec^2 x \cdot \sec^2 x$. Using the identity $\sec^2 x = 1 + \tan^2 x$, we get:

= $\frac{\sec^2 x \cdot (1 + \tan^2 x)}{\tan x}$

= $\frac{\sec^2 x + \tan^2 x \sec^2 x}{\tan x}$

Split the fraction into two terms:

= $\frac{\sec^2 x}{\tan x} + \frac{\tan^2 x \sec^2 x}{\tan x}$

= $\frac{\sec^2 x}{\tan x} + \tan x \sec^2 x$

Now, the integral becomes:

$\int \left( \frac{\sec^2 x}{\tan x} + \tan x \sec^2 x \right) \, dx$

We can integrate each term separately.

For both integrals, we use the substitution method. Let $u = \tan x$.

Differentiating $u$ with respect to $x$:

$\frac{du}{dx} = \frac{d}{dx}(\tan x) = \sec^2 x$

So, $du = \sec^2 x \, dx$.

Substitute $u = \tan x$ and $du = \sec^2 x \, dx$ into the integral:

= $\int \left( \frac{1}{u} + u \right) \, du$

Now, integrate with respect to $u$:

= $\int \frac{1}{u} \, du + \int u \, du$

= $\log_e |u| + \frac{u^{1+1}}{1+1} + C'$

= $\log_e |u| + \frac{u^2}{2} + C'$

Substitute back $u = \tan x$:

= $\log_e |\tan x| + \frac{\tan^2 x}{2} + C$ (where $C$ is the constant of integration)


The final answer is:

$\boxed{\log_e |\tan x| + \frac{\tan^2 x}{2} + C}$

Question 20. $\frac{\cos 2x}{(\cos x + \sin x)^2}$

Answer:

To evaluate the integral $\int \frac{\cos 2x}{(\cos x + \sin x)^2} dx$, we can simplify the integrand first.


We use the trigonometric identity for $\cos 2x$: $\cos 2x = \cos^2 x - \sin^2 x$.

The denominator is $(\cos x + \sin x)^2$.


So, the integrand becomes:

$\frac{\cos^2 x - \sin^2 x}{(\cos x + \sin x)^2}$


The numerator $\cos^2 x - \sin^2 x$ is a difference of squares, which can be factored as $(\cos x - \sin x)(\cos x + \sin x)$.

The integrand is now:

$\frac{(\cos x - \sin x)(\cos x + \sin x)}{(\cos x + \sin x)^2}$


Assuming $\cos x + \sin x \neq 0$, we can cancel one term of $(\cos x + \sin x)$ from the numerator and the denominator:

$= \frac{\cos x - \sin x}{\cos x + \sin x}$


Now we need to evaluate the integral $\int \frac{\cos x - \sin x}{\cos x + \sin x} dx$.

We can use the method of substitution.


Let $u = \cos x + \sin x$.


Differentiate $u$ with respect to $x$:

$\frac{du}{dx} = \frac{d}{dx}(\cos x + \sin x)$

$\frac{du}{dx} = -\sin x + \cos x$

$\frac{du}{dx} = \cos x - \sin x$


From this, we get $du = (\cos x - \sin x) dx$.


Now substitute $u = \cos x + \sin x$ and $du = (\cos x - \sin x) dx$ into the integral $\int \frac{\cos x - \sin x}{\cos x + \sin x} dx$:

$\int \frac{du}{u}$


The integral of $\frac{1}{u}$ with respect to $u$ is $\log |u|$.

$\int \frac{du}{u} = \log |u| + C$

where $C$ is the constant of integration.


Finally, substitute back $u = \cos x + \sin x$ to get the result in terms of $x$:

The integral is equal to:

$\log |\cos x + \sin x| + C$


Thus, $\int \frac{\cos 2x}{(\cos x + \sin x)^2} dx = \mathbf{\log |\cos x + \sin x| + C}$.

Question 21. sin–1 (cos x)

Answer:

Solution:

We need to find the integral of $\sin^{-1}(\cos x)$ with respect to $x$.

The integral is given by:

$\int \sin^{-1}(\cos x) \, dx$

We use the trigonometric identity $\cos x = \sin\left(\frac{\pi}{2} - x\right)$.

Substituting this into the integrand:

$\sin^{-1}(\cos x) = \sin^{-1}\left(\sin\left(\frac{\pi}{2} - x\right)\right)$

For $\sin^{-1}(\sin \theta) = \theta$, provided $\theta$ is in the range $[-\frac{\pi}{2}, \frac{\pi}{2}]$. The range of $\frac{\pi}{2} - x$ depends on the domain of $x$. However, for the purpose of finding a general integral, we can use this simplification, assuming $x$ is such that $\frac{\pi}{2} - x$ is within this range. For instance, if $x \in [0, \pi]$, then $\frac{\pi}{2} - x \in [-\frac{\pi}{2}, \frac{\pi}{2}]$.

So, $\sin^{-1}(\cos x) = \frac{\pi}{2} - x$.

Now, we integrate this simplified expression:

$\int \left(\frac{\pi}{2} - x\right) \, dx$

We can integrate term by term:

= $\int \frac{\pi}{2} \, dx - \int x \, dx$

The integral of a constant $\frac{\pi}{2}$ is $\frac{\pi}{2}x$.

The integral of $x$ is $\frac{x^{1+1}}{1+1} = \frac{x^2}{2}$.

= $\frac{\pi}{2}x - \frac{x^2}{2} + C$

where $C$ is the constant of integration.

Note: This solution assumes the principal value of $\sin^{-1}(\cos x)$, which is $\frac{\pi}{2} - x$ for $x \in [0, \pi]$. If the domain of $x$ is different, the expression for $\sin^{-1}(\cos x)$ might be different (e.g., piecewise). However, for a general indefinite integral, $\frac{\pi}{2} - x$ is typically used.


The final answer is:

$\boxed{\frac{\pi}{2}x - \frac{x^2}{2} + C}$

Question 22. $\frac{1}{\cos (x − a) \cos (x − b)}$

Answer:

Solution:

We need to find the integral of $\frac{1}{\cos (x − a) \cos (x − b)}$ with respect to $x$.

The integral is given by:

$\int \frac{1}{\cos (x − a) \cos (x − b)} \, dx$

We can multiply the numerator and the denominator by a constant factor, say $\sin(b-a)$, assuming $a \neq b$. This will allow us to use trigonometric identities to simplify the expression.

$\int \frac{1}{\sin(b-a)} \frac{\sin(b-a)}{\cos (x − a) \cos (x − b)} \, dx$

We can rewrite the term $\sin(b-a)$ in the numerator using the difference of the arguments in the denominator:

$b - a = (x - a) - (x - b)$

So, $\sin(b-a) = \sin((x-a) - (x-b))$.

Using the identity $\sin(A-B) = \sin A \cos B - \cos A \sin B$, where $A = x-a$ and $B = x-b$:

$\sin((x-a) - (x-b)) = \sin(x-a)\cos(x-b) - \cos(x-a)\sin(x-b)$

Substitute this back into the integral:

= $\frac{1}{\sin(b-a)} \int \frac{\sin(x-a)\cos(x-b) - \cos(x-a)\sin(x-b)}{\cos (x − a) \cos (x − b)} \, dx$

Now, split the fraction into two terms:

= $\frac{1}{\sin(b-a)} \int \left[ \frac{\sin(x-a)\cos(x-b)}{\cos (x − a) \cos (x − b)} - \frac{\cos(x-a)\sin(x-b)}{\cos (x − a) \cos (x − b)} \right] \, dx$

Simplify each term:

= $\frac{1}{\sin(b-a)} \int \left[ \frac{\sin(x-a)}{\cos (x − a)} - \frac{\sin(x-b)}{\cos (x − b)} \right] \, dx$

Using the identity $\tan \theta = \frac{\sin \theta}{\cos \theta}$:

= $\frac{1}{\sin(b-a)} \int [\tan(x-a) - \tan(x-b)] \, dx$

Now, integrate term by term:

= $\frac{1}{\sin(b-a)} \left[ \int \tan(x-a) \, dx - \int \tan(x-b) \, dx \right]$

Recall the standard integral $\int \tan \theta \, d\theta = -\log_e |\cos \theta| + C'$.

$\int \tan(x-a) \, dx = -\log_e |\cos(x-a)|$ (using substitution $u = x-a$, $du = dx$)

$\int \tan(x-b) \, dx = -\log_e |\cos(x-b)|$ (using substitution $v = x-b$, $dv = dx$)

Substitute these results back into the expression:

= $\frac{1}{\sin(b-a)} \left[ (-\log_e |\cos(x-a)|) - (-\log_e |\cos(x-b)|) \right] + C$

= $\frac{1}{\sin(b-a)} \left[ -\log_e |\cos(x-a)| + \log_e |\cos(x-b)| \right] + C$

Using the logarithm property $\log_e P - \log_e Q = \log_e \frac{P}{Q}$:

= $\frac{1}{\sin(b-a)} \log_e \left| \frac{\cos(x-b)}{\cos(x-a)} \right| + C$

where $C$ is the constant of integration. This result is valid for $a \neq b$. If $a=b$, the integral is $\int \sec^2(x-a) dx = \tan(x-a) + C$.


The final answer (for $a \neq b$) is:

$\boxed{\frac{1}{\sin(b-a)} \log_e \left| \frac{\cos(x-b)}{\cos(x-a)} \right| + C}$

Choose the correct answer in Exercises 23 and 24.

Question 23. $\int \frac{\sin^2 x − \cos^2 x}{\sin^2 x \;\cos^2 x} \;dx$ is equal to

(A) tan x + cot x + C

(B) tan x + cosec x + C

(C) – tan x + cot x + C

(D) tan x + sec x + C

Answer:

Solution:

We need to find the integral of $\frac{\sin^2 x − \cos^2 x}{\sin^2 x \;\cos^2 x}$ with respect to $x$.

The integral is given by:

$\int \frac{\sin^2 x − \cos^2 x}{\sin^2 x \;\cos^2 x} \, dx$

We can split the fraction by dividing each term in the numerator by the denominator:

$\frac{\sin^2 x − \cos^2 x}{\sin^2 x \;\cos^2 x} = \frac{\sin^2 x}{\sin^2 x \cos^2 x} - \frac{\cos^2 x}{\sin^2 x \cos^2 x}$

Simplify each term by cancelling common factors:

= $\frac{1}{\cos^2 x} - \frac{1}{\sin^2 x}$

Using reciprocal identities $\sec x = \frac{1}{\cos x}$ and $\text{ cosec } x = \frac{1}{\sin x}$, we get:

= $\sec^2 x - \text{ cosec}^2 x$

Now, integrate this simplified expression:

$\int (\sec^2 x - \text{ cosec}^2 x) \, dx$

We can integrate term by term:

= $\int \sec^2 x \, dx - \int \text{ cosec}^2 x \, dx$

Using the standard integrals $\int \sec^2 x \, dx = \tan x + C_1$ and $\int \text{ cosec}^2 x \, dx = -\cot x + C_2$:

= $\tan x - (-\cot x) + C$

= $\tan x + \cot x + C$

where $C$ is the constant of integration ($C = C_1 + C_2$).


Comparing the result with the given options:

(A) tan x + cot x + C

(B) tan x + cosec x + C

(C) – tan x + cot x + C

(D) tan x + sec x + C

The calculated integral matches option (A).


The final answer is:

$\boxed{\text{tan x + cot x + C}}$

Question 24. $\int \frac{e^x (1 + x)}{\cos^2 (e^x x)}\; dx$ equals

(A) – cot (exx) + C

(B) tan (xex) + C

(C) tan (ex) + C

(D) cot (ex) + C

Answer:

Solution:

We need to find the integral of $\frac{e^x (1 + x)}{\cos^2 (e^x x)}$ with respect to $x$.

The integral is given by:

$\int \frac{e^x (1 + x)}{\cos^2 (e^x x)} \, dx$

We use the substitution method.

Let $u = e^x x$.

Differentiating $u$ with respect to $x$ using the product rule:

$\frac{du}{dx} = \frac{d}{dx}(e^x x) = \frac{d}{dx}(e^x) \cdot x + e^x \cdot \frac{d}{dx}(x)$

= $e^x \cdot x + e^x \cdot 1$

= $e^x(x + 1)$

So, $du = e^x(1+x) \, dx$.

Substitute $u = e^x x$ and $e^x(1+x) \, dx = du$ into the integral:

$\int \frac{1}{\cos^2 u} \, du$

Using the reciprocal identity $\frac{1}{\cos^2 u} = \sec^2 u$, the integral becomes:

= $\int \sec^2 u \, du$

The integral of $\sec^2 u$ is $\tan u$.

= $\tan u + C'$

Substitute back $u = e^x x$:

= $\tan(e^x x) + C$ (where $C$ is the constant of integration)


Comparing the result with the given options:

(A) – cot (exx) + C

(B) tan (xex) + C

(C) tan (ex) + C

(D) cot (ex) + C

Our result $\tan(e^x x) + C$ matches option (B) (since $e^x x$ is the same as $xe^x$).


The final answer is:

$\boxed{\text{tan } (xe^x) + C}$



Example 8 to 10 (Before Exercise 7.4)

Example 8: Find the following integrals:

(i) $\int \frac{dx}{x^2 − 16}$

(ii) $\int \frac{dx}{\sqrt{2x − x^2}}$

Answer:

Solution:

Let's solve each integral separately.


(i) $\int \frac{dx}{x^2 − 16}$

We can write the denominator as $x^2 - 4^2$.

This integral is in the standard form $\int \frac{dx}{x^2 - a^2}$, where $a=4$.

The formula for this integral is $\int \frac{dx}{x^2 - a^2} = \frac{1}{2a} \log_e \left| \frac{x-a}{x+a} \right| + C$.

Substituting $a=4$ into the formula, we get:

$\int \frac{dx}{x^2 − 16} = \frac{1}{2(4)} \log_e \left| \frac{x-4}{x+4} \right| + C$

= $\frac{1}{8} \log_e \left| \frac{x-4}{x+4} \right| + C$

where $C$ is the constant of integration.


(ii) $\int \frac{dx}{\sqrt{2x − x^2}}$

We need to complete the square in the denominator $2x - x^2$.

$2x - x^2 = -(x^2 - 2x)$

To complete the square for $x^2 - 2x$, we add and subtract $\left(\frac{-2}{2}\right)^2 = (-1)^2 = 1$.

$x^2 - 2x = (x^2 - 2x + 1) - 1 = (x-1)^2 - 1$

So, $2x - x^2 = -((x-1)^2 - 1) = 1 - (x-1)^2$.

The integral becomes:

$\int \frac{dx}{\sqrt{1 - (x-1)^2}}$

This integral is in the standard form $\int \frac{du}{\sqrt{a^2 - u^2}}$, where $a=1$ and $u=x-1$.

Let $u = x-1$. Then $du = dx$.

The integral is $\int \frac{du}{\sqrt{1^2 - u^2}}$.

The formula for this integral is $\int \frac{du}{\sqrt{a^2 - u^2}} = \sin^{-1}\left(\frac{u}{a}\right) + C$.

Substituting $a=1$ and $u=x-1$, we get:

$\int \frac{dx}{\sqrt{2x − x^2}} = \sin^{-1}\left(\frac{x-1}{1}\right) + C$

= $\sin^{-1}(x-1) + C$

where $C$ is the constant of integration.

Example 9: Find the following integrals :

(i) $\int \frac{dx}{x^2 − 6x + 13}$

(ii) $\int \frac{dx}{3x^2 + 13x − 10}$

(iii) $\int \frac{dx}{\sqrt{5x^2 − 2x}}$

Answer:

Solution:

Let's solve each integral separately.


(i) $\int \frac{dx}{x^2 − 6x + 13}$

We complete the square in the denominator $x^2 - 6x + 13$.

$x^2 - 6x + 13 = (x^2 - 6x + 9) + 13 - 9$

= $(x - 3)^2 + 4$

= $(x - 3)^2 + 2^2$

The integral becomes:

$\int \frac{dx}{(x - 3)^2 + 2^2}$

This integral is in the standard form $\int \frac{du}{u^2 + a^2}$, where $u = x-3$ and $a=2$.

Let $u = x-3$. Then $du = dx$.

The integral is $\int \frac{du}{u^2 + 2^2}$.

The formula for this integral is $\int \frac{du}{u^2 + a^2} = \frac{1}{a} \tan^{-1}\left(\frac{u}{a}\right) + C$.

Substituting $u = x-3$ and $a=2$, we get:

$\int \frac{dx}{x^2 − 6x + 13} = \frac{1}{2} \tan^{-1}\left(\frac{x-3}{2}\right) + C$

where $C$ is the constant of integration.


(ii) $\int \frac{dx}{3x^2 + 13x − 10}$

First, factor out 3 from the quadratic in the denominator:

$3x^2 + 13x - 10 = 3\left(x^2 + \frac{13}{3}x - \frac{10}{3}\right)$

The integral becomes:

$\int \frac{dx}{3\left(x^2 + \frac{13}{3}x - \frac{10}{3}\right)} = \frac{1}{3} \int \frac{dx}{x^2 + \frac{13}{3}x - \frac{10}{3}}$

Now, complete the square for $x^2 + \frac{13}{3}x - \frac{10}{3}$.

Add and subtract $\left(\frac{13/3}{2}\right)^2 = \left(\frac{13}{6}\right)^2 = \frac{169}{36}$.

$x^2 + \frac{13}{3}x - \frac{10}{3} = \left(x^2 + \frac{13}{3}x + \frac{169}{36}\right) - \frac{10}{3} - \frac{169}{36}$

= $\left(x + \frac{13}{6}\right)^2 - \left(\frac{10 \cdot 12}{3 \cdot 12} + \frac{169}{36}\right)$

= $\left(x + \frac{13}{6}\right)^2 - \left(\frac{120}{36} + \frac{169}{36}\right)$

= $\left(x + \frac{13}{6}\right)^2 - \frac{289}{36}$

= $\left(x + \frac{13}{6}\right)^2 - \left(\frac{17}{6}\right)^2$

The integral becomes:

$\frac{1}{3} \int \frac{dx}{\left(x + \frac{13}{6}\right)^2 - \left(\frac{17}{6}\right)^2}$

This is in the standard form $\int \frac{du}{u^2 - a^2}$, where $u = x + \frac{13}{6}$ and $a = \frac{17}{6}$.

Let $u = x + \frac{13}{6}$. Then $du = dx$.

The integral is $\frac{1}{3} \int \frac{du}{u^2 - \left(\frac{17}{6}\right)^2}$.

The formula for this integral is $\int \frac{du}{u^2 - a^2} = \frac{1}{2a} \log_e \left| \frac{u-a}{u+a} \right| + C'$.

Substituting $u = x + \frac{13}{6}$ and $a = \frac{17}{6}$, we get:

= $\frac{1}{3} \cdot \frac{1}{2\left(\frac{17}{6}\right)} \log_e \left| \frac{x + \frac{13}{6} - \frac{17}{6}}{x + \frac{13}{6} + \frac{17}{6}} \right| + C$

= $\frac{1}{3} \cdot \frac{1}{\frac{17}{3}} \log_e \left| \frac{x - \frac{4}{6}}{x + \frac{30}{6}} \right| + C$

= $\frac{1}{3} \cdot \frac{3}{17} \log_e \left| \frac{x - \frac{2}{3}}{x + 5} \right| + C$

= $\frac{1}{17} \log_e \left| \frac{\frac{3x-2}{3}}{x + 5} \right| + C$

= $\frac{1}{17} \log_e \left| \frac{3x-2}{3(x + 5)} \right| + C$

Using logarithm properties, $\log_e \left| \frac{3x-2}{3(x + 5)} \right| = \log_e |3x-2| - \log_e |3(x+5)| = \log_e |3x-2| - (\log_e |3| + \log_e |x+5|)$.

So, $\frac{1}{17} (\log_e |3x-2| - \log_e |x+5|) - \frac{1}{17}\log_e |3| + C$. The term $-\frac{1}{17}\log_e |3|$ can be absorbed into the constant $C$.

= $\frac{1}{17} \log_e \left| \frac{3x-2}{x+5} \right| + C$


(iii) $\int \frac{dx}{\sqrt{5x^2 − 2x}}$

First, factor out 5 from the term under the square root:

$5x^2 - 2x = 5\left(x^2 - \frac{2}{5}x\right)$

The integral becomes:

$\int \frac{dx}{\sqrt{5\left(x^2 - \frac{2}{5}x\right)}} = \int \frac{dx}{\sqrt{5} \sqrt{x^2 - \frac{2}{5}x}}$

= $\frac{1}{\sqrt{5}} \int \frac{dx}{\sqrt{x^2 - \frac{2}{5}x}}$

Complete the square for $x^2 - \frac{2}{5}x$.

Add and subtract $\left(\frac{-2/5}{2}\right)^2 = \left(-\frac{1}{5}\right)^2 = \frac{1}{25}$.

$x^2 - \frac{2}{5}x = \left(x^2 - \frac{2}{5}x + \frac{1}{25}\right) - \frac{1}{25} = \left(x - \frac{1}{5}\right)^2 - \left(\frac{1}{5}\right)^2$

The integral becomes:

= $\frac{1}{\sqrt{5}} \int \frac{dx}{\sqrt{\left(x - \frac{1}{5}\right)^2 - \left(\frac{1}{5}\right)^2}}$

This is in the standard form $\int \frac{du}{\sqrt{u^2 - a^2}}$, where $u = x - \frac{1}{5}$ and $a = \frac{1}{5}$.

Let $u = x - \frac{1}{5}$. Then $du = dx$.

The integral is $\frac{1}{\sqrt{5}} \int \frac{du}{\sqrt{u^2 - \left(\frac{1}{5}\right)^2}}$.

The formula for this integral is $\int \frac{du}{\sqrt{u^2 - a^2}} = \log_e \left| u + \sqrt{u^2 - a^2} \right| + C''$.

Substituting $u = x - \frac{1}{5}$ and $a = \frac{1}{5}$, we get:

= $\frac{1}{\sqrt{5}} \log_e \left| \left(x - \frac{1}{5}\right) + \sqrt{\left(x - \frac{1}{5}\right)^2 - \left(\frac{1}{5}\right)^2} \right| + C$

We know that $\left(x - \frac{1}{5}\right)^2 - \left(\frac{1}{5}\right)^2 = x^2 - \frac{2}{5}x$. So, $\sqrt{\left(x - \frac{1}{5}\right)^2 - \left(\frac{1}{5}\right)^2} = \sqrt{x^2 - \frac{2}{5}x}$.

Also, $\sqrt{x^2 - \frac{2}{5}x} = \sqrt{\frac{5x^2 - 2x}{5}} = \frac{\sqrt{5x^2 - 2x}}{\sqrt{5}}$.

So the result is:

= $\frac{1}{\sqrt{5}} \log_e \left| x - \frac{1}{5} + \sqrt{x^2 - \frac{2}{5}x} \right| + C$

This can also be written as:

= $\frac{1}{\sqrt{5}} \log_e \left| \frac{5x - 1}{5} + \frac{\sqrt{5x^2 - 2x}}{\sqrt{5}} \right| + C$

= $\frac{1}{\sqrt{5}} \log_e \left| \frac{\sqrt{5}(5x - 1) + 5\sqrt{5x^2 - 2x}}{5\sqrt{5}} \right| + C$

= $\frac{1}{\sqrt{5}} \left( \log_e \left| \sqrt{5}(5x - 1) + 5\sqrt{5x^2 - 2x} \right| - \log_e |5\sqrt{5}| \right) + C$

The term $-\frac{1}{\sqrt{5}}\log_e |5\sqrt{5}|$ can be absorbed into the constant of integration.

Let's simplify the term inside the absolute value:

$\sqrt{5}(5x - 1) + 5\sqrt{5x^2 - 2x} = \sqrt{5}(5x - 1) + 5\sqrt{\frac{5x^2 - 2x}{5} \cdot 5} = \sqrt{5}(5x - 1) + 5\sqrt{5} \sqrt{x^2 - \frac{2}{5}x}$

= $\sqrt{5}(5x - 1) + 5\sqrt{5} \sqrt{\left(x - \frac{1}{5}\right)^2 - \left(\frac{1}{5}\right)^2}$

= $\sqrt{5} \left[ (5x - 1) + 5 \sqrt{\left(x - \frac{1}{5}\right)^2 - \left(\frac{1}{5}\right)^2} \right]$

= $\sqrt{5} \left[ 5x - 1 + 5 \sqrt{x^2 - \frac{2}{5}x} \right]$

= $\sqrt{5} \left[ 5x - 1 + \sqrt{25x^2 - 10x} \right]$

So the integral is:

= $\frac{1}{\sqrt{5}} \log_e \left| \sqrt{5} (5x - 1 + \sqrt{25x^2 - 10x}) \right| + C$

= $\frac{1}{\sqrt{5}} \left( \log_e |\sqrt{5}| + \log_e |5x - 1 + \sqrt{25x^2 - 10x}| \right) + C$

The term $\frac{1}{\sqrt{5}}\log_e |\sqrt{5}|$ can be absorbed into the constant.

Alternatively, we can use the formula $\int \frac{dx}{\sqrt{ax^2+bx+c}} = \frac{1}{\sqrt{a}} \log_e \left| x + \frac{b}{2a} + \sqrt{x^2 + \frac{b}{a}x + \frac{c}{a}} \right| + C$ for $a > 0$.

Here, $a=5$, $b=-2$, $c=0$.

= $\frac{1}{\sqrt{5}} \log_e \left| x + \frac{-2}{2(5)} + \sqrt{x^2 + \frac{-2}{5}x + \frac{0}{5}} \right| + C$

= $\frac{1}{\sqrt{5}} \log_e \left| x - \frac{1}{5} + \sqrt{x^2 - \frac{2}{5}x} \right| + C$

This matches the previous result.

Example 10: Find the following integrals:

(i) $\int \frac{x + 2}{2x^2 + 6x + 5} \;dx$

(ii) $\int \frac{x + 3}{\sqrt{5 − 4x − x^2}} \;dx$

Answer:

Solution:

We solve each integral separately.


(i) $\int \frac{x + 2}{2x^2 + 6x + 5} \;dx$

Let the integral be $I = \int \frac{x + 2}{2x^2 + 6x + 5} \, dx$.

We use the form $\int \frac{px+q}{ax^2+bx+c} \, dx$. The derivative of the denominator is $\frac{d}{dx}(2x^2 + 6x + 5) = 4x + 6$.

We want to write the numerator $x+2$ in the form $A(4x+6) + B$.

$x+2 = 4Ax + 6A + B$

Comparing coefficients of $x$: $4A = 1 \implies A = \frac{1}{4}$.

Comparing constant terms: $6A + B = 2$. Substitute $A = \frac{1}{4}$: $6\left(\frac{1}{4}\right) + B = 2 \implies \frac{3}{2} + B = 2 \implies B = 2 - \frac{3}{2} = \frac{4-3}{2} = \frac{1}{2}$.

So, $x+2 = \frac{1}{4}(4x+6) + \frac{1}{2}$.

Substitute this into the integral:

$I = \int \frac{\frac{1}{4}(4x+6) + \frac{1}{2}}{2x^2 + 6x + 5} \, dx$

= $\int \left( \frac{\frac{1}{4}(4x+6)}{2x^2 + 6x + 5} + \frac{\frac{1}{2}}{2x^2 + 6x + 5} \right) \, dx$

= $\frac{1}{4} \int \frac{4x+6}{2x^2 + 6x + 5} \, dx + \frac{1}{2} \int \frac{dx}{2x^2 + 6x + 5}$

Let the first integral be $I_1$ and the second integral be $I_2$.

$I_1 = \int \frac{4x+6}{2x^2 + 6x + 5} \, dx$. Let $u = 2x^2 + 6x + 5$. Then $du = (4x+6) \, dx$.

$I_1 = \int \frac{du}{u} = \log_e |u| + C_1 = \log_e |2x^2 + 6x + 5| + C_1$.

For $I_2 = \int \frac{dx}{2x^2 + 6x + 5}$, complete the square in the denominator.

$2x^2 + 6x + 5 = 2\left(x^2 + 3x + \frac{5}{2}\right)$

$x^2 + 3x + \frac{5}{2} = \left(x^2 + 3x + \left(\frac{3}{2}\right)^2\right) + \frac{5}{2} - \left(\frac{3}{2}\right)^2$

= $\left(x + \frac{3}{2}\right)^2 + \frac{5}{2} - \frac{9}{4} = \left(x + \frac{3}{2}\right)^2 + \frac{10-9}{4} = \left(x + \frac{3}{2}\right)^2 + \frac{1}{4}$

= $\left(x + \frac{3}{2}\right)^2 + \left(\frac{1}{2}\right)^2$

So, $2x^2 + 6x + 5 = 2\left(\left(x + \frac{3}{2}\right)^2 + \left(\frac{1}{2}\right)^2\right)$.

$I_2 = \int \frac{dx}{2\left(\left(x + \frac{3}{2}\right)^2 + \left(\frac{1}{2}\right)^2\right)} = \frac{1}{2} \int \frac{dx}{\left(x + \frac{3}{2}\right)^2 + \left(\frac{1}{2}\right)^2}$

This is in the standard form $\int \frac{du}{u^2 + a^2}$, with $u = x + \frac{3}{2}$ ($du=dx$) and $a = \frac{1}{2}$.

$I_2 = \frac{1}{2} \cdot \frac{1}{a} \tan^{-1}\left(\frac{u}{a}\right) + C_2 = \frac{1}{2} \cdot \frac{1}{\frac{1}{2}} \tan^{-1}\left(\frac{x + \frac{3}{2}}{\frac{1}{2}}\right) + C_2$

= $\frac{1}{2} \cdot 2 \tan^{-1}\left(\frac{\frac{2x+3}{2}}{\frac{1}{2}}\right) + C_2 = \tan^{-1}(2x+3) + C_2$.

Combining $I_1$ and $I_2$:

$I = \frac{1}{4} I_1 + \frac{1}{2} I_2$

= $\frac{1}{4} \log_e |2x^2 + 6x + 5| + \frac{1}{2} \tan^{-1}(2x+3) + C$ (where $C = \frac{1}{4}C_1 + \frac{1}{2}C_2$)

The quadratic $2x^2 + 6x + 5$ has discriminant $\Delta = b^2 - 4ac = 6^2 - 4(2)(5) = 36 - 40 = -4 < 0$. Since the leading coefficient is positive, the quadratic is always positive, so we can remove the absolute value.

$I = \frac{1}{4} \log_e (2x^2 + 6x + 5) + \frac{1}{2} \tan^{-1}(2x+3) + C$


(ii) $\int \frac{x + 3}{\sqrt{5 − 4x − x^2}} \;dx$

Let the integral be $I = \int \frac{x + 3}{\sqrt{5 − 4x − x^2}} \, dx$.

We use the form $\int \frac{px+q}{\sqrt{ax^2+bx+c}} \, dx$. The derivative of the quadratic inside the square root is $\frac{d}{dx}(5 - 4x - x^2) = -4 - 2x$.

We want to write the numerator $x+3$ in the form $A(-4-2x) + B$.

$x+3 = -4A - 2Ax + B$

Comparing coefficients of $x$: $-2A = 1 \implies A = -\frac{1}{2}$.

Comparing constant terms: $-4A + B = 3$. Substitute $A = -\frac{1}{2}$: $-4\left(-\frac{1}{2}\right) + B = 3 \implies 2 + B = 3 \implies B = 1$.

So, $x+3 = -\frac{1}{2}(-4-2x) + 1$.

Substitute this into the integral:

$I = \int \frac{-\frac{1}{2}(-4-2x) + 1}{\sqrt{5 − 4x − x^2}} \, dx$

= $\int \left( \frac{-\frac{1}{2}(-4-2x)}{\sqrt{5 − 4x − x^2}} + \frac{1}{\sqrt{5 − 4x − x^2}} \right) \, dx$

= $-\frac{1}{2} \int \frac{-4-2x}{\sqrt{5 − 4x − x^2}} \, dx + \int \frac{dx}{\sqrt{5 − 4x − x^2}}$

Let the first integral be $I_1$ and the second integral be $I_2$.

$I_1 = \int \frac{-4-2x}{\sqrt{5 − 4x − x^2}} \, dx$. Let $u = 5 - 4x - x^2$. Then $du = (-4-2x) \, dx$.

$I_1 = \int \frac{du}{\sqrt{u}} = \int u^{-1/2} \, du = \frac{u^{-1/2+1}}{-1/2+1} + C_1 = \frac{u^{1/2}}{1/2} + C_1 = 2\sqrt{u} + C_1 = 2\sqrt{5 - 4x - x^2} + C_1$.

For $I_2 = \int \frac{dx}{\sqrt{5 − 4x − x^2}}$, complete the square in the term under the square root.

$5 - 4x - x^2 = 5 - (x^2 + 4x)$

Complete the square for $x^2 + 4x$: add and subtract $\left(\frac{4}{2}\right)^2 = 2^2 = 4$.

$x^2 + 4x = (x^2 + 4x + 4) - 4 = (x+2)^2 - 4$

So, $5 - (x^2 + 4x) = 5 - ((x+2)^2 - 4) = 5 - (x+2)^2 + 4 = 9 - (x+2)^2 = 3^2 - (x+2)^2$.

$I_2 = \int \frac{dx}{\sqrt{3^2 - (x+2)^2}}$

This is in the standard form $\int \frac{du}{\sqrt{a^2 - u^2}}$, with $u = x+2$ ($du=dx$) and $a=3$.

$I_2 = \sin^{-1}\left(\frac{u}{a}\right) + C_2 = \sin^{-1}\left(\frac{x+2}{3}\right) + C_2$.

Combining $I_1$ and $I_2$:

$I = -\frac{1}{2} I_1 + I_2$

= $-\frac{1}{2} (2\sqrt{5 - 4x - x^2}) + \sin^{-1}\left(\frac{x+2}{3}\right) + C$ (where $C = -\frac{1}{2}C_1 + C_2$)

= $-\sqrt{5 - 4x - x^2} + \sin^{-1}\left(\frac{x+2}{3}\right) + C$



Exercise 7.4

Integrate the functions in Exercises 1 to 23.

Question 1. $\frac{3x^2}{x^6 + 1}$

Answer:

Let the given integral be $I$.

$I = \int \frac{3x^2}{x^6 + 1} dx$


We can rewrite the denominator as $(x^3)^2 + 1$. The integral becomes:

$I = \int \frac{3x^2}{(x^3)^2 + 1} dx$


Let us use the substitution method.

Let $u = x^3$.

Differentiating both sides with respect to $x$, we get:

$\frac{du}{dx} = 3x^2$

So, $du = 3x^2 dx$.


Now, substitute $u$ and $du$ into the integral:

$I = \int \frac{du}{u^2 + 1^2}$


This is a standard integral of the form $\int \frac{dx}{x^2 + a^2} = \frac{1}{a} \tan^{-1}\left(\frac{x}{a}\right) + C$.

Here, $x$ is $u$ and $a$ is 1.

So, $I = \frac{1}{1} \tan^{-1}\left(\frac{u}{1}\right) + C$

$I = \tan^{-1}(u) + C$


Substitute back $u = x^3$:

$I = \tan^{-1}(x^3) + C$


Therefore, the integral of the given function is:

$\int \frac{3x^2}{x^6 + 1} dx = \tan^{-1}(x^3) + C$

where $C$ is the constant of integration.

Question 2. $\frac{1}{\sqrt{1 + 4x^2}}$

Answer:

Let the given integral be $I$.

$I = \int \frac{1}{\sqrt{1 + 4x^2}} dx$


We can rewrite the term under the square root as $1^2 + (2x)^2$.

$I = \int \frac{1}{\sqrt{1^2 + (2x)^2}} dx$


Let us use the substitution method.

Let $u = 2x$.

Differentiating both sides with respect to $x$, we get:

$\frac{du}{dx} = 2$

So, $du = 2 dx$, which means $dx = \frac{1}{2} du$.


Now, substitute $u$ and $dx$ into the integral:

$I = \int \frac{1}{\sqrt{1^2 + u^2}} \left(\frac{1}{2} du\right)$

$I = \frac{1}{2} \int \frac{1}{\sqrt{1^2 + u^2}} du$


This is a standard integral of the form $\int \frac{1}{\sqrt{a^2 + x^2}} dx = \log_e |x + \sqrt{a^2 + x^2}| + C$.

Here, $x$ is $u$ and $a$ is 1.

So, $I = \frac{1}{2} \log_e |u + \sqrt{1^2 + u^2}| + C$

$I = \frac{1}{2} \log_e |u + \sqrt{1 + u^2}| + C$


Substitute back $u = 2x$:

$I = \frac{1}{2} \log_e |2x + \sqrt{1 + (2x)^2}| + C$

$I = \frac{1}{2} \log_e |2x + \sqrt{1 + 4x^2}| + C$


Therefore, the integral of the given function is:

$\int \frac{1}{\sqrt{1 + 4x^2}} dx = \frac{1}{2} \log_e |2x + \sqrt{1 + 4x^2}| + C$

where $C$ is the constant of integration.

Question 3. $\frac{1}{\sqrt{(2 − x)^2 + 1}}$

Answer:

Let the given integral be $I$.

$I = \int \frac{1}{\sqrt{(2 − x)^2 + 1}} dx$


We can rewrite the term under the square root as $(2-x)^2 + 1^2$.

$I = \int \frac{1}{\sqrt{(2-x)^2 + 1^2}} dx$


Let us use the substitution method.

Let $u = 2 - x$.

Differentiating both sides with respect to $x$, we get:

$\frac{du}{dx} = -1$

So, $du = -dx$, which means $dx = -du$.


Now, substitute $u$ and $dx$ into the integral:

$I = \int \frac{1}{\sqrt{u^2 + 1^2}} (-du)$

$I = - \int \frac{1}{\sqrt{u^2 + 1^2}} du$


This is a standard integral of the form $\int \frac{1}{\sqrt{x^2 + a^2}} dx = \log_e |x + \sqrt{x^2 + a^2}| + C$.

Here, $x$ is $u$ and $a$ is 1.

So, $I = - \left( \log_e |u + \sqrt{u^2 + 1^2}| \right) + C$

$I = - \log_e |u + \sqrt{u^2 + 1}| + C$


Substitute back $u = 2 - x$:

$I = - \log_e |(2 - x) + \sqrt{(2 - x)^2 + 1}| + C$


Alternatively, using properties of logarithms, $- \log_e A = \log_e (A^{-1}) = \log_e \left(\frac{1}{A}\right)$.

$I = \log_e \left| \frac{1}{(2 - x) + \sqrt{(2 - x)^2 + 1}} \right| + C$


Therefore, the integral of the given function is:

$\int \frac{1}{\sqrt{(2 − x)^2 + 1}} dx = - \log_e |(2 - x) + \sqrt{(2 - x)^2 + 1}| + C$

or

$\int \frac{1}{\sqrt{(2 − x)^2 + 1}} dx = \log_e \left| \frac{1}{(2 - x) + \sqrt{(2 - x)^2 + 1}} \right| + C$

where $C$ is the constant of integration.

Question 4. $\frac{1}{\sqrt{9 − 25x^2}}$

Answer:

Let the given integral be $I$.

$I = \int \frac{1}{\sqrt{9 − 25x^2}} dx$


We can rewrite the term under the square root as $3^2 - (5x)^2$.

$I = \int \frac{1}{\sqrt{3^2 - (5x)^2}} dx$


Let us use the substitution method.

Let $u = 5x$.

Differentiating both sides with respect to $x$, we get:

$\frac{du}{dx} = 5$

So, $du = 5 dx$, which means $dx = \frac{1}{5} du$.


Now, substitute $u$ and $dx$ into the integral:

$I = \int \frac{1}{\sqrt{3^2 - u^2}} \left(\frac{1}{5} du\right)$

$I = \frac{1}{5} \int \frac{1}{\sqrt{3^2 - u^2}} du$


This is a standard integral of the form $\int \frac{1}{\sqrt{a^2 - x^2}} dx = \sin^{-1}\left(\frac{x}{a}\right) + C$.

Here, $x$ is $u$ and $a$ is 3.

So, $I = \frac{1}{5} \sin^{-1}\left(\frac{u}{3}\right) + C$


Substitute back $u = 5x$:

$I = \frac{1}{5} \sin^{-1}\left(\frac{5x}{3}\right) + C$


Therefore, the integral of the given function is:

$\int \frac{1}{\sqrt{9 − 25x^2}} dx = \frac{1}{5} \sin^{-1}\left(\frac{5x}{3}\right) + C$

where $C$ is the constant of integration.

Question 5. $\frac{3x}{1 + 2x^4}$

Answer:

Let the given integral be $I$.

$I = \int \frac{3x}{1 + 2x^4} dx$


We can rewrite the denominator as $1^2 + (\sqrt{2}x^2)^2$. The integral becomes:

$I = \int \frac{3x}{1^2 + (\sqrt{2}x^2)^2} dx$


Let us use the substitution method.

Let $u = \sqrt{2}x^2$.

Differentiating both sides with respect to $x$, we get:

$\frac{du}{dx} = \sqrt{2} (2x) = 2\sqrt{2}x$

So, $du = 2\sqrt{2}x dx$. This means $x dx = \frac{1}{2\sqrt{2}} du$.

The numerator has $3x dx$, so $3x dx = 3 \left(\frac{1}{2\sqrt{2}} du\right) = \frac{3}{2\sqrt{2}} du$.


Now, substitute $u$ and $dx$ into the integral:

$I = \int \frac{1}{1^2 + u^2} \left(\frac{3}{2\sqrt{2}} du\right)$

$I = \frac{3}{2\sqrt{2}} \int \frac{1}{1^2 + u^2} du$


This is a standard integral of the form $\int \frac{1}{a^2 + x^2} dx = \frac{1}{a} \tan^{-1}\left(\frac{x}{a}\right) + C$.

Here, $x$ is $u$ and $a$ is 1.

So, $I = \frac{3}{2\sqrt{2}} \left( \frac{1}{1} \tan^{-1}\left(\frac{u}{1}\right) \right) + C$

$I = \frac{3}{2\sqrt{2}} \tan^{-1}(u) + C$


Substitute back $u = \sqrt{2}x^2$:

$I = \frac{3}{2\sqrt{2}} \tan^{-1}(\sqrt{2}x^2) + C$

We can rationalize the denominator $\frac{3}{2\sqrt{2}} = \frac{3\sqrt{2}}{2\sqrt{2}\sqrt{2}} = \frac{3\sqrt{2}}{4}$.

$I = \frac{3\sqrt{2}}{4} \tan^{-1}(\sqrt{2}x^2) + C$


Therefore, the integral of the given function is:

$\int \frac{3x}{1 + 2x^4} dx = \frac{3\sqrt{2}}{4} \tan^{-1}(\sqrt{2}x^2) + C$

where $C$ is the constant of integration.

Question 6. $\frac{x^2}{1 − x^6}$

Answer:

Let the given integral be $I$.

$I = \int \frac{x^2}{1 − x^6} dx$


We can rewrite the denominator as $1^2 - (x^3)^2$. The integral becomes:

$I = \int \frac{x^2}{1^2 - (x^3)^2} dx$


Let us use the substitution method.

Let $u = x^3$.

Differentiating both sides with respect to $x$, we get:

$\frac{du}{dx} = 3x^2$

So, $du = 3x^2 dx$. This means $x^2 dx = \frac{1}{3} du$.


Now, substitute $u$ and $dx$ into the integral:

$I = \int \frac{\frac{1}{3} du}{1^2 - u^2}$

$I = \frac{1}{3} \int \frac{1}{1^2 - u^2} du$


This is a standard integral of the form $\int \frac{1}{a^2 - x^2} dx = \frac{1}{2a} \log_e \left| \frac{a+x}{a-x} \right| + C$.

Here, $x$ is $u$ and $a$ is 1.

So, $I = \frac{1}{3} \left( \frac{1}{2 \times 1} \log_e \left| \frac{1+u}{1-u} \right| \right) + C$

$I = \frac{1}{6} \log_e \left| \frac{1+u}{1-u} \right| + C$


Substitute back $u = x^3$:

$I = \frac{1}{6} \log_e \left| \frac{1+x^3}{1-x^3} \right| + C$


Therefore, the integral of the given function is:

$\int \frac{x^2}{1 − x^6} dx = \frac{1}{6} \log_e \left| \frac{1+x^3}{1-x^3} \right| + C$

where $C$ is the constant of integration.

Question 7. $\frac{x − 1}{\sqrt{x^2 − 1}}$

Answer:

Let the given integral be $I$.

$I = \int \frac{x − 1}{\sqrt{x^2 − 1}} dx$


We can split the integral into two parts:

$I = \int \frac{x}{\sqrt{x^2 − 1}} dx - \int \frac{1}{\sqrt{x^2 − 1}} dx$


Let's evaluate the first integral, $I_1 = \int \frac{x}{\sqrt{x^2 − 1}} dx$.

Let $u = x^2 - 1$.

Differentiating both sides with respect to $x$, we get:

$\frac{du}{dx} = 2x$

So, $du = 2x dx$, which means $x dx = \frac{1}{2} du$.


Substituting $u$ and $dx$ into $I_1$:

$I_1 = \int \frac{1}{\sqrt{u}} \left(\frac{1}{2} du\right)$

$I_1 = \frac{1}{2} \int u^{-1/2} du$

$I_1 = \frac{1}{2} \left( \frac{u^{-1/2 + 1}}{-1/2 + 1} \right) + C_1$

$I_1 = \frac{1}{2} \left( \frac{u^{1/2}}{1/2} \right) + C_1$

$I_1 = \frac{1}{2} (2 \sqrt{u}) + C_1$

$I_1 = \sqrt{u} + C_1$

Substitute back $u = x^2 - 1$:

$I_1 = \sqrt{x^2 - 1} + C_1$


Now let's evaluate the second integral, $I_2 = - \int \frac{1}{\sqrt{x^2 − 1}} dx$.

This is a standard integral of the form $\int \frac{1}{\sqrt{x^2 - a^2}} dx = \log_e |x + \sqrt{x^2 - a^2}| + C$.

Here, $a=1$.

So, $I_2 = - \left( \log_e |x + \sqrt{x^2 - 1^2}| \right) + C_2$

$I_2 = - \log_e |x + \sqrt{x^2 - 1}| + C_2$


Combining $I_1$ and $I_2$ to get the total integral $I$:

$I = I_1 + I_2$

$I = (\sqrt{x^2 - 1} + C_1) + (- \log_e |x + \sqrt{x^2 - 1}| + C_2)$

$I = \sqrt{x^2 - 1} - \log_e |x + \sqrt{x^2 - 1}| + (C_1 + C_2)$


Let $C = C_1 + C_2$ be the constant of integration.

Therefore, the integral of the given function is:

$\int \frac{x − 1}{\sqrt{x^2 − 1}} dx = \sqrt{x^2 - 1} - \log_e |x + \sqrt{x^2 - 1}| + C$

Question 8. $\frac{x^2}{\sqrt{x^6 + a^6}}$

Answer:

Let the given integral be $I$.

$I = \int \frac{x^2}{\sqrt{x^6 + a^6}} dx$


We can rewrite the terms under the square root as $(x^3)^2 + (a^3)^2$. The integral becomes:

$I = \int \frac{x^2}{\sqrt{(x^3)^2 + (a^3)^2}} dx$


Let us use the substitution method.

Let $u = x^3$.

Differentiating both sides with respect to $x$, we get:

$\frac{du}{dx} = 3x^2$

So, $du = 3x^2 dx$. This means $x^2 dx = \frac{1}{3} du$.


Now, substitute $u$ and $dx$ into the integral:

$I = \int \frac{\frac{1}{3} du}{\sqrt{u^2 + (a^3)^2}}$

$I = \frac{1}{3} \int \frac{1}{\sqrt{u^2 + (a^3)^2}} du$


This is a standard integral of the form $\int \frac{1}{\sqrt{x^2 + b^2}} dx = \log_e |x + \sqrt{x^2 + b^2}| + C$.

Here, $x$ is $u$ and $b$ is $a^3$.

So, $I = \frac{1}{3} \left( \log_e |u + \sqrt{u^2 + (a^3)^2}| \right) + C$

$I = \frac{1}{3} \log_e |u + \sqrt{u^2 + a^6}| + C$


Substitute back $u = x^3$:

$I = \frac{1}{3} \log_e |x^3 + \sqrt{(x^3)^2 + a^6}| + C$

$I = \frac{1}{3} \log_e |x^3 + \sqrt{x^6 + a^6}| + C$


Therefore, the integral of the given function is:

$\int \frac{x^2}{\sqrt{x^6 + a^6}} dx = \frac{1}{3} \log_e |x^3 + \sqrt{x^6 + a^6}| + C$

where $C$ is the constant of integration.

Question 9. $\frac{\sec^2 x}{\sqrt{\tan^2 x + 4}}$

Answer:

Let the given integral be $I$.

$I = \int \frac{\sec^2 x}{\sqrt{\tan^2 x + 4}} dx$


Let us use the substitution method.

Let $u = \tan x$.

Differentiating both sides with respect to $x$, we get:

$\frac{du}{dx} = \sec^2 x$

So, $du = \sec^2 x dx$.


Now, substitute $u$ and $du$ into the integral:

$I = \int \frac{du}{\sqrt{u^2 + 4}}$

We can rewrite the term under the square root as $u^2 + 2^2$.

$I = \int \frac{du}{\sqrt{u^2 + 2^2}}$


This is a standard integral of the form $\int \frac{1}{\sqrt{x^2 + a^2}} dx = \log_e |x + \sqrt{x^2 + a^2}| + C$.

Here, $x$ is $u$ and $a$ is 2.

So, $I = \log_e |u + \sqrt{u^2 + 2^2}| + C$

$I = \log_e |u + \sqrt{u^2 + 4}| + C$


Substitute back $u = \tan x$:

$I = \log_e |\tan x + \sqrt{\tan^2 x + 4}| + C$


Therefore, the integral of the given function is:

$\int \frac{\sec^2 x}{\sqrt{\tan^2 x + 4}} dx = \log_e |\tan x + \sqrt{\tan^2 x + 4}| + C$

where $C$ is the constant of integration.

Question 10. $\frac{1}{\sqrt{x^2 + 2x + 2}}$

Answer:

Let the given integral be $I$.

$I = \int \frac{1}{\sqrt{x^2 + 2x + 2}} dx$


First, we complete the square for the quadratic expression in the denominator:

$x^2 + 2x + 2 = (x^2 + 2x + 1) + 1$

$= (x+1)^2 + 1^2$


Substitute this back into the integral:

$I = \int \frac{1}{\sqrt{(x+1)^2 + 1^2}} dx$


Let us use the substitution method.

Let $u = x+1$.

Differentiating both sides with respect to $x$, we get:

$\frac{du}{dx} = 1$

So, $du = dx$.


Now, substitute $u$ and $dx$ into the integral:

$I = \int \frac{1}{\sqrt{u^2 + 1^2}} du$


This is a standard integral of the form $\int \frac{1}{\sqrt{x^2 + a^2}} dx = \log_e |x + \sqrt{x^2 + a^2}| + C$.

Here, $x$ is $u$ and $a$ is 1.

So, $I = \log_e |u + \sqrt{u^2 + 1^2}| + C$

$I = \log_e |u + \sqrt{u^2 + 1}| + C$


Substitute back $u = x+1$:

$I = \log_e |(x+1) + \sqrt{(x+1)^2 + 1}| + C$


Simplify the term under the square root:

$(x+1)^2 + 1 = x^2 + 2x + 1 + 1 = x^2 + 2x + 2$

So, $I = \log_e |x+1 + \sqrt{x^2 + 2x + 2}| + C$


Therefore, the integral of the given function is:

$\int \frac{1}{\sqrt{x^2 + 2x + 2}} dx = \log_e |x+1 + \sqrt{x^2 + 2x + 2}| + C$

where $C$ is the constant of integration.

Question 11. $\frac{1}{9x^2 + 6x + 5}$

Answer:

Let the given integral be $I$.

$I = \int \frac{1}{9x^2 + 6x + 5} dx$


First, we complete the square for the quadratic expression in the denominator.

$9x^2 + 6x + 5 = 9\left(x^2 + \frac{6}{9}x + \frac{5}{9}\right)$

$= 9\left(x^2 + \frac{2}{3}x + \frac{5}{9}\right)$

To complete the square for $x^2 + \frac{2}{3}x$, we add and subtract $\left(\frac{1}{2} \times \frac{2}{3}\right)^2 = \left(\frac{1}{3}\right)^2 = \frac{1}{9}$.

$= 9\left(x^2 + \frac{2}{3}x + \frac{1}{9} - \frac{1}{9} + \frac{5}{9}\right)$

$= 9\left(\left(x + \frac{1}{3}\right)^2 + \frac{4}{9}\right)$

$= 9\left(x + \frac{1}{3}\right)^2 + 9 \times \frac{4}{9}$

$= 9\left(x + \frac{1}{3}\right)^2 + 4$

We can rewrite $9\left(x + \frac{1}{3}\right)^2$ as $\left(3\left(x + \frac{1}{3}\right)\right)^2 = (3x + 1)^2$.

So, the denominator is $(3x + 1)^2 + 4 = (3x+1)^2 + 2^2$.


Substitute this back into the integral:

$I = \int \frac{1}{(3x + 1)^2 + 2^2} dx$


Let us use the substitution method.

Let $u = 3x + 1$.

Differentiating both sides with respect to $x$, we get:

$\frac{du}{dx} = 3$

So, $du = 3 dx$, which means $dx = \frac{1}{3} du$.


Now, substitute $u$ and $dx$ into the integral:

$I = \int \frac{1}{u^2 + 2^2} \left(\frac{1}{3} du\right)$

$I = \frac{1}{3} \int \frac{1}{u^2 + 2^2} du$


This is a standard integral of the form $\int \frac{1}{x^2 + a^2} dx = \frac{1}{a} \tan^{-1}\left(\frac{x}{a}\right) + C$.

Here, $x$ is $u$ and $a$ is 2.

So, $I = \frac{1}{3} \left( \frac{1}{2} \tan^{-1}\left(\frac{u}{2}\right) \right) + C$

$I = \frac{1}{6} \tan^{-1}\left(\frac{u}{2}\right) + C$


Substitute back $u = 3x + 1$:

$I = \frac{1}{6} \tan^{-1}\left(\frac{3x + 1}{2}\right) + C$


Therefore, the integral of the given function is:

$\int \frac{1}{9x^2 + 6x + 5} dx = \frac{1}{6} \tan^{-1}\left(\frac{3x + 1}{2}\right) + C$

where $C$ is the constant of integration.

Question 12. $\frac{1}{\sqrt{7 − 6x − x^2}}$

Answer:

Let the given integral be $I$.

$I = \int \frac{1}{\sqrt{7 − 6x − x^2}} dx$


First, we complete the square for the quadratic expression in the denominator:

$7 - 6x - x^2 = - (x^2 + 6x - 7)$

Complete the square for $x^2 + 6x$ by adding and subtracting $\left(\frac{6}{2}\right)^2 = 3^2 = 9$:

$x^2 + 6x - 7 = (x^2 + 6x + 9) - 9 - 7$

$= (x+3)^2 - 16$

So, $7 - 6x - x^2 = - ((x+3)^2 - 16) = 16 - (x+3)^2$.

We can write $16$ as $4^2$.

The denominator is $\sqrt{4^2 - (x+3)^2}$.


Substitute this back into the integral:

$I = \int \frac{1}{\sqrt{4^2 - (x+3)^2}} dx$


Let us use the substitution method.

Let $u = x+3$.

Differentiating both sides with respect to $x$, we get:

$\frac{du}{dx} = 1$

So, $du = dx$.


Now, substitute $u$ and $dx$ into the integral:

$I = \int \frac{1}{\sqrt{4^2 - u^2}} du$


This is a standard integral of the form $\int \frac{1}{\sqrt{a^2 - x^2}} dx = \sin^{-1}\left(\frac{x}{a}\right) + C$.

Here, $x$ is $u$ and $a$ is 4.

So, $I = \sin^{-1}\left(\frac{u}{4}\right) + C$


Substitute back $u = x+3$:

$I = \sin^{-1}\left(\frac{x+3}{4}\right) + C$


Therefore, the integral of the given function is:

$\int \frac{1}{\sqrt{7 − 6x − x^2}} dx = \sin^{-1}\left(\frac{x+3}{4}\right) + C$

where $C$ is the constant of integration.

Question 13. $\frac{1}{\sqrt{(x − 1) (x − 2)}}$

Answer:

Let the given integral be $I$.

$I = \int \frac{1}{\sqrt{(x − 1) (x − 2)}} dx$


First, expand the expression inside the square root:

$(x − 1) (x − 2) = x^2 - 2x - x + 2 = x^2 - 3x + 2$

The integral becomes:

$I = \int \frac{1}{\sqrt{x^2 - 3x + 2}} dx$


Next, we complete the square for the quadratic expression $x^2 - 3x + 2$.

The coefficient of $x$ is -3. Half of this coefficient is $-\frac{3}{2}$. Squaring it gives $\left(-\frac{3}{2}\right)^2 = \frac{9}{4}$.

$x^2 - 3x + 2 = \left(x^2 - 3x + \frac{9}{4}\right) - \frac{9}{4} + 2$

$= \left(x - \frac{3}{2}\right)^2 + \frac{-9 + 8}{4}$

$= \left(x - \frac{3}{2}\right)^2 - \frac{1}{4}$

We can write $\frac{1}{4}$ as $\left(\frac{1}{2}\right)^2$.

So, $x^2 - 3x + 2 = \left(x - \frac{3}{2}\right)^2 - \left(\frac{1}{2}\right)^2$.


Substitute this back into the integral:

$I = \int \frac{1}{\sqrt{\left(x - \frac{3}{2}\right)^2 - \left(\frac{1}{2}\right)^2}} dx$


This is a standard integral of the form $\int \frac{1}{\sqrt{u^2 - a^2}} du = \log_e |u + \sqrt{u^2 - a^2}| + C$.

Here, $u = x - \frac{3}{2}$ and $a = \frac{1}{2}$.

Note that if we substitute $u = x - \frac{3}{2}$, then $du = dx$.

So, the integral is:

$I = \log_e \left| \left(x - \frac{3}{2}\right) + \sqrt{\left(x - \frac{3}{2}\right)^2 - \left(\frac{1}{2}\right)^2} \right| + C$


Substitute back the original quadratic expression inside the square root:

$\left(x - \frac{3}{2}\right)^2 - \left(\frac{1}{2}\right)^2 = x^2 - 3x + 2 = (x-1)(x-2)$

So, $I = \log_e \left| x - \frac{3}{2} + \sqrt{x^2 - 3x + 2} \right| + C$

or

$I = \log_e \left| x - \frac{3}{2} + \sqrt{(x-1)(x-2)} \right| + C$


Therefore, the integral of the given function is:

$\int \frac{1}{\sqrt{(x − 1) (x − 2)}} dx = \log_e \left| x - \frac{3}{2} + \sqrt{(x-1)(x-2)} \right| + C$

where $C$ is the constant of integration.

Question 14. $\frac{1}{\sqrt{8 + 3x − x^2}}$

Answer:

Let the given integral be $I$.

$I = \int \frac{1}{\sqrt{8 + 3x − x^2}} dx$


First, we complete the square for the quadratic expression under the square root:

$8 + 3x - x^2 = - (x^2 - 3x - 8)$

To complete the square for $x^2 - 3x$, we add and subtract $\left(\frac{-3}{2}\right)^2 = \frac{9}{4}$.

$x^2 - 3x - 8 = \left(x^2 - 3x + \frac{9}{4}\right) - \frac{9}{4} - 8$

$= \left(x - \frac{3}{2}\right)^2 - \frac{9}{4} - \frac{32}{4}$

$= \left(x - \frac{3}{2}\right)^2 - \frac{41}{4}$

Now, substitute back the negative sign:

$8 + 3x - x^2 = - \left( \left(x - \frac{3}{2}\right)^2 - \frac{41}{4} \right) = \frac{41}{4} - \left(x - \frac{3}{2}\right)^2$

We can write $\frac{41}{4}$ as $\left(\frac{\sqrt{41}}{2}\right)^2$.

So, the expression under the square root is $\left(\frac{\sqrt{41}}{2}\right)^2 - \left(x - \frac{3}{2}\right)^2$.


Substitute this back into the integral:

$I = \int \frac{1}{\sqrt{\left(\frac{\sqrt{41}}{2}\right)^2 - \left(x - \frac{3}{2}\right)^2}} dx$


This integral is in the form $\int \frac{1}{\sqrt{a^2 - u^2}} du$, where $a = \frac{\sqrt{41}}{2}$ and $u = x - \frac{3}{2}$.

Let $u = x - \frac{3}{2}$. Then $du = dx$.


Using the standard integral formula $\int \frac{1}{\sqrt{a^2 - u^2}} du = \sin^{-1}\left(\frac{u}{a}\right) + C$, we get:

$I = \sin^{-1}\left(\frac{x - \frac{3}{2}}{\frac{\sqrt{41}}{2}}\right) + C$

$I = \sin^{-1}\left(\frac{\frac{2x - 3}{2}}{\frac{\sqrt{41}}{2}}\right) + C$

$I = \sin^{-1}\left(\frac{2x - 3}{\sqrt{41}}\right) + C$


Therefore, the integral of the given function is:

$\int \frac{1}{\sqrt{8 + 3x − x^2}} dx = \sin^{-1}\left(\frac{2x - 3}{\sqrt{41}}\right) + C$

where $C$ is the constant of integration.

Question 15. $\frac{1}{\sqrt{(x − a) (x − b)}}$

Answer:

Let the given integral be $I$.

$I = \int \frac{1}{\sqrt{(x − a) (x − b)}} dx$


First, we expand the expression inside the square root:

$(x − a) (x − b) = x^2 - bx - ax + ab = x^2 - (a+b)x + ab$

The integral becomes:

$I = \int \frac{1}{\sqrt{x^2 - (a+b)x + ab}} dx$


Next, we complete the square for the quadratic expression $x^2 - (a+b)x + ab$.

The coefficient of $x$ is $-(a+b)$. Half of this coefficient is $-\frac{a+b}{2}$. Squaring it gives $\left(-\frac{a+b}{2}\right)^2 = \left(\frac{a+b}{2}\right)^2$.

$x^2 - (a+b)x + ab = \left(x^2 - (a+b)x + \left(\frac{a+b}{2}\right)^2\right) - \left(\frac{a+b}{2}\right)^2 + ab$

$= \left(x - \frac{a+b}{2}\right)^2 - \frac{(a+b)^2 - 4ab}{4}$

$= \left(x - \frac{a+b}{2}\right)^2 - \frac{a^2 + 2ab + b^2 - 4ab}{4}$

$= \left(x - \frac{a+b}{2}\right)^2 - \frac{a^2 - 2ab + b^2}{4}$

$= \left(x - \frac{a+b}{2}\right)^2 - \left(\frac{a-b}{2}\right)^2$


Substitute this back into the integral:

$I = \int \frac{1}{\sqrt{\left(x - \frac{a+b}{2}\right)^2 - \left(\frac{a-b}{2}\right)^2}} dx$


This is a standard integral of the form $\int \frac{1}{\sqrt{u^2 - c^2}} du = \log_e |u + \sqrt{u^2 - c^2}| + C$.

Here, $u = x - \frac{a+b}{2}$ and $c = \frac{a-b}{2}$ (assuming $a>b$, or we can use $\left|\frac{a-b}{2}\right|$). The substitution $u = x - \frac{a+b}{2}$ gives $du = dx$.


Using the standard integral formula:

$I = \log_e \left| \left(x - \frac{a+b}{2}\right) + \sqrt{\left(x - \frac{a+b}{2}\right)^2 - \left(\frac{a-b}{2}\right)^2} \right| + C$


Substitute back the original quadratic expression inside the square root:

$\left(x - \frac{a+b}{2}\right)^2 - \left(\frac{a-b}{2}\right)^2 = x^2 - (a+b)x + ab = (x-a)(x-b)$

So, $I = \log_e \left| x - \frac{a+b}{2} + \sqrt{(x-a)(x-b)} \right| + C$


Therefore, the integral of the given function is:

$\int \frac{1}{\sqrt{(x − a) (x − b)}} dx = \log_e \left| x - \frac{a+b}{2} + \sqrt{(x-a)(x-b)} \right| + C$

where $C$ is the constant of integration.

Question 16. $\frac{4x + 1}{\sqrt{2x^2 + x - 3}}$

Answer:

Let the given integral be $I$.

$I = \int \frac{4x + 1}{\sqrt{2x^2 + x - 3}} dx$


Observe the numerator and the expression under the square root.

Let the expression under the square root be $f(x) = 2x^2 + x - 3$.

The derivative of $f(x)$ with respect to $x$ is $f'(x) = \frac{d}{dx}(2x^2 + x - 3) = 4x + 1$.

The numerator is exactly the derivative of the expression under the square root.


This integral is of the form $\int \frac{f'(x)}{\sqrt{f(x)}} dx$.

Let us use the substitution method.

Let $u = 2x^2 + x - 3$.

Differentiating both sides with respect to $x$, we get:

$\frac{du}{dx} = 4x + 1$

So, $du = (4x + 1) dx$.


Now, substitute $u$ and $du$ into the integral:

$I = \int \frac{du}{\sqrt{u}}$

$I = \int u^{-1/2} du$


Using the power rule for integration, $\int u^n du = \frac{u^{n+1}}{n+1} + C$ (for $n \neq -1$).

Here, $n = -1/2$.

$I = \frac{u^{-1/2 + 1}}{-1/2 + 1} + C$

$I = \frac{u^{1/2}}{1/2} + C$

$I = 2u^{1/2} + C$

$I = 2\sqrt{u} + C$


Substitute back $u = 2x^2 + x - 3$:

$I = 2\sqrt{2x^2 + x - 3} + C$


Therefore, the integral of the given function is:

$\int \frac{4x + 1}{\sqrt{2x^2 + x - 3}} dx = 2\sqrt{2x^2 + x - 3} + C$

where $C$ is the constant of integration.

Question 17. $\frac{x + 2}{\sqrt{x^2 − 1}}$

Answer:

Let the given integral be $I$.

$I = \int \frac{x + 2}{\sqrt{x^2 − 1}} dx$


We can split the integral into two parts:

$I = \int \frac{x}{\sqrt{x^2 − 1}} dx + \int \frac{2}{\sqrt{x^2 − 1}} dx$


Let's evaluate the first integral, $I_1 = \int \frac{x}{\sqrt{x^2 − 1}} dx$.

Let $u = x^2 - 1$.

Differentiating both sides with respect to $x$, we get:

$\frac{du}{dx} = 2x$

So, $du = 2x dx$, which means $x dx = \frac{1}{2} du$.


Substituting $u$ and $dx$ into $I_1$:

$I_1 = \int \frac{1}{\sqrt{u}} \left(\frac{1}{2} du\right)$

$I_1 = \frac{1}{2} \int u^{-1/2} du$

$I_1 = \frac{1}{2} \left( \frac{u^{-1/2 + 1}}{-1/2 + 1} \right) + C_1$

$I_1 = \frac{1}{2} \left( \frac{u^{1/2}}{1/2} \right) + C_1$

$I_1 = \frac{1}{2} (2 \sqrt{u}) + C_1$

$I_1 = \sqrt{u} + C_1$

Substitute back $u = x^2 - 1$:

$I_1 = \sqrt{x^2 - 1} + C_1$


Now let's evaluate the second integral, $I_2 = \int \frac{2}{\sqrt{x^2 − 1}} dx$.

$I_2 = 2 \int \frac{1}{\sqrt{x^2 − 1^2}} dx$

This is a standard integral of the form $\int \frac{1}{\sqrt{x^2 - a^2}} dx = \log_e |x + \sqrt{x^2 - a^2}| + C$.

Here, $a=1$.

So, $I_2 = 2 \left( \log_e |x + \sqrt{x^2 - 1^2}| \right) + C_2$

$I_2 = 2 \log_e |x + \sqrt{x^2 - 1}| + C_2$


Combining $I_1$ and $I_2$ to get the total integral $I$: a

$I = I_1 + I_2$

$I = (\sqrt{x^2 - 1} + C_1) + (2 \log_e |x + \sqrt{x^2 - 1}| + C_2)$

$I = \sqrt{x^2 - 1} + 2 \log_e |x + \sqrt{x^2 - 1}| + (C_1 + C_2)$


Let $C = C_1 + C_2$ be the constant of integration.

Therefore, the integral of the given function is:

$\int \frac{x + 2}{\sqrt{x^2 − 1}} dx = \sqrt{x^2 - 1} + 2 \log_e |x + \sqrt{x^2 - 1}| + C$

Question 18. $\frac{5x − 2}{1 + 2x + 3x^2}$

Answer:

Let the given integral be $I$.

$I = \int \frac{5x − 2}{3x^2 + 2x + 1} dx$


We write the numerator $5x - 2$ in terms of the derivative of the denominator $3x^2 + 2x + 1$.

The derivative of $3x^2 + 2x + 1$ is $\frac{d}{dx}(3x^2 + 2x + 1) = 6x + 2$.

We need to find constants $A$ and $B$ such that:

$5x - 2 = A(6x + 2) + B$

$5x - 2 = 6Ax + 2A + B$


Comparing the coefficients of $x$ on both sides:

$5 = 6A \implies A = \frac{5}{6}$


Comparing the constant terms on both sides:

$-2 = 2A + B$

Substitute $A = \frac{5}{6}$:

$-2 = 2\left(\frac{5}{6}\right) + B$

$-2 = \frac{10}{6} + B$

$-2 = \frac{5}{3} + B$

$B = -2 - \frac{5}{3} = -\frac{6}{3} - \frac{5}{3} = -\frac{11}{3}$


So, we can write the numerator as $5x - 2 = \frac{5}{6}(6x + 2) - \frac{11}{3}$.

The integral becomes:

$I = \int \frac{\frac{5}{6}(6x + 2) - \frac{11}{3}}{3x^2 + 2x + 1} dx$

$I = \frac{5}{6} \int \frac{6x + 2}{3x^2 + 2x + 1} dx - \frac{11}{3} \int \frac{1}{3x^2 + 2x + 1} dx$


Let's evaluate the first integral: $I_1 = \int \frac{6x + 2}{3x^2 + 2x + 1} dx$.

Let $u = 3x^2 + 2x + 1$. Then $du = (6x + 2) dx$.

$I_1 = \int \frac{du}{u} = \log_e |u| + C_1$

Since the discriminant of $3x^2 + 2x + 1$ is $2^2 - 4(3)(1) = 4 - 12 = -8 < 0$ and the coefficient of $x^2$ is positive, the quadratic is always positive. So, $|3x^2 + 2x + 1| = 3x^2 + 2x + 1$.

$I_1 = \log_e (3x^2 + 2x + 1) + C_1$


Now, let's evaluate the second integral: $I_2 = \int \frac{1}{3x^2 + 2x + 1} dx$.

We complete the square for the denominator $3x^2 + 2x + 1$:

$3x^2 + 2x + 1 = 3\left(x^2 + \frac{2}{3}x + \frac{1}{3}\right)$

$= 3\left(\left(x + \frac{1}{3}\right)^2 - \left(\frac{1}{3}\right)^2 + \frac{1}{3}\right)$

$= 3\left(\left(x + \frac{1}{3}\right)^2 - \frac{1}{9} + \frac{3}{9}\right)$

$= 3\left(\left(x + \frac{1}{3}\right)^2 + \frac{2}{9}\right)$

$= 3\left(x + \frac{1}{3}\right)^2 + \frac{2}{3}$

$= (\sqrt{3}(x + \frac{1}{3}))^2 + \left(\sqrt{\frac{2}{3}}\right)^2$


Substitute this back into $I_2$:

$I_2 = \int \frac{1}{3\left(x + \frac{1}{3}\right)^2 + \frac{2}{3}} dx = \frac{1}{3} \int \frac{1}{\left(x + \frac{1}{3}\right)^2 + \frac{2}{9}} dx$

$I_2 = \frac{1}{3} \int \frac{1}{\left(x + \frac{1}{3}\right)^2 + \left(\frac{\sqrt{2}}{3}\right)^2} dx$


This integral is of the form $\int \frac{1}{u^2 + a^2} du$, where $u = x + \frac{1}{3}$ and $a = \frac{\sqrt{2}}{3}$. Note that $du = dx$.

Using the standard integral $\int \frac{1}{u^2 + a^2} du = \frac{1}{a} \tan^{-1}\left(\frac{u}{a}\right) + C_2$, we get:

$I_2 = \frac{1}{3} \left( \frac{1}{\frac{\sqrt{2}}{3}} \tan^{-1}\left(\frac{x + \frac{1}{3}}{\frac{\sqrt{2}}{3}}\right) \right) + C_2$

$I_2 = \frac{1}{3} \left( \frac{3}{\sqrt{2}} \tan^{-1}\left(\frac{\frac{3x + 1}{3}}{\frac{\sqrt{2}}{3}}\right) \right) + C_2$

$I_2 = \frac{1}{\sqrt{2}} \tan^{-1}\left(\frac{3x + 1}{\sqrt{2}}\right) + C_2$


Combining the results for $I_1$ and $I_2$ for the total integral $I = \frac{5}{6} I_1 - \frac{11}{3} I_2$:

$I = \frac{5}{6} (\log_e (3x^2 + 2x + 1) + C_1) - \frac{11}{3} \left( \frac{1}{\sqrt{2}} \tan^{-1}\left(\frac{3x + 1}{\sqrt{2}}\right) + C_2 \right)$

$I = \frac{5}{6} \log_e (3x^2 + 2x + 1) - \frac{11}{3\sqrt{2}} \tan^{-1}\left(\frac{3x + 1}{\sqrt{2}}\right) + \frac{5}{6}C_1 - \frac{11}{3}C_2$


Let $C = \frac{5}{6}C_1 - \frac{11}{3}C_2$ be the constant of integration.

We can rationalize $\frac{11}{3\sqrt{2}} = \frac{11\sqrt{2}}{3\sqrt{2}\sqrt{2}} = \frac{11\sqrt{2}}{6}$.

Therefore, the integral of the given function is:

$\int \frac{5x − 2}{1 + 2x + 3x^2} dx = \frac{5}{6} \log_e (3x^2 + 2x + 1) - \frac{11\sqrt{2}}{6} \tan^{-1}\left(\frac{3x + 1}{\sqrt{2}}\right) + C$

Question 19. $\frac{6x + 7}{\sqrt{(x − 5)(x − 4)}}$

Answer:

Let the given integral be $I$.

$I = \int \frac{6x + 7}{\sqrt{(x − 5)(x − 4)}} dx$


First, we expand the expression inside the square root:

$(x − 5) (x − 4) = x^2 - 4x - 5x + 20 = x^2 - 9x + 20$

The integral becomes:

$I = \int \frac{6x + 7}{\sqrt{x^2 - 9x + 20}} dx$


We write the numerator $6x + 7$ in terms of the derivative of the quadratic expression under the square root, $x^2 - 9x + 20$.

The derivative of $x^2 - 9x + 20$ is $\frac{d}{dx}(x^2 - 9x + 20) = 2x - 9$.

We need to find constants $A$ and $B$ such that:

$6x + 7 = A(2x - 9) + B$

$6x + 7 = 2Ax - 9A + B$


Comparing the coefficients of $x$ on both sides:

$6 = 2A \implies A = 3$


Comparing the constant terms on both sides:

$7 = -9A + B$

Substitute $A = 3$:

$7 = -9(3) + B$

$7 = -27 + B$

$B = 7 + 27 = 34$


So, we can write the numerator as $6x + 7 = 3(2x - 9) + 34$.

The integral becomes:

$I = \int \frac{3(2x - 9) + 34}{\sqrt{x^2 - 9x + 20}} dx$

We split the integral into two parts:

$I = 3 \int \frac{2x - 9}{\sqrt{x^2 - 9x + 20}} dx + 34 \int \frac{1}{\sqrt{x^2 - 9x + 20}} dx$


Let's evaluate the first integral: $I_1 = 3 \int \frac{2x - 9}{\sqrt{x^2 - 9x + 20}} dx$.

Let $u = x^2 - 9x + 20$. Then $du = (2x - 9) dx$.

$I_1 = 3 \int \frac{du}{\sqrt{u}} = 3 \int u^{-1/2} du$

$I_1 = 3 \left(\frac{u^{-1/2 + 1}}{-1/2 + 1}\right) + C_1 = 3 \left(\frac{u^{1/2}}{1/2}\right) + C_1$

$I_1 = 6\sqrt{u} + C_1$

Substitute back $u = x^2 - 9x + 20$:

$I_1 = 6\sqrt{x^2 - 9x + 20} + C_1$


Now, let's evaluate the second integral: $I_2 = 34 \int \frac{1}{\sqrt{x^2 - 9x + 20}} dx$.

We complete the square for the quadratic expression $x^2 - 9x + 20$:

$x^2 - 9x + 20 = x^2 - 9x + \left(\frac{9}{2}\right)^2 - \left(\frac{9}{2}\right)^2 + 20$

$= \left(x - \frac{9}{2}\right)^2 - \frac{81}{4} + \frac{80}{4}$

$= \left(x - \frac{9}{2}\right)^2 - \frac{1}{4} = \left(x - \frac{9}{2}\right)^2 - \left(\frac{1}{2}\right)^2$


Substitute this back into $I_2$:

$I_2 = 34 \int \frac{1}{\sqrt{\left(x - \frac{9}{2}\right)^2 - \left(\frac{1}{2}\right)^2}} dx$

This is a standard integral of the form $\int \frac{1}{\sqrt{u^2 - a^2}} du = \log_e |u + \sqrt{u^2 - a^2}| + C$.

Here, $u = x - \frac{9}{2}$ and $a = \frac{1}{2}$. The substitution $u = x - \frac{9}{2}$ gives $du = dx$.

Using the standard integral formula:

$I_2 = 34 \log_e \left| \left(x - \frac{9}{2}\right) + \sqrt{\left(x - \frac{9}{2}\right)^2 - \left(\frac{1}{2}\right)^2} \right| + C_2$


Substitute back the original quadratic expression inside the square root:

$\left(x - \frac{9}{2}\right)^2 - \left(\frac{1}{2}\right)^2 = x^2 - 9x + 20 = (x-5)(x-4)$

So, $I_2 = 34 \log_e \left| x - \frac{9}{2} + \sqrt{x^2 - 9x + 20} \right| + C_2$

or

$I_2 = 34 \log_e \left| x - \frac{9}{2} + \sqrt{(x-5)(x-4)} \right| + C_2$


Combining the results for $I_1$ and $I_2$ for the total integral $I = I_1 + I_2$:

$I = (6\sqrt{x^2 - 9x + 20} + C_1) + (34 \log_e \left| x - \frac{9}{2} + \sqrt{x^2 - 9x + 20} \right| + C_2)$

$I = 6\sqrt{x^2 - 9x + 20} + 34 \log_e \left| x - \frac{9}{2} + \sqrt{x^2 - 9x + 20} \right| + C$

where $C = C_1 + C_2$ is the constant of integration.


Therefore, the integral of the given function is:

$\int \frac{6x + 7}{\sqrt{(x − 5)(x − 4)}} dx = 6\sqrt{x^2 - 9x + 20} + 34 \log_e \left| x - \frac{9}{2} + \sqrt{x^2 - 9x + 20} \right| + C$

Question 20. $\frac{x + 2}{\sqrt{4x − x^2}}$

Answer:

Let the given integral be $I$.

$I = \int \frac{x + 2}{\sqrt{4x − x^2}} dx$


We write the numerator $x + 2$ in terms of the derivative of the quadratic expression under the square root, $4x - x^2$.

The derivative of $4x - x^2$ is $\frac{d}{dx}(4x - x^2) = 4 - 2x$.

We need to find constants $A$ and $B$ such that:

$x + 2 = A(4 - 2x) + B$

$x + 2 = 4A - 2Ax + B$

$x + 2 = -2Ax + (4A + B)$


Comparing the coefficients of $x$ on both sides:

$1 = -2A \implies A = -\frac{1}{2}$


Comparing the constant terms on both sides:

$2 = 4A + B$

Substitute $A = -\frac{1}{2}$:

$2 = 4\left(-\frac{1}{2}\right) + B$

$2 = -2 + B$

$B = 2 + 2 = 4$


So, we can write the numerator as $x + 2 = -\frac{1}{2}(4 - 2x) + 4$.

The integral becomes:

$I = \int \frac{-\frac{1}{2}(4 - 2x) + 4}{\sqrt{4x - x^2}} dx$

We split the integral into two parts:

$I = -\frac{1}{2} \int \frac{4 - 2x}{\sqrt{4x - x^2}} dx + 4 \int \frac{1}{\sqrt{4x - x^2}} dx$


Let's evaluate the first integral: $I_1 = -\frac{1}{2} \int \frac{4 - 2x}{\sqrt{4x - x^2}} dx$.

Let $u = 4x - x^2$. Differentiating both sides with respect to $x$, we get $du = (4 - 2x) dx$.

$I_1 = -\frac{1}{2} \int \frac{du}{\sqrt{u}} = -\frac{1}{2} \int u^{-1/2} du$

$I_1 = -\frac{1}{2} \left(\frac{u^{-1/2 + 1}}{-1/2 + 1}\right) + C_1 = -\frac{1}{2} \left(\frac{u^{1/2}}{1/2}\right) + C_1$

$I_1 = -\sqrt{u} + C_1$

Substitute back $u = 4x - x^2$:

$I_1 = -\sqrt{4x - x^2} + C_1$


Now, let's evaluate the second integral: $I_2 = 4 \int \frac{1}{\sqrt{4x - x^2}} dx$.

We complete the square for the quadratic expression $4x - x^2$:

$4x - x^2 = - (x^2 - 4x)$

$= - (x^2 - 4x + 4 - 4)$

$= - ((x - 2)^2 - 4)$

$= 4 - (x - 2)^2$

So, the expression under the square root is $2^2 - (x-2)^2$.


Substitute this back into $I_2$:

$I_2 = 4 \int \frac{1}{\sqrt{2^2 - (x - 2)^2}} dx$

This is a standard integral of the form $\int \frac{1}{\sqrt{a^2 - u^2}} du = \sin^{-1}\left(\frac{u}{a}\right) + C$.

Here, $u = x - 2$ and $a = 2$. The substitution $u = x - 2$ gives $du = dx$.

Using the standard integral formula:

$I_2 = 4 \sin^{-1}\left(\frac{x - 2}{2}\right) + C_2$


Combining the results for $I_1$ and $I_2$ for the total integral $I = I_1 + I_2$:

$I = (-\sqrt{4x - x^2} + C_1) + (4 \sin^{-1}\left(\frac{x - 2}{2}\right) + C_2)$

$I = -\sqrt{4x - x^2} + 4 \sin^{-1}\left(\frac{x - 2}{2}\right) + C$

where $C = C_1 + C_2$ is the constant of integration.


Therefore, the integral of the given function is:

$\int \frac{x + 2}{\sqrt{4x − x^2}} dx = -\sqrt{4x - x^2} + 4 \sin^{-1}\left(\frac{x - 2}{2}\right) + C$

Question 21. $\frac{x + 2}{\sqrt{x^2 + 2x + 3}}$

Answer:

Let the given integral be $I$.

$I = \int \frac{x + 2}{\sqrt{x^2 + 2x + 3}} dx$


We write the numerator $x + 2$ in terms of the derivative of the quadratic expression under the square root, $x^2 + 2x + 3$.

The derivative of $x^2 + 2x + 3$ is $\frac{d}{dx}(x^2 + 2x + 3) = 2x + 2$.

We need to find constants $A$ and $B$ such that:

$x + 2 = A(2x + 2) + B$

$x + 2 = 2Ax + 2A + B$


Comparing the coefficients of $x$ on both sides:

$1 = 2A \implies A = \frac{1}{2}$


Comparing the constant terms on both sides:

$2 = 2A + B$

Substitute $A = \frac{1}{2}$:

$2 = 2\left(\frac{1}{2}\right) + B$

$2 = 1 + B$

$B = 2 - 1 = 1$


So, we can write the numerator as $x + 2 = \frac{1}{2}(2x + 2) + 1$.

The integral becomes:

$I = \int \frac{\frac{1}{2}(2x + 2) + 1}{\sqrt{x^2 + 2x + 3}} dx$

We split the integral into two parts:

$I = \frac{1}{2} \int \frac{2x + 2}{\sqrt{x^2 + 2x + 3}} dx + \int \frac{1}{\sqrt{x^2 + 2x + 3}} dx$


Let's evaluate the first integral: $I_1 = \frac{1}{2} \int \frac{2x + 2}{\sqrt{x^2 + 2x + 3}} dx$.

Let $u = x^2 + 2x + 3$. Differentiating both sides with respect to $x$, we get $du = (2x + 2) dx$.

$I_1 = \frac{1}{2} \int \frac{du}{\sqrt{u}} = \frac{1}{2} \int u^{-1/2} du$

$I_1 = \frac{1}{2} \left(\frac{u^{-1/2 + 1}}{-1/2 + 1}\right) + C_1 = \frac{1}{2} \left(\frac{u^{1/2}}{1/2}\right) + C_1$

$I_1 = \sqrt{u} + C_1$

Substitute back $u = x^2 + 2x + 3$:

$I_1 = \sqrt{x^2 + 2x + 3} + C_1$


Now, let's evaluate the second integral: $I_2 = \int \frac{1}{\sqrt{x^2 + 2x + 3}} dx$.

We complete the square for the quadratic expression $x^2 + 2x + 3$:

$x^2 + 2x + 3 = (x^2 + 2x + 1) + 2$

$= (x+1)^2 + (\sqrt{2})^2$


Substitute this back into $I_2$:

$I_2 = \int \frac{1}{\sqrt{(x+1)^2 + (\sqrt{2})^2}} dx$

This is a standard integral of the form $\int \frac{1}{\sqrt{u^2 + a^2}} du = \log_e |u + \sqrt{u^2 + a^2}| + C$.

Here, $u = x+1$ and $a = \sqrt{2}$. The substitution $u = x+1$ gives $du = dx$.

Using the standard integral formula:

$I_2 = \log_e |(x+1) + \sqrt{(x+1)^2 + (\sqrt{2})^2}| + C_2$


Substitute back the original quadratic expression inside the square root:

$(x+1)^2 + (\sqrt{2})^2 = x^2 + 2x + 1 + 2 = x^2 + 2x + 3$

So, $I_2 = \log_e |x+1 + \sqrt{x^2 + 2x + 3}| + C_2$


Combining the results for $I_1$ and $I_2$ for the total integral $I = I_1 + I_2$:

$I = (\sqrt{x^2 + 2x + 3} + C_1) + (\log_e |x+1 + \sqrt{x^2 + 2x + 3}| + C_2)$

$I = \sqrt{x^2 + 2x + 3} + \log_e |x+1 + \sqrt{x^2 + 2x + 3}| + C$

where $C = C_1 + C_2$ is the constant of integration.


Therefore, the integral of the given function is:

$\int \frac{x + 2}{\sqrt{x^2 + 2x + 3}} dx = \sqrt{x^2 + 2x + 3} + \log_e |x+1 + \sqrt{x^2 + 2x + 3}| + C$

Question 22. $\frac{x + 3}{x^2 − 2x − 5}$

Answer:

Let the given integral be $I$.

$I = \int \frac{x + 3}{x^2 − 2x − 5} dx$


We write the numerator $x + 3$ in terms of the derivative of the denominator $x^2 - 2x - 5$.

The derivative of $x^2 - 2x - 5$ is $\frac{d}{dx}(x^2 - 2x - 5) = 2x - 2$.

We need to find constants $A$ and $B$ such that:

$x + 3 = A(2x - 2) + B$

$x + 3 = 2Ax - 2A + B$


Comparing the coefficients of $x$ on both sides:

$1 = 2A$

... (i)

From (i), $A = \frac{1}{2}$.


Comparing the constant terms on both sides:

$3 = -2A + B$

... (ii)

Substitute $A = \frac{1}{2}$ into (ii):

$3 = -2\left(\frac{1}{2}\right) + B$

$3 = -1 + B$

$B = 3 + 1 = 4$


So, we can write the numerator as $x + 3 = \frac{1}{2}(2x - 2) + 4$.

The integral becomes:

$I = \int \frac{\frac{1}{2}(2x - 2) + 4}{x^2 - 2x - 5} dx$

We split the integral into two parts:

$I = \frac{1}{2} \int \frac{2x - 2}{x^2 - 2x - 5} dx + 4 \int \frac{1}{x^2 - 2x - 5} dx$

$I = I_1 + I_2$ (say)


Let's evaluate the first integral: $I_1 = \frac{1}{2} \int \frac{2x - 2}{x^2 - 2x - 5} dx$.

Let $u = x^2 - 2x - 5$. Differentiating with respect to $x$, $du = (2x - 2) dx$.

$I_1 = \frac{1}{2} \int \frac{du}{u}$

$I_1 = \frac{1}{2} \log_e |u| + C_1$

$I_1 = \frac{1}{2} \log_e |x^2 - 2x - 5| + C_1$


Now, let's evaluate the second integral: $I_2 = 4 \int \frac{1}{x^2 - 2x - 5} dx$.

We complete the square for the denominator $x^2 - 2x - 5$:

$x^2 - 2x - 5 = (x^2 - 2x + 1) - 1 - 5$

$= (x - 1)^2 - 6$

$= (x - 1)^2 - (\sqrt{6})^2$


Substitute this back into $I_2$:

$I_2 = 4 \int \frac{1}{(x - 1)^2 - (\sqrt{6})^2} dx$

This is a standard integral of the form $\int \frac{1}{u^2 - a^2} du = \frac{1}{2a} \log_e \left| \frac{u-a}{u+a} \right| + C$.

Here, $u = x - 1$ and $a = \sqrt{6}$. If $u = x-1$, then $du = dx$.

Using the standard integral formula:

$I_2 = 4 \left( \frac{1}{2\sqrt{6}} \log_e \left| \frac{(x - 1) - \sqrt{6}}{(x - 1) + \sqrt{6}} \right| \right) + C_2$

$I_2 = \frac{4}{2\sqrt{6}} \log_e \left| \frac{x - 1 - \sqrt{6}}{x - 1 + \sqrt{6}} \right| + C_2$

$I_2 = \frac{2}{\sqrt{6}} \log_e \left| \frac{x - 1 - \sqrt{6}}{x - 1 + \sqrt{6}} \right| + C_2$

Rationalizing the coefficient:

$I_2 = \frac{2\sqrt{6}}{6} \log_e \left| \frac{x - 1 - \sqrt{6}}{x - 1 + \sqrt{6}} \right| + C_2$

$I_2 = \frac{\sqrt{6}}{3} \log_e \left| \frac{x - 1 - \sqrt{6}}{x - 1 + \sqrt{6}} \right| + C_2$


Combining the results for $I_1$ and $I_2$ for the total integral $I = I_1 + I_2$:

$I = \left(\frac{1}{2} \log_e |x^2 - 2x - 5| + C_1\right) + \left(\frac{\sqrt{6}}{3} \log_e \left| \frac{x - 1 - \sqrt{6}}{x - 1 + \sqrt{6}} \right| + C_2\right)$

$I = \frac{1}{2} \log_e |x^2 - 2x - 5| + \frac{\sqrt{6}}{3} \log_e \left| \frac{x - 1 - \sqrt{6}}{x - 1 + \sqrt{6}} \right| + C$

where $C = C_1 + C_2$ is the constant of integration.


Therefore, the integral of the given function is:

$\int \frac{x + 3}{x^2 − 2x − 5} dx = \frac{1}{2} \log_e |x^2 - 2x - 5| + \frac{\sqrt{6}}{3} \log_e \left| \frac{x - 1 - \sqrt{6}}{x - 1 + \sqrt{6}} \right| + C$

Question 23. $\frac{5x + 3}{\sqrt{x^2 + 4x + 10}}$

Answer:

Let the given integral be $I$.

$I = \int \frac{5x + 3}{\sqrt{x^2 + 4x + 10}} dx$


We write the numerator $5x + 3$ in terms of the derivative of the quadratic expression under the square root, $x^2 + 4x + 10$.

The derivative of $x^2 + 4x + 10$ is $\frac{d}{dx}(x^2 + 4x + 10) = 2x + 4$.

We need to find constants $A$ and $B$ such that:

$5x + 3 = A(2x + 4) + B$

$5x + 3 = 2Ax + 4A + B$


Comparing the coefficients of $x$ on both sides:

$5 = 2A$

... (i)

From (i), $A = \frac{5}{2}$.


Comparing the constant terms on both sides:

$3 = 4A + B$

... (ii)

Substitute $A = \frac{5}{2}$ into (ii):

$3 = 4\left(\frac{5}{2}\right) + B$

$3 = 10 + B$

$B = 3 - 10 = -7$


So, we can write the numerator as $5x + 3 = \frac{5}{2}(2x + 4) - 7$.

The integral becomes:

$I = \int \frac{\frac{5}{2}(2x + 4) - 7}{\sqrt{x^2 + 4x + 10}} dx$

We split the integral into two parts:

$I = \frac{5}{2} \int \frac{2x + 4}{\sqrt{x^2 + 4x + 10}} dx - 7 \int \frac{1}{\sqrt{x^2 + 4x + 10}} dx$

$I = I_1 + I_2$ (say)


Let's evaluate the first integral: $I_1 = \frac{5}{2} \int \frac{2x + 4}{\sqrt{x^2 + 4x + 10}} dx$.

Let $u = x^2 + 4x + 10$. Differentiating with respect to $x$, $du = (2x + 4) dx$.

$I_1 = \frac{5}{2} \int \frac{du}{\sqrt{u}} = \frac{5}{2} \int u^{-1/2} du$

$I_1 = \frac{5}{2} \left(\frac{u^{-1/2 + 1}}{-1/2 + 1}\right) + C_1 = \frac{5}{2} \left(\frac{u^{1/2}}{1/2}\right) + C_1$

$I_1 = \frac{5}{2} (2\sqrt{u}) + C_1 = 5\sqrt{u} + C_1$

Substitute back $u = x^2 + 4x + 10$:

$I_1 = 5\sqrt{x^2 + 4x + 10} + C_1$


Now, let's evaluate the second integral: $I_2 = - 7 \int \frac{1}{\sqrt{x^2 + 4x + 10}} dx$.

We complete the square for the denominator $x^2 + 4x + 10$:

$x^2 + 4x + 10 = (x^2 + 4x + 4) + 6$

$= (x + 2)^2 + (\sqrt{6})^2$


Substitute this back into $I_2$:

$I_2 = - 7 \int \frac{1}{\sqrt{(x + 2)^2 + (\sqrt{6})^2}} dx$

This is a standard integral of the form $\int \frac{1}{\sqrt{u^2 + a^2}} du = \log_e |u + \sqrt{u^2 + a^2}| + C$.

Here, $u = x + 2$ and $a = \sqrt{6}$. If $u = x+2$, then $du = dx$.

Using the standard integral formula:

$I_2 = - 7 \left( \log_e |(x + 2) + \sqrt{(x + 2)^2 + (\sqrt{6})^2}| \right) + C_2$


Substitute back the original quadratic expression inside the square root:

$(x + 2)^2 + (\sqrt{6})^2 = x^2 + 4x + 4 + 6 = x^2 + 4x + 10$

So, $I_2 = - 7 \log_e |x + 2 + \sqrt{x^2 + 4x + 10}| + C_2$


Combining the results for $I_1$ and $I_2$ for the total integral $I = I_1 + I_2$:

$I = (5\sqrt{x^2 + 4x + 10} + C_1) + (- 7 \log_e |x + 2 + \sqrt{x^2 + 4x + 10}| + C_2)$

$I = 5\sqrt{x^2 + 4x + 10} - 7 \log_e |x + 2 + \sqrt{x^2 + 4x + 10}| + C$

where $C = C_1 + C_2$ is the constant of integration.


Therefore, the integral of the given function is:

$\int \frac{5x + 3}{\sqrt{x^2 + 4x + 10}} dx = 5\sqrt{x^2 + 4x + 10} - 7 \log_e |x + 2 + \sqrt{x^2 + 4x + 10}| + C$

Choose the correct answer in Exercises 24 and 25.

Question 24. $\int \frac{dx}{x^2 + 2x + 2}$ equals

(A) x tan–1 (x + 1) + C

(B) tan–1 (x + 1) + C

(C) (x + 1) tan–1 x + C

(D) tan–1 x + C

Answer:

Let the given integral be $I$.

$I = \int \frac{1}{x^2 + 2x + 2} dx$


First, we complete the square for the quadratic expression in the denominator:

$x^2 + 2x + 2 = (x^2 + 2x + 1) + 1$

$= (x+1)^2 + 1^2$


Substitute this back into the integral:

$I = \int \frac{1}{(x+1)^2 + 1^2} dx$


Let us use the substitution method.

Let $u = x+1$.

Differentiating both sides with respect to $x$, we get:

$\frac{du}{dx} = 1$

So, $du = dx$.


Now, substitute $u$ and $dx$ into the integral:

$I = \int \frac{1}{u^2 + 1^2} du$


This is a standard integral of the form $\int \frac{1}{x^2 + a^2} dx = \frac{1}{a} \tan^{-1}\left(\frac{x}{a}\right) + C$.

Here, $x$ is $u$ and $a$ is 1.

So, $I = \frac{1}{1} \tan^{-1}\left(\frac{u}{1}\right) + C$

$I = \tan^{-1}(u) + C$


Substitute back $u = x+1$:

$I = \tan^{-1}(x+1) + C$


Comparing this result with the given options, we find that it matches option (B).

The correct answer is (B) $\tan^{-1} (x + 1) + C$.

Question 25. $\int \frac{dx}{\sqrt{9x − 4x^2}}$

(A) $\frac{1}{9} \sin^{-1} \left( \frac{9x − 8}{8} \right) + C$

(B) $\frac{1}{2} \sin^{-1} \left( \frac{8x − 9}{9} \right) + C$

(C) $\frac{1}{3} \sin^{-1} \left( \frac{9x − 8}{8} \right) + C$

(D) $\frac{1}{2} \sin^{-1} \left( \frac{9x − 8}{9} \right) + C$

Answer:

Let the given integral be $I$.

$I = \int \frac{dx}{\sqrt{9x − 4x^2}}$


First, we work with the expression under the square root, $9x - 4x^2$. We complete the square.

$9x - 4x^2 = -4(x^2 - \frac{9}{4}x)$

To complete the square for $x^2 - \frac{9}{4}x$, we add and subtract $\left(\frac{1}{2} \times -\frac{9}{4}\right)^2 = \left(-\frac{9}{8}\right)^2 = \frac{81}{64}$.

$-4\left(x^2 - \frac{9}{4}x + \frac{81}{64} - \frac{81}{64}\right)$

$= -4\left(\left(x - \frac{9}{8}\right)^2 - \frac{81}{64}\right)$

$= -4\left(x - \frac{9}{8}\right)^2 + 4 \times \frac{81}{64}$

$= \frac{81}{16} - 4\left(x - \frac{9}{8}\right)^2$

We can rewrite this as $\left(\frac{9}{4}\right)^2 - \left(2\left(x - \frac{9}{8}\right)\right)^2 = \left(\frac{9}{4}\right)^2 - \left(2x - \frac{18}{8}\right)^2 = \left(\frac{9}{4}\right)^2 - \left(2x - \frac{9}{4}\right)^2$.


Substitute this completed square form back into the integral:

$I = \int \frac{dx}{\sqrt{\left(\frac{9}{4}\right)^2 - \left(2x - \frac{9}{4}\right)^2}}$


This integral is in the form $\int \frac{1}{\sqrt{a^2 - u^2}} du$.

Let $u = 2x - \frac{9}{4}$.

Differentiating both sides with respect to $x$, we get:

$\frac{du}{dx} = 2$

So, $du = 2 dx$, which means $dx = \frac{1}{2} du$.

Also, let $a = \frac{9}{4}$.


Now, substitute $u$, $a$, and $dx$ into the integral:

$I = \int \frac{\frac{1}{2} du}{\sqrt{a^2 - u^2}}$

$I = \frac{1}{2} \int \frac{1}{\sqrt{a^2 - u^2}} du$


Using the standard integral formula $\int \frac{1}{\sqrt{a^2 - u^2}} du = \sin^{-1}\left(\frac{u}{a}\right) + C$, we get:

$I = \frac{1}{2} \sin^{-1}\left(\frac{u}{a}\right) + C$


Substitute back $u = 2x - \frac{9}{4}$ and $a = \frac{9}{4}$:

$I = \frac{1}{2} \sin^{-1}\left(\frac{2x - \frac{9}{4}}{\frac{9}{4}}\right) + C$

$I = \frac{1}{2} \sin^{-1}\left(\frac{\frac{8x - 9}{4}}{\frac{9}{4}}\right) + C$

$I = \frac{1}{2} \sin^{-1}\left(\frac{8x - 9}{9}\right) + C$


Comparing this result with the given options, we find that it matches option (B).

The correct answer is (B) $\frac{1}{2} \sin^{-1} \left( \frac{8x − 9}{9} \right) + C$



Example 11 to 16 (Before Exercise 7.5)

Example 11: Find $\int \frac{dx}{(x + 1)(x + 2)}$

Answer:

Given:

We are asked to find the integral $\int \frac{dx}{(x + 1)(x + 2)}$.


Solution:

The integrand is a rational function $\frac{1}{(x + 1)(x + 2)}$. Since the denominator is a product of distinct linear factors, we use the method of partial fraction decomposition.

We write the integrand in the form:

$\frac{1}{(x + 1)(x + 2)} = \frac{A}{x + 1} + \frac{B}{x + 2}$

... (1)

Multiplying both sides of equation (1) by $(x + 1)(x + 2)$, we get:

$1 = A(x + 2) + B(x + 1)$


To find the values of A and B, we can substitute convenient values for $x$ in the equation $1 = A(x + 2) + B(x + 1)$.

Setting $x = -1$:

$1 = A(-1 + 2) + B(-1 + 1)$

$1 = A(1) + B(0)$

$1 = A$

Thus, $\mathbf{A = 1}$.


Setting $x = -2$:

$1 = A(-2 + 2) + B(-2 + 1)$

$1 = A(0) + B(-1)$

$1 = -B$

$B = -1$

Thus, $\mathbf{B = -1}$.


Substituting the values of A and B back into equation (1), we get:

$\frac{1}{(x + 1)(x + 2)} = \frac{1}{x + 1} - \frac{1}{x + 2}$

Now, we can integrate the expression:

$\int \frac{dx}{(x + 1)(x + 2)} = \int \left( \frac{1}{x + 1} - \frac{1}{x + 2} \right) dx$

Using the linearity of the integral, we have:

$\int \left( \frac{1}{x + 1} - \frac{1}{x + 2} \right) dx = \int \frac{1}{x + 1} dx - \int \frac{1}{x + 2} dx$

We know that $\int \frac{1}{ax+b} dx = \frac{1}{a} \log_e |ax+b| + C$.

So, $\int \frac{1}{x + 1} dx = \log_e |x + 1|$ (with $a=1, b=1$)

and $\int \frac{1}{x + 2} dx = \log_e |x + 2|$ (with $a=1, b=2$)

(We add the constant of integration at the end).


Therefore, the integral is:

$\int \frac{dx}{(x + 1)(x + 2)} = \log_e |x + 1| - \log_e |x + 2| + C$

where $C$ is the constant of integration.


Using the logarithm property $\log_e a - \log_e b = \log_e \frac{a}{b}$, we can simplify the result:

$\int \frac{dx}{(x + 1)(x + 2)} = \log_e \left|\frac{x + 1}{x + 2}\right| + C$


Final Answer:

The integral is $\int \frac{dx}{(x + 1)(x + 2)} = \log_e \left|\frac{x + 1}{x + 2}\right| + C$.

Example 12: Find $\int \frac{x^2 + 1}{x^2 − 5x + 6} \;dx$

Answer:

Given:

We are asked to find the integral $\int \frac{x^2 + 1}{x^2 − 5x + 6} \;dx$.


Solution:

The integrand is a rational function where the degree of the numerator ($2$) is equal to the degree of the denominator ($2$). In such cases, we first perform polynomial long division.

Let's divide $x^2 + 1$ by $x^2 - 5x + 6$.

$\begin{array}{r} 1 \\ x^2-5x+6{\overline{\smash{\big)}\,x^2+0x+1}} \\ \underline{-~\phantom{(}(x^2-5x+6)\phantom{-b)}} \\ 0+5x-5\phantom{)} \end{array}$

So, we can write the integrand as:

$\frac{x^2 + 1}{x^2 − 5x + 6} = 1 + \frac{5x - 5}{x^2 − 5x + 6}$

... (1)


Now, we need to integrate the remainder term, which requires partial fraction decomposition for $\frac{5x - 5}{x^2 − 5x + 6}$.

First, factor the denominator: $x^2 - 5x + 6 = (x - 2)(x - 3)$.

So, we decompose $\frac{5x - 5}{(x - 2)(x - 3)}$:

$\frac{5x - 5}{(x - 2)(x - 3)} = \frac{A}{x - 2} + \frac{B}{x - 3}$

Multiplying both sides by $(x - 2)(x - 3)$, we get:

$5x - 5 = A(x - 3) + B(x - 2)$


To find A and B, substitute values for $x$:

Set $x = 2$:

$5(2) - 5 = A(2 - 3) + B(2 - 2)$

$10 - 5 = A(-1) + B(0)$

$5 = -A$

$A = -5$

Set $x = 3$:

$5(3) - 5 = A(3 - 3) + B(3 - 2)$

$15 - 5 = A(0) + B(1)$

$10 = B$

Thus, $\mathbf{A = -5}$ and $\mathbf{B = 10}$.


Substituting these values back into the partial fraction decomposition:

$\frac{5x - 5}{(x - 2)(x - 3)} = \frac{-5}{x - 2} + \frac{10}{x - 3}$

... (2)

Now, substitute the result of the long division (1) and partial fraction decomposition (2) back into the original integral:

$\int \frac{x^2 + 1}{x^2 − 5x + 6} \;dx = \int \left( 1 + \frac{-5}{x - 2} + \frac{10}{x - 3} \right) dx$

Using the linearity of the integral:

$\int \frac{x^2 + 1}{x^2 − 5x + 6} \;dx = \int 1 \;dx - 5 \int \frac{1}{x - 2} \;dx + 10 \int \frac{1}{x - 3} \;dx$

We know that $\int 1 \;dx = x$, $\int \frac{1}{x - a} \;dx = \log_e |x - a| + C$.

So, $\int 1 \;dx = x$

$\int \frac{1}{x - 2} \;dx = \log_e |x - 2|$

$\int \frac{1}{x - 3} \;dx = \log_e |x - 3|$

(We add the constant of integration at the end).


Combining these results:

$\int \frac{x^2 + 1}{x^2 − 5x + 6} \;dx = x - 5 \log_e |x - 2| + 10 \log_e |x - 3| + C$

where $C$ is the constant of integration.


Final Answer:

The integral is $\int \frac{x^2 + 1}{x^2 − 5x + 6} \;dx = x - 5 \log_e |x - 2| + 10 \log_e |x - 3| + C$.

Example 13: Find $\int \frac{3x − 2}{(x + 1)^2 (x + 3)} \;dx$

Answer:

Given:

We are asked to find the integral $\int \frac{3x − 2}{(x + 1)^2 (x + 3)} \;dx$.


Solution:

The integrand is a rational function with a repeated linear factor $(x+1)^2$ and a distinct linear factor $(x+3)$ in the denominator. We use the method of partial fraction decomposition.

We write the integrand in the form:

$\frac{3x - 2}{(x + 1)^2 (x + 3)} = \frac{A}{x + 1} + \frac{B}{(x + 1)^2} + \frac{C}{x + 3}$

... (1)

Multiplying both sides of equation (1) by $(x + 1)^2 (x + 3)$, we get:

$3x - 2 = A(x + 1)(x + 3) + B(x + 3) + C(x + 1)^2$

... (2)

We can find the values of A, B, and C by substituting convenient values for $x$ or by comparing coefficients.


Setting $x = -1$ in equation (2):

$3(-1) - 2 = A(-1 + 1)(-1 + 3) + B(-1 + 3) + C(-1 + 1)^2$

$-3 - 2 = A(0)(2) + B(2) + C(0)^2$

$-5 = 2B$

$B = -\frac{5}{2}$

Thus, $\mathbf{B = -\frac{5}{2}}$.


Setting $x = -3$ in equation (2):

$3(-3) - 2 = A(-3 + 1)(-3 + 3) + B(-3 + 3) + C(-3 + 1)^2$

$-9 - 2 = A(-2)(0) + B(0) + C(-2)^2$

$-11 = 4C$

$C = -\frac{11}{4}$

Thus, $\mathbf{C = -\frac{11}{4}}$.


To find A, we can compare the coefficients of $x^2$ in equation (2). Expanding the right side:

$3x - 2 = A(x^2 + 4x + 3) + B(x + 3) + C(x^2 + 2x + 1)$

$3x - 2 = Ax^2 + 4Ax + 3A + Bx + 3B + Cx^2 + 2Cx + C$

$3x - 2 = (A + C)x^2 + (4A + B + 2C)x + (3A + 3B + C)$

Comparing the coefficients of $x^2$ on both sides:

$0 = A + C$

Since $C = -\frac{11}{4}$, we have:

$0 = A - \frac{11}{4}$

$A = \frac{11}{4}$

Thus, $\mathbf{A = \frac{11}{4}}$.


Substituting the values of A, B, and C back into equation (1), we get:

$\frac{3x - 2}{(x + 1)^2 (x + 3)} = \frac{11/4}{x + 1} + \frac{-5/2}{(x + 1)^2} + \frac{-11/4}{x + 3}$

$\frac{3x - 2}{(x + 1)^2 (x + 3)} = \frac{11}{4(x + 1)} - \frac{5}{2(x + 1)^2} - \frac{11}{4(x + 3)}$

Now, we integrate each term:

$\int \frac{3x − 2}{(x + 1)^2 (x + 3)} \;dx = \int \left( \frac{11}{4(x + 1)} - \frac{5}{2(x + 1)^2} - \frac{11}{4(x + 3)} \right) dx$

Using the linearity of the integral:

$\int \frac{3x − 2}{(x + 1)^2 (x + 3)} \;dx = \frac{11}{4} \int \frac{1}{x + 1} \;dx - \frac{5}{2} \int (x + 1)^{-2} \;dx - \frac{11}{4} \int \frac{1}{x + 3} \;dx$

We know that $\int \frac{1}{u} du = \log_e |u|$ and $\int u^n du = \frac{u^{n+1}}{n+1}$ for $n \neq -1$.

$\int \frac{1}{x + 1} \;dx = \log_e |x + 1|$

$\int (x + 1)^{-2} \;dx = \frac{(x + 1)^{-1}}{-1} = -\frac{1}{x + 1}$

$\int \frac{1}{x + 3} \;dx = \log_e |x + 3|$

(We add the constant of integration at the end).


Combining these results:

$\int \frac{3x − 2}{(x + 1)^2 (x + 3)} \;dx = \frac{11}{4} \log_e |x + 1| - \frac{5}{2} \left( -\frac{1}{x + 1} \right) - \frac{11}{4} \log_e |x + 3| + C$

$\int \frac{3x − 2}{(x + 1)^2 (x + 3)} \;dx = \frac{11}{4} \log_e |x + 1| + \frac{5}{2(x + 1)} - \frac{11}{4} \log_e |x + 3| + C$


Using the logarithm property $\log_e a - \log_e b = \log_e \frac{a}{b}$, we can combine the logarithm terms:

$\int \frac{3x − 2}{(x + 1)^2 (x + 3)} \;dx = \frac{11}{4} (\log_e |x + 1| - \log_e |x + 3|) + \frac{5}{2(x + 1)} + C$

$\int \frac{3x − 2}{(x + 1)^2 (x + 3)} \;dx = \frac{11}{4} \log_e \left|\frac{x + 1}{x + 3}\right| + \frac{5}{2(x + 1)} + C$

where $C$ is the constant of integration.


Final Answer:

The integral is $\int \frac{3x − 2}{(x + 1)^2 (x + 3)} \;dx = \frac{11}{4} \log_e \left|\frac{x + 1}{x + 3}\right| + \frac{5}{2(x + 1)} + C$.

Example 14: Find $\int \frac{x^2}{(x^2 + 1) (x^2 + 4)} \;dx$

Answer:

Given:

We are asked to find the integral $\int \frac{x^2}{(x^2 + 1) (x^2 + 4)} \;dx$.


Solution:

The integrand is a rational function $\frac{x^2}{(x^2 + 1) (x^2 + 4)}$. The denominator consists of distinct irreducible quadratic factors. We use the method of partial fraction decomposition.

We write the integrand in the form:

$\frac{x^2}{(x^2 + 1) (x^2 + 4)} = \frac{Ax + B}{x^2 + 1} + \frac{Cx + D}{x^2 + 4}$

... (1)

Multiplying both sides of equation (1) by $(x^2 + 1)(x^2 + 4)$, we get:

$x^2 = (Ax + B)(x^2 + 4) + (Cx + D)(x^2 + 1)$

Expanding the right side:

$x^2 = (Ax^3 + 4Ax + Bx^2 + 4B) + (Cx^3 + Cx + Dx^2 + D)$

$x^2 = Ax^3 + Bx^2 + 4Ax + 4B + Cx^3 + Dx^2 + Cx + D$

Grouping terms by powers of $x$:

$x^2 = (A + C)x^3 + (B + D)x^2 + (4A + C)x + (4B + D)$


Comparing the coefficients of the powers of $x$ on both sides:

Coefficient of $x^3$: $A + C = 0$

Coefficient of $x^2$: $B + D = 1$

Coefficient of $x$: $4A + C = 0$

Constant term: $4B + D = 0$


From the equations for the coefficients of $x^3$ and $x$:
$A + C = 0$
$4A + C = 0$

Subtracting the first equation from the second gives $(4A + C) - (A + C) = 0 - 0$, which simplifies to $3A = 0$. Thus, $\mathbf{A = 0}$.

Substituting $A = 0$ into $A + C = 0$, we get $0 + C = 0$. Thus, $\mathbf{C = 0}$.


From the equations for the coefficients of $x^2$ and the constant term:
$B + D = 1$
$4B + D = 0$

Subtracting the first equation from the second gives $(4B + D) - (B + D) = 0 - 1$, which simplifies to $3B = -1$. Thus, $\mathbf{B = -\frac{1}{3}}$.

Substituting $B = -\frac{1}{3}$ into $B + D = 1$, we get $-\frac{1}{3} + D = 1$. Thus, $D = 1 + \frac{1}{3} = \frac{3}{3} + \frac{1}{3} = \frac{4}{3}$. Thus, $\mathbf{D = \frac{4}{3}}$.


Substituting the values of A, B, C, and D back into equation (1), we get:

$\frac{x^2}{(x^2 + 1) (x^2 + 4)} = \frac{0x - 1/3}{x^2 + 1} + \frac{0x + 4/3}{x^2 + 4}$

$\frac{x^2}{(x^2 + 1) (x^2 + 4)} = -\frac{1}{3(x^2 + 1)} + \frac{4}{3(x^2 + 4)}$

Now, we integrate each term:

$\int \frac{x^2}{(x^2 + 1) (x^2 + 4)} \;dx = \int \left( -\frac{1}{3(x^2 + 1)} + \frac{4}{3(x^2 + 4)} \right) dx$

Using the linearity of the integral:

$\int \frac{x^2}{(x^2 + 1) (x^2 + 4)} \;dx = -\frac{1}{3} \int \frac{1}{x^2 + 1} \;dx + \frac{4}{3} \int \frac{1}{x^2 + 4} \;dx$

We use the standard integral formula $\int \frac{1}{x^2 + a^2} \;dx = \frac{1}{a} \tan^{-1}\left(\frac{x}{a}\right) + C$.

For the first integral, $a^2 = 1$, so $a = 1$.

$\int \frac{1}{x^2 + 1} \;dx = \frac{1}{1} \tan^{-1}\left(\frac{x}{1}\right) = \tan^{-1}(x)$

For the second integral, $a^2 = 4$, so $a = 2$.

$\int \frac{1}{x^2 + 4} \;dx = \frac{1}{2} \tan^{-1}\left(\frac{x}{2}\right)$

(We add the constant of integration at the end).


Combining these results:

$\int \frac{x^2}{(x^2 + 1) (x^2 + 4)} \;dx = -\frac{1}{3} \tan^{-1}(x) + \frac{4}{3} \left( \frac{1}{2} \tan^{-1}\left(\frac{x}{2}\right) \right) + C$

$\int \frac{x^2}{(x^2 + 1) (x^2 + 4)} \;dx = -\frac{1}{3} \tan^{-1}(x) + \frac{4}{6} \tan^{-1}\left(\frac{x}{2}\right) + C$

$\int \frac{x^2}{(x^2 + 1) (x^2 + 4)} \;dx = -\frac{1}{3} \tan^{-1}(x) + \frac{2}{3} \tan^{-1}\left(\frac{x}{2}\right) + C$

where $C$ is the constant of integration.


Final Answer:

The integral is $\int \frac{x^2}{(x^2 + 1) (x^2 + 4)} \;dx = -\frac{1}{3} \tan^{-1}(x) + \frac{2}{3} \tan^{-1}\left(\frac{x}{2}\right) + C$.

Example 15: Find $\int \frac{(3\sin φ − 2) \cos φ}{5 − \cos^2 φ − 4 \sin φ} \;dφ$

Answer:

Given:

We are asked to find the integral $\int \frac{(3\sin φ − 2) \cos φ}{5 − \cos^2 φ − 4 \sin φ} \;dφ$.


Solution:

Let's use the substitution method. Let $t = \sin φ$.

Then, differentiating with respect to $φ$, we get $dt = \cos φ \;dφ$.

Also, we can express $\cos^2 φ$ in terms of $t$ using the identity $\sin^2 φ + \cos^2 φ = 1$.

$\cos^2 φ = 1 - \sin^2 φ = 1 - t^2$.

Now substitute $t = \sin φ$, $dt = \cos φ \;dφ$, and $\cos^2 φ = 1 - t^2$ into the integral:

The numerator becomes $(3\sin φ − 2) \cos φ \;dφ = (3t - 2) dt$.

The denominator becomes $5 - \cos^2 φ - 4 \sin φ = 5 - (1 - t^2) - 4t = 5 - 1 + t^2 - 4t = t^2 - 4t + 4$.

The integral transforms to:

$\int \frac{3t - 2}{t^2 - 4t + 4} \;dt$

... (1)

The denominator is a perfect square: $t^2 - 4t + 4 = (t - 2)^2$.

So the integral is $\int \frac{3t - 2}{(t - 2)^2} \;dt$.

We use partial fraction decomposition for the integrand $\frac{3t - 2}{(t - 2)^2}$.

We write it in the form:

$\frac{3t - 2}{(t - 2)^2} = \frac{A}{t - 2} + \frac{B}{(t - 2)^2}$

... (2)

Multiplying both sides of equation (2) by $(t - 2)^2$, we get:

$3t - 2 = A(t - 2) + B$

To find the values of A and B:

Set $t = 2$:

$3(2) - 2 = A(2 - 2) + B$

$6 - 2 = A(0) + B$

$4 = B$

Thus, $\mathbf{B = 4}$.


Comparing the coefficients of $t$ in the equation $3t - 2 = A(t - 2) + B = At - 2A + B$:

$3 = A$

Thus, $\mathbf{A = 3}$.


Substitute the values of A and B back into the partial fraction decomposition:

$\frac{3t - 2}{(t - 2)^2} = \frac{3}{t - 2} + \frac{4}{(t - 2)^2}$

Now, integrate the decomposed expression with respect to $t$ (from equation (1)):

$\int \frac{3t - 2}{(t - 2)^2} \;dt = \int \left( \frac{3}{t - 2} + \frac{4}{(t - 2)^2} \right) dt$

Using the linearity of the integral:

$\int \left( \frac{3}{t - 2} + \frac{4}{(t - 2)^2} \right) dt = \int \frac{3}{t - 2} dt + \int \frac{4}{(t - 2)^2} dt$

We evaluate each integral:

$\int \frac{3}{t - 2} dt = 3 \int \frac{1}{t - 2} dt = 3 \log_e |t - 2|$

$\int \frac{4}{(t - 2)^2} dt = 4 \int (t - 2)^{-2} dt = 4 \frac{(t - 2)^{-2 + 1}}{-2 + 1} + C' = 4 \frac{(t - 2)^{-1}}{-1} + C' = -\frac{4}{t - 2} + C'$

(We combine constants of integration at the end).


So, the integral with respect to $t$ is:

$3 \log_e |t - 2| - \frac{4}{t - 2} + C$

Now, substitute back $t = \sin φ$:

The integral is $3 \log_e |\sin φ - 2| - \frac{4}{\sin φ - 2} + C$.

Since $-1 \leq \sin φ \leq 1$ for real $φ$, the value of $\sin φ - 2$ is always negative (ranging from $-1 - 2 = -3$ to $1 - 2 = -1$).

Therefore, $|\sin φ - 2| = -(\sin φ - 2) = 2 - \sin φ$.

So, the integral can be written as:

$3 \log_e (2 - \sin φ) - \frac{4}{\sin φ - 2} + C$

Alternatively, we can write $-\frac{4}{\sin φ - 2} = \frac{-4}{- (2 - \sin φ)} = \frac{4}{2 - \sin φ}$.

So the result is also $3 \log_e (2 - \sin φ) + \frac{4}{2 - \sin φ} + C$.

where $C$ is the constant of integration.


Final Answer:

The integral is $\int \frac{(3\sin φ − 2) \cos φ}{5 − \cos^2 φ − 4 \sin φ} \;dφ = 3 \log_e (2 - \sin φ) + \frac{4}{2 - \sin φ} + C$.

Example 16: Find $\int \frac{x^2 + x + 1 \; dx}{(x + 2) (x^2 + 1)}$

Answer:

Given:

We are asked to find the integral $\int \frac{x^2 + x + 1}{(x + 2) (x^2 + 1)} \;dx$.


Solution:

The integrand is a rational function $\frac{x^2 + x + 1}{(x + 2) (x^2 + 1)}$. The denominator consists of a distinct linear factor $(x+2)$ and an irreducible quadratic factor $(x^2+1)$. We use the method of partial fraction decomposition.

We write the integrand in the form:

$\frac{x^2 + x + 1}{(x + 2) (x^2 + 1)} = \frac{A}{x + 2} + \frac{Bx + C}{x^2 + 1}$

... (1)

Multiplying both sides of equation (1) by $(x + 2)(x^2 + 1)$, we get:

$x^2 + x + 1 = A(x^2 + 1) + (Bx + C)(x + 2)$

... (2)

We can find the values of A, B, and C by substituting a convenient value for $x$ and by comparing coefficients.


Setting $x = -2$ in equation (2):

$(-2)^2 + (-2) + 1 = A((-2)^2 + 1) + (B(-2) + C)(-2 + 2)$

$4 - 2 + 1 = A(4 + 1) + (-2B + C)(0)$

$3 = 5A + 0$

$A = \frac{3}{5}$

Thus, $\mathbf{A = \frac{3}{5}}$.


Now, let's expand equation (2) and compare coefficients:

$x^2 + x + 1 = A(x^2 + 1) + (Bx + C)(x + 2)$

$x^2 + x + 1 = Ax^2 + A + Bx^2 + 2Bx + Cx + 2C$

$x^2 + x + 1 = (A + B)x^2 + (2B + C)x + (A + 2C)$

Comparing the coefficients of the powers of $x$ on both sides:

Coefficient of $x^2$: $1 = A + B$

Coefficient of $x$: $1 = 2B + C$

Constant term: $1 = A + 2C$


We have $A = \frac{3}{5}$. Substitute this into the first equation:

$1 = \frac{3}{5} + B$

$B = 1 - \frac{3}{5} = \frac{5 - 3}{5} = \frac{2}{5}$

Thus, $\mathbf{B = \frac{2}{5}}$.


Substitute $B = \frac{2}{5}$ into the second equation:

$1 = 2\left(\frac{2}{5}\right) + C$

$1 = \frac{4}{5} + C$

$C = 1 - \frac{4}{5} = \frac{5 - 4}{5} = \frac{1}{5}$

Thus, $\mathbf{C = \frac{1}{5}}$.


We can verify this with the third equation using $A = \frac{3}{5}$ and $C = \frac{1}{5}$:

$A + 2C = \frac{3}{5} + 2\left(\frac{1}{5}\right) = \frac{3}{5} + \frac{2}{5} = \frac{5}{5} = 1$

This matches the constant term on the left side, confirming our values for A, B, and C are correct.


Substituting the values of A, B, and C back into equation (1), we get:

$\frac{x^2 + x + 1}{(x + 2) (x^2 + 1)} = \frac{3/5}{x + 2} + \frac{(2/5)x + 1/5}{x^2 + 1}$

$\frac{x^2 + x + 1}{(x + 2) (x^2 + 1)} = \frac{3}{5(x + 2)} + \frac{2x + 1}{5(x^2 + 1)}$

Now, we integrate each term:

$\int \frac{x^2 + x + 1}{(x + 2) (x^2 + 1)} \;dx = \int \left( \frac{3}{5(x + 2)} + \frac{2x + 1}{5(x^2 + 1)} \right) dx$

Using the linearity of the integral:

$\int \frac{x^2 + x + 1}{(x + 2) (x^2 + 1)} \;dx = \frac{3}{5} \int \frac{1}{x + 2} \;dx + \frac{1}{5} \int \frac{2x + 1}{x^2 + 1} \;dx$

We evaluate each integral separately.

For the first integral:

$\int \frac{1}{x + 2} \;dx = \log_e |x + 2|$

For the second integral, we split it into two parts:

$\int \frac{2x + 1}{x^2 + 1} \;dx = \int \frac{2x}{x^2 + 1} \;dx + \int \frac{1}{x^2 + 1} \;dx$

For the first part $\int \frac{2x}{x^2 + 1} \;dx$, let $u = x^2 + 1$, then $du = 2x \;dx$.

$\int \frac{2x}{x^2 + 1} \;dx = \int \frac{1}{u} \;du = \log_e |u| = \log_e |x^2 + 1| = \log_e (x^2 + 1)$ (since $x^2+1 > 0$).

For the second part $\int \frac{1}{x^2 + 1} \;dx$, this is a standard integral:

$\int \frac{1}{x^2 + 1} \;dx = \tan^{-1}(x)$

So, $\int \frac{2x + 1}{x^2 + 1} \;dx = \log_e (x^2 + 1) + \tan^{-1}(x)$.

(We add the constant of integration at the end).


Combining all parts of the original integral:

$\int \frac{x^2 + x + 1}{(x + 2) (x^2 + 1)} \;dx = \frac{3}{5} \log_e |x + 2| + \frac{1}{5} (\log_e (x^2 + 1) + \tan^{-1}(x)) + C$

$\int \frac{x^2 + x + 1}{(x + 2) (x^2 + 1)} \;dx = \frac{3}{5} \log_e |x + 2| + \frac{1}{5} \log_e (x^2 + 1) + \frac{1}{5} \tan^{-1}(x) + C$

where $C$ is the constant of integration.


Final Answer:

The integral is $\int \frac{x^2 + x + 1}{(x + 2) (x^2 + 1)} \;dx = \frac{3}{5} \log_e |x + 2| + \frac{1}{5} \log_e (x^2 + 1) + \frac{1}{5} \tan^{-1}(x) + C$.



Exercise 7.5

Integrate the rational functions in Exercises 1 to 21.

Question 1. $\frac{x}{(x + 1) (x + 2)}$

Answer:

Given:

We are asked to find the integral $\int \frac{x}{(x + 1) (x + 2)} \;dx$.


Solution:

The integrand is a rational function $\frac{x}{(x + 1) (x + 2)}$. The denominator is a product of distinct linear factors. We use the method of partial fraction decomposition.

We write the integrand in the form:

$\frac{x}{(x + 1) (x + 2)} = \frac{A}{x + 1} + \frac{B}{x + 2}$

... (1)

Multiplying both sides of equation (1) by $(x + 1)(x + 2)$, we get:

$x = A(x + 2) + B(x + 1)$

... (2)


To find the values of A and B, we substitute convenient values for $x$ in equation (2).

Setting $x = -1$ in equation (2):

$-1 = A(-1 + 2) + B(-1 + 1)$

(Substituting $x=-1$)

$-1 = A(1) + B(0)$

$-1 = A$

Thus, $\mathbf{A = -1}$.


Setting $x = -2$ in equation (2):

$-2 = A(-2 + 2) + B(-2 + 1)$

(Substituting $x=-2$)

$-2 = A(0) + B(-1)$

$-2 = -B$

$B = 2$

Thus, $\mathbf{B = 2}$.


Substituting the values of A and B back into equation (1), we get:

$\frac{x}{(x + 1) (x + 2)} = \frac{-1}{x + 1} + \frac{2}{x + 2}$

Now, we can integrate the expression:

$\int \frac{x}{(x + 1) (x + 2)} \;dx = \int \left( \frac{-1}{x + 1} + \frac{2}{x + 2} \right) \;dx$

Using the linearity of the integral, we have:

$\int \left( \frac{-1}{x + 1} + \frac{2}{x + 2} \right) \;dx = - \int \frac{1}{x + 1} \;dx + 2 \int \frac{1}{x + 2} \;dx$

We know that $\int \frac{1}{u} \;du = \log_e |u| + C'$.

So, $\int \frac{1}{x + 1} \;dx = \log_e |x + 1|$

and $\int \frac{1}{x + 2} \;dx = \log_e |x + 2|$

(We add the constant of integration at the end).


Therefore, the integral is:

$\int \frac{x}{(x + 1) (x + 2)} \;dx = - \log_e |x + 1| + 2 \log_e |x + 2| + C$

where $C$ is the constant of integration.


Using the logarithm property $n \log_e a = \log_e a^n$ and $\log_e a - \log_e b = \log_e \frac{a}{b}$, we can simplify the result:

$\int \frac{x}{(x + 1) (x + 2)} \;dx = \log_e |(x + 2)^2| - \log_e |x + 1| + C$

$\int \frac{x}{(x + 1) (x + 2)} \;dx = \log_e \left|\frac{(x + 2)^2}{x + 1}\right| + C$


Final Answer:

The integral is $\int \frac{x}{(x + 1) (x + 2)} \;dx = 2 \log_e |x + 2| - \log_e |x + 1| + C$ or $\log_e \left|\frac{(x + 2)^2}{x + 1}\right| + C$.

Question 2. $\frac{1}{x^2 − 9}$

Answer:

Given:

We are asked to find the integral $\int \frac{1}{x^2 − 9} \;dx$.


Solution:

The integrand is a rational function $\frac{1}{x^2 − 9}$. The denominator can be factored as a difference of squares: $x^2 − 9 = x^2 - 3^2 = (x - 3)(x + 3)$.

So, the integral is $\int \frac{1}{(x - 3)(x + 3)} \;dx$.

The integrand $\frac{1}{(x - 3)(x + 3)}$ is a rational function with distinct linear factors in the denominator. We use the method of partial fraction decomposition.

We write the integrand in the form:

$\frac{1}{(x - 3)(x + 3)} = \frac{A}{x - 3} + \frac{B}{x + 3}$

... (1)

Multiplying both sides of equation (1) by $(x - 3)(x + 3)$, we get:

$1 = A(x + 3) + B(x - 3)$

... (2)


To find the values of A and B, we substitute convenient values for $x$ in equation (2).

Setting $x = 3$ in equation (2):

$1 = A(3 + 3) + B(3 - 3)$

(Substituting $x=3$)

$1 = A(6) + B(0)$

$1 = 6A$

$A = \frac{1}{6}$

Thus, $\mathbf{A = \frac{1}{6}}$.


Setting $x = -3$ in equation (2):

$1 = A(-3 + 3) + B(-3 - 3)$

(Substituting $x=-3$)

$1 = A(0) + B(-6)$

$1 = -6B$

$B = -\frac{1}{6}$

Thus, $\mathbf{B = -\frac{1}{6}}$.


Substituting the values of A and B back into equation (1), we get:

$\frac{1}{(x - 3)(x + 3)} = \frac{1/6}{x - 3} + \frac{-1/6}{x + 3}$

$\frac{1}{(x - 3)(x + 3)} = \frac{1}{6(x - 3)} - \frac{1}{6(x + 3)}$

Now, we can integrate the expression:

$\int \frac{1}{x^2 - 9} \;dx = \int \left( \frac{1}{6(x - 3)} - \frac{1}{6(x + 3)} \right) \;dx$

Using the linearity of the integral, we have:

$\int \frac{1}{x^2 - 9} \;dx = \frac{1}{6} \int \frac{1}{x - 3} \;dx - \frac{1}{6} \int \frac{1}{x + 3} \;dx$

We know that $\int \frac{1}{u} \;du = \log_e |u| + C'$.

So, $\int \frac{1}{x - 3} \;dx = \log_e |x - 3|$

and $\int \frac{1}{x + 3} \;dx = \log_e |x + 3|$

(We add the constant of integration at the end).


Therefore, the integral is:

$\int \frac{1}{x^2 - 9} \;dx = \frac{1}{6} \log_e |x - 3| - \frac{1}{6} \log_e |x + 3| + C$

where $C$ is the constant of integration.


Using the logarithm property $\log_e a - \log_e b = \log_e \frac{a}{b}$, we can simplify the result:

$\int \frac{1}{x^2 - 9} \;dx = \frac{1}{6} (\log_e |x - 3| - \log_e |x + 3|) + C$

$\int \frac{1}{x^2 - 9} \;dx = \frac{1}{6} \log_e \left|\frac{x - 3}{x + 3}\right| + C$


Alternate Solution using Standard Formula:

We can directly use the standard integration formula for $\int \frac{1}{x^2 - a^2} \;dx$.

The integral is $\int \frac{1}{x^2 - 9} \;dx$. Here, $a^2 = 9$, so $a = 3$.

The formula is $\int \frac{1}{x^2 - a^2} \;dx = \frac{1}{2a} \log_e \left|\frac{x - a}{x + a}\right| + C$.

Substituting $a = 3$:

$\int \frac{1}{x^2 - 3^2} \;dx = \frac{1}{2(3)} \log_e \left|\frac{x - 3}{x + 3}\right| + C$

$\int \frac{1}{x^2 - 9} \;dx = \frac{1}{6} \log_e \left|\frac{x - 3}{x + 3}\right| + C$

This matches the result obtained using partial fractions.


Final Answer:

The integral is $\int \frac{1}{x^2 − 9} \;dx = \frac{1}{6} \log_e \left|\frac{x - 3}{x + 3}\right| + C$.

Question 3. $\frac{3x − 1}{(x − 1) (x − 2) (x − 3)}$

Answer:

Given:

We are asked to find the integral $\int \frac{3x − 1}{(x − 1) (x − 2) (x − 3)} \;dx$.


Solution:

The integrand is a rational function $\frac{3x − 1}{(x − 1) (x − 2) (x − 3)}$. The denominator is a product of distinct linear factors. We use the method of partial fraction decomposition.

We write the integrand in the form:

$\frac{3x - 1}{(x - 1)(x - 2)(x - 3)} = \frac{A}{x - 1} + \frac{B}{x - 2} + \frac{C}{x - 3}$

... (1)

Multiplying both sides of equation (1) by $(x - 1)(x - 2)(x - 3)$, we get:

$3x - 1 = A(x - 2)(x - 3) + B(x - 1)(x - 3) + C(x - 1)(x - 2)$

... (2)


To find the values of A, B, and C, we substitute convenient values for $x$ in equation (2).

Setting $x = 1$ in equation (2):

$3(1) - 1 = A(1 - 2)(1 - 3) + B(1 - 1)(1 - 3) + C(1 - 1)(1 - 2)$

(Substituting $x=1$)

$2 = A(-1)(-2) + B(0)(-2) + C(0)(-1)$

$2 = 2A + 0 + 0$

$2 = 2A$

$A = 1$

Thus, $\mathbf{A = 1}$.


Setting $x = 2$ in equation (2):

$3(2) - 1 = A(2 - 2)(2 - 3) + B(2 - 1)(2 - 3) + C(2 - 1)(2 - 2)$

(Substituting $x=2$)

$6 - 1 = A(0)(-1) + B(1)(-1) + C(1)(0)$

$5 = 0 - B + 0$

$5 = -B$

$B = -5$

Thus, $\mathbf{B = -5}$.


Setting $x = 3$ in equation (2):

$3(3) - 1 = A(3 - 2)(3 - 3) + B(3 - 1)(3 - 3) + C(3 - 1)(3 - 2)$

(Substituting $x=3$)

$9 - 1 = A(1)(0) + B(2)(0) + C(2)(1)$

$8 = 0 + 0 + 2C$

$8 = 2C$

$C = 4$

Thus, $\mathbf{C = 4}$.


Substituting the values of A, B, and C back into equation (1), we get:

$\frac{3x - 1}{(x - 1)(x - 2)(x - 3)} = \frac{1}{x - 1} + \frac{-5}{x - 2} + \frac{4}{x - 3}$

$\frac{3x - 1}{(x - 1)(x - 2)(x - 3)} = \frac{1}{x - 1} - \frac{5}{x - 2} + \frac{4}{x - 3}$

Now, we can integrate the expression:

$\int \frac{3x − 1}{(x − 1) (x − 2) (x − 3)} \;dx = \int \left( \frac{1}{x - 1} - \frac{5}{x - 2} + \frac{4}{x - 3} \right) \;dx$

Using the linearity of the integral, we have:

$\int \frac{3x − 1}{(x − 1) (x − 2) (x − 3)} \;dx = \int \frac{1}{x - 1} \;dx - 5 \int \frac{1}{x - 2} \;dx + 4 \int \frac{1}{x - 3} \;dx$

We know that $\int \frac{1}{u} \;du = \log_e |u| + C'$.

So, $\int \frac{1}{x - 1} \;dx = \log_e |x - 1|$

$\int \frac{1}{x - 2} \;dx = \log_e |x - 2|$

$\int \frac{1}{x - 3} \;dx = \log_e |x - 3|$

(We add the constant of integration at the end).


Therefore, the integral is:

$\int \frac{3x − 1}{(x − 1) (x − 2) (x − 3)} \;dx = \log_e |x - 1| - 5 \log_e |x - 2| + 4 \log_e |x - 3| + C$

where $C$ is the constant of integration.


Using logarithm properties ($n \log_e a = \log_e a^n$ and $\log_e a + \log_e b = \log_e (ab)$), we can also write:

$\int \frac{3x − 1}{(x − 1) (x − 2) (x − 3)} \;dx = \log_e |x - 1| + \log_e |(x - 3)^4| - \log_e |(x - 2)^5| + C$

$\int \frac{3x − 1}{(x − 1) (x − 2) (x − 3)} \;dx = \log_e \left| \frac{(x - 1)(x - 3)^4}{(x - 2)^5} \right| + C$


Final Answer:

The integral is $\int \frac{3x − 1}{(x − 1) (x − 2) (x − 3)} \;dx = \log_e |x - 1| - 5 \log_e |x - 2| + 4 \log_e |x - 3| + C$.

Question 4. $\frac{x}{(x − 1) (x − 2) (x − 3)}$

Answer:

Given:

We are asked to find the integral $\int \frac{x}{(x − 1) (x − 2) (x − 3)} \;dx$.


Solution:

The integrand is a rational function $\frac{x}{(x − 1) (x − 2) (x − 3)}$. The denominator is a product of distinct linear factors. We use the method of partial fraction decomposition.

We write the integrand in the form:

$\frac{x}{(x - 1)(x - 2)(x - 3)} = \frac{A}{x - 1} + \frac{B}{x - 2} + \frac{C}{x - 3}$

... (1)

Multiplying both sides of equation (1) by $(x - 1)(x - 2)(x - 3)$, we get:

$x = A(x - 2)(x - 3) + B(x - 1)(x - 3) + C(x - 1)(x - 2)$

... (2)


To find the values of A, B, and C, we substitute convenient values for $x$ in equation (2).

Setting $x = 1$ in equation (2):

$1 = A(1 - 2)(1 - 3) + B(1 - 1)(1 - 3) + C(1 - 1)(1 - 2)$

(Substitute $x=1$)

$1 = A(-1)(-2) + B(0)(-2) + C(0)(-1)$

$1 = 2A + 0 + 0$

$2A = 1$

$A = \frac{1}{2}$

Thus, $\mathbf{A = \frac{1}{2}}$.


Setting $x = 2$ in equation (2):

$2 = A(2 - 2)(2 - 3) + B(2 - 1)(2 - 3) + C(2 - 1)(2 - 2)$

(Substitute $x=2$)

$2 = A(0)(-1) + B(1)(-1) + C(1)(0)$

$2 = 0 - B + 0$

$2 = -B$

$B = -2$

Thus, $\mathbf{B = -2}$.


Setting $x = 3$ in equation (2):

$3 = A(3 - 2)(3 - 3) + B(3 - 1)(3 - 3) + C(3 - 1)(3 - 2)$

(Substitute $x=3$)

$3 = A(1)(0) + B(2)(0) + C(2)(1)$

$3 = 0 + 0 + 2C$

$2C = 3$

$C = \frac{3}{2}$

Thus, $\mathbf{C = \frac{3}{2}}$.


Substituting the values of A, B, and C back into equation (1), we get:

$\frac{x}{(x - 1)(x - 2)(x - 3)} = \frac{1/2}{x - 1} + \frac{-2}{x - 2} + \frac{3/2}{x - 3}$

$\frac{x}{(x - 1)(x - 2)(x - 3)} = \frac{1}{2(x - 1)} - \frac{2}{x - 2} + \frac{3}{2(x - 3)}$

Now, we can integrate the expression:

$\int \frac{x}{(x - 1) (x - 2) (x - 3)} \;dx = \int \left( \frac{1}{2(x - 1)} - \frac{2}{x - 2} + \frac{3}{2(x - 3)} \right) \;dx$

Using the linearity of the integral, we have:

$\int \frac{x}{(x - 1) (x - 2) (x - 3)} \;dx = \frac{1}{2} \int \frac{1}{x - 1} \;dx - 2 \int \frac{1}{x - 2} \;dx + \frac{3}{2} \int \frac{1}{x - 3} \;dx$

We know that $\int \frac{1}{u} \;du = \log_e |u| + C'$.

So, $\int \frac{1}{x - 1} \;dx = \log_e |x - 1|$

$\int \frac{1}{x - 2} \;dx = \log_e |x - 2|$

$\int \frac{1}{x - 3} \;dx = \log_e |x - 3|$

(We add the constant of integration at the end).


Therefore, the integral is:

$\int \frac{x}{(x - 1) (x - 2) (x - 3)} \;dx = \frac{1}{2} \log_e |x - 1| - 2 \log_e |x - 2| + \frac{3}{2} \log_e |x - 3| + C$

where $C$ is the constant of integration.


Using logarithm properties ($n \log_e a = \log_e a^n$ and $\log_e a + \log_e b = \log_e (ab)$ and $\log_e a - \log_e b = \log_e (a/b)$), we can also write:

$\int \frac{x}{(x - 1) (x - 2) (x - 3)} \;dx = \log_e |x - 1|^{1/2} - \log_e |(x - 2)^2| + \log_e |x - 3|^{3/2} + C$

$\int \frac{x}{(x - 1) (x - 2) (x - 3)} \;dx = \log_e \left| \frac{(x - 1)^{1/2} (x - 3)^{3/2}}{(x - 2)^2} \right| + C$

$\int \frac{x}{(x - 1) (x - 2) (x - 3)} \;dx = \log_e \left| \sqrt{x - 1} \sqrt{(x - 3)^3} \middle/ (x - 2)^2 \right| + C$


Final Answer:

The integral is $\int \frac{x}{(x - 1) (x - 2) (x - 3)} \;dx = \frac{1}{2} \log_e |x - 1| - 2 \log_e |x - 2| + \frac{3}{2} \log_e |x - 3| + C$.

Question 5. $\frac{2x}{x^2 + 3x + 2}$

Answer:

Given:

We are asked to find the integral $\int \frac{2x}{x^2 + 3x + 2} \;dx$.


Solution:

The integrand is a rational function $\frac{2x}{x^2 + 3x + 2}$. First, we factor the denominator.

$x^2 + 3x + 2 = (x + 1)(x + 2)$

So the integral is $\int \frac{2x}{(x + 1) (x + 2)} \;dx$.

The denominator is a product of distinct linear factors. We use the method of partial fraction decomposition.

We write the integrand in the form:

$\frac{2x}{(x + 1) (x + 2)} = \frac{A}{x + 1} + \frac{B}{x + 2}$

... (1)

Multiplying both sides of equation (1) by $(x + 1)(x + 2)$, we get:

$2x = A(x + 2) + B(x + 1)$

... (2)


To find the values of A and B, we substitute convenient values for $x$ in equation (2).

Setting $x = -1$ in equation (2):

$2(-1) = A(-1 + 2) + B(-1 + 1)$

(Substitute $x=-1$)

$-2 = A(1) + B(0)$

$-2 = A$

Thus, $\mathbf{A = -2}$.


Setting $x = -2$ in equation (2):

$2(-2) = A(-2 + 2) + B(-2 + 1)$

(Substitute $x=-2$)

$-4 = A(0) + B(-1)$

$-4 = -B$

$B = 4$

Thus, $\mathbf{B = 4}$.


Substituting the values of A and B back into equation (1), we get:

$\frac{2x}{(x + 1) (x + 2)} = \frac{-2}{x + 1} + \frac{4}{x + 2}$

Now, we can integrate the expression:

$\int \frac{2x}{(x + 1) (x + 2)} \;dx = \int \left( \frac{-2}{x + 1} + \frac{4}{x + 2} \right) \;dx$

Using the linearity of the integral, we have:

$\int \frac{2x}{(x + 1) (x + 2)} \;dx = -2 \int \frac{1}{x + 1} \;dx + 4 \int \frac{1}{x + 2} \;dx$

We know that $\int \frac{1}{u} \;du = \log_e |u| + C'$.

So, $\int \frac{1}{x + 1} \;dx = \log_e |x + 1|$

and $\int \frac{1}{x + 2} \;dx = \log_e |x + 2|$

(We add the constant of integration at the end).


Therefore, the integral is:

$\int \frac{2x}{x^2 + 3x + 2} \;dx = -2 \log_e |x + 1| + 4 \log_e |x + 2| + C$

where $C$ is the constant of integration.


Using the logarithm property $n \log_e a = \log_e a^n$ and $\log_e a - \log_e b = \log_e \frac{a}{b}$, we can simplify the result:

$\int \frac{2x}{x^2 + 3x + 2} \;dx = \log_e |(x + 2)^4| - \log_e |(x + 1)^2| + C$

$\int \frac{2x}{x^2 + 3x + 2} \;dx = \log_e \left|\frac{(x + 2)^4}{(x + 1)^2}\right| + C$


Final Answer:

The integral is $\int \frac{2x}{x^2 + 3x + 2} \;dx = 4 \log_e |x + 2| - 2 \log_e |x + 1| + C$.

Question 6. $\frac{1 − x^2}{x (1 − 2x)}$

Answer:

Given:

We are asked to find the integral $\int \frac{1 − x^2}{x (1 − 2x)} \;dx$.


Solution:

The integrand is a rational function $\frac{1 - x^2}{x (1 - 2x)} = \frac{-x^2 + 1}{-2x^2 + x}$. The degree of the numerator (2) is equal to the degree of the denominator (2). This is an improper rational function, so we must perform polynomial long division first.

Divide $-x^2 + 1$ by $-2x^2 + x$:

$\begin{array}{r} 1/2 \\ -2x^2+x{\overline{\smash{\big)}\,-x^2+0x+1}} \\ \underline{-~\phantom{(}(-x^2+x/2)\phantom{-b)}} \\ 0-x/2+1\phantom{)} \end{array}$

So, $\frac{-x^2 + 1}{-2x^2 + x} = \frac{1}{2} + \frac{-x/2 + 1}{-2x^2 + x}$.

We can rewrite the remainder term as $\frac{1 - x/2}{-x(2x - 1)} = \frac{\frac{2-x}{2}}{x(2x - 1)} = \frac{2-x}{2x(2x - 1)}$.

Thus, the integrand can be written as:

$\frac{1 − x^2}{x (1 − 2x)} = \frac{1}{2} + \frac{2-x}{2x(1 - 2x)}$

... (1)


Now, we need to integrate $\int \left( \frac{1}{2} + \frac{2-x}{2x(1-2x)} \right) dx$. We can integrate the first term easily: $\int \frac{1}{2} dx = \frac{1}{2}x$.

For the second term, we use partial fraction decomposition for $\frac{2-x}{x(1-2x)}$. Note that we can pull out the constant factor $\frac{1}{2}$ from the integral later.

Let's decompose $\frac{2-x}{x(1-2x)}$. The denominator has distinct linear factors $x$ and $1-2x$.

$\frac{2-x}{x(1 - 2x)} = \frac{A}{x} + \frac{B}{1 - 2x}$

... (2)

Multiplying both sides of equation (2) by $x(1 - 2x)$, we get:

$2 - x = A(1 - 2x) + Bx$


To find the values of A and B, we substitute convenient values for $x$.

Setting $x = 0$:

$2 - 0 = A(1 - 2(0)) + B(0)$

(Substitute $x=0$)

$2 = A(1) + 0$

$A = 2$

Thus, $\mathbf{A = 2}$.


Setting $x = \frac{1}{2}$:

$2 - \frac{1}{2} = A\left(1 - 2\left(\frac{1}{2}\right)\right) + B\left(\frac{1}{2}\right)$

(Substitute $x=1/2$)

$\frac{3}{2} = A(1 - 1) + \frac{B}{2}$

$\frac{3}{2} = A(0) + \frac{B}{2}$

$\frac{3}{2} = \frac{B}{2}$

$B = 3$

Thus, $\mathbf{B = 3}$.


Substituting the values of A and B back into equation (2), we get:

$\frac{2-x}{x(1 - 2x)} = \frac{2}{x} + \frac{3}{1 - 2x}$

Now, substitute this back into the original integrand expression (1):

$\frac{1 − x^2}{x (1 − 2x)} = \frac{1}{2} + \frac{1}{2} \left( \frac{2}{x} + \frac{3}{1 - 2x} \right)$

$\frac{1 − x^2}{x (1 − 2x)} = \frac{1}{2} + \frac{1}{x} + \frac{3}{2(1 - 2x)}$

Now, we integrate each term:

$\int \frac{1 − x^2}{x (1 − 2x)} \;dx = \int \left( \frac{1}{2} + \frac{1}{x} + \frac{3}{2(1 - 2x)} \right) \;dx$

Using the linearity of the integral:

$\int \frac{1 − x^2}{x (1 − 2x)} \;dx = \int \frac{1}{2} \;dx + \int \frac{1}{x} \;dx + \frac{3}{2} \int \frac{1}{1 - 2x} \;dx$

We know that $\int c \;dx = cx$, $\int \frac{1}{x} \;dx = \log_e |x|$, and $\int \frac{1}{ax+b} \;dx = \frac{1}{a} \log_e |ax+b|$.

$\int \frac{1}{2} \;dx = \frac{1}{2}x$

$\int \frac{1}{x} \;dx = \log_e |x|$

$\int \frac{1}{1 - 2x} \;dx = \frac{1}{-2} \log_e |1 - 2x| = -\frac{1}{2} \log_e |1 - 2x|$

(We add the constant of integration at the end).


Combining these results:

$\int \frac{1 − x^2}{x (1 − 2x)} \;dx = \frac{1}{2}x + \log_e |x| + \frac{3}{2} \left( -\frac{1}{2} \log_e |1 - 2x| \right) + C$

$\int \frac{1 − x^2}{x (1 − 2x)} \;dx = \frac{1}{2}x + \log_e |x| - \frac{3}{4} \log_e |1 - 2x| + C$

where $C$ is the constant of integration.


Final Answer:

The integral is $\int \frac{1 − x^2}{x (1 − 2x)} \;dx = \frac{x}{2} + \log_e |x| - \frac{3}{4} \log_e |1 - 2x| + C$.

Question 7. $\frac{x}{(x^2 + 1) (x − 1)}$

Answer:

Given:

We are asked to find the integral $\int \frac{x}{(x^2 + 1) (x − 1)} \;dx$.


Solution:

The integrand is a rational function $\frac{x}{(x^2 + 1) (x − 1)}$. The denominator consists of a distinct linear factor $(x-1)$ and an irreducible quadratic factor $(x^2+1)$. We use the method of partial fraction decomposition.

We write the integrand in the form:

$\frac{x}{(x^2 + 1) (x − 1)} = \frac{A}{x - 1} + \frac{Bx + C}{x^2 + 1}$

... (1)

Multiplying both sides of equation (1) by $(x - 1)(x^2 + 1)$, we get:

$x = A(x^2 + 1) + (Bx + C)(x - 1)$

... (2)

We can find the values of A, B, and C by substituting a convenient value for $x$ and by comparing coefficients.


Setting $x = 1$ in equation (2):

$1 = A(1^2 + 1) + (B(1) + C)(1 - 1)$

(Substituting $x=1$)

$1 = A(2) + (B + C)(0)$

$1 = 2A + 0$

$2A = 1$

$A = \frac{1}{2}$

Thus, $\mathbf{A = \frac{1}{2}}$.


Now, let's expand equation (2) and compare coefficients:

$x = A(x^2 + 1) + (Bx + C)(x - 1)$

$x = Ax^2 + A + Bx^2 - Bx + Cx - C$

$x = (A + B)x^2 + (-B + C)x + (A - C)$

Comparing the coefficients of the powers of $x$ on both sides:

Coefficient of $x^2$: $0 = A + B$

Coefficient of $x$: $1 = -B + C$

Constant term: $0 = A - C$


We have $A = \frac{1}{2}$. Substitute this into the first equation:

$0 = \frac{1}{2} + B$

$B = -\frac{1}{2}$

Thus, $\mathbf{B = -\frac{1}{2}}$.


Substitute $B = -\frac{1}{2}$ into the second equation:

$1 = -(-\frac{1}{2}) + C$

$1 = \frac{1}{2} + C$

$C = 1 - \frac{1}{2} = \frac{1}{2}$

Thus, $\mathbf{C = \frac{1}{2}}$.


We can verify this with the third equation using $A = \frac{1}{2}$ and $C = \frac{1}{2}$:

$A - C = \frac{1}{2} - \frac{1}{2} = 0$

This matches the constant term on the left side, confirming our values for A, B, and C are correct.


Substituting the values of A, B, and C back into equation (1), we get:

$\frac{x}{(x^2 + 1) (x − 1)} = \frac{1/2}{x - 1} + \frac{(-1/2)x + 1/2}{x^2 + 1}$

$\frac{x}{(x^2 + 1) (x − 1)} = \frac{1}{2(x - 1)} + \frac{-x + 1}{2(x^2 + 1)}$

Now, we integrate each term:

$\int \frac{x}{(x^2 + 1) (x − 1)} \;dx = \int \left( \frac{1}{2(x - 1)} + \frac{1 - x}{2(x^2 + 1)} \right) dx$

Using the linearity of the integral:

$\int \frac{x}{(x^2 + 1) (x − 1)} \;dx = \frac{1}{2} \int \frac{1}{x - 1} \;dx + \frac{1}{2} \int \frac{1 - x}{x^2 + 1} \;dx$

We evaluate each integral separately.

For the first integral:

$\int \frac{1}{x - 1} \;dx = \log_e |x - 1|$

For the second integral, we split it into two parts:

$\int \frac{1 - x}{x^2 + 1} \;dx = \int \frac{1}{x^2 + 1} \;dx - \int \frac{x}{x^2 + 1} \;dx$

For the first part $\int \frac{1}{x^2 + 1} \;dx$, this is a standard integral:

$\int \frac{1}{x^2 + 1} \;dx = \tan^{-1}(x)$

For the second part $\int \frac{x}{x^2 + 1} \;dx$, let $u = x^2 + 1$, then $du = 2x \;dx$, so $x \;dx = \frac{1}{2} du$.

$\int \frac{x}{x^2 + 1} \;dx = \int \frac{1}{u} \frac{1}{2} \;du = \frac{1}{2} \int \frac{1}{u} \;du = \frac{1}{2} \log_e |u| = \frac{1}{2} \log_e |x^2 + 1| = \frac{1}{2} \log_e (x^2 + 1)$ (since $x^2+1 > 0$).

So, $\int \frac{1 - x}{x^2 + 1} \;dx = \tan^{-1}(x) - \frac{1}{2} \log_e (x^2 + 1)$.

(We add the constant of integration at the end).


Combining all parts of the original integral:

$\int \frac{x}{(x^2 + 1) (x − 1)} \;dx = \frac{1}{2} \log_e |x - 1| + \frac{1}{2} \left( \tan^{-1}(x) - \frac{1}{2} \log_e (x^2 + 1) \right) + C$

$\int \frac{x}{(x^2 + 1) (x − 1)} \;dx = \frac{1}{2} \log_e |x - 1| + \frac{1}{2} \tan^{-1}(x) - \frac{1}{4} \log_e (x^2 + 1) + C$

where $C$ is the constant of integration.


Final Answer:

The integral is $\int \frac{x}{(x^2 + 1) (x − 1)} \;dx = \frac{1}{2} \log_e |x - 1| - \frac{1}{4} \log_e (x^2 + 1) + \frac{1}{2} \tan^{-1}(x) + C$.

Question 8. $\frac{x}{(x − 1)^2 (x + 2)}$

Answer:

Given:

We are asked to find the integral $\int \frac{x}{(x − 1)^2 (x + 2)} \;dx$.


Solution:

The integrand is a rational function $\frac{x}{(x − 1)^2 (x + 2)}$. The denominator consists of a repeated linear factor $(x-1)^2$ and a distinct linear factor $(x+2)$. We use the method of partial fraction decomposition.

We write the integrand in the form:

$\frac{x}{(x - 1)^2 (x + 2)} = \frac{A}{x - 1} + \frac{B}{(x - 1)^2} + \frac{C}{x + 2}$

... (1)

Multiplying both sides of equation (1) by $(x - 1)^2 (x + 2)$, we get:

$x = A(x - 1)(x + 2) + B(x + 2) + C(x - 1)^2$

... (2)

We can find the values of A, B, and C by substituting convenient values for $x$ or by comparing coefficients.


Setting $x = 1$ in equation (2):

$1 = A(1 - 1)(1 + 2) + B(1 + 2) + C(1 - 1)^2$

(Substitute $x=1$)

$1 = A(0)(3) + B(3) + C(0)^2$

$1 = 3B$

$B = \frac{1}{3}$

Thus, $\mathbf{B = \frac{1}{3}}$.


Setting $x = -2$ in equation (2):

$-2 = A(-2 - 1)(-2 + 2) + B(-2 + 2) + C(-2 - 1)^2$

(Substitute $x=-2$)

$-2 = A(-3)(0) + B(0) + C(-3)^2$

$-2 = 9C$

$C = -\frac{2}{9}$

Thus, $\mathbf{C = -\frac{2}{9}}$.


To find A, compare the coefficient of $x^2$ in equation (2). Expanding the right side:

$x = A(x^2 + x - 2) + B(x + 2) + C(x^2 - 2x + 1)$

$x = Ax^2 + Ax - 2A + Bx + 2B + Cx^2 - 2Cx + C$

$x = (A + C)x^2 + (A + B - 2C)x + (-2A + 2B + C)$

Comparing the coefficients of $x^2$ on both sides:

$0 = A + C$

(Comparing $x^2$ coefficients)

Since $C = -\frac{2}{9}$, we have:

$0 = A - \frac{2}{9}$

$A = \frac{2}{9}$

Thus, $\mathbf{A = \frac{2}{9}}$.


Substituting the values of A, B, and C back into equation (1), we get:

$\frac{x}{(x - 1)^2 (x + 2)} = \frac{2/9}{x - 1} + \frac{1/3}{(x - 1)^2} + \frac{-2/9}{x + 2}$

$\frac{x}{(x - 1)^2 (x + 2)} = \frac{2}{9(x - 1)} + \frac{1}{3(x - 1)^2} - \frac{2}{9(x + 2)}$

Now, we integrate each term:

$\int \frac{x}{(x - 1)^2 (x + 2)} \;dx = \int \left( \frac{2}{9(x - 1)} + \frac{1}{3(x - 1)^2} - \frac{2}{9(x + 2)} \right) \;dx$

Using the linearity of the integral:

$\int \frac{x}{(x - 1)^2 (x + 2)} \;dx = \frac{2}{9} \int \frac{1}{x - 1} \;dx + \frac{1}{3} \int (x - 1)^{-2} \;dx - \frac{2}{9} \int \frac{1}{x + 2} \;dx$

We know that $\int \frac{1}{u} \;du = \log_e |u|$ and $\int u^n du = \frac{u^{n+1}}{n+1}$ for $n \neq -1$.

$\int \frac{1}{x - 1} \;dx = \log_e |x - 1|$

$\int (x - 1)^{-2} \;dx = \frac{(x - 1)^{-2 + 1}}{-2 + 1} = \frac{(x - 1)^{-1}}{-1} = -\frac{1}{x - 1}$

$\int \frac{1}{x + 2} \;dx = \log_e |x + 2|$

(We add the constant of integration at the end).


Combining these results:

$\int \frac{x}{(x - 1)^2 (x + 2)} \;dx = \frac{2}{9} \log_e |x - 1| + \frac{1}{3} \left(-\frac{1}{x - 1}\right) - \frac{2}{9} \log_e |x + 2| + C$

$\int \frac{x}{(x - 1)^2 (x + 2)} \;dx = \frac{2}{9} \log_e |x - 1| - \frac{1}{3(x - 1)} - \frac{2}{9} \log_e |x + 2| + C$

where $C$ is the constant of integration.


Using logarithm properties, we can also write:

$\int \frac{x}{(x - 1)^2 (x + 2)} \;dx = \frac{2}{9} (\log_e |x - 1| - \log_e |x + 2|) - \frac{1}{3(x - 1)} + C$

$\int \frac{x}{(x - 1)^2 (x + 2)} \;dx = \frac{2}{9} \log_e \left|\frac{x - 1}{x + 2}\right| - \frac{1}{3(x - 1)} + C$


Final Answer:

The integral is $\int \frac{x}{(x − 1)^2 (x + 2)} \;dx = \frac{2}{9} \log_e |x - 1| - \frac{1}{3(x - 1)} - \frac{2}{9} \log_e |x + 2| + C$.

Question 9. $\frac{3x + 5}{x^3 − x^2 − x + 1}$

Answer:

Given:

We are asked to find the integral $\int \frac{3x + 5}{x^3 − x^2 − x + 1} \;dx$.


Solution:

The integrand is a rational function. First, we need to factor the denominator: $x^3 − x^2 − x + 1$.

We can factor by grouping:

$x^3 − x^2 − x + 1 = x^2(x - 1) - 1(x - 1)$

$= (x^2 - 1)(x - 1)$

Using the difference of squares formula $a^2 - b^2 = (a - b)(a + b)$, we factor $x^2 - 1$:

$x^2 - 1 = (x - 1)(x + 1)$

So the denominator is $(x - 1)(x + 1)(x - 1) = (x - 1)^2 (x + 1)$.

The integral is $\int \frac{3x + 5}{(x - 1)^2 (x + 1)} \;dx$.

The denominator has a repeated linear factor $(x-1)^2$ and a distinct linear factor $(x+1)$. We use the method of partial fraction decomposition.

We write the integrand in the form:

$\frac{3x + 5}{(x - 1)^2 (x + 1)} = \frac{A}{x - 1} + \frac{B}{(x - 1)^2} + \frac{C}{x + 1}$

... (1)

Multiplying both sides of equation (1) by $(x - 1)^2 (x + 1)$, we get:

$3x + 5 = A(x - 1)(x + 1) + B(x + 1) + C(x - 1)^2$

... (2)

We can find the values of A, B, and C by substituting convenient values for $x$ or by comparing coefficients.


Setting $x = 1$ in equation (2):

$3(1) + 5 = A(1 - 1)(1 + 1) + B(1 + 1) + C(1 - 1)^2$

(Substitute $x=1$)

$8 = A(0)(2) + B(2) + C(0)^2$

$8 = 2B$

$B = 4$

Thus, $\mathbf{B = 4}$.


Setting $x = -1$ in equation (2):

$3(-1) + 5 = A(-1 - 1)(-1 + 1) + B(-1 + 1) + C(-1 - 1)^2$

(Substitute $x=-1$)

$2 = A(-2)(0) + B(0) + C(-2)^2$

$2 = 4C$

$C = \frac{2}{4} = \frac{1}{2}$

Thus, $\mathbf{C = \frac{1}{2}}$.


To find A, compare the coefficients of $x^2$ in equation (2). Expanding the right side:

$3x + 5 = A(x^2 - 1) + B(x + 1) + C(x^2 - 2x + 1)$

$3x + 5 = Ax^2 - A + Bx + B + Cx^2 - 2Cx + C$

$3x + 5 = (A + C)x^2 + (B - 2C)x + (-A + B + C)$

Comparing the coefficients of $x^2$ on both sides:

$0 = A + C$

(Comparing $x^2$ coefficients)

Since $C = \frac{1}{2}$, we have:

$0 = A + \frac{1}{2}$

$A = -\frac{1}{2}$

Thus, $\mathbf{A = -\frac{1}{2}}$.


Substituting the values of A, B, and C back into equation (1), we get:

$\frac{3x + 5}{(x - 1)^2 (x + 1)} = \frac{-1/2}{x - 1} + \frac{4}{(x - 1)^2} + \frac{1/2}{x + 1}$

$\frac{3x + 5}{(x - 1)^2 (x + 1)} = -\frac{1}{2(x - 1)} + \frac{4}{(x - 1)^2} + \frac{1}{2(x + 1)}$

Now, we integrate each term:

$\int \frac{3x + 5}{(x - 1)^2 (x + 1)} \;dx = \int \left( -\frac{1}{2(x - 1)} + \frac{4}{(x - 1)^2} + \frac{1}{2(x + 1)} \right) \;dx$

Using the linearity of the integral:

$\int \frac{3x + 5}{(x - 1)^2 (x + 1)} \;dx = -\frac{1}{2} \int \frac{1}{x - 1} \;dx + 4 \int (x - 1)^{-2} \;dx + \frac{1}{2} \int \frac{1}{x + 1} \;dx$

We know that $\int \frac{1}{u} \;du = \log_e |u|$ and $\int u^n du = \frac{u^{n+1}}{n+1}$ for $n \neq -1$.

$\int \frac{1}{x - 1} \;dx = \log_e |x - 1|$

$\int (x - 1)^{-2} \;dx = \frac{(x - 1)^{-2 + 1}}{-2 + 1} = \frac{(x - 1)^{-1}}{-1} = -\frac{1}{x - 1}$

$\int \frac{1}{x + 1} \;dx = \log_e |x + 1|$

(We add the constant of integration at the end).


Combining these results:

$\int \frac{3x + 5}{(x - 1)^2 (x + 1)} \;dx = -\frac{1}{2} \log_e |x - 1| + 4 \left(-\frac{1}{x - 1}\right) + \frac{1}{2} \log_e |x + 1| + C$

$\int \frac{3x + 5}{(x - 1)^2 (x + 1)} \;dx = -\frac{1}{2} \log_e |x - 1| - \frac{4}{x - 1} + \frac{1}{2} \log_e |x + 1| + C$

where $C$ is the constant of integration.


Using logarithm properties, we can rearrange the terms:

$\int \frac{3x + 5}{(x - 1)^2 (x + 1)} \;dx = \frac{1}{2} \log_e |x + 1| - \frac{1}{2} \log_e |x - 1| - \frac{4}{x - 1} + C$

$\int \frac{3x + 5}{(x - 1)^2 (x + 1)} \;dx = \frac{1}{2} (\log_e |x + 1| - \log_e |x - 1|) - \frac{4}{x - 1} + C$

$\int \frac{3x + 5}{(x - 1)^2 (x + 1)} \;dx = \frac{1}{2} \log_e \left|\frac{x + 1}{x - 1}\right| - \frac{4}{x - 1} + C$


Final Answer:

The integral is $\int \frac{3x + 5}{x^3 − x^2 − x + 1} \;dx = \frac{1}{2} \log_e \left|\frac{x + 1}{x - 1}\right| - \frac{4}{x - 1} + C$.

Question 10. $\frac{2x − 3}{(x^2 − 1) (2x + 3)}$

Answer:

Given:

We are asked to find the integral $\int \frac{2x − 3}{(x^2 − 1) (2x + 3)} \;dx$.


Solution:

The integrand is a rational function. First, we factor the denominator: $(x^2 − 1) (2x + 3)$.

The term $x^2 - 1$ is a difference of squares: $x^2 - 1 = (x - 1)(x + 1)$.

So the denominator is $(x - 1)(x + 1)(2x + 3)$.

The integral is $\int \frac{2x − 3}{(x - 1) (x + 1) (2x + 3)} \;dx$.

The denominator is a product of distinct linear factors. We use the method of partial fraction decomposition.

We write the integrand in the form:

$\frac{2x - 3}{(x - 1)(x + 1)(2x + 3)} = \frac{A}{x - 1} + \frac{B}{x + 1} + \frac{C}{2x + 3}$

... (1)

Multiplying both sides of equation (1) by $(x - 1)(x + 1)(2x + 3)$, we get:

$2x - 3 = A(x + 1)(2x + 3) + B(x - 1)(2x + 3) + C(x - 1)(x + 1)$

... (2)

We can find the values of A, B, and C by substituting convenient values for $x$ in equation (2).


Setting $x = 1$ in equation (2):

$2(1) - 3 = A(1 + 1)(2(1) + 3) + B(1 - 1)(2(1) + 3) + C(1 - 1)(1 + 1)$

(Substitute $x=1$)

$-1 = A(2)(5) + B(0)(5) + C(0)(2)$

$-1 = 10A + 0 + 0$

$10A = -1$

$A = -\frac{1}{10}$

Thus, $\mathbf{A = -\frac{1}{10}}$.


Setting $x = -1$ in equation (2):

$2(-1) - 3 = A(-1 + 1)(2(-1) + 3) + B(-1 - 1)(2(-1) + 3) + C(-1 - 1)(-1 + 1)$

(Substitute $x=-1$)

$-5 = A(0)(1) + B(-2)(1) + C(-2)(0)$

$-5 = 0 - 2B + 0$

$-2B = -5$

$B = \frac{5}{2}$

Thus, $\mathbf{B = \frac{5}{2}}$.


Setting $x = -\frac{3}{2}$ in equation (2):

$2\left(-\frac{3}{2}\right) - 3 = A\left(-\frac{3}{2} + 1\right)\left(2\left(-\frac{3}{2}\right) + 3\right) + B\left(-\frac{3}{2} - 1\right)\left(2\left(-\frac{3}{2}\right) + 3\right) + C\left(-\frac{3}{2} - 1\right)\left(-\frac{3}{2} + 1\right)$

(Substitute $x=-3/2$)

$-3 - 3 = A\left(-\frac{1}{2}\right)(-3 + 3) + B\left(-\frac{5}{2}\right)(-3 + 3) + C\left(-\frac{5}{2}\right)\left(-\frac{1}{2}\right)$

$-6 = A\left(-\frac{1}{2}\right)(0) + B\left(-\frac{5}{2}\right)(0) + C\left(\frac{5}{4}\right)$

$-6 = \frac{5}{4}C$

$C = -6 \times \frac{4}{5} = -\frac{24}{5}$

Thus, $\mathbf{C = -\frac{24}{5}}$.


Substituting the values of A, B, and C back into equation (1), we get:

$\frac{2x - 3}{(x - 1)(x + 1)(2x + 3)} = \frac{-1/10}{x - 1} + \frac{5/2}{x + 1} + \frac{-24/5}{2x + 3}$

$\frac{2x - 3}{(x - 1)(x + 1)(2x + 3)} = -\frac{1}{10(x - 1)} + \frac{5}{2(x + 1)} - \frac{24}{5(2x + 3)}$

Now, we can integrate the expression:

$\int \frac{2x − 3}{(x^2 − 1) (2x + 3)} \;dx = \int \left( -\frac{1}{10(x - 1)} + \frac{5}{2(x + 1)} - \frac{24}{5(2x + 3)} \right) \;dx$

Using the linearity of the integral:

$\int \frac{2x − 3}{(x^2 − 1) (2x + 3)} \;dx = -\frac{1}{10} \int \frac{1}{x - 1} \;dx + \frac{5}{2} \int \frac{1}{x + 1} \;dx - \frac{24}{5} \int \frac{1}{2x + 3} \;dx$

We know that $\int \frac{1}{u} \;du = \log_e |u|$ and $\int \frac{1}{ax+b} \;dx = \frac{1}{a} \log_e |ax+b|$.

$\int \frac{1}{x - 1} \;dx = \log_e |x - 1|$

$\int \frac{1}{x + 1} \;dx = \log_e |x + 1|$

$\int \frac{1}{2x + 3} \;dx = \frac{1}{2} \log_e |2x + 3|$

(We add the constant of integration at the end).


Combining these results:

$\int \frac{2x − 3}{(x^2 − 1) (2x + 3)} \;dx = -\frac{1}{10} \log_e |x - 1| + \frac{5}{2} \log_e |x + 1| - \frac{24}{5} \left(\frac{1}{2} \log_e |2x + 3|\right) + C$

$\int \frac{2x − 3}{(x^2 − 1) (2x + 3)} \;dx = -\frac{1}{10} \log_e |x - 1| + \frac{5}{2} \log_e |x + 1| - \frac{12}{5} \log_e |2x + 3| + C$

where $C$ is the constant of integration.


Final Answer:

The integral is $\int \frac{2x − 3}{(x^2 − 1) (2x + 3)} \;dx = -\frac{1}{10} \log_e |x - 1| + \frac{5}{2} \log_e |x + 1| - \frac{12}{5} \log_e |2x + 3| + C$.

Question 11. $\frac{5x}{(x + 1) (x^2 − 4)}$

Answer:

Given:

We are asked to find the integral $\int \frac{5x}{(x + 1) (x^2 − 4)} \;dx$.


Solution:

The integrand is a rational function. First, we need to factor the denominator: $(x + 1) (x^2 − 4)$.

The term $x^2 - 4$ is a difference of squares: $x^2 - 4 = x^2 - 2^2 = (x - 2)(x + 2)$.

So the denominator is $(x + 1)(x - 2)(x + 2)$.

The integral is $\int \frac{5x}{(x + 1) (x - 2) (x + 2)} \;dx$.

The denominator is a product of distinct linear factors. We use the method of partial fraction decomposition.

We write the integrand in the form:

$\frac{5x}{(x + 1) (x - 2) (x + 2)} = \frac{A}{x + 1} + \frac{B}{x - 2} + \frac{C}{x + 2}$

... (1)

Multiplying both sides of equation (1) by $(x + 1)(x - 2)(x + 2)$, we get:

$5x = A(x - 2)(x + 2) + B(x + 1)(x + 2) + C(x + 1)(x - 2)$

... (2)

We can find the values of A, B, and C by substituting convenient values for $x$ in equation (2).


Setting $x = -1$ in equation (2):

$5(-1) = A(-1 - 2)(-1 + 2) + B(-1 + 1)(-1 + 2) + C(-1 + 1)(-1 - 2)$

(Substitute $x=-1$)

$-5 = A(-3)(1) + B(0)(1) + C(0)(-3)$

$-5 = -3A + 0 + 0$

$-3A = -5$

$A = \frac{-5}{-3} = \frac{5}{3}$

Thus, $\mathbf{A = \frac{5}{3}}$.


Setting $x = 2$ in equation (2):

$5(2) = A(2 - 2)(2 + 2) + B(2 + 1)(2 + 2) + C(2 + 1)(2 - 2)$

(Substitute $x=2$)

$10 = A(0)(4) + B(3)(4) + C(3)(0)$

$10 = 0 + 12B + 0$

$12B = 10$

$B = \frac{10}{12} = \frac{5}{6}$

Thus, $\mathbf{B = \frac{5}{6}}$.


Setting $x = -2$ in equation (2):

$5(-2) = A(-2 - 2)(-2 + 2) + B(-2 + 1)(-2 + 2) + C(-2 + 1)(-2 - 2)$

(Substitute $x=-2$)

$-10 = A(-4)(0) + B(-1)(0) + C(-1)(-4)$

$-10 = 0 + 0 + 4C$

$4C = -10$

$C = \frac{-10}{4} = -\frac{5}{2}$

Thus, $\mathbf{C = -\frac{5}{2}}$.


Substituting the values of A, B, and C back into equation (1), we get:

$\frac{5x}{(x + 1) (x - 2) (x + 2)} = \frac{5/3}{x + 1} + \frac{5/6}{x - 2} + \frac{-5/2}{x + 2}$

$\frac{5x}{(x + 1) (x^2 - 4)} = \frac{5}{3(x + 1)} + \frac{5}{6(x - 2)} - \frac{5}{2(x + 2)}$

Now, we can integrate the expression:

$\int \frac{5x}{(x + 1) (x^2 − 4)} \;dx = \int \left( \frac{5}{3(x + 1)} + \frac{5}{6(x - 2)} - \frac{5}{2(x + 2)} \right) \;dx$

Using the linearity of the integral:

$\int \frac{5x}{(x + 1) (x^2 − 4)} \;dx = \frac{5}{3} \int \frac{1}{x + 1} \;dx + \frac{5}{6} \int \frac{1}{x - 2} \;dx - \frac{5}{2} \int \frac{1}{x + 2} \;dx$

We know that $\int \frac{1}{u} \;du = \log_e |u| + C'$.

So, $\int \frac{1}{x + 1} \;dx = \log_e |x + 1|$

$\int \frac{1}{x - 2} \;dx = \log_e |x - 2|$

$\int \frac{1}{x + 2} \;dx = \log_e |x + 2|$

(We add the constant of integration at the end).


Combining these results:

$\int \frac{5x}{(x + 1) (x^2 − 4)} \;dx = \frac{5}{3} \log_e |x + 1| + \frac{5}{6} \log_e |x - 2| - \frac{5}{2} \log_e |x + 2| + C$

where $C$ is the constant of integration.


Final Answer:

The integral is $\int \frac{5x}{(x + 1) (x^2 − 4)} \;dx = \frac{5}{3} \log_e |x + 1| + \frac{5}{6} \log_e |x - 2| - \frac{5}{2} \log_e |x + 2| + C$.

Question 12. $\frac{x^3 + x + 1}{x^2 − 1}$

Answer:

Given:

We are asked to find the integral $\int \frac{x^3 + x + 1}{x^2 − 1} \;dx$.


Solution:

The integrand is a rational function where the degree of the numerator (3) is greater than the degree of the denominator (2). This is an improper rational function, so we first perform polynomial long division.

Divide $x^3 + x + 1$ by $x^2 - 1$:

$\begin{array}{r} x\phantom{x^2-1)} \\ x^2-1{\overline{\smash{\big)}\,x^3+0x^2+x+1}} \\ \underline{-~\phantom{()}(x^3-x)\phantom{+1)}} \\ 0+2x+1\phantom{)} \end{array}$

From the long division, we can write the integrand as the sum of the quotient and the remainder over the divisor:

$\frac{x^3 + x + 1}{x^2 − 1} = x + \frac{2x + 1}{x^2 - 1}$

... (1)


Now, we need to integrate $x + \frac{2x + 1}{x^2 - 1}$. The integral of $x$ is $\int x \;dx = \frac{x^2}{2}$.

For the term $\frac{2x + 1}{x^2 - 1}$, we factor the denominator: $x^2 - 1 = (x - 1)(x + 1)$.

So we have the term $\frac{2x + 1}{(x - 1)(x + 1)}$. We use partial fraction decomposition for this proper rational function.

We write it in the form:

$\frac{2x + 1}{(x - 1)(x + 1)} = \frac{A}{x - 1} + \frac{B}{x + 1}$

... (2)

Multiplying both sides of equation (2) by $(x - 1)(x + 1)$, we get:

$2x + 1 = A(x + 1) + B(x - 1)$


To find the values of A and B, we substitute convenient values for $x$.

Setting $x = 1$:

$2(1) + 1 = A(1 + 1) + B(1 - 1)$

(Substituting $x=1$)

$3 = A(2) + B(0)$

$3 = 2A$

$A = \frac{3}{2}$

Thus, $\mathbf{A = \frac{3}{2}}$.


Setting $x = -1$:

$2(-1) + 1 = A(-1 + 1) + B(-1 - 1)$

(Substituting $x=-1$)

$-1 = A(0) + B(-2)$

$-1 = -2B$

$B = \frac{-1}{-2} = \frac{1}{2}$

Thus, $\mathbf{B = \frac{1}{2}}$.


Substituting the values of A and B back into equation (2), we get:

$\frac{2x + 1}{(x - 1)(x + 1)} = \frac{3/2}{x - 1} + \frac{1/2}{x + 1}$

$\frac{2x + 1}{x^2 - 1} = \frac{3}{2(x - 1)} + \frac{1}{2(x + 1)}$

Now, substitute this back into the integral expression from equation (1):

$\int \frac{x^3 + x + 1}{x^2 − 1} \;dx = \int \left( x + \frac{3}{2(x - 1)} + \frac{1}{2(x + 1)} \right) \;dx$

Using the linearity of the integral:

$\int \frac{x^3 + x + 1}{x^2 − 1} \;dx = \int x \;dx + \frac{3}{2} \int \frac{1}{x - 1} \;dx + \frac{1}{2} \int \frac{1}{x + 1} \;dx$

We evaluate each integral:

$\int x \;dx = \frac{x^2}{2}$

$\int \frac{1}{x - 1} \;dx = \log_e |x - 1|$

$\int \frac{1}{x + 1} \;dx = \log_e |x + 1|$

(We add the constant of integration at the end).


Combining these results:

$\int \frac{x^3 + x + 1}{x^2 − 1} \;dx = \frac{x^2}{2} + \frac{3}{2} \log_e |x - 1| + \frac{1}{2} \log_e |x + 1| + C$

where $C$ is the constant of integration.


Using logarithm properties, we can also write:

$\int \frac{x^3 + x + 1}{x^2 − 1} \;dx = \frac{x^2}{2} + \frac{1}{2} (3 \log_e |x - 1| + \log_e |x + 1|) + C$

$\int \frac{x^3 + x + 1}{x^2 − 1} \;dx = \frac{x^2}{2} + \frac{1}{2} (\log_e |(x - 1)^3| + \log_e |x + 1|) + C$

$\int \frac{x^3 + x + 1}{x^2 − 1} \;dx = \frac{x^2}{2} + \frac{1}{2} \log_e \left|(x - 1)^3 (x + 1)\right| + C$


Final Answer:

The integral is $\int \frac{x^3 + x + 1}{x^2 − 1} \;dx = \frac{x^2}{2} + \frac{3}{2} \log_e |x - 1| + \frac{1}{2} \log_e |x + 1| + C$.

Question 13. $\frac{2}{(1 − x) (1 + x^2)}$

Answer:

Given:

We are asked to find the integral $\int \frac{2}{(1 − x) (1 + x^2)} \;dx$.


Solution:

The integrand is a rational function $\frac{2}{(1 − x) (1 + x^2)}$. The denominator consists of a distinct linear factor $(1-x)$ and an irreducible quadratic factor $(1+x^2)$. We use the method of partial fraction decomposition.

We write the integrand in the form:

$\frac{2}{(1 - x) (1 + x^2)} = \frac{A}{1 - x} + \frac{Bx + C}{1 + x^2}$

... (1)

Multiplying both sides of equation (1) by $(1 - x)(1 + x^2)$, we get:

$2 = A(1 + x^2) + (Bx + C)(1 - x)$

... (2)

We can find the values of A, B, and C by substituting a convenient value for $x$ and by comparing coefficients.


Setting $x = 1$ in equation (2):

$2 = A(1 + 1^2) + (B(1) + C)(1 - 1)$

(Substituting $x=1$)

$2 = A(2) + (B + C)(0)$

$2 = 2A$

$A = 1$

Thus, $\mathbf{A = 1}$.


Now, let's expand equation (2) and compare coefficients:

$2 = A(1 + x^2) + (Bx + C)(1 - x)$

$2 = A + Ax^2 + Bx - Bx^2 + C - Cx$

$2 = (A - B)x^2 + (B - C)x + (A + C)$

Comparing the coefficients of the powers of $x$ on both sides:

Coefficient of $x^2$: $0 = A - B$

Coefficient of $x$: $0 = B - C$

Constant term: $2 = A + C$


We have $A = 1$. Substitute this into the first equation:

$0 = 1 - B$

$B = 1$

Thus, $\mathbf{B = 1}$.


Substitute $B = 1$ into the second equation:

$0 = 1 - C$

$C = 1$

Thus, $\mathbf{C = 1}$.


We can verify this with the third equation using $A = 1$ and $C = 1$:

$A + C = 1 + 1 = 2$

This matches the constant term on the left side, confirming our values for A, B, and C are correct.


Substituting the values of A, B, and C back into equation (1), we get:

$\frac{2}{(1 - x) (1 + x^2)} = \frac{1}{1 - x} + \frac{1x + 1}{1 + x^2}$

$\frac{2}{(1 - x) (1 + x^2)} = \frac{1}{1 - x} + \frac{x + 1}{x^2 + 1}$

Now, we integrate each term:

$\int \frac{2}{(1 - x) (1 + x^2)} \;dx = \int \left( \frac{1}{1 - x} + \frac{x + 1}{x^2 + 1} \right) dx$

Using the linearity of the integral:

$\int \frac{2}{(1 - x) (1 + x^2)} \;dx = \int \frac{1}{1 - x} \;dx + \int \frac{x + 1}{x^2 + 1} \;dx$

We evaluate each integral separately.

For the first integral, let $u = 1 - x$, then $du = -dx$:

$\int \frac{1}{1 - x} \;dx = \int \frac{1}{u} (-du) = - \int \frac{1}{u} \;du = -\log_e |u| = -\log_e |1 - x|$.

For the second integral, we split it into two parts:

$\int \frac{x + 1}{x^2 + 1} \;dx = \int \frac{x}{x^2 + 1} \;dx + \int \frac{1}{x^2 + 1} \;dx$

For the first part $\int \frac{x}{x^2 + 1} \;dx$, let $v = x^2 + 1$, then $dv = 2x \;dx$:

$\int \frac{x}{x^2 + 1} \;dx = \int \frac{1}{v} \frac{1}{2} \;dv = \frac{1}{2} \int \frac{1}{v} \;dv = \frac{1}{2} \log_e |v| = \frac{1}{2} \log_e |x^2 + 1| = \frac{1}{2} \log_e (x^2 + 1)$ (since $x^2+1 > 0$).

For the second part $\int \frac{1}{x^2 + 1} \;dx$, this is a standard integral:

$\int \frac{1}{x^2 + 1} \;dx = \tan^{-1}(x)$

So, $\int \frac{x + 1}{x^2 + 1} \;dx = \frac{1}{2} \log_e (x^2 + 1) + \tan^{-1}(x)$.

(We add the constant of integration at the end).


Combining all parts of the original integral:

$\int \frac{2}{(1 - x) (1 + x^2)} \;dx = -\log_e |1 - x| + \frac{1}{2} \log_e (x^2 + 1) + \tan^{-1}(x) + C$

where $C$ is the constant of integration.


Final Answer:

The integral is $\int \frac{2}{(1 − x) (1 + x^2)} \;dx = -\log_e |1 - x| + \frac{1}{2} \log_e (x^2 + 1) + \tan^{-1}(x) + C$.

Question 14. $\frac{3x − 1}{(x + 2)^2}$

Answer:

Given:

We are asked to find the integral $\int \frac{3x − 1}{(x + 2)^2} \;dx$.


Solution:

The integrand is a rational function $\frac{3x − 1}{(x + 2)^2}$. The denominator has a repeated linear factor $(x+2)^2$. We use the method of partial fraction decomposition.

We write the integrand in the form:

$\frac{3x - 1}{(x + 2)^2} = \frac{A}{x + 2} + \frac{B}{(x + 2)^2}$

... (1)

Multiplying both sides of equation (1) by $(x + 2)^2$, we get:

$3x - 1 = A(x + 2) + B$


To find the values of A and B, we substitute a convenient value for $x$ or compare coefficients.

Setting $x = -2$:

$3(-2) - 1 = A(-2 + 2) + B$

(Substituting $x=-2$)

$-6 - 1 = A(0) + B$

$-7 = B$

Thus, $\mathbf{B = -7}$.


Compare the coefficients of $x$ in the equation $3x - 1 = A(x + 2) + B = Ax + 2A + B$:

$3 = A$

(Comparing coefficient of $x$)

Thus, $\mathbf{A = 3}$.


Substituting the values of A and B back into equation (1), we get:

$\frac{3x - 1}{(x + 2)^2} = \frac{3}{x + 2} + \frac{-7}{(x + 2)^2}$

$\frac{3x - 1}{(x + 2)^2} = \frac{3}{x + 2} - \frac{7}{(x + 2)^2}$

Now, we can integrate the expression:

$\int \frac{3x − 1}{(x + 2)^2} \;dx = \int \left( \frac{3}{x + 2} - \frac{7}{(x + 2)^2} \right) \;dx$

Using the linearity of the integral:

$\int \frac{3x − 1}{(x + 2)^2} \;dx = 3 \int \frac{1}{x + 2} \;dx - 7 \int (x + 2)^{-2} \;dx$

We know that $\int \frac{1}{u} \;du = \log_e |u|$ and $\int u^n du = \frac{u^{n+1}}{n+1}$ for $n \neq -1$.

$\int \frac{1}{x + 2} \;dx = \log_e |x + 2|$

$\int (x + 2)^{-2} \;dx = \frac{(x + 2)^{-2 + 1}}{-2 + 1} = \frac{(x + 2)^{-1}}{-1} = -\frac{1}{x + 2}$

(We add the constant of integration at the end).


Combining these results:

$\int \frac{3x − 1}{(x + 2)^2} \;dx = 3 \log_e |x + 2| - 7 \left(-\frac{1}{x + 2}\right) + C$

$\int \frac{3x − 1}{(x + 2)^2} \;dx = 3 \log_e |x + 2| + \frac{7}{x + 2} + C$

where $C$ is the constant of integration.


Final Answer:

The integral is $\int \frac{3x − 1}{(x + 2)^2} \;dx = 3 \log_e |x + 2| + \frac{7}{x + 2} + C$.

Question 15. $\frac{1}{x^4 − 1}$

Answer:

Given:

We are asked to find the integral $\int \frac{1}{x^4 − 1} \;dx$.


Solution:

The integrand is a rational function $\frac{1}{x^4 − 1}$. First, we need to factor the denominator $x^4 - 1$.

We can factor it as a difference of squares: $x^4 - 1 = (x^2)^2 - 1^2 = (x^2 - 1)(x^2 + 1)$.

The term $x^2 - 1$ is also a difference of squares: $x^2 - 1 = (x - 1)(x + 1)$.

So, the complete factorization of the denominator is $x^4 - 1 = (x - 1)(x + 1)(x^2 + 1)$.

The integral is $\int \frac{1}{(x - 1)(x + 1)(x^2 + 1)} \;dx$.

The denominator consists of distinct linear factors $(x-1)$ and $(x+1)$, and an irreducible quadratic factor $(x^2+1)$. We use the method of partial fraction decomposition.

We write the integrand in the form:

$\frac{1}{(x - 1)(x + 1)(x^2 + 1)} = \frac{A}{x - 1} + \frac{B}{x + 1} + \frac{Cx + D}{x^2 + 1}$

... (1)

Multiplying both sides of equation (1) by $(x - 1)(x + 1)(x^2 + 1)$, we get:

$1 = A(x + 1)(x^2 + 1) + B(x - 1)(x^2 + 1) + (Cx + D)(x - 1)(x + 1)$

... (2)

We can find the values of A, B, C, and D by substituting convenient values for $x$ or by comparing coefficients.


Setting $x = 1$ in equation (2):

$1 = A(1 + 1)(1^2 + 1) + B(1 - 1)(1^2 + 1) + (C(1) + D)(1 - 1)(1 + 1)$

(Substitute $x=1$)

$1 = A(2)(2) + B(0)(2) + (C + D)(0)(2)$

$1 = 4A + 0 + 0$

$4A = 1$

$A = \frac{1}{4}$

Thus, $\mathbf{A = \frac{1}{4}}$.


Setting $x = -1$ in equation (2):

$1 = A(-1 + 1)((-1)^2 + 1) + B(-1 - 1)((-1)^2 + 1) + (C(-1) + D)(-1 - 1)(-1 + 1)$

(Substitute $x=-1$)

$1 = A(0)(2) + B(-2)(2) + (-C + D)(-2)(0)$

$1 = 0 - 4B + 0$

$-4B = 1$

$B = -\frac{1}{4}$

Thus, $\mathbf{B = -\frac{1}{4}}$.


Now, let's expand equation (2) and compare coefficients:

$1 = A(x^3 + x^2 + x + 1) + B(x^3 - x^2 + x - 1) + (Cx + D)(x^2 - 1)$

$1 = Ax^3 + Ax^2 + Ax + A + Bx^3 - Bx^2 + Bx - B + Cx^3 - Cx + Dx^2 - D$

$1 = (A + B + C)x^3 + (A - B + D)x^2 + (A + B - C)x + (A - B - D)$

Comparing the coefficients of the powers of $x$ on both sides:

Coefficient of $x^3$: $0 = A + B + C$

Coefficient of $x^2$: $0 = A - B + D$

Coefficient of $x$: $0 = A + B - C$

Constant term: $1 = A - B - D$


We have $A = \frac{1}{4}$ and $B = -\frac{1}{4}$. Substitute these into the equation for the coefficient of $x^3$:

$0 = \frac{1}{4} + \left(-\frac{1}{4}\right) + C$

$0 = 0 + C$

$C = 0$

Thus, $\mathbf{C = 0}$.


Now substitute $A = \frac{1}{4}$ and $B = -\frac{1}{4}$ into the equation for the coefficient of $x^2$:

$0 = \frac{1}{4} - \left(-\frac{1}{4}\right) + D$

$0 = \frac{1}{4} + \frac{1}{4} + D$

$0 = \frac{2}{4} + D$

$0 = \frac{1}{2} + D$

$D = -\frac{1}{2}$

Thus, $\mathbf{D = -\frac{1}{2}}$.


We can verify these values with the constant term equation using $A = \frac{1}{4}$, $B = -\frac{1}{4}$, and $D = -\frac{1}{2}$:

$A - B - D = \frac{1}{4} - \left(-\frac{1}{4}\right) - \left(-\frac{1}{2}\right) = \frac{1}{4} + \frac{1}{4} + \frac{1}{2} = \frac{2}{4} + \frac{1}{2} = \frac{1}{2} + \frac{1}{2} = 1$

This matches the constant term on the left side, confirming our values for A, B, C, and D are correct.


Substituting the values of A, B, C, and D back into equation (1), we get:

$\frac{1}{(x - 1)(x + 1)(x^2 + 1)} = \frac{1/4}{x - 1} + \frac{-1/4}{x + 1} + \frac{0x - 1/2}{x^2 + 1}$

$\frac{1}{x^4 - 1} = \frac{1}{4(x - 1)} - \frac{1}{4(x + 1)} - \frac{1}{2(x^2 + 1)}$

Now, we can integrate the expression:

$\int \frac{1}{x^4 - 1} \;dx = \int \left( \frac{1}{4(x - 1)} - \frac{1}{4(x + 1)} - \frac{1}{2(x^2 + 1)} \right) \;dx$

Using the linearity of the integral:

$\int \frac{1}{x^4 - 1} \;dx = \frac{1}{4} \int \frac{1}{x - 1} \;dx - \frac{1}{4} \int \frac{1}{x + 1} \;dx - \frac{1}{2} \int \frac{1}{x^2 + 1} \;dx$

We use the standard integral formulas:

$\int \frac{1}{u} \;du = \log_e |u|$

$\int \frac{1}{x^2 + a^2} \;dx = \frac{1}{a} \tan^{-1}\left(\frac{x}{a}\right)$

Applying these formulas:

$\int \frac{1}{x - 1} \;dx = \log_e |x - 1|$

$\int \frac{1}{x + 1} \;dx = \log_e |x + 1|$

$\int \frac{1}{x^2 + 1} \;dx = \tan^{-1}(x)$ (Here $a=1$).

(We add the constant of integration at the end).


Combining these results:

$\int \frac{1}{x^4 - 1} \;dx = \frac{1}{4} \log_e |x - 1| - \frac{1}{4} \log_e |x + 1| - \frac{1}{2} \tan^{-1}(x) + C$

where $C$ is the constant of integration.


Using the logarithm property $\log_e a - \log_e b = \log_e \frac{a}{b}$, we can simplify the result:

$\int \frac{1}{x^4 - 1} \;dx = \frac{1}{4} (\log_e |x - 1| - \log_e |x + 1|) - \frac{1}{2} \tan^{-1}(x) + C$

$\int \frac{1}{x^4 - 1} \;dx = \frac{1}{4} \log_e \left|\frac{x - 1}{x + 1}\right| - \frac{1}{2} \tan^{-1}(x) + C$


Final Answer:

The integral is $\int \frac{1}{x^4 − 1} \;dx = \frac{1}{4} \log_e \left|\frac{x - 1}{x + 1}\right| - \frac{1}{2} \tan^{-1}(x) + C$.

Question 16. $\frac{1}{x(x^n + 1)}$

[Hint: multiply numerator and denominator by xn–1 and put xn = t]

Answer:

Given:

We are asked to find the integral $\int \frac{1}{x(x^n + 1)} \;dx$.


Solution:

We follow the hint provided.

Multiply the numerator and the denominator by $x^{n-1}$.

$\frac{1}{x(x^n + 1)} = \frac{1}{x(x^n + 1)} \times \frac{x^{n-1}}{x^{n-1}} = \frac{x^{n-1}}{x \cdot x^{n-1} (x^n + 1)} = \frac{x^{n-1}}{x^n (x^n + 1)}$

So, the integral becomes:

$\int \frac{x^{n-1}}{x^n (x^n + 1)} \;dx$

... (1)


Now, let's use the substitution suggested: $t = x^n$.

Differentiate $t$ with respect to $x$:

$\frac{dt}{dx} = nx^{n-1}$

This implies $dt = nx^{n-1} \;dx$.

So, $x^{n-1} \;dx = \frac{1}{n} \;dt$.


Substitute $t = x^n$ and $x^{n-1} \;dx = \frac{1}{n} \;dt$ into integral (1):

$\int \frac{1}{t (t + 1)} \frac{1}{n} \;dt = \frac{1}{n} \int \frac{1}{t(t + 1)} \;dt$

... (2)


Now, we need to evaluate the integral $\int \frac{1}{t(t + 1)} \;dt$. We use partial fraction decomposition for the integrand $\frac{1}{t(t + 1)}$.

We write it in the form:

$\frac{1}{t(t + 1)} = \frac{A}{t} + \frac{B}{t + 1}$

... (3)

Multiply both sides of equation (3) by $t(t + 1)$:

$1 = A(t + 1) + Bt$


To find the values of A and B:

Set $t = 0$:

$1 = A(0 + 1) + B(0)$

(Substitute $t=0$)

$1 = A$

Thus, $\mathbf{A = 1}$.


Set $t = -1$:

$1 = A(-1 + 1) + B(-1)$

(Substitute $t=-1$)

$1 = 0 - B$

$B = -1$

Thus, $\mathbf{B = -1}$.


Substitute the values of A and B back into equation (3):

$\frac{1}{t(t + 1)} = \frac{1}{t} + \frac{-1}{t + 1} = \frac{1}{t} - \frac{1}{t + 1}$

Now, substitute this back into integral (2):

$\frac{1}{n} \int \left( \frac{1}{t} - \frac{1}{t + 1} \right) \;dt$

Using the linearity of the integral:

$\frac{1}{n} \left( \int \frac{1}{t} \;dt - \int \frac{1}{t + 1} \;dt \right)$

We know that $\int \frac{1}{u} \;du = \log_e |u| + C'$.

$\int \frac{1}{t} \;dt = \log_e |t|$

$\int \frac{1}{t + 1} \;dt = \log_e |t + 1|$

(We add the constant of integration at the end).


Combining these results for the integral with respect to $t$:

$\frac{1}{n} (\log_e |t| - \log_e |t + 1|) + C$

Using the logarithm property $\log_e a - \log_e b = \log_e \frac{a}{b}$:

$\frac{1}{n} \log_e \left|\frac{t}{t + 1}\right| + C$


Finally, substitute back $t = x^n$:

$\int \frac{1}{x(x^n + 1)} \;dx = \frac{1}{n} \log_e \left|\frac{x^n}{x^n + 1}\right| + C$

where $C$ is the constant of integration.


Final Answer:

The integral is $\int \frac{1}{x(x^n + 1)} \;dx = \frac{1}{n} \log_e \left|\frac{x^n}{x^n + 1}\right| + C$.

Question 17. $\frac{\cos x}{(1 − \sin x) (2 − \sin x)}$

[Hint : Put sin x = t]

Answer:

Given:

We are asked to find the integral $\int \frac{\cos x}{(1 − \sin x) (2 − \sin x)} \;dx$.


Solution:

We use the substitution suggested in the hint. Let $t = \sin x$.

Differentiating both sides with respect to $x$, we get:

$\frac{dt}{dx} = \cos x$

This implies $dt = \cos x \;dx$.


Substitute $t = \sin x$ and $dt = \cos x \;dx$ into the given integral:

$\int \frac{dt}{(1 - t)(2 - t)}$

... (1)


Now, we evaluate the integral $\int \frac{1}{(1 - t)(2 - t)} \;dt$ using partial fraction decomposition.

We write the integrand in the form:

$\frac{1}{(1 - t)(2 - t)} = \frac{A}{1 - t} + \frac{B}{2 - t}$

... (2)

Multiplying both sides of equation (2) by $(1 - t)(2 - t)$, we get:

$1 = A(2 - t) + B(1 - t)$


To find the values of A and B, we substitute convenient values for $t$:

Setting $t = 1$:

$1 = A(2 - 1) + B(1 - 1)$

(Substitute $t=1$)

$1 = A(1) + B(0)$

$A = 1$

Thus, $\mathbf{A = 1}$.


Setting $t = 2$:

$1 = A(2 - 2) + B(2 - 1)$

(Substitute $t=2$)

$1 = A(0) + B(1)$

$B = 1$

Thus, $\mathbf{B = 1}$.


Substituting the values of A and B back into equation (2), we get:

$\frac{1}{(1 - t)(2 - t)} = \frac{1}{1 - t} + \frac{1}{2 - t}$

Now, we integrate this expression with respect to $t$ (from integral (1)):

$\int \frac{dt}{(1 - t)(2 - t)} = \int \left( \frac{1}{1 - t} + \frac{1}{2 - t} \right) \;dt$

Using the linearity of the integral:

$\int \left( \frac{1}{1 - t} + \frac{1}{2 - t} \right) \;dt = \int \frac{1}{1 - t} \;dt + \int \frac{1}{2 - t} \;dt$

We know that $\int \frac{1}{ax+b} \;dx = \frac{1}{a} \log_e |ax+b| + C'$.

For the first integral, let $u = 1 - t$, then $du = -dt$, so $\int \frac{1}{1 - t} \;dt = - \int \frac{1}{u} \;du = -\log_e |1 - t|$.

For the second integral, let $v = 2 - t$, then $dv = -dt$, so $\int \frac{1}{2 - t} \;dt = - \int \frac{1}{v} \;dv = -\log_e |2 - t|$.

(We add the constant of integration at the end).


So, the integral with respect to $t$ is:

$-\log_e |1 - t| - \log_e |2 - t| + C$

This simplifies to $-\log_e (|1 - t| |2 - t|) + C$.

Let's double check the partial fraction calculation. $1 = A(2 - t) + B(1 - t)$ If $t=1$, $1 = A(1) + B(0) \implies A = 1$. If $t=2$, $1 = A(0) + B(-1) \implies B = -1$. The decomposition should be $\frac{1}{1 - t} - \frac{1}{2 - t}$.

Let's recalculate the integral using the correct decomposition:

$\int \left( \frac{1}{1 - t} - \frac{1}{2 - t} \right) \;dt = \int \frac{1}{1 - t} \;dt - \int \frac{1}{2 - t} \;dt$

Using $\int \frac{1}{ax+b} \;dx = \frac{1}{a} \log_e |ax+b| + C'$:

$\int \frac{1}{1 - t} \;dt = \frac{1}{-1} \log_e |1 - t| = -\log_e |1 - t|$

$\int \frac{1}{2 - t} \;dt = \frac{1}{-1} \log_e |2 - t| = -\log_e |2 - t|$

So, the integral is $-\log_e |1 - t| - (-\log_e |2 - t|) + C = -\log_e |1 - t| + \log_e |2 - t| + C$.


Using the logarithm property $\log_e a - \log_e b = \log_e \frac{a}{b}$:

$\int \frac{dt}{(1 - t)(2 - t)} = \log_e \left|\frac{2 - t}{1 - t}\right| + C$

Finally, substitute back $t = \sin x$:

$\int \frac{\cos x}{(1 − \sin x) (2 − \sin x)} \;dx = \log_e \left|\frac{2 - \sin x}{1 - \sin x}\right| + C$

where $C$ is the constant of integration.


Final Answer:

The integral is $\int \frac{\cos x}{(1 − \sin x) (2 − \sin x)} \;dx = \log_e \left|\frac{2 - \sin x}{1 - \sin x}\right| + C$.

Question 18. $\frac{(x^2 + 1) (x^2 + 2)}{(x^2 + 3) (x^2 + 4)}$

Answer:

Given:

We are asked to find the integral $\int \frac{(x^2 + 1) (x^2 + 2)}{(x^2 + 3) (x^2 + 4)} \;dx$.


Solution:

The integrand is a rational function. First, let's expand the numerator and the denominator:

Numerator = $(x^2 + 1)(x^2 + 2) = x^4 + 2x^2 + x^2 + 2 = x^4 + 3x^2 + 2$.

Denominator = $(x^2 + 3)(x^2 + 4) = x^4 + 4x^2 + 3x^2 + 12 = x^4 + 7x^2 + 12$.

The integrand is $\frac{x^4 + 3x^2 + 2}{x^4 + 7x^2 + 12}$. Since the degree of the numerator is equal to the degree of the denominator, this is an improper rational function. We can perform algebraic manipulation or long division.

Let's rewrite the numerator in terms of the denominator:

$x^4 + 3x^2 + 2 = (x^4 + 7x^2 + 12) - 4x^2 - 10$.

So, the integrand becomes:

$\frac{x^4 + 3x^2 + 2}{x^4 + 7x^2 + 12} = \frac{(x^4 + 7x^2 + 12) - (4x^2 + 10)}{x^4 + 7x^2 + 12} = 1 - \frac{4x^2 + 10}{x^4 + 7x^2 + 12}$.

The integral is $\int \left( 1 - \frac{4x^2 + 10}{(x^2 + 3)(x^2 + 4)} \right) dx = \int 1 \;dx - \int \frac{4x^2 + 10}{(x^2 + 3)(x^2 + 4)} \;dx$.


Now we need to evaluate the integral of the second term: $\int \frac{4x^2 + 10}{(x^2 + 3)(x^2 + 4)} \;dx$.

The integrand $\frac{4x^2 + 10}{(x^2 + 3)(x^2 + 4)}$ is a proper rational function with irreducible quadratic factors in the denominator.

To perform partial fraction decomposition, we can treat $x^2$ as a variable temporarily. Let $y = x^2$.

$\frac{4y + 10}{(y + 3)(y + 4)} = \frac{A}{y + 3} + \frac{B}{y + 4}$.

Multiplying both sides by $(y + 3)(y + 4)$, we get:

$4y + 10 = A(y + 4) + B(y + 3)$

Set $y = -3$:

$4(-3) + 10 = A(-3 + 4) + B(-3 + 3)$

$-12 + 10 = A(1) + B(0)$

$-2 = A$

Thus, $\mathbf{A = -2}$.

Set $y = -4$:

$4(-4) + 10 = A(-4 + 4) + B(-4 + 3)$

$-16 + 10 = A(0) + B(-1)$

$-6 = -B$

$B = 6$

Thus, $\mathbf{B = 6}$.


Substituting the values of A and B back into the partial fraction decomposition with $y = x^2$:

$\frac{4x^2 + 10}{(x^2 + 3)(x^2 + 4)} = \frac{-2}{x^2 + 3} + \frac{6}{x^2 + 4}$

Now substitute this back into the original integral:

$\int \frac{(x^2 + 1) (x^2 + 2)}{(x^2 + 3) (x^2 + 4)} \;dx = \int \left( 1 - \left( \frac{-2}{x^2 + 3} + \frac{6}{x^2 + 4} \right) \right) dx$

$\int \frac{(x^2 + 1) (x^2 + 2)}{(x^2 + 3) (x^2 + 4)} \;dx = \int \left( 1 + \frac{2}{x^2 + 3} - \frac{6}{x^2 + 4} \right) dx$

Using the linearity of the integral:

$\int \frac{(x^2 + 1) (x^2 + 2)}{(x^2 + 3) (x^2 + 4)} \;dx = \int 1 \;dx + 2 \int \frac{1}{x^2 + 3} \;dx - 6 \int \frac{1}{x^2 + 4} \;dx$

We use the standard integral formula $\int \frac{1}{x^2 + a^2} \;dx = \frac{1}{a} \tan^{-1}\left(\frac{x}{a}\right) + C'$.

For $\int \frac{1}{x^2 + 3} \;dx$, we have $a^2 = 3$, so $a = \sqrt{3}$.

$\int \frac{1}{x^2 + 3} \;dx = \frac{1}{\sqrt{3}} \tan^{-1}\left(\frac{x}{\sqrt{3}}\right)$

For $\int \frac{1}{x^2 + 4} \;dx$, we have $a^2 = 4$, so $a = 2$.

$\int \frac{1}{x^2 + 4} \;dx = \frac{1}{2} \tan^{-1}\left(\frac{x}{2}\right)$

Also, $\int 1 \;dx = x$.

(We add the constant of integration at the end).


Combining these results:

$\int \frac{(x^2 + 1) (x^2 + 2)}{(x^2 + 3) (x^2 + 4)} \;dx = x + 2 \left( \frac{1}{\sqrt{3}} \tan^{-1}\left(\frac{x}{\sqrt{3}}\right) \right) - 6 \left( \frac{1}{2} \tan^{-1}\left(\frac{x}{2}\right) \right) + C$

$\int \frac{(x^2 + 1) (x^2 + 2)}{(x^2 + 3) (x^2 + 4)} \;dx = x + \frac{2}{\sqrt{3}} \tan^{-1}\left(\frac{x}{\sqrt{3}}\right) - 3 \tan^{-1}\left(\frac{x}{2}\right) + C$

where $C$ is the constant of integration.


Final Answer:

The integral is $\int \frac{(x^2 + 1) (x^2 + 2)}{(x^2 + 3) (x^2 + 4)} \;dx = x + \frac{2}{\sqrt{3}} \tan^{-1}\left(\frac{x}{\sqrt{3}}\right) - 3 \tan^{-1}\left(\frac{x}{2}\right) + C$.

Question 19. $\frac{2x}{(x^2 + 1) (x^2 + 3)}$

Answer:

Given:

We are asked to find the integral $\int \frac{2x}{(x^2 + 1) (x^2 + 3)} \;dx$.


Solution:

Let's use the method of substitution to simplify the integral. Let $u = x^2$.

Differentiating $u$ with respect to $x$, we get:

$\frac{du}{dx} = 2x$

This implies $du = 2x \;dx$.


Now, substitute $u = x^2$ and $du = 2x \;dx$ into the given integral:

$\int \frac{du}{(u + 1) (u + 3)}$

... (1)

This is a proper rational function in terms of $u$ with distinct linear factors in the denominator. We use partial fraction decomposition.

We write the integrand in the form:

$\frac{1}{(u + 1)(u + 3)} = \frac{A}{u + 1} + \frac{B}{u + 3}$

... (2)

Multiplying both sides of equation (2) by $(u + 1)(u + 3)$, we get:

$1 = A(u + 3) + B(u + 1)$


To find the values of A and B, we substitute convenient values for $u$:

Setting $u = -1$:

$1 = A(-1 + 3) + B(-1 + 1)$

(Substituting $u=-1$)

$1 = A(2) + B(0)$

$1 = 2A$

$A = \frac{1}{2}$

Thus, $\mathbf{A = \frac{1}{2}}$.


Setting $u = -3$:

$1 = A(-3 + 3) + B(-3 + 1)$

(Substituting $u=-3$)

$1 = A(0) + B(-2)$

$1 = -2B$

$B = -\frac{1}{2}$

Thus, $\mathbf{B = -\frac{1}{2}}$.


Substituting the values of A and B back into equation (2), we get:

$\frac{1}{(u + 1)(u + 3)} = \frac{1/2}{u + 1} + \frac{-1/2}{u + 3}$

$\frac{1}{(u + 1)(u + 3)} = \frac{1}{2(u + 1)} - \frac{1}{2(u + 3)}$

Now, integrate this expression with respect to $u$ (from integral (1)):

$\int \frac{du}{(u + 1)(u + 3)} = \int \left( \frac{1}{2(u + 1)} - \frac{1}{2(u + 3)} \right) \;du$

Using the linearity of the integral:

$\int \left( \frac{1}{2(u + 1)} - \frac{1}{2(u + 3)} \right) \;du = \frac{1}{2} \int \frac{1}{u + 1} \;du - \frac{1}{2} \int \frac{1}{u + 3} \;du$

We know that $\int \frac{1}{v} \;dv = \log_e |v| + C'$.

$\int \frac{1}{u + 1} \;du = \log_e |u + 1|$

$\int \frac{1}{u + 3} \;du = \log_e |u + 3|$

(We add the constant of integration at the end).


So, the integral with respect to $u$ is:

$\frac{1}{2} \log_e |u + 1| - \frac{1}{2} \log_e |u + 3| + C$

Using the logarithm property $\log_e a - \log_e b = \log_e \frac{a}{b}$:

$\frac{1}{2} (\log_e |u + 1| - \log_e |u + 3|) + C = \frac{1}{2} \log_e \left|\frac{u + 1}{u + 3}\right| + C$


Finally, substitute back $u = x^2$:

$\int \frac{2x}{(x^2 + 1) (x^2 + 3)} \;dx = \frac{1}{2} \log_e \left|\frac{x^2 + 1}{x^2 + 3}\right| + C$

Since $x^2 + 1 > 0$ and $x^2 + 3 > 0$ for all real $x$, we can remove the absolute value signs from the fraction inside the logarithm:

$\int \frac{2x}{(x^2 + 1) (x^2 + 3)} \;dx = \frac{1}{2} \log_e \left(\frac{x^2 + 1}{x^2 + 3}\right) + C$

where $C$ is the constant of integration.


Final Answer:

The integral is $\int \frac{2x}{(x^2 + 1) (x^2 + 3)} \;dx = \frac{1}{2} \log_e \left(\frac{x^2 + 1}{x^2 + 3}\right) + C$.

Question 20. $\frac{1}{x(x^4 − 1)}$

Answer:

Given:

We are asked to find the integral $\int \frac{1}{x(x^4 − 1)} \;dx$.


Solution:

The integrand is a rational function $\frac{1}{x(x^4 − 1)}$. First, we need to factor the denominator $x(x^4 - 1)$.

The term $x^4 - 1$ can be factored as a difference of squares: $x^4 - 1 = (x^2)^2 - 1^2 = (x^2 - 1)(x^2 + 1)$.

The term $x^2 - 1$ can be further factored as a difference of squares: $x^2 - 1 = (x - 1)(x + 1)$.

So, the complete factorization of the denominator is $x(x - 1)(x + 1)(x^2 + 1)$.

The integral is $\int \frac{1}{x(x - 1)(x + 1)(x^2 + 1)} \;dx$.

The denominator consists of distinct linear factors ($x$, $x-1$, $x+1$) and an irreducible quadratic factor ($x^2+1$). We use the method of partial fraction decomposition.

We write the integrand in the form:

$\frac{1}{x(x - 1)(x + 1)(x^2 + 1)} = \frac{A}{x} + \frac{B}{x - 1} + \frac{C}{x + 1} + \frac{Dx + E}{x^2 + 1}$

... (1)

Multiplying both sides of equation (1) by $x(x - 1)(x + 1)(x^2 + 1)$, we get:

$1 = A(x - 1)(x + 1)(x^2 + 1) + Bx(x + 1)(x^2 + 1) + Cx(x - 1)(x^2 + 1) + (Dx + E)x(x - 1)(x + 1)$

... (2)

We can find the values of A, B, C, D, and E by substituting convenient values for $x$ and by comparing coefficients.


Setting $x = 0$ in equation (2):

$1 = A(0 - 1)(0 + 1)(0^2 + 1) + B(0) + C(0) + (D(0) + E)(0)$

(Substitute $x=0$)

$1 = A(-1)(1)(1)$

$1 = -A$

$A = -1$

Thus, $\mathbf{A = -1}$.


Setting $x = 1$ in equation (2):

$1 = A(0)(2)(2) + B(1)(2)(2) + C(1)(0)(2) + (D + E)(1)(0)(2)$

(Substitute $x=1$)

$1 = 4B$

$B = \frac{1}{4}$

Thus, $\mathbf{B = \frac{1}{4}}$.


Setting $x = -1$ in equation (2):

$1 = A(-2)(0)(2) + B(-1)(0)(2) + C(-1)(-2)(2) + (-D + E)(-1)(-2)(0)$

(Substitute $x=-1$)

$1 = 4C$

$C = \frac{1}{4}$

Thus, $\mathbf{C = \frac{1}{4}}$.


Expanding equation (2) and comparing coefficients of $x^3$:
$1 = A(x^4 - 1) + Bx(x^3 + x^2 + x + 1) + Cx(x^3 - x^2 + x - 1) + (Dx + E)(x^2 - 1)x$
$1 = A(x^4 - 1) + B(x^4 + x^3 + x^2 + x) + C(x^4 - x^3 + x^2 - x) + (Dx + E)(x^3 - x)$
$1 = Ax^4 - A + Bx^4 + Bx^3 + Bx^2 + Bx + Cx^4 - Cx^3 + Cx^2 - Cx + Dx^4 + Ex^3 - Dx^2 - Ex$
$1 = (A + B + C + D)x^4 + (B - C + E)x^3 + (B + C - D)x^2 + (B - C - E)x - A$

Comparing the coefficients of $x^3$ on both sides (left side has $0x^3$):

$0 = B - C + E$

(Comparing $x^3$ coefficients)

Substitute $B = \frac{1}{4}$ and $C = \frac{1}{4}$:

$0 = \frac{1}{4} - \frac{1}{4} + E$

$0 = E$

Thus, $\mathbf{E = 0}$.


Comparing the coefficients of $x^2$ on both sides (left side has $0x^2$):

$0 = B + C - D$

(Comparing $x^2$ coefficients)

Substitute $B = \frac{1}{4}$ and $C = \frac{1}{4}$:

$0 = \frac{1}{4} + \frac{1}{4} - D$

$0 = \frac{2}{4} - D$

$0 = \frac{1}{2} - D$

$D = \frac{1}{2}$

Thus, $\mathbf{D = \frac{1}{2}}$.


Substituting the values of A, B, C, D, and E back into equation (1), we get:

$\frac{1}{x(x - 1)(x + 1)(x^2 + 1)} = \frac{-1}{x} + \frac{1/4}{x - 1} + \frac{1/4}{x + 1} + \frac{(1/2)x + 0}{x^2 + 1}$

$\frac{1}{x(x^4 - 1)} = -\frac{1}{x} + \frac{1}{4(x - 1)} + \frac{1}{4(x + 1)} + \frac{x}{2(x^2 + 1)}$

Now, we can integrate the expression:

$\int \frac{1}{x(x^4 - 1)} \;dx = \int \left( -\frac{1}{x} + \frac{1}{4(x - 1)} + \frac{1}{4(x + 1)} + \frac{x}{2(x^2 + 1)} \right) \;dx$

Using the linearity of the integral:

$\int \frac{1}{x(x^4 - 1)} \;dx = -\int \frac{1}{x} \;dx + \frac{1}{4} \int \frac{1}{x - 1} \;dx + \frac{1}{4} \int \frac{1}{x + 1} \;dx + \frac{1}{2} \int \frac{x}{x^2 + 1} \;dx$

We use the standard integral formulas:

$\int \frac{1}{u} \;du = \log_e |u|$

For the last integral $\int \frac{x}{x^2 + 1} \;dx$, let $u = x^2 + 1$, then $du = 2x \;dx$, so $x \;dx = \frac{1}{2} du$.

$\int \frac{x}{x^2 + 1} \;dx = \int \frac{1}{u} \frac{1}{2} \;du = \frac{1}{2} \int \frac{1}{u} \;du = \frac{1}{2} \log_e |u| = \frac{1}{2} \log_e |x^2 + 1| = \frac{1}{2} \log_e (x^2 + 1)$ (since $x^2+1 > 0$).

(We add the constant of integration at the end).


Combining these results:

$\int \frac{1}{x(x^4 - 1)} \;dx = -\log_e |x| + \frac{1}{4} \log_e |x - 1| + \frac{1}{4} \log_e |x + 1| + \frac{1}{2} \left(\frac{1}{2} \log_e (x^2 + 1)\right) + C$

$\int \frac{1}{x(x^4 - 1)} \;dx = -\log_e |x| + \frac{1}{4} \log_e |x - 1| + \frac{1}{4} \log_e |x + 1| + \frac{1}{4} \log_e (x^2 + 1) + C$

where $C$ is the constant of integration.


Using logarithm properties ($n \log_e a = \log_e a^n$ and $\log_e a + \log_e b = \log_e (ab)$ and $\log_e a - \log_e b = \log_e (a/b)$), we can simplify the result:

$\int \frac{1}{x(x^4 - 1)} \;dx = \frac{1}{4} (\log_e |x - 1| + \log_e |x + 1| + \log_e (x^2 + 1)) - \log_e |x| + C$

$\int \frac{1}{x(x^4 - 1)} \;dx = \frac{1}{4} \log_e (|x - 1||x + 1|(x^2 + 1)) - \frac{4}{4}\log_e |x| + C$

$\int \frac{1}{x(x^4 - 1)} \;dx = \frac{1}{4} \log_e |(x^2 - 1)(x^2 + 1)| - \frac{1}{4}\log_e |x^4| + C$

$\int \frac{1}{x(x^4 - 1)} \;dx = \frac{1}{4} \log_e |x^4 - 1| - \frac{1}{4}\log_e |x^4| + C$

$\int \frac{1}{x(x^4 - 1)} \;dx = \frac{1}{4} \log_e \left|\frac{x^4 - 1}{x^4}\right| + C$


Final Answer:

The integral is $\int \frac{1}{x(x^4 − 1)} \;dx = \frac{1}{4} \log_e \left|\frac{x^4 - 1}{x^4}\right| + C$.

Question 21. $\frac{1}{(e^x - 1)}$

[Hint : Put ex = t]

Answer:

Given:

We are asked to find the integral $\int \frac{1}{e^x - 1} \;dx$.


Solution:

We use the substitution suggested in the hint. Let $t = e^x$.

Differentiating both sides with respect to $x$, we get:

$\frac{dt}{dx} = e^x$

So, $dt = e^x \;dx$.

Since $t = e^x$, we have $dx = \frac{dt}{e^x} = \frac{dt}{t}$.


Now, substitute $t = e^x$ and $dx = \frac{dt}{t}$ into the given integral:

$\int \frac{1}{t - 1} \cdot \frac{dt}{t}$

$\int \frac{1}{t(t - 1)} \;dt$

... (1)


Now, we evaluate the integral $\int \frac{1}{t(t - 1)} \;dt$ using partial fraction decomposition.

We write the integrand in the form:

$\frac{1}{t(t - 1)} = \frac{A}{t} + \frac{B}{t - 1}$

... (2)

Multiplying both sides of equation (2) by $t(t - 1)$, we get:

$1 = A(t - 1) + Bt$


To find the values of A and B, we substitute convenient values for $t$:

Setting $t = 0$:

$1 = A(0 - 1) + B(0)$

(Substitute $t=0$)

$1 = -A$

$A = -1$

Thus, $\mathbf{A = -1}$.


Setting $t = 1$:

$1 = A(1 - 1) + B(1)$

(Substitute $t=1$)

$1 = A(0) + B$

$B = 1$

Thus, $\mathbf{B = 1}$.


Substituting the values of A and B back into equation (2), we get:

$\frac{1}{t(t - 1)} = \frac{-1}{t} + \frac{1}{t - 1}$

$\frac{1}{t(t - 1)} = \frac{1}{t - 1} - \frac{1}{t}$

Now, integrate this expression with respect to $t$ (from integral (1)):

$\int \frac{1}{t(t - 1)} \;dt = \int \left( \frac{1}{t - 1} - \frac{1}{t} \right) \;dt$

Using the linearity of the integral:

$\int \left( \frac{1}{t - 1} - \frac{1}{t} \right) \;dt = \int \frac{1}{t - 1} \;dt - \int \frac{1}{t} \;dt$

We know that $\int \frac{1}{u} \;du = \log_e |u| + C'$.

$\int \frac{1}{t - 1} \;dt = \log_e |t - 1|$

$\int \frac{1}{t} \;dt = \log_e |t|$

(We add the constant of integration at the end).


So, the integral with respect to $t$ is:

$\log_e |t - 1| - \log_e |t| + C$

Using the logarithm property $\log_e a - \log_e b = \log_e \frac{a}{b}$:

$\log_e \left|\frac{t - 1}{t}\right| + C$


Finally, substitute back $t = e^x$:

$\int \frac{1}{e^x - 1} \;dx = \log_e \left|\frac{e^x - 1}{e^x}\right| + C$

Since $e^x > 0$ for all real $x$, we have $|e^x| = e^x$. The term $e^x - 1$ can be positive, negative, or zero (when $x=0$). The absolute value around $e^x - 1$ is necessary. The absolute value around $\frac{e^x-1}{e^x}$ is equivalent to $\frac{|e^x - 1|}{e^x}$.

The integral can also be written as $\log_e |e^x - 1| - \log_e |e^x| + C = \log_e |e^x - 1| - x + C$.

where $C$ is the constant of integration.


Final Answer:

The integral is $\int \frac{1}{e^x - 1} \;dx = \log_e \left|\frac{e^x - 1}{e^x}\right| + C$ or $\log_e |e^x - 1| - x + C$.

Choose the correct answer in each of the Exercises 22 and 23.

Question 22. $\int \frac{x \;dx}{(x − 1) (x − 2)}$ equals

(A) $\log \left| \frac{(x − 1)^2}{x − 2} \right| + C$

(B) $\log \left| \frac{(x − 2)^2}{x − 1} \right| + C$

(C) $\log \left| \left( \frac{x − 1}{x − 2} \right)^2 \right| + C$

(D) log |(x - 1) (x - 2)| + c

Answer:

Given:

We are asked to find the integral $\int \frac{x}{(x − 1) (x − 2)} \;dx$ and choose the correct option.


Solution:

The integrand is a rational function $\frac{x}{(x − 1) (x − 2)}$. The denominator is a product of distinct linear factors. We use the method of partial fraction decomposition.

We write the integrand in the form:

$\frac{x}{(x - 1)(x - 2)} = \frac{A}{x - 1} + \frac{B}{x - 2}$

... (1)

Multiplying both sides of equation (1) by $(x - 1)(x - 2)$, we get:

$x = A(x - 2) + B(x - 1)$

... (2)


To find the values of A and B, we substitute convenient values for $x$ in equation (2).

Setting $x = 1$ in equation (2):

$1 = A(1 - 2) + B(1 - 1)$

(Substituting $x=1$)

$1 = A(-1) + B(0)$

$1 = -A$

$A = -1$

Thus, $\mathbf{A = -1}$.


Setting $x = 2$ in equation (2):

$2 = A(2 - 2) + B(2 - 1)$

(Substituting $x=2$)

$2 = A(0) + B(1)$

$2 = B$

Thus, $\mathbf{B = 2}$.


Substituting the values of A and B back into equation (1), we get:

$\frac{x}{(x - 1)(x - 2)} = \frac{-1}{x - 1} + \frac{2}{x - 2}$

Now, we can integrate the expression:

$\int \frac{x}{(x - 1)(x - 2)} \;dx = \int \left( \frac{-1}{x - 1} + \frac{2}{x - 2} \right) \;dx$

Using the linearity of the integral, we have:

$\int \frac{x}{(x - 1)(x - 2)} \;dx = - \int \frac{1}{x - 1} \;dx + 2 \int \frac{1}{x - 2} \;dx$

We know that $\int \frac{1}{u} \;du = \log_e |u| + C'$.

So, $\int \frac{1}{x - 1} \;dx = \log_e |x - 1|$

and $\int \frac{1}{x - 2} \;dx = \log_e |x - 2|$

(We add the constant of integration at the end).


Therefore, the integral is:

$\int \frac{x}{(x - 1)(x - 2)} \;dx = - \log_e |x - 1| + 2 \log_e |x - 2| + C$

where $C$ is the constant of integration.


Using the logarithm property $n \log_e a = \log_e a^n$ and $\log_e a - \log_e b = \log_e \frac{a}{b}$, we can simplify the result:

$\int \frac{x}{(x - 1)(x - 2)} \;dx = \log_e |(x - 2)^2| - \log_e |x - 1| + C$

$\int \frac{x}{(x - 1)(x - 2)} \;dx = \log_e \left|\frac{(x - 2)^2}{x - 1}\right| + C$


Comparing this result with the given options, we see that it matches option (B).

Option (A) is $\log_e \left|\frac{(x - 1)^2}{x - 2}\right| + C = 2\log_e |x - 1| - \log_e |x - 2| + C$. This is incorrect.

Option (C) is $\log_e \left|\left(\frac{x - 1}{x - 2}\right)^2\right| + C = 2\log_e \left|\frac{x - 1}{x - 2}\right| + C = 2(\log_e |x - 1| - \log_e |x - 2|) + C = 2\log_e |x - 1| - 2\log_e |x - 2| + C$. This is incorrect.

Option (D) is $\log_e |(x - 1)(x - 2)| + C = \log_e |x - 1| + \log_e |x - 2| + C$. This is incorrect.


Final Answer:

The correct answer is (B).

Question 23. $\int \frac{dx}{x (x^2 + 1)}$ equals

(A) $\log |x| - \frac{1}{2} \log (x^2 + 1) + C$

(B) $\log |x| + \frac{1}{2} \log (x^2 + 1) + C$

(C) $-\log |x| + \frac{1}{2} \log (x^2 + 1) + C$

(D) $\frac{1}{2} \log |x| + \log (x^2 + 1) + C$

Answer:

Given:

We are asked to find the integral $\int \frac{1}{x (x^2 + 1)} \;dx$ and choose the correct option.


Solution:

The integrand is a rational function $\frac{1}{x (x^2 + 1)}$. The denominator consists of a distinct linear factor $x$ and an irreducible quadratic factor $x^2 + 1$. We use the method of partial fraction decomposition.

We write the integrand in the form:

$\frac{1}{x(x^2 + 1)} = \frac{A}{x} + \frac{Bx + C}{x^2 + 1}$

... (1)

Multiplying both sides of equation (1) by $x(x^2 + 1)$, we get:

$1 = A(x^2 + 1) + (Bx + C)x$

... (2)

Expanding equation (2):

$1 = Ax^2 + A + Bx^2 + Cx$

$1 = (A + B)x^2 + Cx + A$


Comparing the coefficients of the powers of $x$ on both sides:

Coefficient of $x^2$: $A + B = 0$

Coefficient of $x$: $C = 0$

Constant term: $A = 1$


From the constant term, we have $\mathbf{A = 1}$.

From the coefficient of $x$, we have $\mathbf{C = 0}$.

Substitute $A = 1$ into the coefficient of $x^2$ equation:

$1 + B = 0$

$B = -1$

Thus, $\mathbf{B = -1}$.


Substituting the values of A, B, and C back into equation (1), we get:

$\frac{1}{x(x^2 + 1)} = \frac{1}{x} + \frac{-1x + 0}{x^2 + 1}$

$\frac{1}{x(x^2 + 1)} = \frac{1}{x} - \frac{x}{x^2 + 1}$

Now, we integrate the expression:

$\int \frac{1}{x(x^2 + 1)} \;dx = \int \left( \frac{1}{x} - \frac{x}{x^2 + 1} \right) \;dx$

Using the linearity of the integral:

$\int \frac{1}{x(x^2 + 1)} \;dx = \int \frac{1}{x} \;dx - \int \frac{x}{x^2 + 1} \;dx$

We evaluate each integral:

$\int \frac{1}{x} \;dx = \log_e |x|$

For the second integral $\int \frac{x}{x^2 + 1} \;dx$, let $u = x^2 + 1$. Then $du = 2x \;dx$. So $x \;dx = \frac{1}{2} du$.

$\int \frac{x}{x^2 + 1} \;dx = \int \frac{1}{u} \frac{1}{2} \;du = \frac{1}{2} \int \frac{1}{u} \;du = \frac{1}{2} \log_e |u| = \frac{1}{2} \log_e |x^2 + 1|$

Since $x^2 + 1 > 0$ for all real $x$, $|x^2 + 1| = x^2 + 1$. So, $\int \frac{x}{x^2 + 1} \;dx = \frac{1}{2} \log_e (x^2 + 1)$.

(We add the constant of integration at the end).


Combining these results:

$\int \frac{1}{x(x^2 + 1)} \;dx = \log_e |x| - \frac{1}{2} \log_e (x^2 + 1) + C$

where $C$ is the constant of integration.


Comparing this result with the given options, we see that it matches option (A).

Final Answer:

The correct answer is (A).



Example 17 to 22 (Before Exercise 7.6)

Example 17: Find $\int x \cos x \;dx$

Answer:

Given:

We are asked to find the integral $\int x \cos x \;dx$.


Solution:

The integral involves a product of two different types of functions: an algebraic function ($x$) and a trigonometric function ($\cos x$). We can solve this integral using the method of integration by parts.

The formula for integration by parts is:

$\int u \;dv = uv - \int v \;du$

... (1)

We need to choose which part of the integrand will be $u$ and which will be $dv$. We can use the LIATE rule (Logarithmic, Inverse trigonometric, Algebraic, Trigonometric, Exponential) to help make this choice. We generally choose $u$ as the function that comes first in the LIATE order.

In this case, we have $x$ (Algebraic) and $\cos x$ (Trigonometric). 'A' comes before 'T' in LIATE. So we choose $u = x$ and $dv = \cos x \;dx$.

Let $u = x$

Differentiating with respect to $x$ to find $du$:

$\frac{du}{dx} = \frac{d}{dx}(x) = 1$

$du = dx$


Let $dv = \cos x \;dx$

Integrating with respect to $x$ to find $v$:

$v = \int dv = \int \cos x \;dx$

$v = \sin x$

(We omit the constant of integration for $v$ at this step, as we will add a single constant at the end of the entire integral).


Now, substitute $u=x$, $dv=\cos x \;dx$, $du=dx$, and $v=\sin x$ into the integration by parts formula (1):

$\int x \cos x \;dx = (x)(\sin x) - \int (\sin x) \;dx$

$\int x \cos x \;dx = x \sin x - \int \sin x \;dx$

Now, we evaluate the remaining integral $\int \sin x \;dx$:

$\int \sin x \;dx = -\cos x$

(We add the constant of integration here for the overall result).


Substitute this back into the equation for the main integral:

$\int x \cos x \;dx = x \sin x - (-\cos x) + C$

$\int x \cos x \;dx = x \sin x + \cos x + C$

where $C$ is the constant of integration.


Final Answer:

The integral is $\int x \cos x \;dx = x \sin x + \cos x + C$.

Example 18: Find $\int \log x \;dx$

Answer:

Given:

We are asked to find the integral $\int \log x \;dx$. We assume $\log x$ refers to the natural logarithm, $\log_e x$. So, we need to find $\int \log_e x \;dx$.


Solution:

This integral can be evaluated using the method of integration by parts by considering the integrand as a product of two functions: $\log_e x$ and $1$.

The formula for integration by parts is:

$\int u \;dv = uv - \int v \;du$

... (1)

We choose $u = \log_e x$ (as it comes first in the LIATE rule) and $dv = 1 \;dx$.

Let $u = \log_e x$

Differentiating with respect to $x$ to find $du$:

$\frac{du}{dx} = \frac{d}{dx}(\log_e x) = \frac{1}{x}$

$du = \frac{1}{x} \;dx$


Let $dv = 1 \;dx$

Integrating with respect to $x$ to find $v$:

$v = \int dv = \int 1 \;dx$

$v = x$

(We omit the constant of integration for $v$ at this step).


Now, substitute $u=\log_e x$, $dv=1 \;dx$, $du=\frac{1}{x} \;dx$, and $v=x$ into the integration by parts formula (1):

$\int \log_e x \;dx = (\log_e x)(x) - \int (x)\left(\frac{1}{x}\right) \;dx$

$\int \log_e x \;dx = x \log_e x - \int 1 \;dx$

Now, we evaluate the remaining integral $\int 1 \;dx$:

$\int 1 \;dx = x$

(We add the constant of integration here for the overall result).


Substitute this back into the equation for the main integral:

$\int \log_e x \;dx = x \log_e x - x + C$

This can also be written by factoring out $x$:

$\int \log_e x \;dx = x(\log_e x - 1) + C$

where $C$ is the constant of integration.


Final Answer:

The integral is $\int \log x \;dx = x \log x - x + C$.

Example 19: Find $\int x e^x \;dx$

Answer:

Given:

We are asked to find the integral $\int x e^x \;dx$.


Solution:

The integral involves a product of two different types of functions: an algebraic function ($x$) and an exponential function ($e^x$). We can solve this integral using the method of integration by parts.

The formula for integration by parts is:

$\int u \;dv = uv - \int v \;du$

... (1)

We use the LIATE rule (Logarithmic, Inverse trigonometric, Algebraic, Trigonometric, Exponential) to choose $u$ and $dv$. 'A' (Algebraic) comes before 'E' (Exponential).

So we choose $u = x$ and $dv = e^x \;dx$.

Let $u = x$

Differentiating with respect to $x$ to find $du$:

$\frac{du}{dx} = \frac{d}{dx}(x) = 1$

$du = dx$


Let $dv = e^x \;dx$

Integrating with respect to $x$ to find $v$:

$v = \int dv = \int e^x \;dx$

$v = e^x$

(We omit the constant of integration for $v$ at this step).


Now, substitute $u=x$, $dv=e^x \;dx$, $du=dx$, and $v=e^x$ into the integration by parts formula (1):

$\int x e^x \;dx = (x)(e^x) - \int (e^x) \;dx$

$\int x e^x \;dx = x e^x - \int e^x \;dx$

Now, we evaluate the remaining integral $\int e^x \;dx$:

$\int e^x \;dx = e^x$

(We add the constant of integration here for the overall result).


Substitute this back into the equation for the main integral:

$\int x e^x \;dx = x e^x - e^x + C$

This can also be written by factoring out $e^x$:

$\int x e^x \;dx = e^x(x - 1) + C$

where $C$ is the constant of integration.


Final Answer:

The integral is $\int x e^x \;dx = x e^x - e^x + C$.

Example 20: Find $\int \frac{x \sin^{−1} x}{\sqrt{1 − x^2}} \;dx$

Answer:

Given:

We are asked to find the integral $\int \frac{x \sin^{−1} x}{\sqrt{1 − x^2}} \;dx$.


Solution:

We can use the method of substitution to simplify the integral.

Let $t = \sin^{-1} x$.

Differentiating both sides with respect to $x$, we get:

$\frac{dt}{dx} = \frac{1}{\sqrt{1 - x^2}}$

This implies $dt = \frac{dx}{\sqrt{1 - x^2}}$.

Also, from $t = \sin^{-1} x$, we have $x = \sin t$.


Now, substitute $t = \sin^{-1} x$, $x = \sin t$, and $dt = \frac{dx}{\sqrt{1 - x^2}}$ into the given integral:

$\int \frac{x \sin^{-1} x}{\sqrt{1 − x^2}} \;dx = \int (\sin t) (t) \;dt$

$= \int t \sin t \;dt$

... (1)


Now, we need to evaluate the integral $\int t \sin t \;dt$. This is a product of an algebraic function ($t$) and a trigonometric function ($\sin t$). We use the method of integration by parts.

The formula for integration by parts is:

$\int u \;dv = uv - \int v \;du$

... (2)

Using the LIATE rule (Logarithmic, Inverse trigonometric, Algebraic, Trigonometric, Exponential), we choose $u = t$ and $dv = \sin t \;dt$.

Let $u = t$

Differentiating to find $du$:

$\frac{du}{dt} = \frac{d}{dt}(t) = 1$

$du = dt$


Let $dv = \sin t \;dt$

Integrating to find $v$:

$v = \int dv = \int \sin t \;dt$

$v = -\cos t$

(We omit the constant of integration for $v$ here).


Substitute $u=t$, $dv=\sin t \;dt$, $du=dt$, and $v=-\cos t$ into the integration by parts formula (2):

$\int t \sin t \;dt = (t)(-\cos t) - \int (-\cos t) \;dt$

$\int t \sin t \;dt = -t \cos t + \int \cos t \;dt$

Now, evaluate the remaining integral $\int \cos t \;dt$:

$\int \cos t \;dt = \sin t$

(We add the constant of integration here for the overall result).


Combine the results for the integral with respect to $t$:

$\int t \sin t \;dt = -t \cos t + \sin t + C$


Finally, substitute back to the original variable $x$. Recall $t = \sin^{-1} x$ and $\sin t = x$.

We also need to express $\cos t$ in terms of $x$. Since $t = \sin^{-1} x$, we have $\sin t = x$. Consider a right-angled triangle where the angle is $t$. The opposite side is $x$ and the hypotenuse is $1$. By the Pythagorean theorem, the adjacent side is $\sqrt{1^2 - x^2} = \sqrt{1 - x^2}$.

$\cos t = \sqrt{1 - x^2}$

Substitute $t = \sin^{-1} x$, $\cos t = \sqrt{1 - x^2}$, and $\sin t = x$ back into the result:

$-(\sin^{-1} x) (\sqrt{1 - x^2}) + x + C$

We can rearrange the terms:

$\int \frac{x \sin^{−1} x}{\sqrt{1 − x^2}} \;dx = x - \sqrt{1 - x^2} \sin^{-1} x + C$

where $C$ is the constant of integration.


Final Answer:

The integral is $\int \frac{x \sin^{−1} x}{\sqrt{1 − x^2}} \;dx = x - \sqrt{1 - x^2} \sin^{-1} x + C$.

Example 21: Find $\int e^x \sin x \;dx$

Answer:

Given:

We are asked to find the integral $\int e^x \sin x \;dx$.


Solution:

The integral involves a product of an exponential function ($e^x$) and a trigonometric function ($\sin x$). We can solve this integral using the method of integration by parts.

The formula for integration by parts is:

$\int u \;dv = uv - \int v \;du$

... (1)

We choose $u = \sin x$ and $dv = e^x \;dx$ based on the LIATE rule (though the choice between exponential and trigonometric is less critical as long as it's consistent in subsequent steps). Choosing the trigonometric function as $u$ often works well here.

Let $u = \sin x$.

Differentiating with respect to $x$ to find $du$:

$\frac{du}{dx} = \cos x$

$du = \cos x \;dx$


Let $dv = e^x \;dx$.

Integrating with respect to $x$ to find $v$:

$v = \int dv = \int e^x \;dx$

$v = e^x$

(We omit the constant of integration for $v$ at this step).


Apply the integration by parts formula (1) with these choices:

$\int e^x \sin x \;dx = (\sin x)(e^x) - \int (e^x)(\cos x) \;dx$

$\int e^x \sin x \;dx = e^x \sin x - \int e^x \cos x \;dx$

... (2)

Now, we need to evaluate the new integral $\int e^x \cos x \;dx$. This is also a product of an exponential and a trigonometric function, so we apply integration by parts again.

For $\int e^x \cos x \;dx$, we must make a consistent choice for $u'$ and $dv'$. Since we chose the trigonometric function ($\sin x$) as $u$ and the exponential function ($e^x$) as $dv$ in the first step, we do the same for $\cos x$ and $e^x$.

Let $u' = \cos x$.

Differentiating to find $du'$:

$\frac{du'}{dx} = -\sin x$

$du' = -\sin x \;dx$


Let $dv' = e^x \;dx$.

Integrating to find $v'$:

$v' = \int dv' = \int e^x \;dx$

$v' = e^x$


Apply the integration by parts formula to $\int e^x \cos x \;dx$ with $u'=\cos x$, $dv'=e^x \;dx$, $du'=-\sin x \;dx$, and $v'=e^x$:

$\int e^x \cos x \;dx = (\cos x)(e^x) - \int (e^x)(-\sin x) \;dx$

$\int e^x \cos x \;dx = e^x \cos x + \int e^x \sin x \;dx$

... (3)


Substitute the result from equation (3) back into equation (2):

$\int e^x \sin x \;dx = e^x \sin x - (e^x \cos x + \int e^x \sin x \;dx)$

$\int e^x \sin x \;dx = e^x \sin x - e^x \cos x - \int e^x \sin x \;dx$

Let $I = \int e^x \sin x \;dx$. The equation becomes:

$I = e^x \sin x - e^x \cos x - I$

Now, we solve this equation for $I$ by adding $I$ to both sides:

$I + I = e^x \sin x - e^x \cos x$

$2I = e^x (\sin x - \cos x)$

Divide by 2 to find $I$:

$I = \frac{1}{2} e^x (\sin x - \cos x)$

Finally, add the constant of integration $C$ to the result:

$\int e^x \sin x \;dx = \frac{1}{2} e^x (\sin x - \cos x) + C$

where $C$ is the constant of integration.


Final Answer:

The integral is $\int e^x \sin x \;dx = \frac{1}{2} e^x (\sin x - \cos x) + C$.

Example 22: Find

(i) $\int e^x \left( \tan^{−1} x + \frac{1}{1 + x^2} \right) \;dx$

(ii) $\int \frac{(x^2 + 1) e^x}{(x + 1)^2} \;dx$

Answer:

Part (i)

Given:

We are asked to find the integral $\int e^x \left( \tan^{−1} x + \frac{1}{1 + x^2} \right) \;dx$.


Solution:

We observe that the integrand is in the form $e^x [f(x) + f'(x)]$.

We know the standard integral formula:

$\int e^x [f(x) + f'(x)] \;dx = e^x f(x) + C$

... (1)

In the given integral $\int e^x \left( \tan^{−1} x + \frac{1}{1 + x^2} \right) \;dx$, let's identify $f(x)$ and $f'(x)$.

Let $f(x) = \tan^{-1} x$.

Then, the derivative of $f(x)$ with respect to $x$ is:

$f'(x) = \frac{d}{dx}(\tan^{-1} x) = \frac{1}{1 + x^2}$

We see that the term $\frac{1}{1 + x^2}$ is exactly the other term inside the parenthesis multiplied by $e^x$.

So, the integrand is indeed of the form $e^x [f(x) + f'(x)]$ with $f(x) = \tan^{-1} x$ and $f'(x) = \frac{1}{1 + x^2}$.


Applying the formula (1):

$\int e^x \left( \tan^{−1} x + \frac{1}{1 + x^2} \right) \;dx = e^x (\tan^{-1} x) + C$

where $C$ is the constant of integration.


Final Answer (i):

The integral is $\int e^x \left( \tan^{−1} x + \frac{1}{1 + x^2} \right) \;dx = e^x \tan^{-1} x + C$.


Part (ii)

Given:

We are asked to find the integral $\int \frac{(x^2 + 1) e^x}{(x + 1)^2} \;dx$.


Solution:

We want to see if this integral can also be expressed in the form $\int e^x [f(x) + f'(x)] \;dx$.

We need to manipulate the fraction $\frac{x^2 + 1}{(x + 1)^2}$. The denominator is $(x + 1)^2 = x^2 + 2x + 1$.

Let's rewrite the numerator $x^2 + 1$ in terms of $(x+1)^2$ or $x+1$.

We can write $x^2 + 1 = (x^2 + 2x + 1) - 2x = (x + 1)^2 - 2x$.

So, the fraction is $\frac{(x + 1)^2 - 2x}{(x + 1)^2} = \frac{(x + 1)^2}{(x + 1)^2} - \frac{2x}{(x + 1)^2} = 1 - \frac{2x}{(x + 1)^2}$.

This doesn't seem to directly fit the form $f(x) + f'(x)$. Let's try another manipulation.

Consider $f(x) = \frac{x-1}{x+1}$.

Then the derivative $f'(x)$ is calculated using the quotient rule:

$f'(x) = \frac{\frac{d}{dx}(x-1) \cdot (x+1) - (x-1) \cdot \frac{d}{dx}(x+1)}{(x+1)^2}$

$f'(x) = \frac{1 \cdot (x+1) - (x-1) \cdot 1}{(x+1)^2}$

$f'(x) = \frac{x+1 - x+1}{(x+1)^2}$

$f'(x) = \frac{2}{(x+1)^2}$

Now let's consider the sum $f(x) + f'(x)$:

$f(x) + f'(x) = \frac{x-1}{x+1} + \frac{2}{(x+1)^2}$

To combine these, we find a common denominator $(x+1)^2$:

$f(x) + f'(x) = \frac{(x-1)(x+1)}{(x+1)^2} + \frac{2}{(x+1)^2}$

$f(x) + f'(x) = \frac{x^2 - 1 + 2}{(x+1)^2}$

$f(x) + f'(x) = \frac{x^2 + 1}{(x+1)^2}$

This is exactly the fractional part of our integrand.

So, the integrand is in the form $e^x [f(x) + f'(x)]$ with $f(x) = \frac{x-1}{x+1}$ and $f'(x) = \frac{2}{(x+1)^2}$.


Applying the standard integral formula (1):

$\int e^x \left( \frac{x^2 + 1}{(x + 1)^2} \right) \;dx = \int e^x \left( \frac{x-1}{x+1} + \frac{2}{(x+1)^2} \right) \;dx = e^x \left( \frac{x-1}{x+1} \right) + C$

where $C$ is the constant of integration.


Final Answer (ii):

The integral is $\int \frac{(x^2 + 1) e^x}{(x + 1)^2} \;dx = e^x \frac{x-1}{x+1} + C$.



Exercise 7.6

Integrate the functions in Exercises 1 to 22.

Question 1. x sin x

Answer:

We need to evaluate the integral $\int x \sin x \, dx$.


This integral can be solved using the method of integration by parts.

The formula for integration by parts is:

$\int u \, dv = uv - \int v \, du$


Let's choose our $u$ and $dv$. According to the LIATE rule (Logarithmic, Inverse trigonometric, Algebraic, Trigonometric, Exponential), we choose $u$ as the function that comes first in this order.

In this case, we have an algebraic function ($x$) and a trigonometric function ($\sin x$). Algebraic comes before trigonometric.

So, we set:

$u = x$

$dv = \sin x \, dx$


Now, we find $du$ by differentiating $u$ with respect to $x$, and $v$ by integrating $dv$ with respect to $x$.

Differentiating $u$:
$du = \frac{d}{dx}(x) \, dx = 1 \, dx = dx$

Integrating $dv$:
$v = \int \sin x \, dx = -\cos x$


Now, substitute these into the integration by parts formula $\int u \, dv = uv - \int v \, du$:

$\int x \sin x \, dx = (x)(-\cos x) - \int (-\cos x) \, dx$

$\int x \sin x \, dx = -x \cos x - \int (-\cos x) \, dx$

$\int x \sin x \, dx = -x \cos x + \int \cos x \, dx$


The remaining integral is $\int \cos x \, dx$, which is a standard integral.

$\int \cos x \, dx = \sin x + C$

(where $C$ is the constant of integration).


Substitute this back into the equation:

$\int x \sin x \, dx = -x \cos x + (\sin x + C)$

$\int x \sin x \, dx = -x \cos x + \sin x + C$


Thus, the integral of $x \sin x$ is $\sin x - x \cos x + C$.

Question 2. x sin 3x

Answer:

We need to evaluate the integral $\int x \sin 3x \, dx$.


We will use the method of integration by parts.

The formula for integration by parts is:

$\int u \, dv = uv - \int v \, du$


According to the LIATE rule, we choose:

$u = x$

$dv = \sin 3x \, dx$


Now, we find $du$ and $v$.

Differentiating $u$:

$du = \frac{d}{dx}(x) \, dx = dx$

Integrating $dv$:

$v = \int \sin 3x \, dx$

Using the standard integral $\int \sin(ax) \, dx = -\frac{1}{a} \cos(ax)$, we get:

$v = -\frac{1}{3} \cos 3x$


Substitute these into the integration by parts formula:

$\int x \sin 3x \, dx = (x)(-\frac{1}{3} \cos 3x) - \int (-\frac{1}{3} \cos 3x) \, dx$

$\int x \sin 3x \, dx = -\frac{1}{3} x \cos 3x - \int (-\frac{1}{3} \cos 3x) \, dx$

$\int x \sin 3x \, dx = -\frac{1}{3} x \cos 3x + \frac{1}{3} \int \cos 3x \, dx$


Now, evaluate the remaining integral $\int \cos 3x \, dx$.

Using the standard integral $\int \cos(ax) \, dx = \frac{1}{a} \sin(ax)$, we get:

$\int \cos 3x \, dx = \frac{1}{3} \sin 3x$


Substitute this result back into the expression:

$\int x \sin 3x \, dx = -\frac{1}{3} x \cos 3x + \frac{1}{3} \left( \frac{1}{3} \sin 3x \right) + C$

$\int x \sin 3x \, dx = -\frac{1}{3} x \cos 3x + \frac{1}{9} \sin 3x + C$


Where $C$ is the constant of integration.

The final answer is $\frac{1}{9} \sin 3x - \frac{1}{3} x \cos 3x + C$.

Question 3. x2 ex

Answer:

We need to evaluate the integral $\int x^2 e^x \, dx$.


We will use the method of integration by parts.

The formula for integration by parts is:

$\int u \, dv = uv - \int v \, du$


According to the LIATE rule, we choose $u = x^2$ (Algebraic) and $dv = e^x \, dx$ (Exponential).

So, we set:

$u = x^2$

$dv = e^x \, dx$


Now, we find $du$ and $v$.

Differentiating $u$:

$du = \frac{d}{dx}(x^2) \, dx = 2x \, dx$

Integrating $dv$:

$v = \int e^x \, dx = e^x$


Substitute these into the integration by parts formula:

$\int x^2 e^x \, dx = (x^2)(e^x) - \int (e^x)(2x) \, dx$

$\int x^2 e^x \, dx = x^2 e^x - \int 2x e^x \, dx$

$\int x^2 e^x \, dx = x^2 e^x - 2 \int x e^x \, dx$


Now we need to evaluate the new integral $\int x e^x \, dx$. This also requires integration by parts.

For $\int x e^x \, dx$, we set:

$u = x$

$dv = e^x \, dx$


Differentiating $u$:
$du = \frac{d}{dx}(x) \, dx = dx$

Integrating $dv$:
$v = \int e^x \, dx = e^x$


Apply integration by parts to $\int x e^x \, dx$:

$\int x e^x \, dx = (x)(e^x) - \int (e^x)(dx)$

$\int x e^x \, dx = x e^x - \int e^x \, dx$

$\int x e^x \, dx = x e^x - e^x + C_1$


Now, substitute this result back into the original equation:

$\int x^2 e^x \, dx = x^2 e^x - 2 \left( x e^x - e^x \right) + C$

$\int x^2 e^x \, dx = x^2 e^x - 2x e^x + 2e^x + C$


We can factor out $e^x$:

$\int x^2 e^x \, dx = e^x (x^2 - 2x + 2) + C$

Where $C$ is the constant of integration.

The final answer is $e^x (x^2 - 2x + 2) + C$.

Question 4. x log x

Answer:

We need to evaluate the integral $\int x \log x \, dx$.


We will use the method of integration by parts.

The formula for integration by parts is:

$\int u \, dv = uv - \int v \, du$


According to the LIATE rule, we choose the logarithmic function as $u$ and the algebraic function as $dv$.

So, we set:

$u = \log x$

$dv = x \, dx$


Now, we find $du$ and $v$.

Differentiating $u$:

$du = \frac{d}{dx}(\log x) \, dx = \frac{1}{x} \, dx$

Integrating $dv$:

$v = \int x \, dx = \frac{x^2}{2}$


Substitute these into the integration by parts formula:

$\int x \log x \, dx = (\log x)\left(\frac{x^2}{2}\right) - \int \left(\frac{x^2}{2}\right)\left(\frac{1}{x}\right) \, dx$

$\int x \log x \, dx = \frac{x^2}{2} \log x - \int \frac{x}{2} \, dx$

$\int x \log x \, dx = \frac{x^2}{2} \log x - \frac{1}{2} \int x \, dx$


Now, evaluate the remaining integral $\int x \, dx$.

$\int x \, dx = \frac{x^2}{2}$


Substitute this result back into the expression:

$\int x \log x \, dx = \frac{x^2}{2} \log x - \frac{1}{2} \left( \frac{x^2}{2} \right) + C$

$\int x \log x \, dx = \frac{x^2}{2} \log x - \frac{x^2}{4} + C$


Where $C$ is the constant of integration.

The final answer is $\frac{x^2}{2} \log x - \frac{x^2}{4} + C$.

Question 5. x log 2x

Answer:

We need to evaluate the integral $\int x \log 2x \, dx$.


We will use the method of integration by parts.

The formula for integration by parts is:

$\int u \, dv = uv - \int v \, du$


According to the LIATE rule, we choose the logarithmic function as $u$ and the algebraic function as $dv$.

So, we set:

$u = \log 2x$}

$dv = x \, dx$


Now, we find $du$ and $v$.

Differentiating $u$:

$du = \frac{d}{dx}(\log 2x) \, dx = \frac{1}{2x} \cdot 2 \, dx = \frac{1}{x} \, dx$

Integrating $dv$:

$v = \int x \, dx = \frac{x^2}{2}$


Substitute these into the integration by parts formula:

$\int x \log 2x \, dx = (\log 2x)\left(\frac{x^2}{2}\right) - \int \left(\frac{x^2}{2}\right)\left(\frac{1}{x}\right) \, dx$

$\int x \log 2x \, dx = \frac{x^2}{2} \log 2x - \int \frac{x}{2} \, dx$

$\int x \log 2x \, dx = \frac{x^2}{2} \log 2x - \frac{1}{2} \int x \, dx$


Now, evaluate the remaining integral $\int x \, dx$.

$\int x \, dx = \frac{x^2}{2}$


Substitute this result back into the expression:

$\int x \log 2x \, dx = \frac{x^2}{2} \log 2x - \frac{1}{2} \left( \frac{x^2}{2} \right) + C$

$\int x \log 2x \, dx = \frac{x^2}{2} \log 2x - \frac{x^2}{4} + C$


Where $C$ is the constant of integration.

The final answer is $\frac{x^2}{2} \log 2x - \frac{x^2}{4} + C$.

Question 6. x2 log x

Answer:

We need to evaluate the integral $\int x^2 \log x \, dx$.


We will use the method of integration by parts.

The formula for integration by parts is:

$\int u \, dv = uv - \int v \, du$


According to the LIATE rule, we choose the logarithmic function as $u$ and the algebraic function as $dv$.

So, we set:

$u = \log x$

$dv = x^2 \, dx$


Now, we find $du$ and $v$.

Differentiating $u$:

$du = \frac{d}{dx}(\log x) \, dx = \frac{1}{x} \, dx$

Integrating $dv$:

$v = \int x^2 \, dx = \frac{x^3}{3}$


Substitute these into the integration by parts formula:

$\int x^2 \log x \, dx = (\log x)\left(\frac{x^3}{3}\right) - \int \left(\frac{x^3}{3}\right)\left(\frac{1}{x}\right) \, dx$

$\int x^2 \log x \, dx = \frac{x^3}{3} \log x - \int \frac{x^2}{3} \, dx$

$\int x^2 \log x \, dx = \frac{x^3}{3} \log x - \frac{1}{3} \int x^2 \, dx$


Now, evaluate the remaining integral $\int x^2 \, dx$.

$\int x^2 \, dx = \frac{x^3}{3}$


Substitute this result back into the expression:

$\int x^2 \log x \, dx = \frac{x^3}{3} \log x - \frac{1}{3} \left( \frac{x^3}{3} \right) + C$

$\int x^2 \log x \, dx = \frac{x^3}{3} \log x - \frac{x^3}{9} + C$


Where $C$ is the constant of integration.

The final answer is $\frac{x^3}{3} \log x - \frac{x^3}{9} + C$.

Question 7. x sin–1 x

Answer:

We need to evaluate the integral $\int x \sin^{-1} x \, dx$.


We will use the method of integration by parts.

The formula for integration by parts is:

$\int u \, dv = uv - \int v \, du$


According to the LIATE rule (Logarithmic, Inverse trigonometric, Algebraic, Trigonometric, Exponential), the inverse trigonometric function comes before the algebraic function.

So, we set:

$u = \sin^{-1} x$

$dv = x \, dx$


Now, we find $du$ and $v$.

Differentiating $u$:

$du = \frac{d}{dx}(\sin^{-1} x) \, dx = \frac{1}{\sqrt{1 - x^2}} \, dx$

Integrating $dv$:

$v = \int x \, dx = \frac{x^2}{2}$


Substitute these into the integration by parts formula:

$\int x \sin^{-1} x \, dx = (\sin^{-1} x)\left(\frac{x^2}{2}\right) - \int \left(\frac{x^2}{2}\right)\left(\frac{1}{\sqrt{1 - x^2}}\right) \, dx$

$\int x \sin^{-1} x \, dx = \frac{x^2}{2} \sin^{-1} x - \frac{1}{2} \int \frac{x^2}{\sqrt{1 - x^2}} \, dx$}


Now we need to evaluate the integral $I_1 = \int \frac{x^2}{\sqrt{1 - x^2}} \, dx$. We can use a trigonometric substitution.

Let $x = \sin \theta$. Then $dx = \cos \theta \, d\theta$.

Also, $\sqrt{1 - x^2} = \sqrt{1 - \sin^2 \theta} = \sqrt{\cos^2 \theta} = |\cos \theta|$. Assuming $-\frac{\pi}{2} \leq \theta \leq \frac{\pi}{2}$, $\cos \theta \geq 0$, so $\sqrt{1 - x^2} = \cos \theta$.

The integral becomes:

$I_1 = \int \frac{\sin^2 \theta}{\cos \theta} (\cos \theta \, d\theta) = \int \sin^2 \theta \, d\theta$


Use the identity $\sin^2 \theta = \frac{1 - \cos 2\theta}{2}$:

$I_1 = \int \frac{1 - \cos 2\theta}{2} \, d\theta = \frac{1}{2} \int (1 - \cos 2\theta) \, d\theta$

$I_1 = \frac{1}{2} \left( \theta - \frac{1}{2} \sin 2\theta \right)$

Using the identity $\sin 2\theta = 2 \sin \theta \cos \theta$:

$I_1 = \frac{1}{2} \left( \theta - \frac{1}{2} (2 \sin \theta \cos \theta) \right) = \frac{1}{2} (\theta - \sin \theta \cos \theta)$


Substitute back in terms of $x$. Since $x = \sin \theta$, we have $\theta = \sin^{-1} x$. Also, $\cos \theta = \sqrt{1 - x^2}$.

$I_1 = \frac{1}{2} (\sin^{-1} x - x \sqrt{1 - x^2})$


Now, substitute $I_1$ back into the original integral expression:

$\int x \sin^{-1} x \, dx = \frac{x^2}{2} \sin^{-1} x - \frac{1}{2} I_1$

$\int x \sin^{-1} x \, dx = \frac{x^2}{2} \sin^{-1} x - \frac{1}{2} \left[ \frac{1}{2} (\sin^{-1} x - x \sqrt{1 - x^2}) \right] + C$

$\int x \sin^{-1} x \, dx = \frac{x^2}{2} \sin^{-1} x - \frac{1}{4} \sin^{-1} x + \frac{x \sqrt{1 - x^2}}{4} + C$


Combine the terms with $\sin^{-1} x$:

$\int x \sin^{-1} x \, dx = \left(\frac{x^2}{2} - \frac{1}{4}\right) \sin^{-1} x + \frac{x \sqrt{1 - x^2}}{4} + C$

$\int x \sin^{-1} x \, dx = \frac{2x^2 - 1}{4} \sin^{-1} x + \frac{x \sqrt{1 - x^2}}{4} + C$


Where $C$ is the constant of integration.

The final answer is $\frac{2x^2 - 1}{4} \sin^{-1} x + \frac{x \sqrt{1 - x^2}}{4} + C$.

Question 8. x tan–1 x

Answer:

We need to evaluate the integral $\int x \tan^{-1} x \, dx$.


We will use the method of integration by parts.

The formula for integration by parts is:

$\int u \, dv = uv - \int v \, du$


According to the LIATE rule, we choose the inverse trigonometric function as $u$ and the algebraic function as $dv$.

So, we set:

$u = \tan^{-1} x$

$dv = x \, dx$


Now, we find $du$ and $v$.

Differentiating $u$:

$du = \frac{d}{dx}(\tan^{-1} x) \, dx = \frac{1}{1 + x^2} \, dx$

Integrating $dv$:

$v = \int x \, dx = \frac{x^2}{2}$


Substitute these into the integration by parts formula:

$\int x \tan^{-1} x \, dx = (\tan^{-1} x)\left(\frac{x^2}{2}\right) - \int \left(\frac{x^2}{2}\right)\left(\frac{1}{1 + x^2}\right) \, dx$

$\int x \tan^{-1} x \, dx = \frac{x^2}{2} \tan^{-1} x - \frac{1}{2} \int \frac{x^2}{1 + x^2} \, dx$}


Now we need to evaluate the integral $I_1 = \int \frac{x^2}{1 + x^2} \, dx$.

We can rewrite the integrand:

$\frac{x^2}{1 + x^2} = \frac{(1 + x^2) - 1}{1 + x^2} = \frac{1 + x^2}{1 + x^2} - \frac{1}{1 + x^2} = 1 - \frac{1}{1 + x^2}$


Now integrate this expression:

$I_1 = \int \left(1 - \frac{1}{1 + x^2}\right) \, dx$}

$I_1 = \int 1 \, dx - \int \frac{1}{1 + x^2} \, dx$

$I_1 = x - \tan^{-1} x$


Substitute $I_1$ back into the main equation:

$\int x \tan^{-1} x \, dx = \frac{x^2}{2} \tan^{-1} x - \frac{1}{2} (x - \tan^{-1} x) + C$

$\int x \tan^{-1} x \, dx = \frac{x^2}{2} \tan^{-1} x - \frac{x}{2} + \frac{1}{2} \tan^{-1} x + C$


Combine the terms with $\tan^{-1} x$:

$\int x \tan^{-1} x \, dx = \left(\frac{x^2}{2} + \frac{1}{2}\right) \tan^{-1} x - \frac{x}{2} + C$

$\int x \tan^{-1} x \, dx = \frac{x^2 + 1}{2} \tan^{-1} x - \frac{x}{2} + C$


Where $C$ is the constant of integration.

The final answer is $\frac{x^2 + 1}{2} \tan^{-1} x - \frac{x}{2} + C$.

Question 9. x cos–1 x

Answer:

We need to evaluate the integral $\int x \cos^{-1} x \, dx$.


We will use the method of integration by parts.

The formula for integration by parts is:

$\int u \, dv = uv - \int v \, du$


According to the LIATE rule (Logarithmic, Inverse trigonometric, Algebraic, Trigonometric, Exponential), the inverse trigonometric function comes before the algebraic function.

So, we set:

$u = \cos^{-1} x$

$dv = x \, dx$


Now, we find $du$ and $v$.

Differentiating $u$:

$du = \frac{d}{dx}(\cos^{-1} x) \, dx = -\frac{1}{\sqrt{1 - x^2}} \, dx$}

Integrating $dv$:

$v = \int x \, dx = \frac{x^2}{2}$


Substitute these into the integration by parts formula:

$\int x \cos^{-1} x \, dx = (\cos^{-1} x)\left(\frac{x^2}{2}\right) - \int \left(\frac{x^2}{2}\right)\left(-\frac{1}{\sqrt{1 - x^2}}\right) \, dx$}

$\int x \cos^{-1} x \, dx = \frac{x^2}{2} \cos^{-1} x - \int -\frac{x^2}{2\sqrt{1 - x^2}} \, dx$}

$\int x \cos^{-1} x \, dx = \frac{x^2}{2} \cos^{-1} x + \frac{1}{2} \int \frac{x^2}{\sqrt{1 - x^2}} \, dx$}


Now we need to evaluate the integral $I_1 = \int \frac{x^2}{\sqrt{1 - x^2}} \, dx$. We can use a trigonometric substitution.

Let $x = \sin \theta$. Then $dx = \cos \theta \, d\theta$.

Also, $\sqrt{1 - x^2} = \sqrt{1 - \sin^2 \theta} = \sqrt{\cos^2 \theta} = |\cos \theta|$. Assuming $-\frac{\pi}{2} \leq \theta \leq \frac{\pi}{2}$ (which corresponds to $-1 \leq x \leq 1$), $\cos \theta \geq 0$, so $\sqrt{1 - x^2} = \cos \theta$.

The integral becomes:

$I_1 = \int \frac{\sin^2 \theta}{\cos \theta} (\cos \theta \, d\theta) = \int \sin^2 \theta \, d\theta$


Use the identity $\sin^2 \theta = \frac{1 - \cos 2\theta}{2}$:

$I_1 = \int \frac{1 - \cos 2\theta}{2} \, d\theta = \frac{1}{2} \int (1 - \cos 2\theta) \, d\theta$

$I_1 = \frac{1}{2} \left( \int 1 \, d\theta - \int \cos 2\theta \, d\theta \right)$

$I_1 = \frac{1}{2} \left( \theta - \frac{1}{2} \sin 2\theta \right)$

Using the identity $\sin 2\theta = 2 \sin \theta \cos \theta$:

$I_1 = \frac{1}{2} \left( \theta - \frac{1}{2} (2 \sin \theta \cos \theta) \right) = \frac{1}{2} (\theta - \sin \theta \cos \theta)$


Substitute back in terms of $x$. Since $x = \sin \theta$, we have $\theta = \sin^{-1} x$. Also, $\cos \theta = \sqrt{1 - x^2}$.

$I_1 = \frac{1}{2} (\sin^{-1} x - x \sqrt{1 - x^2})$


Now, substitute $I_1$ back into the original integral expression:

$\int x \cos^{-1} x \, dx = \frac{x^2}{2} \cos^{-1} x + \frac{1}{2} I_1$

$\int x \cos^{-1} x \, dx = \frac{x^2}{2} \cos^{-1} x + \frac{1}{2} \left[ \frac{1}{2} (\sin^{-1} x - x \sqrt{1 - x^2}) \right] + C$

$\int x \cos^{-1} x \, dx = \frac{x^2}{2} \cos^{-1} x + \frac{1}{4} \sin^{-1} x - \frac{x \sqrt{1 - x^2}}{4} + C$


We can use the identity $\sin^{-1} x = \frac{\pi}{2} - \cos^{-1} x$ to express the answer solely in terms of $\cos^{-1} x$.

$\int x \cos^{-1} x \, dx = \frac{x^2}{2} \cos^{-1} x + \frac{1}{4} \left(\frac{\pi}{2} - \cos^{-1} x\right) - \frac{x \sqrt{1 - x^2}}{4} + C$

$\int x \cos^{-1} x \, dx = \frac{x^2}{2} \cos^{-1} x + \frac{\pi}{8} - \frac{1}{4} \cos^{-1} x - \frac{x \sqrt{1 - x^2}}{4} + C$

Combine the terms with $\cos^{-1} x$:

$\int x \cos^{-1} x \, dx = \left(\frac{x^2}{2} - \frac{1}{4}\right) \cos^{-1} x - \frac{x \sqrt{1 - x^2}}{4} + \frac{\pi}{8} + C$

$\int x \cos^{-1} x \, dx = \frac{2x^2 - 1}{4} \cos^{-1} x - \frac{x \sqrt{1 - x^2}}{4} + C'$


Where $C$ is the constant of integration and $C' = C + \frac{\pi}{8}$ is also a constant of integration.

The final answer is $\frac{2x^2 - 1}{4} \cos^{-1} x - \frac{x \sqrt{1 - x^2}}{4} + C$.

Question 10. (sin–1 x)2

Answer:

We need to evaluate the integral $\int (\sin^{-1} x)^2 \, dx$.


We will use the method of integration by parts. We can write the integrand as $(\sin^{-1} x)^2 \cdot 1$.

The formula for integration by parts is:

$\int u \, dv = uv - \int v \, du$


According to the LIATE rule, we choose the inverse trigonometric function as $u$ and the algebraic function (constant 1) as $dv$.

So, we set:

$u = (\sin^{-1} x)^2$

$dv = 1 \, dx$


Now, we find $du$ and $v$.

Differentiating $u$ using the chain rule:

$du = \frac{d}{dx}((\sin^{-1} x)^2) \, dx = 2 (\sin^{-1} x) \cdot \frac{d}{dx}(\sin^{-1} x) \, dx$

$du = 2 (\sin^{-1} x) \cdot \frac{1}{\sqrt{1 - x^2}} \, dx$

Integrating $dv$:

$v = \int 1 \, dx = x$


Substitute these into the integration by parts formula:

$\int (\sin^{-1} x)^2 \cdot 1 \, dx = ((\sin^{-1} x)^2)(x) - \int (x)\left(2 (\sin^{-1} x) \frac{1}{\sqrt{1 - x^2}}\right) \, dx$

$\int (\sin^{-1} x)^2 \, dx = x (\sin^{-1} x)^2 - 2 \int \frac{x \sin^{-1} x}{\sqrt{1 - x^2}} \, dx$}


Now we need to evaluate the new integral $I_1 = \int \frac{x \sin^{-1} x}{\sqrt{1 - x^2}} \, dx$.

We can use a substitution. Let $y = \sin^{-1} x$. Then $dy = \frac{1}{\sqrt{1 - x^2}} \, dx$. Also, $x = \sin y$.

Substitute these into the integral $I_1$:

$I_1 = \int x \cdot \sin^{-1} x \cdot \frac{1}{\sqrt{1 - x^2}} \, dx = \int (\sin y) \cdot (y) \, dy$

$I_1 = \int y \sin y \, dy$


This is an integral we evaluated in Question 1. Using integration by parts with $u' = y$ and $dv' = \sin y \, dy$, we get $du' = dy$ and $v' = -\cos y$.

$\int y \sin y \, dy = y(-\cos y) - \int (-\cos y) \, dy$

$\int y \sin y \, dy = -y \cos y + \int \cos y \, dy$

$\int y \sin y \, dy = -y \cos y + \sin y + C_1$


Now, substitute back $y = \sin^{-1} x$. Since $x = \sin y$, we have $\sin y = x$. For $\cos y$, we use $\cos y = \sqrt{1 - \sin^2 y} = \sqrt{1 - x^2}$ (assuming the principal value of $\sin^{-1} x$).

$I_1 = -(\sin^{-1} x) \sqrt{1 - x^2} + x + C_1$


Substitute $I_1$ back into the original integral expression:

$\int (\sin^{-1} x)^2 \, dx = x (\sin^{-1} x)^2 - 2 I_1$

$\int (\sin^{-1} x)^2 \, dx = x (\sin^{-1} x)^2 - 2 \left( -(\sin^{-1} x) \sqrt{1 - x^2} + x \right) + C$

$\int (\sin^{-1} x)^2 \, dx = x (\sin^{-1} x)^2 + 2 (\sin^{-1} x) \sqrt{1 - x^2} - 2x + C$


Where $C$ is the constant of integration (absorbing $-2C_1$).

The final answer is $x (\sin^{-1} x)^2 + 2 \sqrt{1 - x^2} \sin^{-1} x - 2x + C$.

Question 11. $\frac{x \cos^{−1} x}{\sqrt{1 − x^2}}$

Answer:

We need to evaluate the integral $\int \frac{x \cos^{-1} x}{\sqrt{1 - x^2}} \, dx$.


This integral can be solved using a substitution method followed by integration by parts.


Let $u = \cos^{-1} x$.

Differentiating both sides with respect to $x$, we get:

$\frac{du}{dx} = \frac{d}{dx}(\cos^{-1} x) = -\frac{1}{\sqrt{1 - x^2}}$

So, $du = -\frac{1}{\sqrt{1 - x^2}} \, dx$, which means $\frac{1}{\sqrt{1 - x^2}} \, dx = -du$.

From the substitution $u = \cos^{-1} x$, we also have $x = \cos u$.}


Now, substitute these into the integral:

$\int \frac{x \cos^{-1} x}{\sqrt{1 - x^2}} \, dx = \int (\cos^{-1} x) \cdot x \cdot \left(\frac{1}{\sqrt{1 - x^2}} \, dx\right)$

$= \int (u) \cdot (\cos u) \cdot (-du)$

$= - \int u \cos u \, du$


Now we need to evaluate the integral $- \int u \cos u \, du$. We use integration by parts for this integral.

The formula for integration by parts is:

$\int w \, dz = wz - \int z \, dw$


For the integral $\int u \cos u \, du$, let:

$w = u$

$dz = \cos u \, du$


Differentiating $w$ and integrating $dz$:

$dw = \frac{d}{du}(u) \, du = du$

$z = \int \cos u \, du = \sin u$


Substitute these into the integration by parts formula:

$\int u \cos u \, du = (u)(\sin u) - \int (\sin u) \, du$

$\int u \cos u \, du = u \sin u - (-\cos u) + C_1$

$\int u \cos u \, du = u \sin u + \cos u + C_1$


Now, we need to find $- \int u \cos u \, du$:

$- \int u \cos u \, du = -(u \sin u + \cos u) + C$

$- \int u \cos u \, du = -u \sin u - \cos u + C$


Finally, substitute back $u = \cos^{-1} x$. We also need to express $\sin u$ in terms of $x$.

Since $u = \cos^{-1} x$, we have $\cos u = x$.

Using the identity $\sin^2 u + \cos^2 u = 1$, we get $\sin^2 u = 1 - \cos^2 u = 1 - x^2$.

For the principal value of $\cos^{-1} x$, $0 \leq u \leq \pi$, which means $\sin u \geq 0$.

So, $\sin u = \sqrt{1 - x^2}$.


Substitute $u = \cos^{-1} x$, $\sin u = \sqrt{1 - x^2}$, and $\cos u = x$ back into the result:

$-u \sin u - \cos u + C = -(\cos^{-1} x) \sqrt{1 - x^2} - x + C$


Where $C$ is the constant of integration.

The final answer is $-x - \sqrt{1 - x^2} \cos^{-1} x + C$.

Question 12. x sec2 x

Answer:

We need to evaluate the integral $\int x \sec^2 x \, dx$.


We will use the method of integration by parts for this integral.

The formula for integration by parts is given by:

$\int u \, dv = uv - \int v \, du$


According to the LIATE rule (Logarithmic, Inverse trigonometric, Algebraic, Trigonometric, Exponential) for choosing $u$, we have an algebraic term ($x$) and a trigonometric term ($\sec^2 x$). The algebraic term comes before the trigonometric term in LIATE.

So, we set:

$u = x$

$dv = \sec^2 x \, dx$


Now, we need to find $du$ by differentiating $u$ with respect to $x$, and find $v$ by integrating $dv$ with respect to $x$.

Differentiating $u$:

$du = \frac{d}{dx}(x) \, dx = 1 \, dx = dx$

Integrating $dv$:

$v = \int \sec^2 x \, dx$

The integral of $\sec^2 x$ is $\tan x$.

$v = \tan x$


Now, substitute $u=x$, $dv=\sec^2 x \, dx$, $du=dx$, and $v=\tan x$ into the integration by parts formula $\int u \, dv = uv - \int v \, du$:

$\int x \sec^2 x \, dx = (x)(\tan x) - \int (\tan x) \, dx$

$\int x \sec^2 x \, dx = x \tan x - \int \tan x \, dx$


Next, we need to evaluate the remaining integral $\int \tan x \, dx$.

We know that $\tan x = \frac{\sin x}{\cos x}$. So the integral is $\int \frac{\sin x}{\cos x} \, dx$.

Let $t = \cos x$. Then $dt = \frac{d}{dx}(\cos x) \, dx = -\sin x \, dx$. This means $\sin x \, dx = -dt$.

The integral becomes $\int \frac{-dt}{t} = - \int \frac{1}{t} \, dt = -\log |t| + C_1$.

Substituting back $t = \cos x$, we get $\int \tan x \, dx = -\log |\cos x|$.


Substitute this result back into the expression from the integration by parts formula:

$\int x \sec^2 x \, dx = x \tan x - (-\log |\cos x|) + C$

$\int x \sec^2 x \, dx = x \tan x + \log |\cos x| + C$

Where $C$ is the constant of integration.


The final answer is $\mathbf{x \tan x + \log |\cos x| + C}$.

Question 13. tan–1 x

Answer:

We need to evaluate the integral $\int \tan^{-1} x \, dx$.


We will use the method of integration by parts. We can write the integrand as $\tan^{-1} x \cdot 1$.

The formula for integration by parts is:

$\int u \, dv = uv - \int v \, du$


According to the LIATE rule, we choose the inverse trigonometric function as $u$ and the algebraic function (constant 1) as $dv$.

So, we set:

$u = \tan^{-1} x$

$dv = 1 \, dx$


Now, we find $du$ and $v$.

Differentiating $u$:

$du = \frac{d}{dx}(\tan^{-1} x) \, dx = \frac{1}{1 + x^2} \, dx$}

Integrating $dv$:

$v = \int 1 \, dx = x$


Substitute these into the integration by parts formula:

$\int \tan^{-1} x \cdot 1 \, dx = (\tan^{-1} x)(x) - \int (x)\left(\frac{1}{1 + x^2}\right) \, dx$}

$\int \tan^{-1} x \, dx = x \tan^{-1} x - \int \frac{x}{1 + x^2} \, dx$}


Now we need to evaluate the remaining integral $I_1 = \int \frac{x}{1 + x^2} \, dx$.

We can use a substitution method. Let $w = 1 + x^2$.

Differentiating $w$ with respect to $x$: $dw = 2x \, dx$.

So, $x \, dx = \frac{1}{2} dw$.


Substitute these into the integral $I_1$:

$I_1 = \int \frac{1}{1 + x^2} (x \, dx) = \int \frac{1}{w} \left(\frac{1}{2} dw\right)$

$I_1 = \frac{1}{2} \int \frac{1}{w} \, dw$

$I_1 = \frac{1}{2} \log |w| + C_1$

Substitute back $w = 1 + x^2$:

$I_1 = \frac{1}{2} \log |1 + x^2| + C_1$. Since $1 + x^2 > 0$ for all real $x$, we have $|1 + x^2| = 1 + x^2$.

$I_1 = \frac{1}{2} \log (1 + x^2) + C_1$


Substitute $I_1$ back into the main equation:

$\int \tan^{-1} x \, dx = x \tan^{-1} x - \left(\frac{1}{2} \log (1 + x^2)\right) + C$

$\int \tan^{-1} x \, dx = x \tan^{-1} x - \frac{1}{2} \log (1 + x^2) + C$


Where $C$ is the constant of integration (absorbing $-C_1$).

The final answer is $x \tan^{-1} x - \frac{1}{2} \log (1 + x^2) + C$.

Question 14. x (log x)2

Answer:

We need to evaluate the integral $\int x (\log x)^2 \, dx$.


We will use the method of integration by parts.

The formula for integration by parts is:

$\int u \, dv = uv - \int v \, du$


According to the LIATE rule, we choose the logarithmic function as $u$ and the algebraic function as $dv$.

So, we set:

$u = (\log x)^2$

$dv = x \, dx$


Now, we find $du$ and $v$.

Differentiating $u$ using the chain rule:

$du = \frac{d}{dx}((\log x)^2) \, dx = 2 (\log x) \cdot \frac{d}{dx}(\log x) \, dx$

$du = 2 (\log x) \cdot \frac{1}{x} \, dx = \frac{2 \log x}{x} \, dx$

Integrating $dv$:

$v = \int x \, dx = \frac{x^2}{2}$


Substitute these into the integration by parts formula:

$\int x (\log x)^2 \, dx = ((\log x)^2)\left(\frac{x^2}{2}\right) - \int \left(\frac{x^2}{2}\right)\left(\frac{2 \log x}{x}\right) \, dx$

$\int x (\log x)^2 \, dx = \frac{x^2}{2} (\log x)^2 - \int x \log x \, dx$}


Now we need to evaluate the new integral $I_1 = \int x \log x \, dx$. This also requires integration by parts.

For $\int x \log x \, dx$, we set $u' = \log x$ and $dv' = x \, dx$.

Differentiating $u'$: $du' = \frac{1}{x} \, dx$.

Integrating $dv'$: $v' = \frac{x^2}{2}$.


Apply integration by parts to $\int x \log x \, dx$:

$I_1 = (\log x)\left(\frac{x^2}{2}\right) - \int \left(\frac{x^2}{2}\right)\left(\frac{1}{x}\right) \, dx$

$I_1 = \frac{x^2}{2} \log x - \int \frac{x}{2} \, dx$

$I_1 = \frac{x^2}{2} \log x - \frac{1}{2} \int x \, dx$

$I_1 = \frac{x^2}{2} \log x - \frac{1}{2} \left(\frac{x^2}{2}\right) + C_1$

$I_1 = \frac{x^2}{2} \log x - \frac{x^2}{4} + C_1$


Now, substitute $I_1$ back into the original equation:

$\int x (\log x)^2 \, dx = \frac{x^2}{2} (\log x)^2 - \left( \frac{x^2}{2} \log x - \frac{x^2}{4} \right) + C$

$\int x (\log x)^2 \, dx = \frac{x^2}{2} (\log x)^2 - \frac{x^2}{2} \log x + \frac{x^2}{4} + C$


Where $C$ is the constant of integration (absorbing $-C_1$).

The final answer is $\frac{x^2}{2} (\log x)^2 - \frac{x^2}{2} \log x + \frac{x^2}{4} + C$.

Question 15. (x2 + 1) log x

Answer:

We need to evaluate the integral $\int (x^2 + 1) \log x \, dx$.


We can rewrite the integral as $\int (x^2 \log x + \log x) \, dx = \int x^2 \log x \, dx + \int \log x \, dx$.

We will evaluate each integral separately using integration by parts.


Integral 1: $\int x^2 \log x \, dx$

Using integration by parts $\int u \, dv = uv - \int v \, du$, with $u = \log x$ and $dv = x^2 \, dx$.

$du = \frac{1}{x} \, dx$ and $v = \int x^2 \, dx = \frac{x^3}{3}$.

$\int x^2 \log x \, dx = (\log x)\left(\frac{x^3}{3}\right) - \int \left(\frac{x^3}{3}\right)\left(\frac{1}{x}\right) \, dx$

$\int x^2 \log x \, dx = \frac{x^3}{3} \log x - \int \frac{x^2}{3} \, dx$

$\int x^2 \log x \, dx = \frac{x^3}{3} \log x - \frac{1}{3} \int x^2 \, dx$

$\int x^2 \log x \, dx = \frac{x^3}{3} \log x - \frac{1}{3} \left(\frac{x^3}{3}\right) + C_1$}

$\int x^2 \log x \, dx = \frac{x^3}{3} \log x - \frac{x^3}{9} + C_1$


Integral 2: $\int \log x \, dx$

Using integration by parts $\int u \, dv = uv - \int v \, du$, with $u = \log x$ and $dv = 1 \, dx$.

$du = \frac{1}{x} \, dx$ and $v = \int 1 \, dx = x$.

$\int \log x \, dx = (\log x)(x) - \int (x)\left(\frac{1}{x}\right) \, dx$

$\int \log x \, dx = x \log x - \int 1 \, dx$

$\int \log x \, dx = x \log x - x + C_2$


Now, combine the results of the two integrals:

$\int (x^2 + 1) \log x \, dx = \left(\frac{x^3}{3} \log x - \frac{x^3}{9}\right) + (x \log x - x) + C$

$\int (x^2 + 1) \log x \, dx = \frac{x^3}{3} \log x + x \log x - \frac{x^3}{9} - x + C$


We can factor out $\log x$ from the first two terms:

$\int (x^2 + 1) \log x \, dx = \left(\frac{x^3}{3} + x\right) \log x - \frac{x^3}{9} - x + C$

$\int (x^2 + 1) \log x \, dx = x\left(\frac{x^2}{3} + 1\right) \log x - \frac{x^3}{9} - x + C$


Where $C = C_1 + C_2$ is the constant of integration.

The final answer is $x\left(\frac{x^2}{3} + 1\right) \log x - \frac{x^3}{9} - x + C$.

Question 16. ex (sin x + cos x)

Answer:

We need to evaluate the integral $\int e^x (\sin x + \cos x) \, dx$.


This integral is of the form $\int e^x (f(x) + f'(x)) \, dx$.}


We know the standard result for this form is:

$\int e^x (f(x) + f'(x)) \, dx = e^x f(x) + C$


Let's identify $f(x)$ and $f'(x)$ in the given integral $e^x (\sin x + \cos x)$.

Let $f(x) = \sin x$.}

Then, the derivative of $f(x)$ is $f'(x) = \frac{d}{dx}(\sin x) = \cos x$.


The integral is $\int e^x (\sin x + \cos x) \, dx$. This perfectly matches the form with $f(x) = \sin x$ and $f'(x) = \cos x$.}


Using the standard result, we have:

$\int e^x (\sin x + \cos x) \, dx = e^x (\sin x) + C$

$\int e^x (\sin x + \cos x) \, dx = e^x \sin x + C$


Where $C$ is the constant of integration.

The final answer is $e^x \sin x + C$.

Question 17. $\frac{x e^x}{(1+ x)^2}$

Answer:

We need to evaluate the integral $\int \frac{x e^x}{(1 + x)^2} \, dx$.


This integral is of the form $\int e^x f(x) \, dx$. We want to manipulate the expression to fit the form $\int e^x (f(x) + f'(x)) \, dx$.


Let's rewrite the term $\frac{x}{(1 + x)^2}$:

$\frac{x}{(1 + x)^2} = \frac{(1 + x) - 1}{(1 + x)^2} = \frac{1 + x}{(1 + x)^2} - \frac{1}{(1 + x)^2}$

$\frac{x}{(1 + x)^2} = \frac{1}{1 + x} - \frac{1}{(1 + x)^2}$


So the integral becomes:

$\int \frac{x e^x}{(1 + x)^2} \, dx = \int e^x \left( \frac{1}{1 + x} - \frac{1}{(1 + x)^2} \right) \, dx$}

$\int \frac{x e^x}{(1 + x)^2} \, dx = \int e^x \left( \frac{1}{1 + x} + \left(-\frac{1}{(1 + x)^2}\right) \right) \, dx$}


This integral is now in the form $\int e^x (f(x) + f'(x)) \, dx$, where:

$f(x) = \frac{1}{1 + x}$

Let's check the derivative of $f(x)$:

$f'(x) = \frac{d}{dx}\left(\frac{1}{1 + x}\right) = \frac{d}{dx}((1 + x)^{-1})$

$f'(x) = -1 (1 + x)^{-2} \cdot \frac{d}{dx}(1 + x) = -1 (1 + x)^{-2} \cdot 1$

$f'(x) = -\frac{1}{(1 + x)^2}$


Indeed, the derivative of $f(x) = \frac{1}{1 + x}$ is $f'(x) = -\frac{1}{(1 + x)^2}$.


Using the standard result $\int e^x (f(x) + f'(x)) \, dx = e^x f(x) + C$, we have:

$\int e^x \left( \frac{1}{1 + x} + \left(-\frac{1}{(1 + x)^2}\right) \right) \, dx = e^x \left(\frac{1}{1 + x}\right) + C$}

$\int \frac{x e^x}{(1 + x)^2} \, dx = \frac{e^x}{1 + x} + C$


Where $C$ is the constant of integration.

The final answer is $\frac{e^x}{1 + x} + C$.

Question 18. $e^x \left( \frac{1 + \sin x}{1 + \cos x} \right)$

Answer:

We need to evaluate the integral $\int e^x \left( \frac{1 + \sin x}{1 + \cos x} \right) \, dx$.


This integral is of the form $\int e^x (f(x)) \, dx$. We will try to rewrite the expression $\frac{1 + \sin x}{1 + \cos x}$ to fit the form $f(x) + f'(x)$.


Let's simplify the fraction using half-angle formulas:

Recall that $\sin x = 2 \sin \frac{x}{2} \cos \frac{x}{2}$ and $1 + \cos x = 2 \cos^2 \frac{x}{2}$.

So, $\frac{1 + \sin x}{1 + \cos x} = \frac{1 + 2 \sin \frac{x}{2} \cos \frac{x}{2}}{2 \cos^2 \frac{x}{2}}$


Split the fraction into two terms:

$\frac{1 + 2 \sin \frac{x}{2} \cos \frac{x}{2}}{2 \cos^2 \frac{x}{2}} = \frac{1}{2 \cos^2 \frac{x}{2}} + \frac{2 \sin \frac{x}{2} \cos \frac{x}{2}}{2 \cos^2 \frac{x}{2}}$


Simplify each term:

$\frac{1}{2 \cos^2 \frac{x}{2}} = \frac{1}{2} \sec^2 \frac{x}{2}$

$\frac{2 \sin \frac{x}{2} \cos \frac{x}{2}}{2 \cos^2 \frac{x}{2}} = \frac{\sin \frac{x}{2}}{\cos \frac{x}{2}} = \tan \frac{x}{2}$


Thus, $\frac{1 + \sin x}{1 + \cos x} = \tan \frac{x}{2} + \frac{1}{2} \sec^2 \frac{x}{2}$.


The integral becomes:

$\int e^x \left( \tan \frac{x}{2} + \frac{1}{2} \sec^2 \frac{x}{2} \right) \, dx$


This integral is of the form $\int e^x (f(x) + f'(x)) \, dx$.

Let $f(x) = \tan \frac{x}{2}$.

Then, the derivative $f'(x)$ is:

$f'(x) = \frac{d}{dx}\left(\tan \frac{x}{2}\right)$

Using the chain rule, $\frac{d}{dx}(\tan u) = \sec^2 u \frac{du}{dx}$. Here $u = \frac{x}{2}$, so $\frac{du}{dx} = \frac{1}{2}$.

$f'(x) = \sec^2 \frac{x}{2} \cdot \frac{1}{2} = \frac{1}{2} \sec^2 \frac{x}{2}$


We see that the integrand is $e^x (f(x) + f'(x))$ with $f(x) = \tan \frac{x}{2}$ and $f'(x) = \frac{1}{2} \sec^2 \frac{x}{2}$.


Using the standard result for this form:

$\int e^x (f(x) + f'(x)) \, dx = e^x f(x) + C$


Applying the result:

$\int e^x \left( \tan \frac{x}{2} + \frac{1}{2} \sec^2 \frac{x}{2} \right) \, dx = e^x \tan \frac{x}{2} + C$


Where $C$ is the constant of integration.

The final answer is $e^x \tan \frac{x}{2} + C$.

Question 19. $e^x \left( \frac{1}{x} − \frac{1}{x^2} \right)$

Answer:

We need to evaluate the integral $\int e^x \left( \frac{1}{x} - \frac{1}{x^2} \right) \, dx$.


This integral is of the form $\int e^x (f(x)) \, dx$. We should check if it is in the form $\int e^x (f(x) + f'(x)) \, dx$.}


Let's identify the terms in the parenthesis:

$f(x) = \frac{1}{x}$

Now, find the derivative of $f(x)$:

$f'(x) = \frac{d}{dx}\left(\frac{1}{x}\right) = \frac{d}{dx}(x^{-1})$

$f'(x) = -1 \cdot x^{-2} = -\frac{1}{x^2}$


The expression inside the parenthesis is $\frac{1}{x} - \frac{1}{x^2}$, which can be written as $\frac{1}{x} + \left(-\frac{1}{x^2}\right)$.

This perfectly matches $f(x) + f'(x)$ with $f(x) = \frac{1}{x}$.


Using the standard result for integrals of the form $\int e^x (f(x) + f'(x)) \, dx$:

$\int e^x (f(x) + f'(x)) \, dx = e^x f(x) + C$


Applying the result to our integral:

$\int e^x \left( \frac{1}{x} + \left(-\frac{1}{x^2}\right) \right) \, dx = e^x \left(\frac{1}{x}\right) + C$}

$\int e^x \left( \frac{1}{x} - \frac{1}{x^2} \right) \, dx = \frac{e^x}{x} + C$}


Where $C$ is the constant of integration.

The final answer is $\frac{e^x}{x} + C$.

Question 20. $\frac{(x − 3) e^x}{(x − 1)^3}$

Answer:

We need to evaluate the integral $\int \frac{(x - 3) e^x}{(x - 1)^3} \, dx$.


This integral is of the form $\int e^x g(x) \, dx$, where $g(x) = \frac{x - 3}{(x - 1)^3}$. We will try to rewrite $g(x)$ in the form $f(x) + f'(x)$.


Let's manipulate the fraction $\frac{x - 3}{(x - 1)^3}$. We can rewrite the numerator in terms of $(x - 1)$:

$x - 3 = (x - 1) - 2$


Substitute this into the fraction:

$\frac{x - 3}{(x - 1)^3} = \frac{(x - 1) - 2}{(x - 1)^3}$


Now, split the fraction into two terms:

$\frac{(x - 1) - 2}{(x - 1)^3} = \frac{x - 1}{(x - 1)^3} - \frac{2}{(x - 1)^3}$


Simplify the first term:

$\frac{x - 1}{(x - 1)^3} = \frac{1}{(x - 1)^2}$


So, the expression inside the integral becomes:

$\frac{x - 3}{(x - 1)^3} = \frac{1}{(x - 1)^2} - \frac{2}{(x - 1)^3}$


The integral is now:

$\int e^x \left( \frac{1}{(x - 1)^2} - \frac{2}{(x - 1)^3} \right) \, dx$}

We can write this as:

$\int e^x \left( \frac{1}{(x - 1)^2} + \left(-\frac{2}{(x - 1)^3}\right) \right) \, dx$}


This integral is in the standard form $\int e^x (f(x) + f'(x)) \, dx$.

Let $f(x) = \frac{1}{(x - 1)^2} = (x - 1)^{-2}$.

Let's find the derivative of $f(x)$:

$f'(x) = \frac{d}{dx}((x - 1)^{-2})$

Using the power rule and chain rule, $\frac{d}{dx}(u^n) = n u^{n-1} \frac{du}{dx}$ where $u = x - 1$ and $n = -2$.

$f'(x) = -2 (x - 1)^{-2 - 1} \cdot \frac{d}{dx}(x - 1)$

$f'(x) = -2 (x - 1)^{-3} \cdot 1$

$f'(x) = -\frac{2}{(x - 1)^3}$


Indeed, the expression is of the form $e^x (f(x) + f'(x))$, where $f(x) = \frac{1}{(x - 1)^2}$ and $f'(x) = -\frac{2}{(x - 1)^3}$.


Using the standard result for this type of integral:

$\int e^x (f(x) + f'(x)) \, dx = e^x f(x) + C$


Applying the result to our integral:

$\int e^x \left( \frac{1}{(x - 1)^2} + \left(-\frac{2}{(x - 1)^3}\right) \right) \, dx = e^x \left(\frac{1}{(x - 1)^2}\right) + C$}

$\int \frac{(x - 3) e^x}{(x - 1)^3} \, dx = \frac{e^x}{(x - 1)^2} + C$


Where $C$ is the constant of integration.

The final answer is $\frac{e^x}{(x - 1)^2} + C$.

Question 21. e2x sin x

Answer:

We need to evaluate the integral $\int e^{2x} \sin x \, dx$.


We will use the method of integration by parts. This integral requires applying integration by parts twice.

The formula for integration by parts is:

$\int u \, dv = uv - \int v \, du$


Let $I = \int e^{2x} \sin x \, dx$.

For the first application of integration by parts, let's choose:

$u = \sin x$

$dv = e^{2x} \, dx$


Now, we find $du$ and $v$.

Differentiating $u$:

$du = \frac{d}{dx}(\sin x) \, dx = \cos x \, dx$

Integrating $dv$:

$v = \int e^{2x} \, dx = \frac{1}{2} e^{2x}$


Substitute these into the integration by parts formula:

$I = (\sin x)\left(\frac{1}{2} e^{2x}\right) - \int \left(\frac{1}{2} e^{2x}\right)(\cos x \, dx)$

$I = \frac{1}{2} e^{2x} \sin x - \frac{1}{2} \int e^{2x} \cos x \, dx$}

$$I = \frac{1}{2} e^{2x} \sin x - \frac{1}{2} \int e^{2x} \cos x \, dx$$

... (1)


Now we need to evaluate the new integral $\int e^{2x} \cos x \, dx$. We apply integration by parts again.

For $\int e^{2x} \cos x \, dx$, let:

$u' = \cos x$

$dv' = e^{2x} \, dx$


Differentiating $u'$ and integrating $dv'$:

$du' = \frac{d}{dx}(\cos x) \, dx = -\sin x \, dx$

$v' = \int e^{2x} \, dx = \frac{1}{2} e^{2x}$


Substitute these into the integration by parts formula for $\int u' \, dv'$:

$\int e^{2x} \cos x \, dx = (\cos x)\left(\frac{1}{2} e^{2x}\right) - \int \left(\frac{1}{2} e^{2x}\right)(-\sin x \, dx)$

$\int e^{2x} \cos x \, dx = \frac{1}{2} e^{2x} \cos x - \int -\frac{1}{2} e^{2x} \sin x \, dx$}

$\int e^{2x} \cos x \, dx = \frac{1}{2} e^{2x} \cos x + \frac{1}{2} \int e^{2x} \sin x \, dx$}


Notice that the integral on the right side is the original integral $I = \int e^{2x} \sin x \, dx$.

So, $\int e^{2x} \cos x \, dx = \frac{1}{2} e^{2x} \cos x + \frac{1}{2} I$.}

$$\int e^{2x} \cos x \, dx = \frac{1}{2} e^{2x} \cos x + \frac{1}{2} I$$

... (2)


Substitute the result from equation (2) back into equation (1):

$I = \frac{1}{2} e^{2x} \sin x - \frac{1}{2} \left( \frac{1}{2} e^{2x} \cos x + \frac{1}{2} I \right)$}

$I = \frac{1}{2} e^{2x} \sin x - \frac{1}{4} e^{2x} \cos x - \frac{1}{4} I$}


Now, solve for $I$ by moving the terms involving $I$ to one side:

$I + \frac{1}{4} I = \frac{1}{2} e^{2x} \sin x - \frac{1}{4} e^{2x} \cos x$}

$\frac{5}{4} I = \frac{1}{2} e^{2x} \sin x - \frac{1}{4} e^{2x} \cos x$}


Multiply both sides by $\frac{4}{5}$ to find $I$:

$I = \frac{4}{5} \left( \frac{1}{2} e^{2x} \sin x - \frac{1}{4} e^{2x} \cos x \right) + C$

$I = \frac{4}{5} \cdot \frac{1}{2} e^{2x} \sin x - \frac{4}{5} \cdot \frac{1}{4} e^{2x} \cos x + C$

$I = \frac{2}{5} e^{2x} \sin x - \frac{1}{5} e^{2x} \cos x + C$


We can factor out $\frac{e^{2x}}{5}$:

$I = \frac{e^{2x}}{5} (2 \sin x - \cos x) + C$


Where $C$ is the constant of integration.

The final answer is $\frac{e^{2x}}{5} (2 \sin x - \cos x) + C$.

Question 22. $\sin^{-1} \left( \frac{2x}{1 + x^2} \right)$

Answer:

We need to evaluate the integral $\int \sin^{-1} \left( \frac{2x}{1 + x^2} \right) \, dx$.


Let's use a trigonometric substitution to simplify the term inside the inverse sine function.

Let $x = \tan \theta$.}

Then, $dx = \sec^2 \theta \, d\theta$.}


Substitute $x = \tan \theta$ into the expression $\frac{2x}{1 + x^2}$:

$\frac{2x}{1 + x^2} = \frac{2 \tan \theta}{1 + \tan^2 \theta}$

Using the identity $1 + \tan^2 \theta = \sec^2 \theta$, we get:

$\frac{2 \tan \theta}{\sec^2 \theta} = \frac{2 \frac{\sin \theta}{\cos \theta}}{\frac{1}{\cos^2 \theta}} = 2 \frac{\sin \theta}{\cos \theta} \cos^2 \theta = 2 \sin \theta \cos \theta$}

Using the double angle identity $\sin 2\theta = 2 \sin \theta \cos \theta$, we have:

$\frac{2x}{1 + x^2} = \sin 2\theta$}


So the integrand becomes $\sin^{-1}(\sin 2\theta)$.

For the principal value branch of $\sin^{-1} y$, $\sin^{-1}(\sin u) = u$ if $-\frac{\pi}{2} \leq u \leq \frac{\pi}{2}$.

Assuming $-1 \leq x \leq 1$, we have $-\frac{\pi}{4} \leq \tan^{-1} x \leq \frac{\pi}{4}$, which means $-\frac{\pi}{4} \leq \theta \leq \frac{\pi}{4}$.

Multiplying by 2, we get $-\frac{\pi}{2} \leq 2\theta \leq \frac{\pi}{2}$.

In this range, $\sin^{-1}(\sin 2\theta) = 2\theta$.}


Since $x = \tan \theta$, we have $\theta = \tan^{-1} x$.}

So, $\sin^{-1} \left( \frac{2x}{1 + x^2} \right) = 2 \tan^{-1} x$, under the assumption $-1 \leq x \leq 1$.


The integral becomes $\int 2 \tan^{-1} x \, dx = 2 \int \tan^{-1} x \, dx$.}


We need to evaluate $\int \tan^{-1} x \, dx$. We use integration by parts, setting $u = \tan^{-1} x$ and $dv = 1 \, dx$.

$du = \frac{1}{1 + x^2} \, dx$}

$v = \int 1 \, dx = x$}


Applying the integration by parts formula $\int u \, dv = uv - \int v \, du$:

$\int \tan^{-1} x \, dx = (\tan^{-1} x)(x) - \int (x)\left(\frac{1}{1 + x^2}\right) \, dx$}

$\int \tan^{-1} x \, dx = x \tan^{-1} x - \int \frac{x}{1 + x^2} \, dx$}


To evaluate $\int \frac{x}{1 + x^2} \, dx$, we use the substitution $w = 1 + x^2$, so $dw = 2x \, dx$, which means $x \, dx = \frac{1}{2} dw$.}

$\int \frac{x}{1 + x^2} \, dx = \int \frac{1}{w} \left(\frac{1}{2} dw\right) = \frac{1}{2} \int \frac{1}{w} \, dw = \frac{1}{2} \log |w| + C_1 = \frac{1}{2} \log (1 + x^2) + C_1$ (since $1 + x^2 > 0$).


Substituting this back into the expression for $\int \tan^{-1} x \, dx$:

$\int \tan^{-1} x \, dx = x \tan^{-1} x - \frac{1}{2} \log (1 + x^2) + C'$ (where $C'$ includes $-C_1$).


Finally, multiply by 2 to get the original integral:

$\int \sin^{-1} \left( \frac{2x}{1 + x^2} \right) \, dx = 2 \int \tan^{-1} x \, dx$}

$= 2 \left( x \tan^{-1} x - \frac{1}{2} \log (1 + x^2) \right) + C$}

$= 2x \tan^{-1} x - \log (1 + x^2) + C$}


Where $C$ is the constant of integration.

Note: This solution is valid assuming $-1 \leq x \leq 1$, where $\sin^{-1} \left( \frac{2x}{1 + x^2} \right) = 2 \tan^{-1} x$. For $|x| > 1$, the identity is different.

The final answer is $2x \tan^{-1} x - \log (1 + x^2) + C$.

Choose the correct answer in Exercises 23 and 24.

Question 23. $\int x^2 e^{x^{3}} \;dx$ eqyals

(A) $\frac{1}{3} e^{x^{3}}+ C$

(B) $\frac{1}{3} e^{x^{2}}+ C$

(C) $\frac{1}{2} e^{x^{3}} + C$

(D) $\frac{1}{2} e^{x^{2}} + C$

Answer:

We need to evaluate the integral $\int x^2 e^{x^{3}} \, dx$.


We can solve this integral using the substitution method.

Let $u = x^3$.


Differentiate $u$ with respect to $x$:

$\frac{du}{dx} = \frac{d}{dx}(x^3) = 3x^2$


Rearranging the differential, we get:

$du = 3x^2 \, dx$

So, $x^2 \, dx = \frac{1}{3} du$.


Substitute $u = x^3$ and $x^2 \, dx = \frac{1}{3} du$ into the integral:

$\int x^2 e^{x^3} \, dx = \int e^{x^3} (x^2 \, dx)$

$= \int e^u \left(\frac{1}{3} du\right)$

$= \frac{1}{3} \int e^u \, du$


Now, integrate with respect to $u$:

$\frac{1}{3} \int e^u \, du = \frac{1}{3} e^u + C$


Substitute back $u = x^3$:

$\frac{1}{3} e^{x^3} + C$


Comparing this result with the given options, we see that it matches option (A).


The correct answer is:

(A) $\frac{1}{3} e^{x^{3}}+ C$

Question 24. $\int e^x \sec x (1 + \tan x) \;dx$ equals

(A) $e^x \cos x + C$

(B) $e^x \sec x + C$

(C) $e^x \sin x + C$

(D) $e^x \tan x + C$

Answer:

We need to evaluate the integral $\int e^x \sec x (1 + \tan x) \, dx$.


First, let's expand the term $\sec x (1 + \tan x)$:

$\sec x (1 + \tan x) = \sec x \cdot 1 + \sec x \cdot \tan x = \sec x + \sec x \tan x$


So the integral becomes:

$\int e^x (\sec x + \sec x \tan x) \, dx$


This integral is of the form $\int e^x (f(x) + f'(x)) \, dx$.

Let's check if we can identify a function $f(x)$ such that its derivative $f'(x)$ is the other term in the parenthesis.

Consider $f(x) = \sec x$.}

Let's find the derivative of $f(x)$:

$f'(x) = \frac{d}{dx}(\sec x) = \sec x \tan x$


The integrand is $e^x (\sec x + \sec x \tan x)$, which matches the form $e^x (f(x) + f'(x))$ with $f(x) = \sec x$ and $f'(x) = \sec x \tan x$.}


We use the standard result for integrals of this form:

$\int e^x (f(x) + f'(x)) \, dx = e^x f(x) + C$


Applying this result, the integral is:

$\int e^x (\sec x + \sec x \tan x) \, dx = e^x (\sec x) + C$

$\int e^x \sec x (1 + \tan x) \, dx = e^x \sec x + C$


Where $C$ is the constant of integration.

Comparing this result with the given options, we see that it matches option (B).


The correct answer is:

(B) $e^x \sec x + C$



Example 23 & 24 (Before Exercise 7.7)

Example 23: Find $\int \sqrt{x^2 + 2x + 5} \;dx$

Answer:

We need to evaluate the integral $\int \sqrt{x^2 + 2x + 5} \, dx$.


We will first complete the square for the quadratic expression inside the square root.

$x^2 + 2x + 5 = (x^2 + 2x + 1) - 1 + 5$

$x^2 + 2x + 5 = (x + 1)^2 + 4$


The integral becomes $\int \sqrt{(x + 1)^2 + 4} \, dx$.


This integral is in the form $\int \sqrt{u^2 + a^2} \, du$, where $u = x + 1$ and $a^2 = 4$, so $a = 2$.

Note that if $u = x + 1$, then $du = dx$.


We use the standard integral formula:

$\int \sqrt{u^2 + a^2} \, du = \frac{u}{2} \sqrt{u^2 + a^2} + \frac{a^2}{2} \log |u + \sqrt{u^2 + a^2}| + C$


Substitute $u = x + 1$ and $a = 2$ into the formula:

$\int \sqrt{(x + 1)^2 + 4} \, dx = \frac{x + 1}{2} \sqrt{(x + 1)^2 + 4} + \frac{2^2}{2} \log |(x + 1) + \sqrt{(x + 1)^2 + 4}| + C$


Simplify the expression under the square root:

$(x + 1)^2 + 4 = x^2 + 2x + 1 + 4 = x^2 + 2x + 5$


So, the integral is:

$\int \sqrt{x^2 + 2x + 5} \, dx = \frac{x + 1}{2} \sqrt{x^2 + 2x + 5} + \frac{4}{2} \log |x + 1 + \sqrt{x^2 + 2x + 5}| + C$

$\int \sqrt{x^2 + 2x + 5} \, dx = \frac{x + 1}{2} \sqrt{x^2 + 2x + 5} + 2 \log |x + 1 + \sqrt{x^2 + 2x + 5}| + C$


Where $C$ is the constant of integration.

The final answer is $\frac{x + 1}{2} \sqrt{x^2 + 2x + 5} + 2 \log |x + 1 + \sqrt{x^2 + 2x + 5}| + C$.

Example 24: Find $\int \sqrt{3 − 2x − x^2} \;dx$

Answer:

We need to evaluate the integral $\int \sqrt{3 - 2x - x^2} \, dx$.


We will first complete the square for the quadratic expression inside the square root.

$3 - 2x - x^2 = -(x^2 + 2x - 3)$

$= -(x^2 + 2x + 1 - 1 - 3)$

$= -((x + 1)^2 - 4)$

$= 4 - (x + 1)^2$


The integral becomes $\int \sqrt{4 - (x + 1)^2} \, dx$.


This integral is in the form $\int \sqrt{a^2 - u^2} \, du$, where $u = x + 1$ and $a^2 = 4$, so $a = 2$.

If $u = x + 1$, then $du = dx$.


We use the standard integral formula:

$\int \sqrt{a^2 - u^2} \, du = \frac{u}{2} \sqrt{a^2 - u^2} + \frac{a^2}{2} \sin^{-1} \frac{u}{a} + C$


Substitute $u = x + 1$ and $a = 2$ into the formula:

$\int \sqrt{4 - (x + 1)^2} \, dx = \frac{x + 1}{2} \sqrt{4 - (x + 1)^2} + \frac{2^2}{2} \sin^{-1} \left(\frac{x + 1}{2}\right) + C$


Substitute back the original expression for $4 - (x + 1)^2$:

$4 - (x + 1)^2 = 4 - (x^2 + 2x + 1) = 4 - x^2 - 2x - 1 = 3 - 2x - x^2$


So, the integral is:

$\int \sqrt{3 - 2x - x^2} \, dx = \frac{x + 1}{2} \sqrt{3 - 2x - x^2} + \frac{4}{2} \sin^{-1} \left(\frac{x + 1}{2}\right) + C$

$\int \sqrt{3 - 2x - x^2} \, dx = \frac{x + 1}{2} \sqrt{3 - 2x - x^2} + 2 \sin^{-1} \left(\frac{x + 1}{2}\right) + C$


Where $C$ is the constant of integration.

The final answer is $\frac{x + 1}{2} \sqrt{3 - 2x - x^2} + 2 \sin^{-1} \left(\frac{x + 1}{2}\right) + C$.



Exercise 7.7

Integrate the functions in Exercises 1 to 9.

Question 1. $\sqrt{4 − x^2}$

Answer:

Let the given integral be $I$.

$I = \int \sqrt{4 - x^2} dx$


We can write $4$ as $2^2$. So the integral becomes:

$I = \int \sqrt{2^2 - x^2} dx$


This integral is of the form $\int \sqrt{a^2 - x^2} dx$.

The standard formula for this type of integral is:

$\int \sqrt{a^2 - x^2} dx = \frac{x}{2}\sqrt{a^2 - x^2} + \frac{a^2}{2}\sin^{-1}(\frac{x}{a}) + C$


In our case, $a^2 = 4$, which implies $a = 2$.

Substituting $a=2$ into the formula, we get:

$I = \frac{x}{2}\sqrt{2^2 - x^2} + \frac{2^2}{2}\sin^{-1}(\frac{x}{2}) + C$


Simplifying the expression:

$I = \frac{x}{2}\sqrt{4 - x^2} + \frac{4}{2}\sin^{-1}(\frac{x}{2}) + C$

$I = \frac{x}{2}\sqrt{4 - x^2} + 2\sin^{-1}(\frac{x}{2}) + C$


Where $C$ is the constant of integration.


Thus, the integral of $\sqrt{4 - x^2}$ is:

$\int \sqrt{4 - x^2} dx = \frac{x}{2}\sqrt{4 - x^2} + 2\sin^{-1}(\frac{x}{2}) + C$

Question 2. $\sqrt{1 − 4x^2}$

Answer:

Let the given integral be $I$.

$I = \int \sqrt{1 - 4x^2} dx$


We can rewrite the integrand as $\sqrt{1^2 - (2x)^2}$.

This integral is of the form $\int \sqrt{a^2 - (bx)^2} dx$.


To evaluate this, we can use a substitution.

Let $u = 2x$.

Then, the differential $du$ is given by $du = 2 dx$.

This implies $dx = \frac{1}{2} du$.


Substituting $u = 2x$ and $dx = \frac{1}{2} du$ into the integral, we get:

$I = \int \sqrt{1^2 - u^2} \left(\frac{1}{2} du\right)$

$I = \frac{1}{2} \int \sqrt{1^2 - u^2} du$


The integral is now in the standard form $\int \sqrt{a^2 - u^2} du$ with $a = 1$.

Using the standard formula:

$\int \sqrt{a^2 - u^2} du = \frac{u}{2}\sqrt{a^2 - u^2} + \frac{a^2}{2}\sin^{-1}(\frac{u}{a}) + C$


Applying this formula with $a = 1$, we get:

$I = \frac{1}{2} \left( \frac{u}{2}\sqrt{1^2 - u^2} + \frac{1^2}{2}\sin^{-1}(\frac{u}{1}) \right) + C'$

$I = \frac{1}{2} \left( \frac{u}{2}\sqrt{1 - u^2} + \frac{1}{2}\sin^{-1}(u) \right) + C'$


Now, substitute back $u = 2x$ into the expression:

$I = \frac{1}{2} \left( \frac{2x}{2}\sqrt{1 - (2x)^2} + \frac{1}{2}\sin^{-1}(2x) \right) + C'$

$I = \frac{1}{2} \left( x\sqrt{1 - 4x^2} + \frac{1}{2}\sin^{-1}(2x) \right) + C'$


Distribute the $\frac{1}{2}$ outside the parenthesis:

$I = \frac{x}{2}\sqrt{1 - 4x^2} + \frac{1}{4}\sin^{-1}(2x) + \frac{1}{2}C'$

Let $C = \frac{1}{2}C'$, which is still an arbitrary constant of integration.

$I = \frac{x}{2}\sqrt{1 - 4x^2} + \frac{1}{4}\sin^{-1}(2x) + C$


Where $C$ is the constant of integration.


Thus, the integral of $\sqrt{1 - 4x^2}$ is:

$\int \sqrt{1 - 4x^2} dx = \frac{x}{2}\sqrt{1 - 4x^2} + \frac{1}{4}\sin^{-1}(2x) + C$

Question 3. $\sqrt{x^2 + 4x + 6}$

Answer:

Let the given integral be $I$.

$I = \int \sqrt{x^2 + 4x + 6} dx$


We complete the square for the quadratic expression inside the square root:

$x^2 + 4x + 6 = (x^2 + 4x + 4) - 4 + 6$

$x^2 + 4x + 6 = (x+2)^2 + 2$


Substitute this back into the integral:

$I = \int \sqrt{(x+2)^2 + 2} dx$


This integral is of the form $\int \sqrt{u^2 + a^2} du$.

Let $u = x+2$. Then $du = dx$.

Also, $a^2 = 2$, so $a = \sqrt{2}$.


The integral becomes:

$I = \int \sqrt{u^2 + (\sqrt{2})^2} du$


Using the standard formula for integrals of the form $\int \sqrt{u^2 + a^2} du$:

$\int \sqrt{u^2 + a^2} du = \frac{u}{2}\sqrt{u^2 + a^2} + \frac{a^2}{2}\log|u + \sqrt{u^2 + a^2}| + C'$


Substitute $u = x+2$ and $a^2 = 2$ back into the formula:

$I = \frac{x+2}{2}\sqrt{(x+2)^2 + 2} + \frac{2}{2}\log|(x+2) + \sqrt{(x+2)^2 + 2}| + C$


Simplify the expression. Recall that $(x+2)^2 + 2 = x^2 + 4x + 6$.

$I = \frac{x+2}{2}\sqrt{x^2 + 4x + 6} + 1 \cdot \log|(x+2) + \sqrt{x^2 + 4x + 6}| + C$

$I = \frac{x+2}{2}\sqrt{x^2 + 4x + 6} + \log|x+2 + \sqrt{x^2 + 4x + 6}| + C$


Where $C$ is the constant of integration.


Thus, the integral of $\sqrt{x^2 + 4x + 6}$ is:

$\int \sqrt{x^2 + 4x + 6} dx = \frac{x+2}{2}\sqrt{x^2 + 4x + 6} + \log|x+2 + \sqrt{x^2 + 4x + 6}| + C$

Question 4. $\sqrt{x^2 + 4x + 1}$

Answer:

Let the given integral be $I$.

$I = \int \sqrt{x^2 + 4x + 1} dx$


We complete the square for the quadratic expression inside the square root:

$x^2 + 4x + 1 = (x^2 + 4x + 4) - 4 + 1$

$x^2 + 4x + 1 = (x+2)^2 - 3$


Substitute this back into the integral:

$I = \int \sqrt{(x+2)^2 - 3} dx$


This integral is of the form $\int \sqrt{u^2 - a^2} du$.

Let $u = x+2$. Then $du = dx$.

Also, $a^2 = 3$, so $a = \sqrt{3}$.


The integral becomes:

$I = \int \sqrt{u^2 - (\sqrt{3})^2} du$


Using the standard formula for integrals of the form $\int \sqrt{u^2 - a^2} du$:

$\int \sqrt{u^2 - a^2} du = \frac{u}{2}\sqrt{u^2 - a^2} - \frac{a^2}{2}\log|u + \sqrt{u^2 - a^2}| + C'$


Substitute $u = x+2$ and $a^2 = 3$ back into the formula:

$I = \frac{x+2}{2}\sqrt{(x+2)^2 - 3} - \frac{3}{2}\log|(x+2) + \sqrt{(x+2)^2 - 3}| + C$


Simplify the expression. Recall that $(x+2)^2 - 3 = x^2 + 4x + 1$.

$I = \frac{x+2}{2}\sqrt{x^2 + 4x + 1} - \frac{3}{2}\log|x+2 + \sqrt{x^2 + 4x + 1}| + C$


Where $C$ is the constant of integration.


Thus, the integral of $\sqrt{x^2 + 4x + 1}$ is:

$\int \sqrt{x^2 + 4x + 1} dx = \frac{x+2}{2}\sqrt{x^2 + 4x + 1} - \frac{3}{2}\log|x+2 + \sqrt{x^2 + 4x + 1}| + C$

Question 5. $\sqrt{1 − 4x − x^2}$

Answer:

Let the given integral be $I$.

$I = \int \sqrt{1 - 4x - x^2} dx$


We complete the square for the quadratic expression inside the square root. First, rewrite the expression by taking out the negative sign from the $x$ terms:

$1 - 4x - x^2 = 1 - (x^2 + 4x)$

Now, complete the square for $x^2 + 4x$:

$x^2 + 4x = x^2 + 4x + 2^2 - 2^2 = (x+2)^2 - 4$

Substitute this back into the original expression:

$1 - (x^2 + 4x) = 1 - ((x+2)^2 - 4)$

$1 - (x+2)^2 + 4 = 5 - (x+2)^2$


Substitute this back into the integral:

$I = \int \sqrt{5 - (x+2)^2} dx$


This integral is of the form $\int \sqrt{a^2 - u^2} du$.

Let $u = x+2$. Then $du = dx$.

Also, $a^2 = 5$, so $a = \sqrt{5}$.


The integral becomes:

$I = \int \sqrt{(\sqrt{5})^2 - u^2} du$


Using the standard formula for integrals of the form $\int \sqrt{a^2 - u^2} du$:

$\int \sqrt{a^2 - u^2} du = \frac{u}{2}\sqrt{a^2 - u^2} + \frac{a^2}{2}\sin^{-1}(\frac{u}{a}) + C'$


Substitute $u = x+2$ and $a^2 = 5$ (and $a = \sqrt{5}$) back into the formula:

$I = \frac{x+2}{2}\sqrt{5 - (x+2)^2} + \frac{5}{2}\sin^{-1}\left(\frac{x+2}{\sqrt{5}}\right) + C$


Simplify the expression. Recall that $5 - (x+2)^2 = 1 - 4x - x^2$.

$I = \frac{x+2}{2}\sqrt{1 - 4x - x^2} + \frac{5}{2}\sin^{-1}\left(\frac{x+2}{\sqrt{5}}\right) + C$


Where $C$ is the constant of integration.


Thus, the integral of $\sqrt{1 - 4x - x^2}$ is:

$\int \sqrt{1 - 4x - x^2} dx = \frac{x+2}{2}\sqrt{1 - 4x - x^2} + \frac{5}{2}\sin^{-1}\left(\frac{x+2}{\sqrt{5}}\right) + C$

Question 6. $\sqrt{x^2 + 4x − 5}$

Answer:

Let the given integral be $I$.

$I = \int \sqrt{x^2 + 4x - 5} dx$


We complete the square for the quadratic expression inside the square root:

$x^2 + 4x - 5 = (x^2 + 4x + 4) - 4 - 5$

$x^2 + 4x - 5 = (x+2)^2 - 9$


Substitute this back into the integral:

$I = \int \sqrt{(x+2)^2 - 9} dx$


We can write $9$ as $3^2$.

$I = \int \sqrt{(x+2)^2 - 3^2} dx$


This integral is of the form $\int \sqrt{u^2 - a^2} du$.

Let $u = x+2$. Then $du = dx$.

Also, $a = 3$.


The integral becomes:

$I = \int \sqrt{u^2 - a^2} du$


Using the standard formula for integrals of the form $\int \sqrt{u^2 - a^2} du$:

$\int \sqrt{u^2 - a^2} du = \frac{u}{2}\sqrt{u^2 - a^2} - \frac{a^2}{2}\log|u + \sqrt{u^2 - a^2}| + C'$


Substitute $u = x+2$ and $a=3$ back into the formula:

$I = \frac{x+2}{2}\sqrt{(x+2)^2 - 3^2} - \frac{3^2}{2}\log|(x+2) + \sqrt{(x+2)^2 - 3^2}| + C$


Simplify the expression. Recall that $(x+2)^2 - 3^2 = x^2 + 4x - 5$.

$I = \frac{x+2}{2}\sqrt{x^2 + 4x - 5} - \frac{9}{2}\log|x+2 + \sqrt{x^2 + 4x - 5}| + C$


Where $C$ is the constant of integration.


Thus, the integral of $\sqrt{x^2 + 4x - 5}$ is:

$\int \sqrt{x^2 + 4x - 5} dx = \frac{x+2}{2}\sqrt{x^2 + 4x - 5} - \frac{9}{2}\log|x+2 + \sqrt{x^2 + 4x - 5}| + C$

Question 7. $\sqrt{1 + 3x − x^2}$

Answer:

Let the given integral be $I$.

$I = \int \sqrt{1 + 3x - x^2} dx$


We complete the square for the quadratic expression inside the square root.

First, rewrite the expression by taking out the negative sign from the $x$ terms:

$1 + 3x - x^2 = 1 - (x^2 - 3x)$

Complete the square for $x^2 - 3x$ by adding and subtracting $(\frac{-3}{2})^2 = \frac{9}{4}$:

$x^2 - 3x = x^2 - 3x + \left(\frac{3}{2}\right)^2 - \left(\frac{3}{2}\right)^2 = \left(x - \frac{3}{2}\right)^2 - \frac{9}{4}$

Substitute this back into the original expression:

$1 - (x^2 - 3x) = 1 - \left(\left(x - \frac{3}{2}\right)^2 - \frac{9}{4}\right)$

$= 1 - \left(x - \frac{3}{2}\right)^2 + \frac{9}{4}$

$= \frac{4}{4} + \frac{9}{4} - \left(x - \frac{3}{2}\right)^2$

$= \frac{13}{4} - \left(x - \frac{3}{2}\right)^2$


Substitute this back into the integral:

$I = \int \sqrt{\frac{13}{4} - \left(x - \frac{3}{2}\right)^2} dx$


This integral is of the form $\int \sqrt{a^2 - u^2} du$.

Let $u = x - \frac{3}{2}$. Then $du = dx$.

Let $a^2 = \frac{13}{4}$, so $a = \sqrt{\frac{13}{4}} = \frac{\sqrt{13}}{2}$.


The integral becomes:

$I = \int \sqrt{a^2 - u^2} du$


Using the standard formula for integrals of the form $\int \sqrt{a^2 - u^2} du$:

$\int \sqrt{a^2 - u^2} du = \frac{u}{2}\sqrt{a^2 - u^2} + \frac{a^2}{2}\sin^{-1}\left(\frac{u}{a}\right) + C'$


Substitute $u = x - \frac{3}{2}$, $a^2 = \frac{13}{4}$, and $a = \frac{\sqrt{13}}{2}$ back into the formula:

$I = \frac{x - \frac{3}{2}}{2}\sqrt{\frac{13}{4} - \left(x - \frac{3}{2}\right)^2} + \frac{\frac{13}{4}}{2}\sin^{-1}\left(\frac{x - \frac{3}{2}}{\frac{\sqrt{13}}{2}}\right) + C$


Simplify the expression:

$\frac{x - \frac{3}{2}}{2} = \frac{\frac{2x-3}{2}}{2} = \frac{2x-3}{4}$

$\sqrt{\frac{13}{4} - \left(x - \frac{3}{2}\right)^2} = \sqrt{1 + 3x - x^2}$

$\frac{\frac{13}{4}}{2} = \frac{13}{8}$

$\frac{x - \frac{3}{2}}{\frac{\sqrt{13}}{2}} = \frac{\frac{2x-3}{2}}{\frac{\sqrt{13}}{2}} = \frac{2x-3}{\sqrt{13}}$


Substituting these back into the formula for $I$:

$I = \frac{2x-3}{4}\sqrt{1 + 3x - x^2} + \frac{13}{8}\sin^{-1}\left(\frac{2x-3}{\sqrt{13}}\right) + C$


Where $C$ is the constant of integration.


Thus, the integral of $\sqrt{1 + 3x - x^2}$ is:

$\int \sqrt{1 + 3x - x^2} dx = \frac{2x-3}{4}\sqrt{1 + 3x - x^2} + \frac{13}{8}\sin^{-1}\left(\frac{2x-3}{\sqrt{13}}\right) + C$

Question 8. $\sqrt{x^2 + 3x}$

Answer:

Let the given integral be $I$.

$I = \int \sqrt{x^2 + 3x} dx$


We complete the square for the expression inside the square root.

To complete the square for $x^2 + 3x$, we add and subtract the square of half of the coefficient of $x$, which is $\left(\frac{3}{2}\right)^2 = \frac{9}{4}$.

$x^2 + 3x = x^2 + 3x + \frac{9}{4} - \frac{9}{4}$

$x^2 + 3x = \left(x + \frac{3}{2}\right)^2 - \frac{9}{4}$


Substitute this back into the integral:

$I = \int \sqrt{\left(x + \frac{3}{2}\right)^2 - \frac{9}{4}} dx$


We can write $\frac{9}{4}$ as $\left(\frac{3}{2}\right)^2$.

$I = \int \sqrt{\left(x + \frac{3}{2}\right)^2 - \left(\frac{3}{2}\right)^2} dx$


This integral is of the form $\int \sqrt{u^2 - a^2} du$.

Let $u = x + \frac{3}{2}$. Then $du = dx$.

Let $a = \frac{3}{2}$.


The integral becomes:

$I = \int \sqrt{u^2 - a^2} du$


Using the standard formula for integrals of the form $\int \sqrt{u^2 - a^2} du$:

$\int \sqrt{u^2 - a^2} du = \frac{u}{2}\sqrt{u^2 - a^2} - \frac{a^2}{2}\log|u + \sqrt{u^2 - a^2}| + C'$


Substitute $u = x + \frac{3}{2}$ and $a = \frac{3}{2}$ back into the formula:

$I = \frac{x + \frac{3}{2}}{2}\sqrt{\left(x + \frac{3}{2}\right)^2 - \left(\frac{3}{2}\right)^2} - \frac{\left(\frac{3}{2}\right)^2}{2}\log\left|x + \frac{3}{2} + \sqrt{\left(x + \frac{3}{2}\right)^2 - \left(\frac{3}{2}\right)^2}\right| + C$


Simplify the expression:

$\frac{x + \frac{3}{2}}{2} = \frac{\frac{2x+3}{2}}{2} = \frac{2x+3}{4}$

$\sqrt{\left(x + \frac{3}{2}\right)^2 - \left(\frac{3}{2}\right)^2} = \sqrt{x^2 + 3x}$

$\frac{\left(\frac{3}{2}\right)^2}{2} = \frac{\frac{9}{4}}{2} = \frac{9}{8}$


Substituting these back into the formula for $I$:

$I = \frac{2x+3}{4}\sqrt{x^2 + 3x} - \frac{9}{8}\log\left|x + \frac{3}{2} + \sqrt{x^2 + 3x}\right| + C$


Where $C$ is the constant of integration.


Thus, the integral of $\sqrt{x^2 + 3x}$ is:

$\int \sqrt{x^2 + 3x} dx = \frac{2x+3}{4}\sqrt{x^2 + 3x} - \frac{9}{8}\log\left|x + \frac{3}{2} + \sqrt{x^2 + 3x}\right| + C$

Question 9. $\sqrt{1 + \frac{x^2}{9}}$

Answer:

Let the given integral be $I$.

$I = \int \sqrt{1 + \frac{x^2}{9}} dx$


We can rewrite the expression inside the square root:

$1 + \frac{x^2}{9} = \frac{9 + x^2}{9} = \frac{1}{9}(9 + x^2)$


Substitute this back into the integral:

$I = \int \sqrt{\frac{1}{9}(9 + x^2)} dx$

$I = \int \sqrt{\frac{1}{9}}\sqrt{9 + x^2} dx$

$I = \int \frac{1}{3}\sqrt{x^2 + 9} dx$

$I = \frac{1}{3} \int \sqrt{x^2 + 3^2} dx$


This integral is of the form $\int \sqrt{x^2 + a^2} dx$.

Here, $a = 3$.


Using the standard formula for integrals of the form $\int \sqrt{x^2 + a^2} dx$:

$\int \sqrt{x^2 + a^2} dx = \frac{x}{2}\sqrt{x^2 + a^2} + \frac{a^2}{2}\log|x + \sqrt{x^2 + a^2}| + C'$


Substitute $a=3$ into the formula:

$I = \frac{1}{3} \left( \frac{x}{2}\sqrt{x^2 + 3^2} + \frac{3^2}{2}\log|x + \sqrt{x^2 + 3^2}| \right) + C''$

$I = \frac{1}{3} \left( \frac{x}{2}\sqrt{x^2 + 9} + \frac{9}{2}\log|x + \sqrt{x^2 + 9}| \right) + C''$


Distribute the $\frac{1}{3}$ outside the parenthesis:

$I = \frac{x}{6}\sqrt{x^2 + 9} + \frac{9}{6}\log|x + \sqrt{x^2 + 9}| + \frac{1}{3}C''$

$I = \frac{x}{6}\sqrt{x^2 + 9} + \frac{3}{2}\log|x + \sqrt{x^2 + 9}| + C$


Alternatively, we can write $\sqrt{x^2+9}$ as $\sqrt{9(1 + \frac{x^2}{9})} = 3\sqrt{1 + \frac{x^2}{9}}$.

So, $I = \frac{x}{6} \cdot 3\sqrt{1 + \frac{x^2}{9}} + \frac{3}{2}\log|x + 3\sqrt{1 + \frac{x^2}{9}}| + C$

$I = \frac{x}{2}\sqrt{1 + \frac{x^2}{9}} + \frac{3}{2}\log|x + 3\sqrt{1 + \frac{x^2}{9}}| + C$

Both forms are equivalent and correct.


Where $C$ is the constant of integration.


Thus, the integral of $\sqrt{1 + \frac{x^2}{9}}$ is:

$\int \sqrt{1 + \frac{x^2}{9}} dx = \frac{x}{6}\sqrt{x^2 + 9} + \frac{3}{2}\log|x + \sqrt{x^2 + 9}| + C$

Choose the correct answer in Exercises 10 to 11.

Question 10. $\int \sqrt{1 + x^2} \;dx$ is equal to

(A) $\frac{x}{2} \sqrt{1 + x^2} + \frac{1}{2} \log |(x + \sqrt{1 + x^2})| + C$

(B) $\frac{2}{3} (1 + x^2)^{\frac{3}{2}} + C$

(C) $\frac{2}{3}x (1 + x^2)^{\frac{3}{2}} + C$

(D) $\frac{x^2}{2} \sqrt{1 + x^2} + \frac{1}{2} x^2 \log |x + \sqrt{1 + x^2}| + C$

Answer:

Let the given integral be $I$.

$I = \int \sqrt{1 + x^2} dx$


We can write the integrand as $\sqrt{1^2 + x^2}$.

This integral is of the form $\int \sqrt{a^2 + x^2} dx$, where $a=1$.


Using the standard formula for integrals of the form $\int \sqrt{a^2 + x^2} dx$:

$\int \sqrt{a^2 + x^2} dx = \frac{x}{2}\sqrt{a^2 + x^2} + \frac{a^2}{2}\log|x + \sqrt{a^2 + x^2}| + C'$


Substitute $a=1$ into the formula:

$I = \frac{x}{2}\sqrt{1^2 + x^2} + \frac{1^2}{2}\log|x + \sqrt{1^2 + x^2}| + C$

$I = \frac{x}{2}\sqrt{1 + x^2} + \frac{1}{2}\log|x + \sqrt{1 + x^2}| + C$


Where $C$ is the constant of integration.


Comparing this result with the given options, we see that it matches option (A).


The correct answer is (A).

Question 11. $\int \sqrt{x^2 − 8x + 7} \;dx$ is equal to

(A) $\frac{1}{2} (x - 4) \sqrt{x^2 − 8x + 7} + 9 \log |x − 4 + \sqrt{x^2 − 8x + 7}| + C$

(B) $\frac{1}{2} (x + 4) \sqrt{x^2 − 8x + 7} + 9 \log |x + 4 + \sqrt{x^2 − 8x + 7}| + C$

(C) $\frac{1}{2} (x - 4) \sqrt{x^2 − 8x + 7} - 3\sqrt{2} \log |x − 4 + \sqrt{x^2 − 8x + 7}| + C$

(D) $\frac{1}{2} (x - 4) \sqrt{x^2 − 8x + 7} - \frac{9}{2} \log |x − 4 + \sqrt{x^2 − 8x + 7}| + C$

Answer:

Let the given integral be $I$.

$I = \int \sqrt{x^2 - 8x + 7} dx$


We complete the square for the quadratic expression inside the square root:

$x^2 - 8x + 7 = (x^2 - 8x + (-4)^2) - (-4)^2 + 7$

$x^2 - 8x + 7 = (x^2 - 8x + 16) - 16 + 7$

$x^2 - 8x + 7 = (x-4)^2 - 9$


Substitute this back into the integral:

$I = \int \sqrt{(x-4)^2 - 9} dx$


We can write $9$ as $3^2$.

$I = \int \sqrt{(x-4)^2 - 3^2} dx$


This integral is of the form $\int \sqrt{u^2 - a^2} du$.

Let $u = x-4$. Then $du = dx$.

Also, $a = 3$.


The integral becomes:

$I = \int \sqrt{u^2 - a^2} du$


Using the standard formula for integrals of the form $\int \sqrt{u^2 - a^2} du$:

$\int \sqrt{u^2 - a^2} du = \frac{u}{2}\sqrt{u^2 - a^2} - \frac{a^2}{2}\log|u + \sqrt{u^2 - a^2}| + C'$


Substitute $u = x-4$ and $a=3$ back into the formula:

$I = \frac{x-4}{2}\sqrt{(x-4)^2 - 3^2} - \frac{3^2}{2}\log|(x-4) + \sqrt{(x-4)^2 - 3^2}| + C$


Simplify the expression. Recall that $(x-4)^2 - 3^2 = x^2 - 8x + 7$.

$I = \frac{x-4}{2}\sqrt{x^2 - 8x + 7} - \frac{9}{2}\log|x - 4 + \sqrt{x^2 - 8x + 7}| + C$


Where $C$ is the constant of integration.


Comparing this result with the given options, we see that it matches option (D).


The correct answer is (D).



Example 25 (Before Exercise 7.8)

Example 25: Evaluate the following integrals:

(i) $\int\limits_2^3 x^2 \;dx$

(ii) $\int\limits_4^9 \frac{\sqrt{x}}{(30− x^{\frac{3}{2}})^2} \;dx$

(iii) $\int\limits_1^2 \frac{x \;dx}{(x + 1) (x + 2)}$

(iv) $\int\limits_0^{\frac{π}{4}} \sin^3 2t \;\cos 2t \;dt$

Answer:

(i) Evaluate $\int\limits_2^3 x^2 \;dx$


The integral of $x^2$ is $\frac{x^3}{3}$.

We need to evaluate this definite integral from $2$ to $3$.

$\int\limits_2^3 x^2 \;dx = \left[\frac{x^3}{3}\right]_2^3$

$= \frac{(3)^3}{3} - \frac{(2)^3}{3}$

$= \frac{27}{3} - \frac{8}{3}$

$= \frac{27 - 8}{3}$

$= \frac{19}{3}$


(ii) Evaluate $\int\limits_4^9 \frac{\sqrt{x}}{(30− x^{\frac{3}{2}})^2} \;dx$


Let $u = 30 - x^{\frac{3}{2}}$.

Differentiating $u$ with respect to $x$, we get:

$\frac{du}{dx} = -\frac{3}{2} x^{\frac{3}{2}-1} = -\frac{3}{2} x^{\frac{1}{2}} = -\frac{3}{2} \sqrt{x}$

So, $du = -\frac{3}{2} \sqrt{x} \;dx$, which means $\sqrt{x} \;dx = -\frac{2}{3} \;du$.


We need to change the limits of integration according to the substitution:

When $x = 4$, $u = 30 - (4)^{\frac{3}{2}} = 30 - (\sqrt{4})^3 = 30 - (2)^3 = 30 - 8 = 22$.

When $x = 9$, $u = 30 - (9)^{\frac{3}{2}} = 30 - (\sqrt{9})^3 = 30 - (3)^3 = 30 - 27 = 3$.


Substituting $u$ and the new limits into the integral:

$\int\limits_4^9 \frac{\sqrt{x}}{(30− x^{\frac{3}{2}})^2} \;dx = \int\limits_{22}^3 \frac{1}{u^2} \left(-\frac{2}{3}\right) \;du$

$= -\frac{2}{3} \int\limits_{22}^3 u^{-2} \;du$

The integral of $u^{-2}$ is $\frac{u^{-1}}{-1} = -\frac{1}{u}$.

$= -\frac{2}{3} \left[-\frac{1}{u}\right]_{22}^3$

$= -\frac{2}{3} \left(-\frac{1}{3} - \left(-\frac{1}{22}\right)\right)$

$= -\frac{2}{3} \left(-\frac{1}{3} + \frac{1}{22}\right)$

$= -\frac{2}{3} \left(\frac{-22 + 3}{66}\right)$

$= -\frac{2}{3} \left(\frac{-19}{66}\right)$

$= \frac{2 \times 19}{3 \times 66}$

$= \frac{19}{3 \times 33} = \frac{19}{99}$


(iii) Evaluate $\int\limits_1^2 \frac{x \;dx}{(x + 1) (x + 2)}$


We use partial fraction decomposition for the integrand $\frac{x}{(x+1)(x+2)}$.

Let $\frac{x}{(x+1)(x+2)} = \frac{A}{x+1} + \frac{B}{x+2}$.

Multiplying by $(x+1)(x+2)$, we get:

$x = A(x+2) + B(x+1)$

To find $A$, set $x = -1$:

$-1 = A(-1+2) + B(-1+1)$

$-1 = A(1) + B(0) \implies A = -1$

To find $B$, set $x = -2$:

$-2 = A(-2+2) + B(-2+1)$

$-2 = A(0) + B(-1) \implies -2 = -B \implies B = 2$

So, $\frac{x}{(x+1)(x+2)} = \frac{-1}{x+1} + \frac{2}{x+2}$.


Now, integrate the decomposed expression:

$\int\limits_1^2 \left(\frac{-1}{x+1} + \frac{2}{x+2}\right) dx$

$= \int\limits_1^2 \frac{-1}{x+1} dx + \int\limits_1^2 \frac{2}{x+2} dx$

$= [-\log|x+1|]_1^2 + [2\log|x+2|]_1^2$

Evaluate at the limits:

$= (-\log|2+1| - (-\log|1+1|)) + (2\log|2+2| - 2\log|1+2|)$

$= (-\log 3 + \log 2) + (2\log 4 - 2\log 3)$

$= \log 2 - \log 3 + 2\log (2^2) - 2\log 3$

$= \log 2 - \log 3 + 4\log 2 - 2\log 3$

$= (\log 2 + 4\log 2) + (-\log 3 - 2\log 3)$

$= 5\log 2 - 3\log 3$

Using logarithm properties, $n\log m = \log m^n$ and $\log m - \log n = \log \frac{m}{n}$:

$= \log (2^5) - \log (3^3)$

$= \log 32 - \log 27$

$= \log\left(\frac{32}{27}\right)$


(iv) Evaluate $\int\limits_0^{\frac{π}{4}} \sin^3 2t \;\cos 2t \;dt$


Let $u = \sin 2t$.

Differentiating $u$ with respect to $t$, we get:

$\frac{du}{dt} = \cos 2t \cdot \frac{d}{dt}(2t) = \cos 2t \cdot 2 = 2\cos 2t$

So, $du = 2\cos 2t \;dt$, which means $\cos 2t \;dt = \frac{1}{2} \;du$.


We need to change the limits of integration according to the substitution:

When $t = 0$, $u = \sin (2 \cdot 0) = \sin 0 = 0$.

When $t = \frac{\pi}{4}$, $u = \sin \left(2 \cdot \frac{\pi}{4}\right) = \sin \left(\frac{\pi}{2}\right) = 1$.


Substituting $u$ and the new limits into the integral:

$\int\limits_0^{\frac{π}{4}} \sin^3 2t \;\cos 2t \;dt = \int\limits_0^1 u^3 \left(\frac{1}{2}\right) \;du$

$= \frac{1}{2} \int\limits_0^1 u^3 \;du$

The integral of $u^3$ is $\frac{u^4}{4}$.

$= \frac{1}{2} \left[\frac{u^4}{4}\right]_0^1$

Evaluate at the limits:

$= \frac{1}{2} \left(\frac{(1)^4}{4} - \frac{(0)^4}{4}\right)$

$= \frac{1}{2} \left(\frac{1}{4} - 0\right)$

$= \frac{1}{2} \times \frac{1}{4} = \frac{1}{8}$



Exercise 7.8

Evaluate the definite integrals in Exercises 1 to 20

Question 1. $\int\limits_{−1}^1 (x + 1) \;dx$

Answer:

We need to evaluate the definite integral $\int\limits_{-1}^{1} (x+1) \;dx$.

First, find the indefinite integral of the function $(x+1)$.

The integral of $x$ is $\frac{x^{1+1}}{1+1} = \frac{x^2}{2}$.

The integral of a constant $1$ is $x$.

So, the indefinite integral is $\int (x+1) \;dx = \frac{x^2}{2} + x + C$.


Now, we evaluate the definite integral using the Fundamental Theorem of Calculus:

$\int\limits_{a}^{b} f(x) \;dx = F(b) - F(a)$, where $F(x)$ is an antiderivative of $f(x)$.

Here, $f(x) = x+1$, $a = -1$, $b = 1$, and $F(x) = \frac{x^2}{2} + x$.

Evaluate $F(b)$:

$F(1) = \frac{(1)^2}{2} + 1 = \frac{1}{2} + 1 = \frac{1+2}{2} = \frac{3}{2}$.

Evaluate $F(a)$:

$F(-1) = \frac{(-1)^2}{2} + (-1) = \frac{1}{2} - 1 = \frac{1-2}{2} = -\frac{1}{2}$.

Calculate $F(b) - F(a)$:

$\int\limits_{-1}^{1} (x+1) \;dx = F(1) - F(-1) = \frac{3}{2} - \left(-\frac{1}{2}\right) = \frac{3}{2} + \frac{1}{2} = \frac{3+1}{2} = \frac{4}{2} = 2$.


The value of the definite integral is:

$\int\limits_{-1}^1 (x + 1) \;dx = \mathbf{2}$.

Question 2. $\int\limits_2^3 \frac{1}{x} \;dx$

Answer:

We are asked to evaluate the definite integral $\int\limits_{2}^{3} \frac{1}{x} \;dx$.

First, we find the indefinite integral of the function $f(x) = \frac{1}{x}$.

The indefinite integral of $\frac{1}{x}$ is $\log_e |x|$. We can ignore the constant of integration for definite integrals.

So, the antiderivative $F(x)$ is $F(x) = \log_e |x|$.


Now, we apply the Fundamental Theorem of Calculus:

$\int\limits_{a}^{b} f(x) \;dx = F(b) - F(a)$.

Here, $f(x) = \frac{1}{x}$, $a = 2$, $b = 3$, and $F(x) = \log_e |x|$.

Evaluate $F(b)$ at the upper limit $x=3$:

$F(3) = \log_e |3| = \log_e 3$.

Evaluate $F(a)$ at the lower limit $x=2$:

$F(2) = \log_e |2| = \log_e 2$.


Subtract $F(a)$ from $F(b)$:

$\int\limits_{2}^{3} \frac{1}{x} \;dx = F(3) - F(2) = \log_e 3 - \log_e 2$.

Using the property of logarithms, $\log_e A - \log_e B = \log_e \left(\frac{A}{B}\right)$, we can simplify the result:

$\log_e 3 - \log_e 2 = \log_e \left(\frac{3}{2}\right)$.


Thus, the value of the definite integral is:

$\int\limits_2^3 \frac{1}{x} \;dx = \mathbf{\log_e \left(\frac{3}{2}\right)}$.

Question 3. $\int\limits_1^2 (4x^3 − 5x^2 + 6x + 9) \;dx$

Answer:

We need to evaluate the definite integral $\int\limits_1^2 (4x^3 − 5x^2 + 6x + 9) \;dx$.

First, find the indefinite integral of the function $f(x) = 4x^3 − 5x^2 + 6x + 9$.

Using the power rule for integration $\int x^n \;dx = \frac{x^{n+1}}{n+1}$ and the linearity of integrals:

$\int (4x^3 − 5x^2 + 6x + 9) \;dx = \int 4x^3 \;dx - \int 5x^2 \;dx + \int 6x \;dx + \int 9 \;dx$

$= 4 \int x^3 \;dx - 5 \int x^2 \;dx + 6 \int x \;dx + 9 \int 1 \;dx$

$= 4 \left(\frac{x^{3+1}}{3+1}\right) - 5 \left(\frac{x^{2+1}}{2+1}\right) + 6 \left(\frac{x^{1+1}}{1+1}\right) + 9x$

$= 4 \left(\frac{x^4}{4}\right) - 5 \left(\frac{x^3}{3}\right) + 6 \left(\frac{x^2}{2}\right) + 9x$

$= x^4 - \frac{5}{3}x^3 + 3x^2 + 9x$

Let the antiderivative be $F(x) = x^4 - \frac{5}{3}x^3 + 3x^2 + 9x$.


Now, apply the Fundamental Theorem of Calculus: $\int\limits_a^b f(x) \;dx = F(b) - F(a)$.

Here, $a=1$ and $b=2$.

Evaluate $F(x)$ at the upper limit $x=2$:

$F(2) = (2)^4 - \frac{5}{3}(2)^3 + 3(2)^2 + 9(2)$

$F(2) = 16 - \frac{5}{3}(8) + 3(4) + 18$

$F(2) = 16 - \frac{40}{3} + 12 + 18$

$F(2) = 46 - \frac{40}{3}$

$F(2) = \frac{46 \times 3 - 40}{3} = \frac{138 - 40}{3} = \frac{98}{3}$


Evaluate $F(x)$ at the lower limit $x=1$:

$F(1) = (1)^4 - \frac{5}{3}(1)^3 + 3(1)^2 + 9(1)$

$F(1) = 1 - \frac{5}{3}(1) + 3(1) + 9(1)$

$F(1) = 1 - \frac{5}{3} + 3 + 9$

$F(1) = 13 - \frac{5}{3}$

$F(1) = \frac{13 \times 3 - 5}{3} = \frac{39 - 5}{3} = \frac{34}{3}$


Now subtract $F(1)$ from $F(2)$:

$\int\limits_1^2 (4x^3 − 5x^2 + 6x + 9) \;dx = F(2) - F(1)$

$= \frac{98}{3} - \frac{34}{3}$

$= \frac{98 - 34}{3}$

$= \frac{64}{3}$


The value of the definite integral is:

$\int\limits_1^2 (4x^3 − 5x^2 + 6x + 9) \;dx = \mathbf{\frac{64}{3}}$.

Question 4. $\int\limits_0^{\frac{π}{4}} \sin 2x \;dx$

Answer:

We are asked to evaluate the definite integral $\int\limits_0^{\frac{π}{4}} \sin 2x \;dx$.

First, find the indefinite integral of $f(x) = \sin 2x$.

The integral of $\sin(ax)$ is $-\frac{1}{a}\cos(ax)$.

So, $\int \sin 2x \;dx = -\frac{1}{2}\cos 2x$.

Let the antiderivative be $F(x) = -\frac{1}{2}\cos 2x$.


Now, apply the Fundamental Theorem of Calculus: $\int\limits_a^b f(x) \;dx = F(b) - F(a)$.

Here, the lower limit is $a = 0$ and the upper limit is $b = \frac{π}{4}$.

Evaluate $F(x)$ at the upper limit $x = \frac{π}{4}$:

$F\left(\frac{π}{4}\right) = -\frac{1}{2}\cos\left(2 \times \frac{π}{4}\right)$

$= -\frac{1}{2}\cos\left(\frac{π}{2}\right)$

Since $\cos\left(\frac{π}{2}\right) = 0$,

$F\left(\frac{π}{4}\right) = -\frac{1}{2} \times 0 = 0$.


Evaluate $F(x)$ at the lower limit $x = 0$:

$F(0) = -\frac{1}{2}\cos(2 \times 0)$

$= -\frac{1}{2}\cos(0)$

Since $\cos(0) = 1$,

$F(0) = -\frac{1}{2} \times 1 = -\frac{1}{2}$.


Subtract $F(a)$ from $F(b)$:

$\int\limits_0^{\frac{π}{4}} \sin 2x \;dx = F\left(\frac{π}{4}\right) - F(0)$

$= 0 - \left(-\frac{1}{2}\right)$

$= 0 + \frac{1}{2}$

$= \frac{1}{2}$


The value of the definite integral is:

$\int\limits_0^{\frac{π}{4}} \sin 2x \;dx = \mathbf{\frac{1}{2}}$.

Question 5. $\int\limits_0^{\frac{π}{2}} \cos 2x \;dx$

Answer:

We need to evaluate the definite integral $\int\limits_0^{\frac{π}{2}} \cos 2x \;dx$.

First, find the indefinite integral of the function $f(x) = \cos 2x$.

The integral of $\cos(ax)$ is $\frac{1}{a}\sin(ax)$.

So, $\int \cos 2x \;dx = \frac{1}{2}\sin 2x$.

Let the antiderivative be $F(x) = \frac{1}{2}\sin 2x$.


Now, apply the Fundamental Theorem of Calculus: $\int\limits_a^b f(x) \;dx = F(b) - F(a)$.

Here, the lower limit is $a = 0$ and the upper limit is $b = \frac{π}{2}$.

Evaluate $F(x)$ at the upper limit $x = \frac{π}{2}$:

$F\left(\frac{π}{2}\right) = \frac{1}{2}\sin\left(2 \times \frac{π}{2}\right)$

$= \frac{1}{2}\sin(π)$

Since $\sin(π) = 0$,

$F\left(\frac{π}{2}\right) = \frac{1}{2} \times 0 = 0$.


Evaluate $F(x)$ at the lower limit $x = 0$:

$F(0) = \frac{1}{2}\sin(2 \times 0)$

$= \frac{1}{2}\sin(0)$

Since $\sin(0) = 0$,

$F(0) = \frac{1}{2} \times 0 = 0$.


Subtract $F(a)$ from $F(b)$:

$\int\limits_0^{\frac{π}{2}} \cos 2x \;dx = F\left(\frac{π}{2}\right) - F(0)$

$= 0 - 0$

$= 0$


The value of the definite integral is:

$\int\limits_0^{\frac{π}{2}} \cos 2x \;dx = \mathbf{0}$.

Question 6. $\int\limits_4^5 e^x \;dx$

Answer:

We need to evaluate the definite integral $\int\limits_4^5 e^x \;dx$.

First, find the indefinite integral of the function $f(x) = e^x$.

The indefinite integral of $e^x$ is $e^x$. We can ignore the constant of integration for definite integrals.

So, the antiderivative $F(x)$ is $F(x) = e^x$.


Now, apply the Fundamental Theorem of Calculus: $\int\limits_a^b f(x) \;dx = F(b) - F(a)$.

Here, the lower limit is $a = 4$ and the upper limit is $b = 5$.

Evaluate $F(x)$ at the upper limit $x=5$:

$F(5) = e^5$.


Evaluate $F(x)$ at the lower limit $x=4$:

$F(4) = e^4$.


Subtract $F(a)$ from $F(b)$:

$\int\limits_4^5 e^x \;dx = F(5) - F(4) = e^5 - e^4$.

We can factor out $e^4$ for a slightly different form of the answer:

$e^5 - e^4 = e^4(e - 1)$.


The value of the definite integral is:

$\int\limits_4^5 e^x \;dx = \mathbf{e^5 - e^4}$ or $\mathbf{e^4(e - 1)}$.

Question 7. $\int\limits_0^{\frac{π}{4}} \tan x \;dx$

Answer:

We need to evaluate the definite integral $\int\limits_0^{\frac{π}{4}} \tan x \;dx$.

First, find the indefinite integral of the function $f(x) = \tan x$.

The indefinite integral of $\tan x$ is $\int \tan x \;dx = -\log_e |\cos x|$.

Let the antiderivative be $F(x) = -\log_e |\cos x|$.


Now, apply the Fundamental Theorem of Calculus: $\int\limits_a^b f(x) \;dx = F(b) - F(a)$.

Here, the lower limit is $a = 0$ and the upper limit is $b = \frac{π}{4}$.

Evaluate $F(x)$ at the upper limit $x = \frac{π}{4}$:

$F\left(\frac{π}{4}\right) = -\log_e \left|\cos\left(\frac{π}{4}\right)\right|$

Since $\cos\left(\frac{π}{4}\right) = \frac{1}{\sqrt{2}} = \frac{\sqrt{2}}{2}$, and $\frac{\sqrt{2}}{2} > 0$ in the interval $[0, \frac{\pi}{4}]$, the absolute value can be removed.

$F\left(\frac{π}{4}\right) = -\log_e \left(\frac{1}{\sqrt{2}}\right)$

Using logarithm properties, $-\log_e \left(\frac{1}{\sqrt{2}}\right) = -(\log_e 1 - \log_e \sqrt{2}) = -(0 - \log_e 2^{1/2}) = -(-\frac{1}{2}\log_e 2) = \frac{1}{2}\log_e 2$.

So, $F\left(\frac{π}{4}\right) = \frac{1}{2}\log_e 2$.


Evaluate $F(x)$ at the lower limit $x = 0$:

$F(0) = -\log_e |\cos(0)|$

Since $\cos(0) = 1$, and $1 > 0$, the absolute value can be removed.

$F(0) = -\log_e (1)$

Since $\log_e (1) = 0$,

$F(0) = -0 = 0$.


Subtract $F(a)$ from $F(b)$:

$\int\limits_0^{\frac{π}{4}} \tan x \;dx = F\left(\frac{π}{4}\right) - F(0)$

$= \frac{1}{2}\log_e 2 - 0$

$= \frac{1}{2}\log_e 2$.


The value of the definite integral is:

$\int\limits_0^{\frac{π}{4}} \tan x \;dx = \mathbf{\frac{1}{2}\log_e 2}$.

Question 8. $\int\limits_{\frac{π}{6}}^{\frac{π}{4}} cosec\;x \;dx$

Answer:

We need to evaluate the definite integral $\int\limits_{\frac{π}{6}}^{\frac{π}{4}} \text{cosec}\;x \;dx$.

First, find the indefinite integral of the function $f(x) = \text{cosec}\;x$.

The indefinite integral of $\text{cosec}\;x$ is $\int \text{cosec}\;x \;dx = \log_e |\text{cosec}\;x - \cot x|$.

Let the antiderivative be $F(x) = \log_e |\text{cosec}\;x - \cot x|$.


Now, apply the Fundamental Theorem of Calculus: $\int\limits_a^b f(x) \;dx = F(b) - F(a)$.

Here, the lower limit is $a = \frac{π}{6}$ and the upper limit is $b = \frac{π}{4}$.

Evaluate $F(x)$ at the upper limit $x = \frac{π}{4}$:

$F\left(\frac{π}{4}\right) = \log_e \left|\text{cosec}\left(\frac{π}{4}\right) - \cot\left(\frac{π}{4}\right)\right|$

We know that $\sin\left(\frac{π}{4}\right) = \frac{1}{\sqrt{2}}$ and $\cos\left(\frac{π}{4}\right) = \frac{1}{\sqrt{2}}$.

So, $\text{cosec}\left(\frac{π}{4}\right) = \frac{1}{\sin(π/4)} = \sqrt{2}$ and $\cot\left(\frac{π}{4}\right) = \frac{\cos(π/4)}{\sin(π/4)} = 1$.

$F\left(\frac{π}{4}\right) = \log_e |\sqrt{2} - 1|$

Since $\sqrt{2} \approx 1.414$, $\sqrt{2} - 1$ is positive, so $|\sqrt{2} - 1| = \sqrt{2} - 1$.

$F\left(\frac{π}{4}\right) = \log_e (\sqrt{2} - 1)$.


Evaluate $F(x)$ at the lower limit $x = \frac{π}{6}$:

$F\left(\frac{π}{6}\right) = \log_e \left|\text{cosec}\left(\frac{π}{6}\right) - \cot\left(\frac{π}{6}\right)\right|$

We know that $\sin\left(\frac{π}{6}\right) = \frac{1}{2}$ and $\cos\left(\frac{π}{6}\right) = \frac{\sqrt{3}}{2}$.

So, $\text{cosec}\left(\frac{π}{6}\right) = \frac{1}{\sin(π/6)} = 2$ and $\cot\left(\frac{π}{6}\right) = \frac{\cos(π/6)}{\sin(π/6)} = \frac{\sqrt{3}/2}{1/2} = \sqrt{3}$.

$F\left(\frac{π}{6}\right) = \log_e |2 - \sqrt{3}|$

Since $\sqrt{3} \approx 1.732$, $2 - \sqrt{3}$ is positive, so $|2 - \sqrt{3}| = 2 - \sqrt{3}$.

$F\left(\frac{π}{6}\right) = \log_e (2 - \sqrt{3})$.


Subtract $F(a)$ from $F(b)$:

$\int\limits_{\frac{π}{6}}^{\frac{π}{4}} \text{cosec}\;x \;dx = F\left(\frac{π}{4}\right) - F\left(\frac{π}{6}\right)$

$= \log_e (\sqrt{2} - 1) - \log_e (2 - \sqrt{3})$.

Using the logarithm property $\log_e A - \log_e B = \log_e \left(\frac{A}{B}\right)$, we can write:

$= \log_e \left(\frac{\sqrt{2} - 1}{2 - \sqrt{3}}\right)$.


The value of the definite integral is:

$\int\limits_{\frac{π}{6}}^{\frac{π}{4}} \text{cosec}\;x \;dx = \mathbf{\log_e (\sqrt{2} - 1) - \log_e (2 - \sqrt{3})}$ or $\mathbf{\log_e \left(\frac{\sqrt{2} - 1}{2 - \sqrt{3}}\right)}$.

Question 9. $\int\limits_0^1 \frac{dx}{\sqrt{1 − x^2}}$

Answer:

We need to evaluate the definite integral $\int\limits_0^1 \frac{dx}{\sqrt{1 − x^2}}$.

First, find the indefinite integral of the function $f(x) = \frac{1}{\sqrt{1 − x^2}}$.

We know that the integral of $\frac{1}{\sqrt{1 - x^2}}$ is $\sin^{-1}(x)$.

So, the antiderivative is $F(x) = \sin^{-1}(x)$.


Now, apply the Fundamental Theorem of Calculus: $\int\limits_a^b f(x) \;dx = F(b) - F(a)$.

Here, the lower limit is $a = 0$ and the upper limit is $b = 1$.

Evaluate $F(x)$ at the upper limit $x = 1$:

$F(1) = \sin^{-1}(1)$.

The value of $y$ such that $\sin(y) = 1$ is $y = \frac{π}{2}$.

So, $F(1) = \frac{π}{2}$.


Evaluate $F(x)$ at the lower limit $x = 0$:

$F(0) = \sin^{-1}(0)$.

The value of $y$ such that $\sin(y) = 0$ is $y = 0$.

So, $F(0) = 0$.


Subtract $F(a)$ from $F(b)$:

$\int\limits_0^1 \frac{dx}{\sqrt{1 − x^2}} = F(1) - F(0)$

$= \frac{π}{2} - 0$

$= \frac{π}{2}$.


The value of the definite integral is:

$\int\limits_0^1 \frac{dx}{\sqrt{1 − x^2}} = \mathbf{\frac{π}{2}}$.

Question 10. $\int\limits_0^1 \frac{dx}{1 + x^2}$

Answer:

We need to evaluate the definite integral $\int\limits_0^1 \frac{dx}{1 + x^2}$.

First, find the indefinite integral of the function $f(x) = \frac{1}{1 + x^2}$.

We know that the indefinite integral of $\frac{1}{1 + x^2}$ is $\tan^{-1}(x)$ (or $\tan^{-1}(x)$).

So, the antiderivative is $F(x) = \tan^{-1}(x)$.


Now, apply the Fundamental Theorem of Calculus: $\int\limits_a^b f(x) \;dx = F(b) - F(a)$.

Here, the lower limit is $a = 0$ and the upper limit is $b = 1$.

Evaluate $F(x)$ at the upper limit $x = 1$:

$F(1) = \tan^{-1}(1)$.

The value of the angle whose tangent is $1$ is $\frac{π}{4}$ radians.

So, $F(1) = \frac{π}{4}$.


Evaluate $F(x)$ at the lower limit $x = 0$:

$F(0) = \tan^{-1}(0)$.

The value of the angle whose tangent is $0$ is $0$ radians.

So, $F(0) = 0$.


Subtract $F(a)$ from $F(b)$:

$\int\limits_0^1 \frac{dx}{1 + x^2} = F(1) - F(0)$

$= \frac{π}{4} - 0$

$= \frac{π}{4}$.


The value of the definite integral is:

$\int\limits_0^1 \frac{dx}{1 + x^2} = \mathbf{\frac{π}{4}}$.

Question 11. $\int\limits_2^3 \frac{dx}{x^2 − 1}$

Answer:

We need to evaluate the definite integral $\int\limits_2^3 \frac{dx}{x^2 − 1}$.

First, find the indefinite integral of the function $f(x) = \frac{1}{x^2 − 1}$.

This integral is of the form $\int \frac{dx}{x^2 - a^2}$, where $a^2 = 1$, so $a=1$.

The standard integral formula is $\int \frac{dx}{x^2 - a^2} = \frac{1}{2a} \log_e \left|\frac{x-a}{x+a}\right|$.

Using $a=1$, the indefinite integral is:

$\int \frac{dx}{x^2 − 1} = \frac{1}{2(1)} \log_e \left|\frac{x-1}{x+1}\right| = \frac{1}{2} \log_e \left|\frac{x-1}{x+1}\right|$.

Let the antiderivative be $F(x) = \frac{1}{2} \log_e \left|\frac{x-1}{x+1}\right|$.


Now, apply the Fundamental Theorem of Calculus: $\int\limits_a^b f(x) \;dx = F(b) - F(a)$.

Here, the lower limit is $a = 2$ and the upper limit is $b = 3$.

Evaluate $F(x)$ at the upper limit $x = 3$:

$F(3) = \frac{1}{2} \log_e \left|\frac{3-1}{3+1}\right| = \frac{1}{2} \log_e \left|\frac{2}{4}\right| = \frac{1}{2} \log_e \left|\frac{1}{2}\right|$.

Since $\frac{1}{2} > 0$, $|\frac{1}{2}| = \frac{1}{2}$.

$F(3) = \frac{1}{2} \log_e \left(\frac{1}{2}\right)$.

Using logarithm property $\log_e (\frac{A}{B}) = \log_e A - \log_e B$, we get $\log_e (\frac{1}{2}) = \log_e 1 - \log_e 2 = 0 - \log_e 2 = -\log_e 2$.

So, $F(3) = \frac{1}{2}(-\log_e 2) = -\frac{1}{2}\log_e 2$.


Evaluate $F(x)$ at the lower limit $x = 2$:

$F(2) = \frac{1}{2} \log_e \left|\frac{2-1}{2+1}\right| = \frac{1}{2} \log_e \left|\frac{1}{3}\right|$.

Since $\frac{1}{3} > 0$, $|\frac{1}{3}| = \frac{1}{3}$.

$F(2) = \frac{1}{2} \log_e \left(\frac{1}{3}\right)$.

Using logarithm property $\log_e (\frac{A}{B}) = \log_e A - \log_e B$, we get $\log_e (\frac{1}{3}) = \log_e 1 - \log_e 3 = 0 - \log_e 3 = -\log_e 3$.

So, $F(2) = \frac{1}{2}(-\log_e 3) = -\frac{1}{2}\log_e 3$.


Subtract $F(a)$ from $F(b)$:

$\int\limits_2^3 \frac{dx}{x^2 − 1} = F(3) - F(2)$

$= \left(-\frac{1}{2}\log_e 2\right) - \left(-\frac{1}{2}\log_e 3\right)$

$= -\frac{1}{2}\log_e 2 + \frac{1}{2}\log_e 3$

$= \frac{1}{2}(\log_e 3 - \log_e 2)$.

Using logarithm property $\log_e A - \log_e B = \log_e (\frac{A}{B})$, we can write:

$= \frac{1}{2}\log_e \left(\frac{3}{2}\right)$.


The value of the definite integral is:

$\int\limits_2^3 \frac{dx}{x^2 − 1} = \mathbf{\frac{1}{2}\log_e \left(\frac{3}{2}\right)}$.

Question 12. $\int\limits_0^{\frac{π}{2}} \cos^2 x \;dx$

Answer:

We need to evaluate the definite integral $\int\limits_0^{\frac{π}{2}} \cos^2 x \;dx$.

To integrate $\cos^2 x$, we use the trigonometric identity $\cos 2x = 2\cos^2 x - 1$, which can be rearranged to $\cos^2 x = \frac{1 + \cos 2x}{2}$.

Substitute this identity into the integral:

$\int\limits_0^{\frac{π}{2}} \cos^2 x \;dx = \int\limits_0^{\frac{π}{2}} \frac{1 + \cos 2x}{2} \;dx$

$= \frac{1}{2} \int\limits_0^{\frac{π}{2}} (1 + \cos 2x) \;dx$


Now, find the indefinite integral of $(1 + \cos 2x)$.

$\int (1 + \cos 2x) \;dx = \int 1 \;dx + \int \cos 2x \;dx$

$= x + \frac{1}{2}\sin 2x$.

So, the antiderivative of $\cos^2 x$ is $F(x) = \frac{1}{2}\left(x + \frac{1}{2}\sin 2x\right) = \frac{x}{2} + \frac{1}{4}\sin 2x$.


Apply the Fundamental Theorem of Calculus: $\int\limits_a^b f(x) \;dx = F(b) - F(a)$.

Here, $a=0$ and $b=\frac{π}{2}$.

Evaluate $F(x)$ at the upper limit $x = \frac{π}{2}$:

$F\left(\frac{π}{2}\right) = \frac{(π/2)}{2} + \frac{1}{4}\sin\left(2 \times \frac{π}{2}\right)$

$= \frac{π}{4} + \frac{1}{4}\sin(π)$.

Since $\sin(π) = 0$,

$F\left(\frac{π}{2}\right) = \frac{π}{4} + \frac{1}{4}(0) = \frac{π}{4}$.


Evaluate $F(x)$ at the lower limit $x = 0$:

$F(0) = \frac{0}{2} + \frac{1}{4}\sin(2 \times 0)$

$= 0 + \frac{1}{4}\sin(0)$.

Since $\sin(0) = 0$,

$F(0) = 0 + \frac{1}{4}(0) = 0$.


Subtract $F(a)$ from $F(b)$:

$\int\limits_0^{\frac{π}{2}} \cos^2 x \;dx = F\left(\frac{π}{2}\right) - F(0)$

$= \frac{π}{4} - 0$

$= \frac{π}{4}$.


The value of the definite integral is:

$\int\limits_0^{\frac{π}{2}} \cos^2 x \;dx = \mathbf{\frac{π}{4}}$.

Question 13. $\int\limits_2^3 \frac{x \;dx}{x^2 + 1} $

Answer:

We need to evaluate the definite integral $\int\limits_2^3 \frac{x}{x^2 + 1} \;dx$.

First, find the indefinite integral $\int \frac{x}{x^2 + 1} \;dx$.

We can use the substitution method. Let $u = x^2 + 1$.

Differentiating with respect to $x$, we get $\frac{du}{dx} = 2x$.

So, $du = 2x \;dx$, which means $x \;dx = \frac{1}{2} \;du$.

Substitute $u$ and $x \;dx$ into the integral:

$\int \frac{x}{x^2 + 1} \;dx = \int \frac{1}{u} \left(\frac{1}{2} \;du\right) = \frac{1}{2} \int \frac{1}{u} \;du$.

The integral of $\frac{1}{u}$ is $\log_e |u|$.

So, the indefinite integral is $\frac{1}{2} \log_e |u| + C$.

Substitute back $u = x^2 + 1$:

$\frac{1}{2} \log_e |x^2 + 1| + C$.

Since $x^2 + 1$ is always positive for real values of $x$, we can remove the absolute value sign: $\frac{1}{2} \log_e (x^2 + 1)$.

Let the antiderivative be $F(x) = \frac{1}{2} \log_e (x^2 + 1)$.


Now, apply the Fundamental Theorem of Calculus: $\int\limits_a^b f(x) \;dx = F(b) - F(a)$.

Here, the lower limit is $a = 2$ and the upper limit is $b = 3$.

Evaluate $F(x)$ at the upper limit $x = 3$:

$F(3) = \frac{1}{2} \log_e (3^2 + 1) = \frac{1}{2} \log_e (9 + 1) = \frac{1}{2} \log_e (10)$.


Evaluate $F(x)$ at the lower limit $x = 2$:

$F(2) = \frac{1}{2} \log_e (2^2 + 1) = \frac{1}{2} \log_e (4 + 1) = \frac{1}{2} \log_e (5)$.


Subtract $F(a)$ from $F(b)$:

$\int\limits_2^3 \frac{x}{x^2 + 1} \;dx = F(3) - F(2)$

$= \frac{1}{2} \log_e (10) - \frac{1}{2} \log_e (5)$.

Factor out $\frac{1}{2}$:

$= \frac{1}{2} (\log_e (10) - \log_e (5))$.

Using the logarithm property $\log_e A - \log_e B = \log_e \left(\frac{A}{B}\right)$:

$= \frac{1}{2} \log_e \left(\frac{10}{5}\right) = \frac{1}{2} \log_e (2)$.


The value of the definite integral is:

$\int\limits_2^3 \frac{x \;dx}{x^2 + 1} = \mathbf{\frac{1}{2} \log_e (2)}$.

Question 14. $\int\limits_0^1 \frac{2x + 3}{5x^2 + 1} \;dx$

Answer:

We need to evaluate the definite integral $\int\limits_0^1 \frac{2x + 3}{5x^2 + 1} \;dx$.

We can split the integrand into two parts:

$\frac{2x + 3}{5x^2 + 1} = \frac{2x}{5x^2 + 1} + \frac{3}{5x^2 + 1}$.

So, the integral becomes $\int\limits_0^1 \left(\frac{2x}{5x^2 + 1} + \frac{3}{5x^2 + 1}\right) \;dx = \int\limits_0^1 \frac{2x}{5x^2 + 1} \;dx + \int\limits_0^1 \frac{3}{5x^2 + 1} \;dx$.


Let's evaluate the first integral: $\int \frac{2x}{5x^2 + 1} \;dx$.

Use substitution: Let $u = 5x^2 + 1$. Then $du = 10x \;dx$, which means $2x \;dx = \frac{1}{5} \;du$.

The indefinite integral is $\int \frac{1}{u} \left(\frac{1}{5} \;du\right) = \frac{1}{5} \int \frac{1}{u} \;du = \frac{1}{5} \log_e |u|$.

Substitute back $u = 5x^2 + 1$: $\frac{1}{5} \log_e (5x^2 + 1)$ (since $5x^2 + 1 > 0$).


Now, let's evaluate the second integral: $\int \frac{3}{5x^2 + 1} \;dx$.

This can be written as $3 \int \frac{1}{5x^2 + 1} \;dx = 3 \int \frac{1}{5(x^2 + \frac{1}{5})} \;dx = \frac{3}{5} \int \frac{1}{x^2 + \left(\frac{1}{\sqrt{5}}\right)^2} \;dx$.

This is in the form $\int \frac{dx}{x^2 + a^2}$ with $a = \frac{1}{\sqrt{5}}$. The integral is $\frac{1}{a} \tan^{-1}\left(\frac{x}{a}\right)$.

So, $\frac{3}{5} \times \frac{1}{1/\sqrt{5}} \tan^{-1}\left(\frac{x}{1/\sqrt{5}}\right) = \frac{3}{5} \times \sqrt{5} \tan^{-1}(\sqrt{5}x) = \frac{3}{\sqrt{5}} \tan^{-1}(\sqrt{5}x)$.


Combining the two indefinite integrals, the antiderivative of the original function is $F(x) = \frac{1}{5} \log_e (5x^2 + 1) + \frac{3}{\sqrt{5}} \tan^{-1}(\sqrt{5}x)$.


Now, apply the Fundamental Theorem of Calculus $\int\limits_a^b f(x) \;dx = F(b) - F(a)$ with $a=0$ and $b=1$.

Evaluate $F(1)$:

$F(1) = \frac{1}{5} \log_e (5(1)^2 + 1) + \frac{3}{\sqrt{5}} \tan^{-1}(\sqrt{5}(1)) = \frac{1}{5} \log_e (6) + \frac{3}{\sqrt{5}} \tan^{-1}(\sqrt{5})$.


Evaluate $F(0)$:

$F(0) = \frac{1}{5} \log_e (5(0)^2 + 1) + \frac{3}{\sqrt{5}} \tan^{-1}(\sqrt{5}(0)) = \frac{1}{5} \log_e (1) + \frac{3}{\sqrt{5}} \tan^{-1}(0)$.

Since $\log_e (1) = 0$ and $\tan^{-1}(0) = 0$, $F(0) = \frac{1}{5}(0) + \frac{3}{\sqrt{5}}(0) = 0$.


Calculate $F(1) - F(0)$:

$\int\limits_0^1 \frac{2x + 3}{5x^2 + 1} \;dx = F(1) - F(0) = \left(\frac{1}{5} \log_e (6) + \frac{3}{\sqrt{5}} \tan^{-1}(\sqrt{5})\right) - 0 = \frac{1}{5} \log_e (6) + \frac{3}{\sqrt{5}} \tan^{-1}(\sqrt{5})$.


The value of the definite integral is:

$\int\limits_0^1 \frac{2x + 3}{5x^2 + 1} \;dx = \mathbf{\frac{1}{5} \log_e (6) + \frac{3}{\sqrt{5}} \tan^{-1}(\sqrt{5})}$.

Question 15. $\int\limits_0^1 x \;e^{x^{2}} \;dx$

Answer:

We need to evaluate the definite integral $\int\limits_0^1 x \;e^{x^{2}} \;dx$.

We will use the substitution method to find the indefinite integral $\int x \;e^{x^{2}} \;dx$.

Let $u = x^2$.

Differentiating both sides with respect to $x$, we get $\frac{du}{dx} = 2x$.

This implies $du = 2x \;dx$. We have $x \;dx$ in the integral, so $x \;dx = \frac{1}{2} \;du$.


Substitute $u$ and $x \;dx$ into the indefinite integral:

$\int x \;e^{x^{2}} \;dx = \int e^{x^2} (x \;dx) = \int e^u \left(\frac{1}{2} \;du\right)$

$= \frac{1}{2} \int e^u \;du$.

The integral of $e^u$ is $e^u$.

So, the indefinite integral is $\frac{1}{2} e^u + C$.

Substitute back $u = x^2$:

The antiderivative is $F(x) = \frac{1}{2} e^{x^2}$.


Now, apply the Fundamental Theorem of Calculus: $\int\limits_a^b f(x) \;dx = F(b) - F(a)$.

Here, the lower limit is $a = 0$ and the upper limit is $b = 1$.

Evaluate $F(x)$ at the upper limit $x = 1$:

$F(1) = \frac{1}{2} e^{(1)^2} = \frac{1}{2} e^1 = \frac{e}{2}$.


Evaluate $F(x)$ at the lower limit $x = 0$:

$F(0) = \frac{1}{2} e^{(0)^2} = \frac{1}{2} e^0$.

Since $e^0 = 1$,

$F(0) = \frac{1}{2} \times 1 = \frac{1}{2}$.


Subtract $F(a)$ from $F(b)$:

$\int\limits_0^1 x \;e^{x^{2}} \;dx = F(1) - F(0)$

$= \frac{e}{2} - \frac{1}{2}$.

This can also be written as $\frac{1}{2}(e - 1)$.


The value of the definite integral is:

$\int\limits_0^1 x \;e^{x^{2}} \;dx = \mathbf{\frac{e - 1}{2}}$.

Question 16. $\int\limits_1^2 \frac{5x^2}{x^2 + 4x + 3} \;dx$

Answer:

We need to evaluate the definite integral $\int\limits_1^2 \frac{5x^2}{x^2 + 4x + 3} \;dx$.

The degree of the numerator is equal to the degree of the denominator. We perform polynomial long division or algebraic manipulation.

$\frac{5x^2}{x^2 + 4x + 3} = \frac{5(x^2 + 4x + 3) - 20x - 15}{x^2 + 4x + 3} = 5 - \frac{20x + 15}{x^2 + 4x + 3}$.

The integral becomes $\int\limits_1^2 \left(5 - \frac{20x + 15}{x^2 + 4x + 3}\right) \;dx = \int\limits_1^2 5 \;dx - \int\limits_1^2 \frac{20x + 15}{x^2 + 4x + 3} \;dx$.


Evaluate the first part: $\int\limits_1^2 5 \;dx$.

The antiderivative of $5$ is $5x$.

$\int\limits_1^2 5 \;dx = [5x]_1^2 = 5(2) - 5(1) = 10 - 5 = 5$.


Evaluate the second part: $\int\limits_1^2 \frac{20x + 15}{x^2 + 4x + 3} \;dx$.

Factor the denominator: $x^2 + 4x + 3 = (x+1)(x+3)$.

Use partial fraction decomposition for the integrand:

$\frac{20x + 15}{(x+1)(x+3)} = \frac{A}{x+1} + \frac{B}{x+3}$.

Multiply by $(x+1)(x+3)$: $20x + 15 = A(x+3) + B(x+1)$.

Set $x = -1$: $20(-1) + 15 = A(-1+3) + B(-1+1) \implies -20 + 15 = 2A + 0 \implies -5 = 2A \implies A = -\frac{5}{2}$.

Set $x = -3$: $20(-3) + 15 = A(-3+3) + B(-3+1) \implies -60 + 15 = 0 + B(-2) \implies -45 = -2B \implies B = \frac{45}{2}$.

So, $\frac{20x + 15}{x^2 + 4x + 3} = \frac{-5/2}{x+1} + \frac{45/2}{x+3}$.


Integrate the partial fractions:

$\int \left(\frac{-5/2}{x+1} + \frac{45/2}{x+3}\right) \;dx = -\frac{5}{2} \int \frac{1}{x+1} \;dx + \frac{45}{2} \int \frac{1}{x+3} \;dx$

$= -\frac{5}{2}\log_e |x+1| + \frac{45}{2}\log_e |x+3|$.

For the interval $[1, 2]$, $x+1 > 0$ and $x+3 > 0$, so we can remove the absolute values.

The antiderivative is $G(x) = -\frac{5}{2}\log_e (x+1) + \frac{45}{2}\log_e (x+3)$.


Evaluate the definite integral of the second part from $x=1$ to $x=2$:

$\int\limits_1^2 \frac{20x + 15}{x^2 + 4x + 3} \;dx = G(2) - G(1)$.

$G(2) = -\frac{5}{2}\log_e (2+1) + \frac{45}{2}\log_e (2+3) = -\frac{5}{2}\log_e 3 + \frac{45}{2}\log_e 5$.

$G(1) = -\frac{5}{2}\log_e (1+1) + \frac{45}{2}\log_e (1+3) = -\frac{5}{2}\log_e 2 + \frac{45}{2}\log_e 4$.

$G(2) - G(1) = \left(-\frac{5}{2}\log_e 3 + \frac{45}{2}\log_e 5\right) - \left(-\frac{5}{2}\log_e 2 + \frac{45}{2}\log_e 4\right)$

$= -\frac{5}{2}\log_e 3 + \frac{45}{2}\log_e 5 + \frac{5}{2}\log_e 2 - \frac{45}{2}(2\log_e 2)$

$= -\frac{5}{2}\log_e 3 + \frac{45}{2}\log_e 5 + \frac{5}{2}\log_e 2 - 45\log_e 2$

$= \frac{45}{2}\log_e 5 - \frac{5}{2}\log_e 3 + \left(\frac{5}{2} - 45\right)\log_e 2$

$= \frac{45}{2}\log_e 5 - \frac{5}{2}\log_e 3 - \frac{85}{2}\log_e 2$.


Combine the results from the two parts:

$\int\limits_1^2 \frac{5x^2}{x^2 + 4x + 3} \;dx = 5 - \left(\frac{45}{2}\log_e 5 - \frac{5}{2}\log_e 3 - \frac{85}{2}\log_e 2\right)$

$= 5 - \frac{45}{2}\log_e 5 + \frac{5}{2}\log_e 3 + \frac{85}{2}\log_e 2$.

This can be written as $5 + \frac{1}{2}(85\log_e 2 + 5\log_e 3 - 45\log_e 5)$.

Using logarithm properties, this is $5 + \frac{1}{2}(\log_e (2^{85}) + \log_e (3^5) - \log_e (5^{45})) = 5 + \frac{1}{2}\log_e \left(\frac{2^{85} \cdot 3^5}{5^{45}}\right)$.


The value of the definite integral is:

$\int\limits_1^2 \frac{5x^2}{x^2 + 4x + 3} \;dx = \mathbf{5 + \frac{85}{2}\log_e 2 + \frac{5}{2}\log_e 3 - \frac{45}{2}\log_e 5}$ or $\mathbf{5 + \frac{1}{2}\log_e \left(\frac{2^{85} \cdot 3^5}{5^{45}}\right)}$.

Question 17. $\int\limits_0^{\frac{π}{4}} (2\sec^2 x + x^3 + 2) \;dx$

Answer:

We need to evaluate the definite integral $\int\limits_0^{\frac{π}{4}} (2\sec^2 x + x^3 + 2) \;dx$.

First, find the indefinite integral of the function $f(x) = 2\sec^2 x + x^3 + 2$.

Using the linearity of integrals, we integrate each term separately:

$\int (2\sec^2 x + x^3 + 2) \;dx = \int 2\sec^2 x \;dx + \int x^3 \;dx + \int 2 \;dx$


The integral of $\sec^2 x$ is $\tan x$. So, $\int 2\sec^2 x \;dx = 2\tan x$.

The integral of $x^3$ is $\frac{x^{3+1}}{3+1} = \frac{x^4}{4}$ using the power rule.

The integral of a constant $2$ is $2x$.


Combining these, the indefinite integral is $2\tan x + \frac{x^4}{4} + 2x$.

Let the antiderivative be $F(x) = 2\tan x + \frac{x^4}{4} + 2x$.


Now, apply the Fundamental Theorem of Calculus: $\int\limits_a^b f(x) \;dx = F(b) - F(a)$.

Here, the lower limit is $a = 0$ and the upper limit is $b = \frac{π}{4}$.

Evaluate $F(x)$ at the upper limit $x = \frac{π}{4}$:

$F\left(\frac{π}{4}\right) = 2\tan\left(\frac{π}{4}\right) + \frac{(π/4)^4}{4} + 2\left(\frac{π}{4}\right)$.

We know that $\tan\left(\frac{π}{4}\right) = 1$.

$F\left(\frac{π}{4}\right) = 2(1) + \frac{π^4/256}{4} + \frac{2π}{4}$

$F\left(\frac{π}{4}\right) = 2 + \frac{π^4}{1024} + \frac{π}{2}$.


Evaluate $F(x)$ at the lower limit $x = 0$:

$F(0) = 2\tan(0) + \frac{(0)^4}{4} + 2(0)$.

We know that $\tan(0) = 0$.

$F(0) = 2(0) + \frac{0}{4} + 0$

$F(0) = 0 + 0 + 0 = 0$.


Subtract $F(a)$ from $F(b)$:

$\int\limits_0^{\frac{π}{4}} (2\sec^2 x + x^3 + 2) \;dx = F\left(\frac{π}{4}\right) - F(0)$

$= \left(2 + \frac{π^4}{1024} + \frac{π}{2}\right) - 0$

$= 2 + \frac{π^4}{1024} + \frac{π}{2}$.


The value of the definite integral is:

$\int\limits_0^{\frac{π}{4}} (2\sec^2 x + x^3 + 2) \;dx = \mathbf{2 + \frac{π}{2} + \frac{π^4}{1024}}$.

Question 18. $\int\limits_0^π \left( \sin^2 \frac{x}{2} − \cos^2 \frac{x}{2} \right) \;dx$

Answer:

We need to evaluate the definite integral $\int\limits_0^π \left( \sin^2 \frac{x}{2} − \cos^2 \frac{x}{2} \right) \;dx$.

We can use the trigonometric identity $\cos 2\theta = \cos^2 \theta - \sin^2 \theta$.

In our case, $\theta = \frac{x}{2}$. So, $\cos(2 \times \frac{x}{2}) = \cos x = \cos^2 \frac{x}{2} - \sin^2 \frac{x}{2}$.

The integrand is $\sin^2 \frac{x}{2} − \cos^2 \frac{x}{2} = -(\cos^2 \frac{x}{2} - \sin^2 \frac{x}{2}) = -\cos x$.


Substitute the identity into the integral:

$\int\limits_0^π (-\cos x) \;dx = - \int\limits_0^π \cos x \;dx$.

First, find the indefinite integral of $\cos x$.

The indefinite integral of $\cos x$ is $\sin x$.

So, the antiderivative is $F(x) = \sin x$.


Now, evaluate the definite integral using the Fundamental Theorem of Calculus: $\int\limits_a^b \cos x \;dx = [\sin x]_a^b = \sin(b) - \sin(a)$.

Here, $a=0$ and $b=π$.

$\int\limits_0^π \cos x \;dx = \sin(π) - \sin(0)$.

We know that $\sin(π) = 0$ and $\sin(0) = 0$.

So, $\int\limits_0^π \cos x \;dx = 0 - 0 = 0$.


Now, consider the original integral with the negative sign:

$\int\limits_0^π \left( \sin^2 \frac{x}{2} − \cos^2 \frac{x}{2} \right) \;dx = - \int\limits_0^π \cos x \;dx = - (0) = 0$.


The value of the definite integral is:

$\int\limits_0^π \left( \sin^2 \frac{x}{2} − \cos^2 \frac{x}{2} \right) \;dx = \mathbf{0}$.

Question 19. $\int\limits_0^2 \frac{6x + 3}{x^2 + 4} \;dx$

Answer:

We need to evaluate the definite integral $\int\limits_0^2 \frac{6x + 3}{x^2 + 4} \;dx$.

We can split the integrand into two fractions:

$\frac{6x + 3}{x^2 + 4} = \frac{6x}{x^2 + 4} + \frac{3}{x^2 + 4}$.

The integral can be written as the sum of two integrals:

$\int\limits_0^2 \left(\frac{6x}{x^2 + 4} + \frac{3}{x^2 + 4}\right) \;dx = \int\limits_0^2 \frac{6x}{x^2 + 4} \;dx + \int\limits_0^2 \frac{3}{x^2 + 4} \;dx$.


Evaluate the first integral: $\int \frac{6x}{x^2 + 4} \;dx$.

Use substitution: Let $u = x^2 + 4$. Then $du = 2x \;dx$. So, $6x \;dx = 3(2x \;dx) = 3 \;du$.

The indefinite integral is $\int \frac{3 \;du}{u} = 3 \int \frac{1}{u} \;du = 3 \log_e |u|$.

Substitute back $u = x^2 + 4$: $3 \log_e |x^2 + 4|$.

Since $x^2 + 4$ is always positive, we can write $3 \log_e (x^2 + 4)$.


Evaluate the second integral: $\int \frac{3}{x^2 + 4} \;dx$.

This integral is of the form $\int \frac{c}{x^2 + a^2} \;dx$, where $c=3$ and $a^2 = 4$, so $a=2$.

The integral is $3 \int \frac{1}{x^2 + 2^2} \;dx = 3 \times \frac{1}{2} \tan^{-1}\left(\frac{x}{2}\right) = \frac{3}{2} \tan^{-1}\left(\frac{x}{2}\right)$.


Combining the results, the antiderivative of the original function is $F(x) = 3 \log_e (x^2 + 4) + \frac{3}{2} \tan^{-1}\left(\frac{x}{2}\right)$.


Now, apply the Fundamental Theorem of Calculus: $\int\limits_a^b f(x) \;dx = F(b) - F(a)$ with $a=0$ and $b=2$.

Evaluate $F(x)$ at the upper limit $x=2$:

$F(2) = 3 \log_e (2^2 + 4) + \frac{3}{2} \tan^{-1}\left(\frac{2}{2}\right) = 3 \log_e (4 + 4) + \frac{3}{2} \tan^{-1}(1)$.

$F(2) = 3 \log_e (8) + \frac{3}{2} \left(\frac{π}{4}\right)$ (since $\tan^{-1}(1) = \frac{π}{4}$).

$F(2) = 3 \log_e (8) + \frac{3π}{8}$.


Evaluate $F(x)$ at the lower limit $x=0$:

$F(0) = 3 \log_e (0^2 + 4) + \frac{3}{2} \tan^{-1}\left(\frac{0}{2}\right) = 3 \log_e (4) + \frac{3}{2} \tan^{-1}(0)$.

$F(0) = 3 \log_e (4) + \frac{3}{2} (0)$ (since $\tan^{-1}(0) = 0$).

$F(0) = 3 \log_e (4)$.


Calculate $F(2) - F(0)$:

$\int\limits_0^2 \frac{6x + 3}{x^2 + 4} \;dx = F(2) - F(0)$

$= \left(3 \log_e (8) + \frac{3π}{8}\right) - 3 \log_e (4)$.

$= 3 \log_e (8) - 3 \log_e (4) + \frac{3π}{8}$.

Use the logarithm property $\log_e A - \log_e B = \log_e (\frac{A}{B})$:

$= 3 (\log_e 8 - \log_e 4) + \frac{3π}{8}$.

$= 3 \log_e \left(\frac{8}{4}\right) + \frac{3π}{8}$.

$= 3 \log_e (2) + \frac{3π}{8}$.


The value of the definite integral is:

$\int\limits_0^2 \frac{6x + 3}{x^2 + 4} \;dx = \mathbf{3 \log_e (2) + \frac{3π}{8}}$.

Question 20. $\int\limits_0^1 \left( x e^x + \sin \frac{πx}{4} \right) \;dx$

Answer:

We need to evaluate the definite integral $\int\limits_0^1 \left( x e^x + \sin \frac{πx}{4} \right) \;dx$.

We can split the integral into two parts using the linearity property:

$\int\limits_0^1 \left( x e^x + \sin \frac{πx}{4} \right) \;dx = \int\limits_0^1 x e^x \;dx + \int\limits_0^1 \sin \frac{πx}{4} \;dx$.


First, let's evaluate the integral $\int\limits_0^1 x e^x \;dx$ using integration by parts.

The formula for integration by parts is $\int u \;dv = uv - \int v \;du$.

Let $u = x$ and $dv = e^x \;dx$.

Then $du = dx$ and $v = \int e^x \;dx = e^x$.

The indefinite integral is $\int x e^x \;dx = x e^x - \int e^x \;dx = x e^x - e^x = e^x(x - 1)$.

Now evaluate the definite integral:

$\int\limits_0^1 x e^x \;dx = [e^x(x - 1)]_0^1 = (e^1(1 - 1)) - (e^0(0 - 1))$

$= (e \times 0) - (1 \times (-1))$

$= 0 - (-1) = 1$.


Next, let's evaluate the integral $\int\limits_0^1 \sin \frac{πx}{4} \;dx$.

We use substitution. Let $u = \frac{πx}{4}$.

Then $du = \frac{π}{4} \;dx$, which means $dx = \frac{4}{π} \;du$.

When $x = 0$, $u = \frac{π(0)}{4} = 0$.

When $x = 1$, $u = \frac{π(1)}{4} = \frac{π}{4}$.

The integral becomes $\int\limits_0^{\frac{π}{4}} \sin u \left(\frac{4}{π}\right) \;du = \frac{4}{π} \int\limits_0^{\frac{π}{4}} \sin u \;du$.

The indefinite integral of $\sin u$ is $-\cos u$.

Now evaluate the definite integral:

$\frac{4}{π} [-\cos u]_0^{\frac{π}{4}} = \frac{4}{π} \left(-\cos\left(\frac{π}{4}\right) - (-\cos(0))\right)$

$= \frac{4}{π} \left(-\frac{\sqrt{2}}{2} + 1\right)$

$= \frac{4}{π} \left(1 - \frac{\sqrt{2}}{2}\right) = \frac{4}{π} - \frac{4}{π} \frac{\sqrt{2}}{2} = \frac{4}{π} - \frac{2\sqrt{2}}{π}$.


Finally, sum the results of the two integrals:

$\int\limits_0^1 \left( x e^x + \sin \frac{πx}{4} \right) \;dx = \int\limits_0^1 x e^x \;dx + \int\limits_0^1 \sin \frac{πx}{4} \;dx$

$= 1 + \left(\frac{4}{π} - \frac{2\sqrt{2}}{π}\right)$.

$= 1 + \frac{4 - 2\sqrt{2}}{π}$.


The value of the definite integral is:

$\int\limits_0^1 \left( x e^x + \sin \frac{πx}{4} \right) \;dx = \mathbf{1 + \frac{4 - 2\sqrt{2}}{π}}$.

Choose the correct answer in Exercises 21 and 22

Question 21. $\int\limits_1^{\sqrt{3}} \frac{dx}{1 + x^2} $ equals

(A) $\frac{π}{3}$

(B) $\frac{2π}{3}$

(C) $\frac{π}{6}$

(D) $\frac{π}{12}$

Answer:

We need to evaluate the definite integral $\int\limits_1^{\sqrt{3}} \frac{dx}{1 + x^2}$.

The indefinite integral of $\frac{1}{1 + x^2}$ is $\tan^{-1}(x)$ (or $\tan^{-1}(x)$).

Let the antiderivative be $F(x) = \tan^{-1}(x)$.


Applying the Fundamental Theorem of Calculus $\int\limits_a^b f(x) \;dx = F(b) - F(a)$, with $a=1$ and $b=\sqrt{3}$:

$\int\limits_1^{\sqrt{3}} \frac{dx}{1 + x^2} = [\tan^{-1}(x)]_1^{\sqrt{3}}$

$= \tan^{-1}(\sqrt{3}) - \tan^{-1}(1)$.


We know the values of $\tan^{-1}(x)$ for standard angles:

$\tan^{-1}(\sqrt{3}) = \frac{π}{3}$ (since $\tan(\frac{π}{3}) = \sqrt{3}$).

$\tan^{-1}(1) = \frac{π}{4}$ (since $\tan(\frac{π}{4}) = 1$).


Substitute these values into the expression:

$\int\limits_1^{\sqrt{3}} \frac{dx}{1 + x^2} = \frac{π}{3} - \frac{π}{4}$.

To subtract the fractions, find a common denominator, which is 12:

$\frac{π}{3} - \frac{π}{4} = \frac{4π}{12} - \frac{3π}{12} = \frac{4π - 3π}{12} = \frac{π}{12}$.


The value of the definite integral is $\frac{π}{12}$.

Comparing this with the given options, we see that it matches option (D).


The final answer is $\mathbf{(D) \;\frac{π}{12}}$.

Question 22. $\int\limits_0^{\frac{2}{3}} \frac{dx}{4 + 9x^2} $ equals

(A) $\frac{π}{6}$

(B) $\frac{π}{12}$

(C) $\frac{π}{24}$

(D) $\frac{π}{4}$

Answer:

We need to evaluate the definite integral $\int\limits_0^{\frac{2}{3}} \frac{dx}{4 + 9x^2}$.

First, we find the indefinite integral of the function $f(x) = \frac{1}{4 + 9x^2}$.

We can rewrite the denominator as $4 + 9x^2 = 2^2 + (3x)^2$.

This integral is of the form $\int \frac{dx}{a^2 + (bx)^2}$. We use the formula $\int \frac{dx}{a^2 + u^2} = \frac{1}{a} \tan^{-1}\left(\frac{u}{a}\right) + C$ with substitution $u = bx$.

Let $u = 3x$. Then $du = 3 \;dx$, which means $dx = \frac{1}{3} \;du$.

The integral becomes $\int \frac{\frac{1}{3} \;du}{2^2 + u^2} = \frac{1}{3} \int \frac{du}{2^2 + u^2}$.

Using the formula with $a=2$ and $u$, we get:

$\frac{1}{3} \times \frac{1}{2} \tan^{-1}\left(\frac{u}{2}\right) = \frac{1}{6} \tan^{-1}\left(\frac{u}{2}\right)$.

Substitute back $u = 3x$:

The indefinite integral is $\frac{1}{6} \tan^{-1}\left(\frac{3x}{2}\right)$.

Let the antiderivative be $F(x) = \frac{1}{6} \tan^{-1}\left(\frac{3x}{2}\right)$.


Now, apply the Fundamental Theorem of Calculus: $\int\limits_a^b f(x) \;dx = F(b) - F(a)$.

Here, the lower limit is $a = 0$ and the upper limit is $b = \frac{2}{3}$.

Evaluate $F(x)$ at the upper limit $x = \frac{2}{3}$:

$F\left(\frac{2}{3}\right) = \frac{1}{6} \tan^{-1}\left(\frac{3 \times (2/3)}{2}\right) = \frac{1}{6} \tan^{-1}\left(\frac{2}{2}\right) = \frac{1}{6} \tan^{-1}(1)$.

We know that $\tan^{-1}(1) = \frac{π}{4}$.

$F\left(\frac{2}{3}\right) = \frac{1}{6} \times \frac{π}{4} = \frac{π}{24}$.


Evaluate $F(x)$ at the lower limit $x = 0$:

$F(0) = \frac{1}{6} \tan^{-1}\left(\frac{3 \times 0}{2}\right) = \frac{1}{6} \tan^{-1}(0)$.

We know that $\tan^{-1}(0) = 0$.

$F(0) = \frac{1}{6} \times 0 = 0$.


Subtract $F(a)$ from $F(b)$:

$\int\limits_0^{\frac{2}{3}} \frac{dx}{4 + 9x^2} = F\left(\frac{2}{3}\right) - F(0)$

$= \frac{π}{24} - 0$

$= \frac{π}{24}$.


The value of the definite integral is $\frac{π}{24}$.

Comparing this with the given options, we find that it matches option (C).


The final answer is $\mathbf{(C) \;\frac{π}{24}}$.



Example 26 & 27 (Before Exercise 7.9)

Example 26: Evaluate $\int\limits_{−1}^1 5x^4 \sqrt{x^5 + 1} \;dx$.

Answer:

We need to evaluate the definite integral $\int\limits_{-1}^1 5x^4 \sqrt{x^5 + 1} \;dx$.

We use the substitution method to find the indefinite integral $\int 5x^4 \sqrt{x^5 + 1} \;dx$.

Let $u = x^5 + 1$.

Differentiating with respect to $x$, we get:

$du = 5x^4 \;dx$.


Substitute $u$ and $du$ into the indefinite integral:

$\int \sqrt{x^5 + 1} (5x^4 \;dx) = \int \sqrt{u} \;du$

$= \int u^{1/2} \;du$.

Using the power rule for integration $\int u^n \;du = \frac{u^{n+1}}{n+1}$:

$= \frac{u^{1/2 + 1}}{1/2 + 1} = \frac{u^{3/2}}{3/2} = \frac{2}{3} u^{3/2}$.

Substitute back $u = x^5 + 1$:

The indefinite integral is $\frac{2}{3} (x^5 + 1)^{3/2}$.

Let the antiderivative be $F(x) = \frac{2}{3} (x^5 + 1)^{3/2}$.


Now, apply the Fundamental Theorem of Calculus: $\int\limits_a^b f(x) \;dx = F(b) - F(a)$.

Here, the lower limit is $a = -1$ and the upper limit is $b = 1$.

Evaluate $F(x)$ at the upper limit $x = 1$:

$F(1) = \frac{2}{3} ((1)^5 + 1)^{3/2}$

$F(1) = \frac{2}{3} (1 + 1)^{3/2}$

$F(1) = \frac{2}{3} (2)^{3/2}$

$F(1) = \frac{2}{3} (2 \sqrt{2}) = \frac{4\sqrt{2}}{3}$.


Evaluate $F(x)$ at the lower limit $x = -1$:

$F(-1) = \frac{2}{3} ((-1)^5 + 1)^{3/2}$

$F(-1) = \frac{2}{3} (-1 + 1)^{3/2}$

$F(-1) = \frac{2}{3} (0)^{3/2}$

$F(-1) = \frac{2}{3} \times 0 = 0$.


Calculate $F(b) - F(a)$:

$\int\limits_{-1}^1 5x^4 \sqrt{x^5 + 1} \;dx = F(1) - F(-1)$

$= \frac{4\sqrt{2}}{3} - 0$

$= \frac{4\sqrt{2}}{3}$.


The value of the definite integral is:

$\int\limits_{−1}^1 5x^4 \sqrt{x^5 + 1} \;dx = \mathbf{\frac{4\sqrt{2}}{3}}$.

Example 27: Evaluate $\int\limits_0^1 \frac{tan^{−1} x}{1 + x^2} \;dx$

Answer:

Solution:


Let the given integral be $I$.

$I = \int\limits_0^1 \frac{\tan^{-1} x}{1 + x^2} \;dx$

We can solve this integral using substitution.

Let $u = \tan^{-1} x$.

Differentiating both sides with respect to $x$, we get:

$\frac{du}{dx} = \frac{1}{1 + x^2}$

So, $du = \frac{1}{1 + x^2} \;dx$.

Now, we need to change the limits of integration according to the substitution $u = \tan^{-1} x$.

When $x = 0$, $u = \tan^{-1}(0) = 0$.

When $x = 1$, $u = \tan^{-1}(1) = \frac{\pi}{4}$.

Substituting $u$ and $du$ into the integral with the new limits:

$I = \int\limits_0^{\pi/4} u \;du$

Now, we evaluate the transformed integral using the power rule for integration $\int u^n \;du = \frac{u^{n+1}}{n+1}$.

$I = \left[\frac{u^{1+1}}{1+1}\right]_0^{\pi/4}$

$I = \left[\frac{u^2}{2}\right]_0^{\pi/4}$

Now, apply the limits of integration:

$I = \frac{(\pi/4)^2}{2} - \frac{(0)^2}{2}$

$I = \frac{\pi^2/16}{2} - 0$

$I = \frac{\pi^2}{16 \times 2}$

$I = \frac{\pi^2}{32}$

Thus, the value of the integral is $\mathbf{\frac{\pi^2}{32}}$.



Exercise 7.9

Evaluate the integrals in Exercises 1 to 8 using substitution.

Question 1. $\int\limits_0^1 \frac{x}{x^2 + 1} \;dx$

Answer:

Solution:


Let the given integral be $I$.

$I = \int\limits_0^1 \frac{x}{x^2 + 1} \;dx$

We use the method of substitution to evaluate this integral.

Let $u = x^2 + 1$.

Differentiate both sides with respect to $x$:

$\frac{du}{dx} = \frac{d}{dx}(x^2 + 1)$

$\frac{du}{dx} = 2x$

This gives us $du = 2x \;dx$.

Rearranging to find $x \;dx$, we get $x \;dx = \frac{1}{2} du$.

Next, we change the limits of integration. The original limits are for $x$. We need to find the corresponding limits for $u$ using the substitution $u = x^2 + 1$.

When the lower limit $x = 0$, the new lower limit for $u$ is $u = (0)^2 + 1 = 0 + 1 = 1$.

When the upper limit $x = 1$, the new upper limit for $u$ is $u = (1)^2 + 1 = 1 + 1 = 2$.

Now, substitute $u$ and $du$ into the integral, along with the new limits:

$I = \int\limits_1^2 \frac{1}{u} \cdot \frac{1}{2} \;du$

$I = \frac{1}{2} \int\limits_1^2 \frac{1}{u} \;du$

The integral of $\frac{1}{u}$ with respect to $u$ is $\log_e |u|$.

$I = \frac{1}{2} [\log_e |u|]_1^2$

Now, apply the limits of integration by substituting the upper limit and subtracting the result of substituting the lower limit:

$I = \frac{1}{2} (\log_e |2| - \log_e |1|)$

Since $2 > 0$, $\log_e |2| = \log_e 2$. The value of $\log_e |1|$ (or $\log_e 1$) is $0$.

$I = \frac{1}{2} (\log_e 2 - 0)$

$I = \frac{1}{2} \log_e 2$

The value of the definite integral is $\mathbf{\frac{1}{2} \log_e 2}$.

Question 2. $\int\limits_0^{\frac{π}{2}} \sqrt{\sin φ} \cos^5 φ \;dφ$

Answer:

Solution:


Let the given integral be $I$.

$I = \int\limits_0^{\frac{π}{2}} \sqrt{\sin φ} \cos^5 φ \;dφ$

We can rewrite $\cos^5 φ$ as $\cos^4 φ \cdot \cos φ$.

$I = \int\limits_0^{\frac{π}{2}} \sqrt{\sin φ} \cos^4 φ \cos φ \;dφ$

Now, we use the identity $\cos^2 φ = 1 - \sin^2 φ$. So, $\cos^4 φ = (\cos^2 φ)^2 = (1 - \sin^2 φ)^2$.

$I = \int\limits_0^{\frac{π}{2}} \sqrt{\sin φ} (1 - \sin^2 φ)^2 \cos φ \;dφ$

We use the method of substitution.

Let $u = \sin φ$.

Differentiate both sides with respect to $φ$:

$\frac{du}{dφ} = \cos φ$

This gives us $du = \cos φ \;dφ$.

Now, we change the limits of integration. The original limits are for $φ$. We need to find the corresponding limits for $u$ using the substitution $u = \sin φ$.

When the lower limit $φ = 0$, the new lower limit for $u$ is $u = \sin(0) = 0$.

When the upper limit $φ = \frac{π}{2}$, the new upper limit for $u$ is $u = \sin(\frac{π}{2}) = 1$.

Substitute $u$ and $du$ into the integral with the new limits:

$I = \int\limits_0^1 \sqrt{u} (1 - u^2)^2 \;du$

Expand the term $(1 - u^2)^2$:

$(1 - u^2)^2 = 1^2 - 2(1)(u^2) + (u^2)^2 = 1 - 2u^2 + u^4$

Substitute this back into the integral:

$I = \int\limits_0^1 u^{1/2} (1 - 2u^2 + u^4) \;du$

Distribute $u^{1/2}$ inside the parenthesis:

$I = \int\limits_0^1 (u^{1/2} \cdot 1 - u^{1/2} \cdot 2u^2 + u^{1/2} \cdot u^4) \;du$

$I = \int\limits_0^1 (u^{1/2} - 2u^{1/2 + 2} + u^{1/2 + 4}) \;du$

$I = \int\limits_0^1 (u^{1/2} - 2u^{5/2} + u^{9/2}) \;du$

Now, integrate term by term using the power rule $\int u^n \;du = \frac{u^{n+1}}{n+1}$:

$I = \left[\frac{u^{1/2 + 1}}{1/2 + 1} - 2\frac{u^{5/2 + 1}}{5/2 + 1} + \frac{u^{9/2 + 1}}{9/2 + 1}\right]_0^1$

$I = \left[\frac{u^{3/2}}{3/2} - 2\frac{u^{7/2}}{7/2} + \frac{u^{11/2}}{11/2}\right]_0^1$

$I = \left[\frac{2}{3} u^{3/2} - \frac{4}{7} u^{7/2} + \frac{2}{11} u^{11/2}\right]_0^1$

Now, apply the limits of integration:

$I = \left(\frac{2}{3} (1)^{3/2} - \frac{4}{7} (1)^{7/2} + \frac{2}{11} (1)^{11/2}\right) - \left(\frac{2}{3} (0)^{3/2} - \frac{4}{7} (0)^{7/2} + \frac{2}{11} (0)^{11/2}\right)$

$I = \left(\frac{2}{3} (1) - \frac{4}{7} (1) + \frac{2}{11} (1)\right) - (0 - 0 + 0)$

$I = \frac{2}{3} - \frac{4}{7} + \frac{2}{11}$

To combine these fractions, find a common denominator, which is the least common multiple of 3, 7, and 11. Since 3, 7, and 11 are prime numbers, their LCM is their product: $3 \times 7 \times 11 = 231$.

$I = \frac{2}{3} \cdot \frac{77}{77} - \frac{4}{7} \cdot \frac{33}{33} + \frac{2}{11} \cdot \frac{21}{21}$

$I = \frac{154}{231} - \frac{132}{231} + \frac{42}{231}$

$I = \frac{154 - 132 + 42}{231}$

$I = \frac{22 + 42}{231}$

$I = \frac{64}{231}$

The value of the definite integral is $\mathbf{\frac{64}{231}}$.

Question 3. $\int\limits_0^1 \sin^{−1} \left( \frac{2x}{1 + x^2} \right) \;dx$

Answer:

Solution:


Let the given integral be $I$.

$I = \int\limits_0^1 \sin^{−1} \left( \frac{2x}{1 + x^2} \right) \;dx$

We use the identity $\sin^{−1} \left( \frac{2x}{1 + x^2} \right) = 2\tan^{-1} x$. This identity is valid for $-1 \leq x \leq 1$, which covers the interval of integration $[0, 1]$.

Substituting the identity into the integral:

$I = \int\limits_0^1 2\tan^{-1} x \;dx$

$I = 2 \int\limits_0^1 \tan^{-1} x \;dx$

Now we evaluate the indefinite integral $\int \tan^{-1} x \;dx$ using integration by parts.

Let $u = \tan^{-1} x$ and $dv = dx$.

Then $du = \frac{1}{1 + x^2} \;dx$ and $v = \int dx = x$.

Using the integration by parts formula $\int u \;dv = uv - \int v \;du$:

$\int \tan^{-1} x \;dx = x \tan^{-1} x - \int x \cdot \frac{1}{1 + x^2} \;dx$

Now, evaluate the integral $\int \frac{x}{1 + x^2} \;dx$ using substitution.

Let $w = 1 + x^2$. Then $dw = 2x \;dx$, which means $x \;dx = \frac{1}{2} dw$.

$\int \frac{x}{1 + x^2} \;dx = \int \frac{1}{w} \frac{1}{2} \;dw = \frac{1}{2} \int \frac{1}{w} \;dw = \frac{1}{2} \log_e |w| + C_1$.

Substituting back $w = 1 + x^2$ (since $1+x^2 > 0$, $|w| = w$):

$\int \frac{x}{1 + x^2} \;dx = \frac{1}{2} \log_e (1 + x^2) + C_1$.

Substitute this result back into the integration by parts for $\int \tan^{-1} x \;dx$:

$\int \tan^{-1} x \;dx = x \tan^{-1} x - \frac{1}{2} \log_e (1 + x^2) + C$.

Now, apply the limits of the definite integral:

$I = 2 \left[ x \tan^{-1} x - \frac{1}{2} \log_e (1 + x^2) \right]_0^1$

Evaluate the expression at the upper limit ($x=1$) and subtract the evaluation at the lower limit ($x=0$):

$I = 2 \left[ \left( 1 \cdot \tan^{-1}(1) - \frac{1}{2} \log_e (1^2 + 1) \right) - \left( 0 \cdot \tan^{-1}(0) - \frac{1}{2} \log_e (0^2 + 1) \right) \right]$

$I = 2 \left[ \left( 1 \cdot \frac{\pi}{4} - \frac{1}{2} \log_e (2) \right) - \left( 0 \cdot 0 - \frac{1}{2} \log_e (1) \right) \right]$

Since $\tan^{-1}(0) = 0$ and $\log_e (1) = 0$:

$I = 2 \left[ \left( \frac{\pi}{4} - \frac{1}{2} \log_e 2 \right) - (0 - 0) \right]$

$I = 2 \left( \frac{\pi}{4} - \frac{1}{2} \log_e 2 \right)$

Distribute the 2:

$I = 2 \cdot \frac{\pi}{4} - 2 \cdot \frac{1}{2} \log_e 2$

$I = \frac{\pi}{2} - \log_e 2$

The value of the definite integral is $\mathbf{\frac{\pi}{2} - \log_e 2}$.

Question 4. $\int\limits_0^2 x \sqrt{x + 2}$

(Put x + 2 = t2)

Answer:

Solution:


Let the given integral be $I$.

$I = \int\limits_0^2 x \sqrt{x + 2} \;dx$

We are asked to use the substitution $x + 2 = t^2$.

From the substitution, we have $x = t^2 - 2$.

Differentiating both sides with respect to $t$:

$\frac{dx}{dt} = \frac{d}{dt}(t^2 - 2)$

$\frac{dx}{dt} = 2t$

So, $dx = 2t \;dt$.

Now, we change the limits of integration. The original limits are for $x$. We need to find the corresponding limits for $t$ using the substitution $x + 2 = t^2$. Since $\sqrt{x+2}$ is involved, we consider $t \geq 0$.

When the lower limit $x = 0$:

$0 + 2 = t^2$

$t^2 = 2$

Since $t \geq 0$, the new lower limit for $t$ is $t = \sqrt{2}$.

When the upper limit $x = 2$:

$2 + 2 = t^2$

$t^2 = 4$

Since $t \geq 0$, the new upper limit for $t$ is $t = \sqrt{4} = 2$.

Substitute $x$, $\sqrt{x+2}$, $dx$, and the new limits into the integral:

$I = \int\limits_{\sqrt{2}}^2 (t^2 - 2) \sqrt{t^2} (2t \;dt)$

Since the limits are $\sqrt{2}$ to 2, $t$ is positive in this range, so $\sqrt{t^2} = t$.

$I = \int\limits_{\sqrt{2}}^2 (t^2 - 2) t (2t \;dt)$

$I = \int\limits_{\sqrt{2}}^2 (t^3 - 2t) (2t \;dt)$

$I = \int\limits_{\sqrt{2}}^2 (2t^4 - 4t^2) \;dt$

Now, integrate term by term with respect to $t$:

$I = \left[2 \frac{t^{4+1}}{4+1} - 4 \frac{t^{2+1}}{2+1}\right]_{\sqrt{2}}^2$

$I = \left[2 \frac{t^5}{5} - 4 \frac{t^3}{3}\right]_{\sqrt{2}}^2$

$I = \left[\frac{2}{5} t^5 - \frac{4}{3} t^3\right]_{\sqrt{2}}^2$

Apply the limits of integration:

$I = \left(\frac{2}{5} (2)^5 - \frac{4}{3} (2)^3\right) - \left(\frac{2}{5} (\sqrt{2})^5 - \frac{4}{3} (\sqrt{2})^3\right)$

Evaluate the powers:

$(2)^5 = 32$

$(2)^3 = 8$

$(\sqrt{2})^5 = (\sqrt{2})^4 \cdot \sqrt{2} = ((\sqrt{2})^2)^2 \cdot \sqrt{2} = (2)^2 \cdot \sqrt{2} = 4\sqrt{2}$

$(\sqrt{2})^3 = (\sqrt{2})^2 \cdot \sqrt{2} = 2\sqrt{2}$

Substitute these values back:

$I = \left(\frac{2}{5} (32) - \frac{4}{3} (8)\right) - \left(\frac{2}{5} (4\sqrt{2}) - \frac{4}{3} (2\sqrt{2})\right)$

$I = \left(\frac{64}{5} - \frac{32}{3}\right) - \left(\frac{8\sqrt{2}}{5} - \frac{8\sqrt{2}}{3}\right)$

Remove the parenthesis and change the signs:

$I = \frac{64}{5} - \frac{32}{3} - \frac{8\sqrt{2}}{5} + \frac{8\sqrt{2}}{3}$

Group terms with common denominators:

$I = \left(\frac{64}{5} - \frac{8\sqrt{2}}{5}\right) + \left(\frac{8\sqrt{2}}{3} - \frac{32}{3}\right)$

$I = \frac{64 - 8\sqrt{2}}{5} + \frac{8\sqrt{2} - 32}{3}$

Find a common denominator (LCM of 5 and 3 is 15):

$I = \frac{3(64 - 8\sqrt{2})}{15} + \frac{5(8\sqrt{2} - 32)}{15}$

$I = \frac{192 - 24\sqrt{2} + 40\sqrt{2} - 160}{15}$

Combine like terms:

$I = \frac{(192 - 160) + (40\sqrt{2} - 24\sqrt{2})}{15}$

$I = \frac{32 + 16\sqrt{2}}{15}$

The value of the definite integral is $\mathbf{\frac{32 + 16\sqrt{2}}{15}}$.

Question 5. $\int\limits_0^{\frac{π}{2}} \frac{\sin x}{1 + \cos^2 x} \;dx$

Answer:

Solution:


Let the given integral be $I$.

$I = \int\limits_0^{\frac{π}{2}} \frac{\sin x}{1 + \cos^2 x} \;dx$

We use the method of substitution to evaluate this integral.

Let $u = \cos x$.

Differentiate both sides with respect to $x$:

$\frac{du}{dx} = \frac{d}{dx}(\cos x)$

$\frac{du}{dx} = -\sin x$

This gives us $du = -\sin x \;dx$, or $\sin x \;dx = -du$.

Next, we change the limits of integration. The original limits are for $x$. We need to find the corresponding limits for $u$ using the substitution $u = \cos x$.

When the lower limit $x = 0$, the new lower limit for $u$ is $u = \cos(0) = 1$.

When the upper limit $x = \frac{π}{2}$, the new upper limit for $u$ is $u = \cos(\frac{π}{2}) = 0$.

Substitute $u$, $du$, and the new limits into the integral:

$I = \int\limits_1^0 \frac{-du}{1 + u^2}$

Using the property of definite integrals $\int_a^b f(x) dx = -\int_b^a f(x) dx$, we can swap the limits by changing the sign:

$I = - \int\limits_1^0 \frac{1}{1 + u^2} \;du$

$I = \int\limits_0^1 \frac{1}{1 + u^2} \;du$

The integral of $\frac{1}{1 + u^2}$ with respect to $u$ is $\tan^{-1} u$.

$I = [\tan^{-1} u]_0^1$

Now, apply the limits of integration by substituting the upper limit and subtracting the result of substituting the lower limit:

$I = \tan^{-1}(1) - \tan^{-1}(0)$

Evaluate the values of $\tan^{-1}$ at these points:

$\tan^{-1}(1) = \frac{π}{4}$

$\tan^{-1}(0) = 0$

Substitute these values back into the expression for $I$:

$I = \frac{π}{4} - 0$

$I = \frac{π}{4}$

The value of the definite integral is $\mathbf{\frac{π}{4}}$.

Question 6. $\int\limits_0^2 \frac{dx}{x + 4 − x^2} $

Answer:

Solution:


Let the given integral be $I$.

$I = \int\limits_0^2 \frac{dx}{x + 4 − x^2}$

We need to evaluate this integral. The denominator is a quadratic expression $4 + x - x^2$. We can rewrite the denominator by completing the square.

$4 + x - x^2 = -(x^2 - x - 4)$

Complete the square for the quadratic term inside the parenthesis:

$x^2 - x = x^2 - 2 \cdot x \cdot \frac{1}{2} + \left(\frac{1}{2}\right)^2 - \left(\frac{1}{2}\right)^2 = \left(x - \frac{1}{2}\right)^2 - \frac{1}{4}$

So, $x^2 - x - 4 = \left(x - \frac{1}{2}\right)^2 - \frac{1}{4} - 4 = \left(x - \frac{1}{2}\right)^2 - \frac{1}{4} - \frac{16}{4} = \left(x - \frac{1}{2}\right)^2 - \frac{17}{4}$

Substitute this back into the denominator:

$4 + x - x^2 = - \left[ \left(x - \frac{1}{2}\right)^2 - \frac{17}{4} \right] = \frac{17}{4} - \left(x - \frac{1}{2}\right)^2$

The integral becomes:

$I = \int\limits_0^2 \frac{dx}{\frac{17}{4} - \left(x - \frac{1}{2}\right)^2}$

This integral is in the form $\int \frac{du}{a^2 - u^2}$, where $a^2 = \frac{17}{4}$ and $u = x - \frac{1}{2}$.

So, $a = \sqrt{\frac{17}{4}} = \frac{\sqrt{17}}{2}$.

Let $u = x - \frac{1}{2}$. Then $du = dx$.

Now we change the limits of integration. The original limits are for $x$. We need to find the corresponding limits for $u = x - \frac{1}{2}$.

When the lower limit $x = 0$, the new lower limit for $u$ is $u = 0 - \frac{1}{2} = -\frac{1}{2}$.

When the upper limit $x = 2$, the new upper limit for $u$ is $u = 2 - \frac{1}{2} = \frac{4}{2} - \frac{1}{2} = \frac{3}{2}$.

Substitute $u$, $du$, and the new limits into the integral:

$I = \int\limits_{-1/2}^{3/2} \frac{du}{\frac{17}{4} - u^2}$

Using the standard integral formula $\int \frac{du}{a^2 - u^2} = \frac{1}{2a} \log_e \left|\frac{a+u}{a-u}\right| + C$:

$I = \left[ \frac{1}{2 \left(\frac{\sqrt{17}}{2}\right)} \log_e \left|\frac{\frac{\sqrt{17}}{2} + u}{\frac{\sqrt{17}}{2} - u}\right| \right]_{-1/2}^{3/2}$

$I = \frac{1}{\sqrt{17}} \left[ \log_e \left|\frac{\sqrt{17} + 2u}{\sqrt{17} - 2u}\right| \right]_{-1/2}^{3/2}$

Now, apply the limits of integration:

$I = \frac{1}{\sqrt{17}} \left( \log_e \left|\frac{\sqrt{17} + 2(\frac{3}{2})}{\sqrt{17} - 2(\frac{3}{2})}\right| - \log_e \left|\frac{\sqrt{17} + 2(-\frac{1}{2})}{\sqrt{17} - 2(-\frac{1}{2})}\right| \right)$

$I = \frac{1}{\sqrt{17}} \left( \log_e \left|\frac{\sqrt{17} + 3}{\sqrt{17} - 3}\right| - \log_e \left|\frac{\sqrt{17} - 1}{\sqrt{17} + 1}\right| \right)$

Using the logarithm property $\log_e A - \log_e B = \log_e \frac{A}{B}$:

$I = \frac{1}{\sqrt{17}} \log_e \left| \frac{\frac{\sqrt{17} + 3}{\sqrt{17} - 3}}{\frac{\sqrt{17} - 1}{\sqrt{17} + 1}} \right|$

$I = \frac{1}{\sqrt{17}} \log_e \left| \frac{\sqrt{17} + 3}{\sqrt{17} - 3} \times \frac{\sqrt{17} + 1}{\sqrt{17} - 1} \right|$

Simplify the expression inside the logarithm:

$(\sqrt{17} + 3)(\sqrt{17} + 1) = (\sqrt{17})^2 + \sqrt{17} + 3\sqrt{17} + 3 = 17 + 4\sqrt{17} + 3 = 20 + 4\sqrt{17}$

$(\sqrt{17} - 3)(\sqrt{17} - 1) = (\sqrt{17})^2 - \sqrt{17} - 3\sqrt{17} + 3 = 17 - 4\sqrt{17} + 3 = 20 - 4\sqrt{17}$

The expression inside the logarithm is $\frac{20 + 4\sqrt{17}}{20 - 4\sqrt{17}}$.

Factor out 4 from the numerator and denominator:

$\frac{4(5 + \sqrt{17})}{4(5 - \sqrt{17})} = \frac{5 + \sqrt{17}}{5 - \sqrt{17}}$

Rationalize the denominator:

$\frac{5 + \sqrt{17}}{5 - \sqrt{17}} \times \frac{5 + \sqrt{17}}{5 + \sqrt{17}} = \frac{(5 + \sqrt{17})^2}{5^2 - (\sqrt{17})^2} = \frac{25 + 10\sqrt{17} + 17}{25 - 17} = \frac{42 + 10\sqrt{17}}{8}$

Simplify the fraction by dividing numerator and denominator by 2:

$\frac{21 + 5\sqrt{17}}{4}$

Since $21 + 5\sqrt{17} > 0$ and $4 > 0$ for the interval $[0, 2]$, the absolute value can be removed.

So, $I = \frac{1}{\sqrt{17}} \log_e \left( \frac{21 + 5\sqrt{17}}{4} \right)$.

The value of the definite integral is $\mathbf{\frac{1}{\sqrt{17}} \log_e \left( \frac{21 + 5\sqrt{17}}{4} \right)}$.

Question 7. $\int\limits_{−1}^1 \frac{dx}{x^2 + 2x + 5} $

Answer:

Solution:


Let the given integral be $I$.

$I = \int\limits_{−1}^1 \frac{dx}{x^2 + 2x + 5}$

We need to evaluate this definite integral. The denominator is a quadratic expression $x^2 + 2x + 5$. We can rewrite the denominator by completing the square.

$x^2 + 2x + 5 = (x^2 + 2x + 1) + 4$

$x^2 + 2x + 5 = (x + 1)^2 + 2^2$

The integral becomes:

$I = \int\limits_{−1}^1 \frac{dx}{(x + 1)^2 + 2^2}$

This integral is in the form $\int \frac{du}{u^2 + a^2}$, where $u = x + 1$ and $a = 2$.

Let $u = x + 1$.

Differentiating both sides with respect to $x$:

$\frac{du}{dx} = \frac{d}{dx}(x + 1)$

$\frac{du}{dx} = 1$

This gives us $du = dx$.

Next, we change the limits of integration. The original limits are for $x$. We need to find the corresponding limits for $u$ using the substitution $u = x + 1$.

When the lower limit $x = -1$, the new lower limit for $u$ is $u = -1 + 1 = 0$.

When the upper limit $x = 1$, the new upper limit for $u$ is $u = 1 + 1 = 2$.

Substitute $u$, $du$, and the new limits into the integral:

$I = \int\limits_0^2 \frac{du}{u^2 + 2^2}$

Using the standard integral formula $\int \frac{du}{u^2 + a^2} = \frac{1}{a} \tan^{-1}\left(\frac{u}{a}\right) + C$:

$I = \left[ \frac{1}{2} \tan^{-1}\left(\frac{u}{2}\right) \right]_0^2$

Now, apply the limits of integration by substituting the upper limit and subtracting the result of substituting the lower limit:

$I = \frac{1}{2} \tan^{-1}\left(\frac{2}{2}\right) - \frac{1}{2} \tan^{-1}\left(\frac{0}{2}\right)$

$I = \frac{1}{2} \tan^{-1}(1) - \frac{1}{2} \tan^{-1}(0)$

Evaluate the values of $\tan^{-1}$ at these points:

$\tan^{-1}(1) = \frac{\pi}{4}$

$\tan^{-1}(0) = 0$

Substitute these values back into the expression for $I$:

$I = \frac{1}{2} \left(\frac{\pi}{4}\right) - \frac{1}{2} (0)$

$I = \frac{\pi}{8} - 0$

$I = \frac{\pi}{8}$

The value of the definite integral is $\mathbf{\frac{\pi}{8}}$.

Question 8. $\int\limits_1^2 \left( \frac{1}{x} − \frac{1}{2x^2} \right) e^{2x} \;dx$

Answer:

Solution:


Let the given integral be $I$.

$I = \int\limits_1^2 \left( \frac{1}{x} − \frac{1}{2x^2} \right) e^{2x} \;dx$

We can rewrite the integrand to match the form $\int e^{ax} [a f(x) + f'(x)] \;dx = e^{ax} f(x) + C$.

Let's observe the terms within the parenthesis and the exponential function.

The exponential part is $e^{2x}$, which suggests $a = 2$.

We need the term inside the parenthesis to be in the form $a f(x) + f'(x)$, which is $2 f(x) + f'(x)$.

Consider $f(x) = \frac{1}{2x}$.

Then $a f(x) = 2 \cdot \left(\frac{1}{2x}\right) = \frac{1}{x}$.

Now, let's find the derivative of $f(x) = \frac{1}{2x}$.

$f(x) = \frac{1}{2} x^{-1}$

$f'(x) = \frac{1}{2} \cdot (-1) x^{-1-1} = -\frac{1}{2} x^{-2} = -\frac{1}{2x^2}$.

So, the integrand $\left( \frac{1}{x} − \frac{1}{2x^2} \right) e^{2x}$ can be written as $e^{2x} \left( 2 \cdot \frac{1}{2x} + \left(-\frac{1}{2x^2}\right) \right)$.

This perfectly matches the form $e^{ax} [a f(x) + f'(x)]$ with $a=2$ and $f(x) = \frac{1}{2x}$.

Using the integration formula $\int e^{ax} [a f(x) + f'(x)] \;dx = e^{ax} f(x) + C$, the indefinite integral is:

$\int \left( \frac{1}{x} − \frac{1}{2x^2} \right) e^{2x} \;dx = e^{2x} \cdot \frac{1}{2x} + C = \frac{e^{2x}}{2x} + C$

Now, we evaluate the definite integral using the limits of integration from 1 to 2:

$I = \left[\frac{e^{2x}}{2x}\right]_1^2$

Apply the Fundamental Theorem of Calculus by substituting the upper limit and subtracting the result of substituting the lower limit:

$I = \left(\frac{e^{2(2)}}{2(2)}\right) - \left(\frac{e^{2(1)}}{2(1)}\right)$

$I = \frac{e^4}{4} - \frac{e^2}{2}$

The value of the definite integral is $\mathbf{\frac{e^4}{4} - \frac{e^2}{2}}$.

Choose the correct answer in Exercises 9 and 10.

Question 9. The value of the integral $\int\limits_{\frac{1}{3}}^1 \frac{(x − x^3)^{\frac{1}{3}}}{x^4} \;dx$ is

(A) 6

(B) 0

(C) 3

(D) 4

Answer:

Solution:


Let the given integral be $I$.

$I = \int\limits_{\frac{1}{3}}^1 \frac{(x − x^3)^{\frac{1}{3}}}{x^4} \;dx$

We can rewrite the term inside the parenthesis in the numerator by factoring out $x^3$:

$(x − x^3)^{\frac{1}{3}} = \left[x^3\left(\frac{x}{x^3} - \frac{x^3}{x^3}\right)\right]^{\frac{1}{3}} = \left[x^3\left(\frac{1}{x^2} - 1\right)\right]^{\frac{1}{3}}$

$= (x^3)^{\frac{1}{3}} \left(\frac{1}{x^2} - 1\right)^{\frac{1}{3}} = x \left(\frac{1}{x^2} - 1\right)^{\frac{1}{3}}$

Substitute this back into the integrand:

$\frac{(x − x^3)^{\frac{1}{3}}}{x^4} = \frac{x \left(\frac{1}{x^2} - 1\right)^{\frac{1}{3}}}{x^4} = \frac{\left(\frac{1}{x^2} - 1\right)^{\frac{1}{3}}}{x^3}$

The integral becomes:

$I = \int\limits_{\frac{1}{3}}^1 \frac{\left(\frac{1}{x^2} - 1\right)^{\frac{1}{3}}}{x^3} \;dx$

Now, we use the method of substitution.

Let $u = \frac{1}{x^2} - 1$.

Differentiate both sides with respect to $x$:

$\frac{du}{dx} = \frac{d}{dx}(x^{-2} - 1) = -2x^{-3} = -\frac{2}{x^3}$.

This gives us $du = -\frac{2}{x^3} \;dx$, which implies $\frac{1}{x^3} \;dx = -\frac{1}{2} du$.

Next, we change the limits of integration. The original limits are for $x$. We need to find the corresponding limits for $u$ using the substitution $u = \frac{1}{x^2} - 1$.

When the lower limit $x = \frac{1}{3}$, the new lower limit for $u$ is $u = \frac{1}{(\frac{1}{3})^2} - 1 = \frac{1}{\frac{1}{9}} - 1 = 9 - 1 = 8$.

When the upper limit $x = 1$, the new upper limit for $u$ is $u = \frac{1}{(1)^2} - 1 = 1 - 1 = 0$.

Substitute $u$, $du$, and the new limits into the integral:

$I = \int\limits_8^0 u^{\frac{1}{3}} \left(-\frac{1}{2}\right) du$

$I = -\frac{1}{2} \int\limits_8^0 u^{\frac{1}{3}} \;du$

Using the property of definite integrals $\int_a^b f(x) dx = -\int_b^a f(x) dx$, we can swap the limits by changing the sign:

$I = -\frac{1}{2} \left[-\int\limits_0^8 u^{\frac{1}{3}} \;du\right] = \frac{1}{2} \int\limits_0^8 u^{\frac{1}{3}} \;du$

Now, integrate $u^{\frac{1}{3}}$ with respect to $u$ using the power rule $\int u^n \;du = \frac{u^{n+1}}{n+1}$:

$\int u^{\frac{1}{3}} \;du = \frac{u^{\frac{1}{3}+1}}{\frac{1}{3}+1} + C = \frac{u^{\frac{4}{3}}}{\frac{4}{3}} + C = \frac{3}{4} u^{\frac{4}{3}} + C$

Evaluate the definite integral:

$I = \frac{1}{2} \left[\frac{3}{4} u^{\frac{4}{3}}\right]_0^8$

Apply the limits of integration:

$I = \frac{1}{2} \left(\frac{3}{4} (8)^{\frac{4}{3}} - \frac{3}{4} (0)^{\frac{4}{3}}\right)$

Evaluate the terms:

$(8)^{\frac{4}{3}} = (\sqrt[3]{8})^4 = (2)^4 = 16$.

$(0)^{\frac{4}{3}} = 0$.

Substitute these values back:

$I = \frac{1}{2} \left(\frac{3}{4} (16) - \frac{3}{4} (0)\right)$

$I = \frac{1}{2} \left(\frac{3 \cdot 16}{4} - 0\right)$

$I = \frac{1}{2} (3 \cdot 4)$

$I = \frac{1}{2} (12)$

$I = 6$


The value of the integral is $\mathbf{6}$.


The correct option is (A) 6.

Question 10. If $f(x) = \int\limits_0^x t \sin t \;dt$, then f’(x) is

(A) cos x + x sin x

(B) x sin x

(C) x cos x

(D) sin x + x cos x

Answer:

Solution:


We are given the function $f(x)$ defined as a definite integral with a variable upper limit:

$f(x) = \int\limits_0^x t \sin t \;dt$

We need to find the derivative of $f(x)$, which is $f'(x)$.

We can use the Fundamental Theorem of Calculus, Part 1.

The theorem states that if $F(x) = \int\limits_a^x g(t) \;dt$, where $g(t)$ is a continuous function on an interval containing $a$ and $x$, then the derivative of $F(x)$ with respect to $x$ is given by $F'(x) = g(x)$.

In our case, the function is $f(x) = \int\limits_0^x t \sin t \;dt$.

Comparing this with the form $F(x) = \int\limits_a^x g(t) \;dt$, we have:

The lower limit $a = 0$.

The upper limit is $x$.

The integrand is $g(t) = t \sin t$.

The function $g(t) = t \sin t$ is continuous for all values of $t$.

According to the Fundamental Theorem of Calculus, Part 1, the derivative $f'(x)$ is obtained by replacing $t$ with $x$ in the integrand $g(t)$.

$f'(x) = g(x) = x \sin x$

The derivative of $f(x)$ is $\mathbf{x \sin x}$.


Comparing this result with the given options, we find that the correct option is (B).

The correct answer is (B) x sin x.



Example 28 to 34 (Before Exercise 7.10)

Example 28: Evaluate $\int\limits_{−1}^2 |x^3 − x| \;dx$

Answer:

Given:

The definite integral $\int\limits_{−1}^2 |x^3 − x| \;dx$


To Evaluate:

The value of the given integral.


Solution:

Let the given integral be $I$.

$I = \int\limits_{−1}^2 |x^3 − x| \;dx$

To evaluate this integral, we need to determine where the expression inside the absolute value, $x^3 - x$, is positive or negative in the interval of integration $[-1, 2]$.

First, find the roots of $x^3 - x = 0$:

$x(x^2 - 1) = 0$

$x(x - 1)(x + 1) = 0$

The roots are $x = -1$, $x = 0$, and $x = 1$. These roots divide the interval $[-1, 2]$ into subintervals.

We analyze the sign of $x^3 - x$ in each subinterval:

  • Interval $[-1, 0]$: For $x \in (-1, 0)$, let's test $x = -0.5$. $(-0.5)^3 - (-0.5) = -0.125 + 0.5 = 0.375 > 0$. So, $x^3 - x \geq 0$ on $[-1, 0]$. Thus, $|x^3 - x| = x^3 - x$.
  • Interval $[0, 1]$: For $x \in (0, 1)$, let's test $x = 0.5$. $(0.5)^3 - (0.5) = 0.125 - 0.5 = -0.375 < 0$. So, $x^3 - x \leq 0$ on $[0, 1]$. Thus, $|x^3 - x| = -(x^3 - x) = x - x^3$.
  • Interval $[1, 2]$: For $x \in (1, 2]$, let's test $x = 1.5$. $(1.5)^3 - (1.5) = 3.375 - 1.5 = 1.875 > 0$. So, $x^3 - x \geq 0$ on $[1, 2]$. Thus, $|x^3 - x| = x^3 - x$.

Now, we can split the integral based on these intervals:

$I = \int\limits_{−1}^0 (x^3 − x) \;dx + \int\limits_0^1 (x - x^3) \;dx + \int\limits_1^2 (x^3 − x) \;dx$

We evaluate each integral separately.

Integral 1: $\int\limits_{−1}^0 (x^3 − x) \;dx$

$\int (x^3 - x) \;dx = \frac{x^4}{4} - \frac{x^2}{2}$

$\int\limits_{−1}^0 (x^3 − x) \;dx = \left[\frac{x^4}{4} - \frac{x^2}{2}\right]_{-1}^0 = \left(\frac{0^4}{4} - \frac{0^2}{2}\right) - \left(\frac{(-1)^4}{4} - \frac{(-1)^2}{2}\right)$

$= (0 - 0) - \left(\frac{1}{4} - \frac{1}{2}\right) = 0 - \left(\frac{1}{4} - \frac{2}{4}\right) = - \left(-\frac{1}{4}\right) = \frac{1}{4}$

Integral 2: $\int\limits_0^1 (x - x^3) \;dx$

$\int (x - x^3) \;dx = \frac{x^2}{2} - \frac{x^4}{4}$

$\int\limits_0^1 (x - x^3) \;dx = \left[\frac{x^2}{2} - \frac{x^4}{4}\right]_0^1 = \left(\frac{1^2}{2} - \frac{1^4}{4}\right) - \left(\frac{0^2}{2} - \frac{0^4}{4}\right)$

$= \left(\frac{1}{2} - \frac{1}{4}\right) - (0 - 0) = \left(\frac{2}{4} - \frac{1}{4}\right) = \frac{1}{4}$

Integral 3: $\int\limits_1^2 (x^3 − x) \;dx$

$\int (x^3 - x) \;dx = \frac{x^4}{4} - \frac{x^2}{2}$

$\int\limits_1^2 (x^3 − x) \;dx = \left[\frac{x^4}{4} - \frac{x^2}{2}\right]_1^2 = \left(\frac{2^4}{4} - \frac{2^2}{2}\right) - \left(\frac{1^4}{4} - \frac{1^2}{2}\right)$

$= \left(\frac{16}{4} - \frac{4}{2}\right) - \left(\frac{1}{4} - \frac{1}{2}\right) = (4 - 2) - \left(\frac{1}{4} - \frac{2}{4}\right)$

$= 2 - \left(-\frac{1}{4}\right) = 2 + \frac{1}{4} = \frac{8}{4} + \frac{1}{4} = \frac{9}{4}$

Now, sum the values of the three integrals to find the total value of $I$:

$I = \frac{1}{4} + \frac{1}{4} + \frac{9}{4}$

$I = \frac{1 + 1 + 9}{4} = \frac{11}{4}$

The value of the integral is $\mathbf{\frac{11}{4}}$.

Example 29: Evaluate $\int\limits_{\frac{−π}{4}}^{\frac{π}{4}} \sin^2 x \;dx$

Answer:

Given:

The definite integral $\int\limits_{\frac{−π}{4}}^{\frac{π}{4}} \sin^2 x \;dx$


To Evaluate:

The value of the given integral.


Solution:

Let the given integral be $I$.

$I = \int\limits_{\frac{−π}{4}}^{\frac{π}{4}} \sin^2 x \;dx$

The interval of integration is $\left[-\frac{\pi}{4}, \frac{\pi}{4}\right]$, which is symmetric about 0.

Let $f(x) = \sin^2 x$. We check if the function is even or odd.

$f(-x) = \sin^2(-x) = (\sin(-x))^2$

Since $\sin(-x) = -\sin x$, we have:

$f(-x) = (-\sin x)^2 = \sin^2 x$

So, $f(-x) = f(x)$, which means $f(x) = \sin^2 x$ is an even function.

For an even function $f(x)$ over a symmetric interval $[-a, a]$, we have the property:

$\int\limits_{-a}^a f(x) \;dx = 2 \int\limits_0^a f(x) \;dx$

Using this property for our integral with $a = \frac{\pi}{4}$ and $f(x) = \sin^2 x$:

$I = 2 \int\limits_0^{\frac{π}{4}} \sin^2 x \;dx$

To integrate $\sin^2 x$, we use the trigonometric identity $\sin^2 x = \frac{1 - \cos(2x)}{2}$.

$I = 2 \int\limits_0^{\frac{π}{4}} \frac{1 - \cos(2x)}{2} \;dx$

$I = 2 \cdot \frac{1}{2} \int\limits_0^{\frac{π}{4}} (1 - \cos(2x)) \;dx$

$I = \int\limits_0^{\frac{π}{4}} (1 - \cos(2x)) \;dx$

Now, we integrate term by term:

$\int (1 - \cos(2x)) \;dx = \int 1 \;dx - \int \cos(2x) \;dx$

$\int 1 \;dx = x$

For $\int \cos(2x) \;dx$, we can use a simple substitution $u = 2x$, $du = 2 dx$, so $dx = \frac{1}{2} du$.

$\int \cos(2x) \;dx = \int \cos(u) \frac{1}{2} \;du = \frac{1}{2} \int \cos(u) \;du = \frac{1}{2} \sin(u) + C$

Substituting back $u = 2x$:

$\int \cos(2x) \;dx = \frac{1}{2} \sin(2x) + C$

So, the indefinite integral is $x - \frac{1}{2} \sin(2x)$.

Now, we evaluate the definite integral with the limits from 0 to $\frac{\pi}{4}$:

$I = \left[x - \frac{1}{2} \sin(2x)\right]_0^{\frac{π}{4}}$

Apply the limits of integration:

$I = \left(\frac{\pi}{4} - \frac{1}{2} \sin\left(2 \cdot \frac{\pi}{4}\right)\right) - \left(0 - \frac{1}{2} \sin(2 \cdot 0)\right)$

$I = \left(\frac{\pi}{4} - \frac{1}{2} \sin\left(\frac{\pi}{2}\right)\right) - \left(0 - \frac{1}{2} \sin(0)\right)$

Evaluate the sine values:

$\sin\left(\frac{\pi}{2}\right) = 1$

$\sin(0) = 0$

Substitute these values back:

$I = \left(\frac{\pi}{4} - \frac{1}{2} (1)\right) - \left(0 - \frac{1}{2} (0)\right)$

$I = \left(\frac{\pi}{4} - \frac{1}{2}\right) - (0 - 0)$

$I = \frac{\pi}{4} - \frac{1}{2}$

The value of the definite integral is $\mathbf{\frac{\pi}{4} - \frac{1}{2}}$.

Example 30: Evaluate $\int\limits_0^π \frac{x \sin x}{1 + \cos^2 x} \;dx$

Answer:

Given:

The definite integral $\int\limits_0^π \frac{x \sin x}{1 + \cos^2 x} \;dx$


To Evaluate:

The value of the given integral.


Solution:

Let the given integral be $I$.

$I = \int\limits_0^π \frac{x \sin x}{1 + \cos^2 x} \;dx$

We use the property of definite integrals:

$\int\limits_0^a f(x) \;dx = \int\limits_0^a f(a-x) \;dx$

Here, $a = \pi$ and $f(x) = \frac{x \sin x}{1 + \cos^2 x}$.

Let's find $f(\pi - x)$:

$f(\pi - x) = \frac{(\pi - x) \sin (\pi - x)}{1 + \cos^2 (\pi - x)}$

Using the trigonometric identities $\sin (\pi - x) = \sin x$ and $\cos (\pi - x) = -\cos x$, we have:

$\sin (\pi - x) = \sin x$

$\cos^2 (\pi - x) = (-\cos x)^2 = \cos^2 x$

So, $f(\pi - x) = \frac{(\pi - x) \sin x}{1 + \cos^2 x} = \frac{\pi \sin x - x \sin x}{1 + \cos^2 x}$.

Apply the property to the integral $I$:

$I = \int\limits_0^π \frac{(\pi - x) \sin x}{1 + \cos^2 x} \;dx$

$I = \int\limits_0^π \frac{\pi \sin x - x \sin x}{1 + \cos^2 x} \;dx$

$I = \int\limits_0^π \left( \frac{\pi \sin x}{1 + \cos^2 x} - \frac{x \sin x}{1 + \cos^2 x} \right) \;dx$

$I = \int\limits_0^π \frac{\pi \sin x}{1 + \cos^2 x} \;dx - \int\limits_0^π \frac{x \sin x}{1 + \cos^2 x} \;dx$

The second integral on the right-hand side is the original integral $I$.

$I = \int\limits_0^π \frac{\pi \sin x}{1 + \cos^2 x} \;dx - I$

Add $I$ to both sides:

$I + I = \int\limits_0^π \frac{\pi \sin x}{1 + \cos^2 x} \;dx$

$2I = \pi \int\limits_0^π \frac{\sin x}{1 + \cos^2 x} \;dx$

Now, let's evaluate the integral on the right side: $\int\limits_0^π \frac{\sin x}{1 + \cos^2 x} \;dx$.

We use the substitution method. Let $u = \cos x$.

Differentiating both sides with respect to $x$, we get $du = -\sin x \;dx$, so $\sin x \;dx = -du$.

Next, change the limits of integration. The original limits are for $x$. We find the corresponding limits for $u = \cos x$.

When $x = 0$, $u = \cos(0) = 1$.

When $x = \pi$, $u = \cos(\pi) = -1$.

Substitute $u$ and $du$ into the integral and change the limits:

$\int\limits_0^π \frac{\sin x}{1 + \cos^2 x} \;dx = \int\limits_1^{-1} \frac{-du}{1 + u^2}$

Using the property $\int_a^b f(x) dx = -\int_b^a f(x) dx$:

$\int\limits_1^{-1} \frac{-du}{1 + u^2} = - \int\limits_1^{-1} \frac{du}{1 + u^2} = \int\limits_{-1}^1 \frac{du}{1 + u^2}$

The integral of $\frac{1}{1 + u^2}$ is $\tan^{-1} u$.

$\int\limits_{-1}^1 \frac{du}{1 + u^2} = [\tan^{-1} u]_{-1}^1$

Evaluate the expression at the limits:

$[\tan^{-1} u]_{-1}^1 = \tan^{-1}(1) - \tan^{-1}(-1)$

$\tan^{-1}(1) = \frac{\pi}{4}$

$\tan^{-1}(-1) = -\frac{\pi}{4}$

So, $[\tan^{-1} u]_{-1}^1 = \frac{\pi}{4} - \left(-\frac{\pi}{4}\right) = \frac{\pi}{4} + \frac{\pi}{4} = \frac{2\pi}{4} = \frac{\pi}{2}$.

Substitute this value back into the equation for $2I$:

$2I = \pi \cdot \frac{\pi}{2}$

$2I = \frac{\pi^2}{2}$

Solve for $I$:

$I = \frac{1}{2} \cdot \frac{\pi^2}{2}$

$I = \frac{\pi^2}{4}$

The value of the definite integral is $\mathbf{\frac{\pi^2}{4}}$.

Example 31: Evaluate $\int\limits_{−1}^1 \sin^5 x \;\cos^4 x \;dx$

Answer:

Given:

The definite integral $\int\limits_{−1}^1 \sin^5 x \;\cos^4 x \;dx$


To Evaluate:

The value of the given integral.


Solution:

Let the given integral be $I$.

$I = \int\limits_{−1}^1 \sin^5 x \;\cos^4 x \;dx$

The interval of integration is $[-1, 1]$, which is symmetric about 0.

Let the integrand be $f(x) = \sin^5 x \;\cos^4 x$. We need to determine if the function is even or odd.

We evaluate $f(-x)$:

$f(-x) = \sin^5 (-x) \;\cos^4 (-x)$

Using the properties of sine and cosine functions: $\sin(-x) = -\sin x$ and $\cos(-x) = \cos x$.

$\sin^5 (-x) = (\sin(-x))^5 = (-\sin x)^5 = (-1)^5 \sin^5 x = -\sin^5 x$

$\cos^4 (-x) = (\cos(-x))^4 = (\cos x)^4 = \cos^4 x$

Substitute these back into $f(-x)$:

$f(-x) = (-\sin^5 x) (\cos^4 x)$

$f(-x) = -\sin^5 x \;\cos^4 x$

So, $f(-x) = -f(x)$, which means $f(x) = \sin^5 x \;\cos^4 x$ is an odd function.

For an odd function $f(x)$ over a symmetric interval $[-a, a]$, we have the property:

$\int\limits_{-a}^a f(x) \;dx = 0$

In our case, the interval is $[-1, 1]$, which is symmetric about 0, and the integrand $f(x) = \sin^5 x \;\cos^4 x$ is an odd function.

Therefore, the value of the integral is 0.

$I = 0$

The value of the definite integral is $\mathbf{0}$.

Example 32: Evaluate $\int\limits_0^{\frac{π}{2}} \frac{\sin^4 x}{\sin^4 x + \cos^4 x} \;dx$

Answer:

Given:

The definite integral $\int\limits_0^{\frac{π}{2}} \frac{\sin^4 x}{\sin^4 x + \cos^4 x} \;dx$


To Evaluate:

The value of the given integral.


Solution:

Let the given integral be $I$.

$I = \int\limits_0^{\frac{π}{2}} \frac{\sin^4 x}{\sin^4 x + \cos^4 x} \;dx$

$\mathbf{I = \int\limits_0^{\frac{π}{2}} \frac{\sin^4 x}{\sin^4 x + \cos^4 x} \;dx}$

... (i)

We use the property of definite integrals:

$\int\limits_0^a f(x) \;dx = \int\limits_0^a f(a-x) \;dx$

Here, $a = \frac{\pi}{2}$ and $f(x) = \frac{\sin^4 x}{\sin^4 x + \cos^4 x}$.

Let's apply the property to the integral $I$:

$I = \int\limits_0^{\frac{π}{2}} f\left(\frac{\pi}{2}-x\right) \;dx = \int\limits_0^{\frac{π}{2}} \frac{\sin^4 \left(\frac{\pi}{2}-x\right)}{\sin^4 \left(\frac{\pi}{2}-x\right) + \cos^4 \left(\frac{\pi}{2}-x\right)} \;dx$

Using the trigonometric identities $\sin(\frac{\pi}{2} - x) = \cos x$ and $\cos(\frac{\pi}{2} - x) = \sin x$, we have:

$I = \int\limits_0^{\frac{π}{2}} \frac{\cos^4 x}{\cos^4 x + \sin^4 x} \;dx$

$\mathbf{I = \int\limits_0^{\frac{π}{2}} \frac{\cos^4 x}{\sin^4 x + \cos^4 x} \;dx}$

... (ii)

Now, add equation (i) and equation (ii):

$I + I = \int\limits_0^{\frac{π}{2}} \frac{\sin^4 x}{\sin^4 x + \cos^4 x} \;dx + \int\limits_0^{\frac{π}{2}} \frac{\cos^4 x}{\sin^4 x + \cos^4 x} \;dx$

$2I = \int\limits_0^{\frac{π}{2}} \left( \frac{\sin^4 x}{ \sin^4 x + \cos^4 x} + \frac{\cos^4 x}{\sin^4 x + \cos^4 x} \right) \;dx$

$2I = \int\limits_0^{\frac{π}{2}} \frac{\sin^4 x + \cos^4 x}{\sin^4 x + \cos^4 x} \;dx$

$2I = \int\limits_0^{\frac{π}{2}} 1 \;dx$

Now, evaluate the integral of 1 with respect to $x$:

$2I = [x]_0^{\frac{π}{2}}$

Apply the limits of integration:

$2I = \frac{\pi}{2} - 0$

$2I = \frac{\pi}{2}$

Solve for $I$:

$I = \frac{1}{2} \cdot \frac{\pi}{2}$

$I = \frac{\pi}{4}$

The value of the integral is $\mathbf{\frac{\pi}{4}}$.

Example 33: Evaluate $\int\limits_{\frac{π}{6}}^{\frac{π}{3}} \frac{dx}{1 + \sqrt{\tan x}} $

Answer:

Given:

The definite integral $\int\limits_{\frac{π}{6}}^{\frac{π}{3}} \frac{dx}{1 + \sqrt{\tan x}} $


To Evaluate:

The value of the given integral.


Solution:

Let the given integral be $I$.

$I = \int\limits_{\frac{π}{6}}^{\frac{π}{3}} \frac{dx}{1 + \sqrt{\tan x}} $

We can rewrite $\sqrt{\tan x}$ in terms of sine and cosine:

$\sqrt{\tan x} = \sqrt{\frac{\sin x}{\cos x}} = \frac{\sqrt{\sin x}}{\sqrt{\cos x}}$

$I = \int\limits_{\frac{π}{6}}^{\frac{π}{3}} \frac{dx}{1 + \frac{\sqrt{\sin x}}{\sqrt{\cos x}}} = \int\limits_{\frac{π}{6}}^{\frac{π}{3}} \frac{dx}{\frac{\sqrt{\cos x} + \sqrt{\sin x}}{\sqrt{\cos x}}}$

$I = \int\limits_{\frac{π}{6}}^{\frac{π}{3}} \frac{\sqrt{\cos x}}{\sqrt{\cos x} + \sqrt{\sin x}} \;dx$

$\mathbf{I = \int\limits_{\frac{π}{6}}^{\frac{π}{3}} \frac{\sqrt{\cos x}}{\sqrt{\sin x} + \sqrt{\cos x}} \;dx}$

... (i)

We use the property of definite integrals:

$\int\limits_a^b f(x) \;dx = \int\limits_a^b f(a+b-x) \;dx$

Here, $a = \frac{\pi}{6}$, $b = \frac{\pi}{3}$, and $a+b = \frac{\pi}{6} + \frac{\pi}{3} = \frac{\pi}{6} + \frac{2\pi}{6} = \frac{3\pi}{6} = \frac{\pi}{2}$.

Let's apply the property to the integral $I$. We substitute $x$ with $\frac{\pi}{2} - x$:

$\sin\left(\frac{\pi}{2} - x\right) = \cos x$

$\cos\left(\frac{\pi}{2} - x\right) = \sin x$

$I = \int\limits_{\frac{π}{6}}^{\frac{π}{3}} \frac{\sqrt{\cos\left(\frac{\pi}{2}-x\right)}}{\sqrt{\sin\left(\frac{\pi}{2}-x\right)} + \sqrt{\cos\left(\frac{\pi}{2}-x\right)}} \;dx$

$I = \int\limits_{\frac{π}{6}}^{\frac{π}{3}} \frac{\sqrt{\sin x}}{\sqrt{\cos x} + \sqrt{\sin x}} \;dx$

$\mathbf{I = \int\limits_{\frac{π}{6}}^{\frac{π}{3}} \frac{\sqrt{\sin x}}{\sqrt{\sin x} + \sqrt{\cos x}} \;dx}$

... (ii)

Now, add equation (i) and equation (ii):

$I + I = \int\limits_{\frac{π}{6}}^{\frac{π}{3}} \frac{\sqrt{\cos x}}{\sqrt{\sin x} + \sqrt{\cos x}} \;dx + \int\limits_{\frac{π}{6}}^{\frac{π}{3}} \frac{\sqrt{\sin x}}{\sqrt{\sin x} + \sqrt{\cos x}} \;dx$

$2I = \int\limits_{\frac{π}{6}}^{\frac{π}{3}} \left( \frac{\sqrt{\cos x}}{\sqrt{\sin x} + \sqrt{\cos x}} + \frac{\sqrt{\sin x}}{\sqrt{\sin x} + \sqrt{\cos x}} \right) \;dx$

$2I = \int\limits_{\frac{π}{6}}^{\frac{π}{3}} \frac{\sqrt{\cos x} + \sqrt{\sin x}}{\sqrt{\sin x} + \sqrt{\cos x}} \;dx$

$2I = \int\limits_{\frac{π}{6}}^{\frac{π}{3}} 1 \;dx$

Now, evaluate the integral of 1 with respect to $x$:

$2I = [x]_{\frac{π}{6}}^{\frac{π}{3}}$

Apply the limits of integration:

$2I = \frac{\pi}{3} - \frac{\pi}{6}$

$2I = \frac{2\pi}{6} - \frac{\pi}{6}$

$2I = \frac{\pi}{6}$

Solve for $I$:

$I = \frac{1}{2} \cdot \frac{\pi}{6}$

$I = \frac{\pi}{12}$

The value of the integral is $\mathbf{\frac{\pi}{12}}$.

Example 34: Evaluate $\int\limits_0^{\frac{π}{2}} \log \;\sin x \;dx$

Answer:

Given:

The definite integral $\int\limits_0^{\frac{π}{2}} \log \;\sin x \;dx$


To Evaluate:

The value of the given integral.


Solution:

Let the given integral be $I$.

$I = \int\limits_0^{\frac{π}{2}} \log \;\sin x \;dx$

$\mathbf{I = \int\limits_0^{\frac{π}{2}} \log (\sin x) \;dx}$

... (i)

We use the property of definite integrals:

$\int\limits_0^a f(x) \;dx = \int\limits_0^a f(a-x) \;dx$

Here, $a = \frac{\pi}{2}$ and $f(x) = \log (\sin x)$.

Let's apply the property to the integral $I$. We substitute $x$ with $\frac{\pi}{2} - x$:

$I = \int\limits_0^{\frac{π}{2}} \log \left(\sin \left(\frac{\pi}{2}-x\right)\right) \;dx$

Using the trigonometric identity $\sin(\frac{\pi}{2} - x) = \cos x$, we have:

$I = \int\limits_0^{\frac{π}{2}} \log (\cos x) \;dx$

$\mathbf{I = \int\limits_0^{\frac{π}{2}} \log (\cos x) \;dx}$

... (ii)

Now, add equation (i) and equation (ii):

$I + I = \int\limits_0^{\frac{π}{2}} \log (\sin x) \;dx + \int\limits_0^{\frac{π}{2}} \log (\cos x) \;dx$

$2I = \int\limits_0^{\frac{π}{2}} (\log (\sin x) + \log (\cos x)) \;dx$

Using the logarithm property $\log A + \log B = \log (A \cdot B)$:

$2I = \int\limits_0^{\frac{π}{2}} \log (\sin x \cos x) \;dx$

We can rewrite $\sin x \cos x$ using the identity $\sin(2x) = 2 \sin x \cos x$, so $\sin x \cos x = \frac{1}{2} \sin(2x)$.

$2I = \int\limits_0^{\frac{π}{2}} \log \left(\frac{1}{2} \sin(2x)\right) \;dx$

Using the logarithm property $\log \frac{A}{B} = \log A - \log B$:

$2I = \int\limits_0^{\frac{π}{2}} \left( \log \left(\frac{1}{2}\right) + \log (\sin(2x)) \right) \;dx$

$2I = \int\limits_0^{\frac{π}{2}} \log \left(\frac{1}{2}\right) \;dx + \int\limits_0^{\frac{π}{2}} \log (\sin(2x)) \;dx$

$2I = \log \left(\frac{1}{2}\right) \int\limits_0^{\frac{π}{2}} 1 \;dx + \int\limits_0^{\frac{π}{2}} \log (\sin(2x)) \;dx$

$2I = -\log 2 \cdot [x]_0^{\frac{π}{2}} + \int\limits_0^{\frac{π}{2}} \log (\sin(2x)) \;dx$

$2I = -\log 2 \cdot \left(\frac{\pi}{2} - 0\right) + \int\limits_0^{\frac{π}{2}} \log (\sin(2x)) \;dx$

$2I = -\frac{\pi}{2} \log 2 + \int\limits_0^{\frac{π}{2}} \log (\sin(2x)) \;dx$

Let's evaluate the integral $\int\limits_0^{\frac{π}{2}} \log (\sin(2x)) \;dx$. We use substitution.

Let $t = 2x$. Then $dt = 2 \;dx$, so $dx = \frac{1}{2} dt$.

Change the limits of integration:

When $x = 0$, $t = 2(0) = 0$.

When $x = \frac{\pi}{2}$, $t = 2\left(\frac{\pi}{2}\right) = \pi$.

The integral becomes:

$\int\limits_0^{\frac{π}{2}} \log (\sin(2x)) \;dx = \int\limits_0^{\pi} \log (\sin t) \frac{1}{2} \;dt = \frac{1}{2} \int\limits_0^{\pi} \log (\sin t) \;dt$

We use the property $\int\limits_0^{2a} f(x) \;dx = 2 \int\limits_0^a f(x) \;dx$ if $f(2a - x) = f(x)$.

Let $g(t) = \log (\sin t)$. Check $g(\pi - t) = \log (\sin(\pi - t)) = \log (\sin t) = g(t)$.

So, $\int\limits_0^{\pi} \log (\sin t) \;dt = 2 \int\limits_0^{\pi/2} \log (\sin t) \;dt$.

The integral $\int\limits_0^{\pi/2} \log (\sin t) \;dt$ is the same as the original integral $I$ (just with a different variable of integration, which doesn't change the value).

So, $\int\limits_0^{\pi} \log (\sin t) \;dt = 2 I$.

Therefore, $\int\limits_0^{\frac{π}{2}} \log (\sin(2x)) \;dx = \frac{1}{2} (2I) = I$.

Substitute this back into the equation for $2I$:

$2I = -\frac{\pi}{2} \log 2 + I$

Subtract $I$ from both sides:

$2I - I = -\frac{\pi}{2} \log 2$

$I = -\frac{\pi}{2} \log 2$

The value of the integral is $\mathbf{-\frac{\pi}{2} \log 2}$.


Note: The value of this integral is a known result in definite integrals.



Exercise 7.10

By using the properties of definite integrals, evaluate the integrals in Exercises 1 to 19.

Question 1. $\int\limits_0^{\frac{π}{2}} \cos^2 x \;dx$

Answer:

Solution:


Let the given integral be $I$.

$I = \int\limits_0^{\frac{π}{2}} \cos^2 x \;dx$


Using the property $\int\limits_0^a f(x) dx = \int\limits_0^a f(a-x) dx$, with $a = \frac{π}{2}$ and $f(x) = \cos^2 x$, we have:

$I = \int\limits_0^{\frac{π}{2}} \cos^2 \left(\frac{π}{2} - x\right) \;dx$


Since $\cos\left(\frac{π}{2} - x\right) = \sin x$, the integral becomes:

$I = \int\limits_0^{\frac{π}{2}} \sin^2 x \;dx$

... (i)


The original integral is:

$I = \int\limits_0^{\frac{π}{2}} \cos^2 x \;dx$

... (ii)


Adding (i) and (ii):

$I + I = \int\limits_0^{\frac{π}{2}} \sin^2 x \;dx + \int\limits_0^{\frac{π}{2}} \cos^2 x \;dx$

$2I = \int\limits_0^{\frac{π}{2}} (\sin^2 x + \cos^2 x) \;dx$


Using the identity $\sin^2 x + \cos^2 x = 1$:

$2I = \int\limits_0^{\frac{π}{2}} 1 \;dx$


Evaluating the integral:

$2I = [x]_0^{\frac{π}{2}}$

$2I = \frac{π}{2} - 0$

$2I = \frac{π}{2}$


Solving for $I$:

$I = \frac{π}{2} \times \frac{1}{2}$

$I = \frac{π}{4}$


Thus, the value of the integral is $\frac{π}{4}$.

Question 2. $\int\limits_0^{\frac{π}{2}} \frac{\sqrt{\sin x}}{\sqrt{\sin x} + \sqrt{\cos x}} \;dx$

Answer:

Solution:


Let the given integral be $I$.

$I = \int\limits_0^{\frac{π}{2}} \frac{\sqrt{\sin x}}{\sqrt{\sin x} + \sqrt{\cos x}} \;dx$


Using the property of definite integrals $\int\limits_0^a f(x) dx = \int\limits_0^a f(a-x) dx$, with $a = \frac{π}{2}$, we replace $x$ with $\frac{π}{2} - x$.

We know that $\sin\left(\frac{π}{2} - x\right) = \cos x$ and $\cos\left(\frac{π}{2} - x\right) = \sin x$.

Applying this property, the integral becomes:

$I = \int\limits_0^{\frac{π}{2}} \frac{\sqrt{\sin(\frac{π}{2} - x)}}{\sqrt{\sin(\frac{π}{2} - x)} + \sqrt{\cos(\frac{π}{2} - x)}} \;dx = \int\limits_0^{\frac{π}{2}} \frac{\sqrt{\cos x}}{\sqrt{\cos x} + \sqrt{\sin x}} \;dx$

... (i)


The original integral is:

$I = \int\limits_0^{\frac{π}{2}} \frac{\sqrt{\sin x}}{\sqrt{\sin x} + \sqrt{\cos x}} \;dx$

... (ii)


Adding equation (i) and equation (ii):

$I + I = \int\limits_0^{\frac{π}{2}} \frac{\sqrt{\cos x}}{\sqrt{\cos x} + \sqrt{\sin x}} \;dx + \int\limits_0^{\frac{π}{2}} \frac{\sqrt{\sin x}}{\sqrt{\sin x} + \sqrt{\cos x}} \;dx$

$2I = \int\limits_0^{\frac{π}{2}} \left( \frac{\sqrt{\cos x}}{\sqrt{\cos x} + \sqrt{\sin x}} + \frac{\sqrt{\sin x}}{\sqrt{\sin x} + \sqrt{\cos x}} \right) \;dx$

$2I = \int\limits_0^{\frac{π}{2}} \frac{\sqrt{\cos x} + \sqrt{\sin x}}{\sqrt{\cos x} + \sqrt{\sin x}} \;dx$


Simplifying the integrand:

$2I = \int\limits_0^{\frac{π}{2}} 1 \;dx$


Evaluating the integral:

$2I = [x]_0^{\frac{π}{2}}$

$2I = \frac{π}{2} - 0$

$2I = \frac{π}{2}$


Solving for $I$:

$I = \frac{π}{2} \times \frac{1}{2}$

$I = \frac{π}{4}$


The value of the integral is $\frac{π}{4}$.

Question 3. $\int\limits_0^{\frac{π}{2}} \frac{\sin^{\frac{3}{2}} x \;dx}{\sin^{\frac{3}{2}} x + \cos^{\frac{3}{2}} x}$

Answer:

Solution:


Let the given integral be $I$.

$I = \int\limits_0^{\frac{π}{2}} \frac{\sin^{\frac{3}{2}} x}{\sin^{\frac{3}{2}} x + \cos^{\frac{3}{2}} x} \;dx$


Using the property of definite integrals $\int\limits_0^a f(x) dx = \int\limits_0^a f(a-x) dx$, with $a = \frac{π}{2}$, we replace $x$ with $\frac{π}{2} - x$.

We know that $\sin\left(\frac{π}{2} - x\right) = \cos x$ and $\cos\left(\frac{π}{2} - x\right) = \sin x$.

Applying this property, the integral becomes:

$I = \int\limits_0^{\frac{π}{2}} \frac{\sin^{\frac{3}{2}}(\frac{π}{2} - x)}{\sin^{\frac{3}{2}}(\frac{π}{2} - x) + \cos^{\frac{3}{2}}(\frac{π}{2} - x)} \;dx = \int\limits_0^{\frac{π}{2}} \frac{\cos^{\frac{3}{2}} x}{\cos^{\frac{3}{2}} x + \sin^{\frac{3}{2}} x} \;dx$

... (i)


The original integral is:

$I = \int\limits_0^{\frac{π}{2}} \frac{\sin^{\frac{3}{2}} x}{\sin^{\frac{3}{2}} x + \cos^{\frac{3}{2}} x} \;dx$

... (ii)


Adding equation (i) and equation (ii):

$I + I = \int\limits_0^{\frac{π}{2}} \frac{\cos^{\frac{3}{2}} x}{\cos^{\frac{3}{2}} x + \sin^{\frac{3}{2}} x} \;dx + \int\limits_0^{\frac{π}{2}} \frac{\sin^{\frac{3}{2}} x}{\sin^{\frac{3}{2}} x + \cos^{\frac{3}{2}} x} \;dx$

$2I = \int\limits_0^{\frac{π}{2}} \left( \frac{\cos^{\frac{3}{2}} x}{\cos^{\frac{3}{2}} x + \sin^{\frac{3}{2}} x} + \frac{\sin^{\frac{3}{2}} x}{\sin^{\frac{3}{2}} x + \cos^{\frac{3}{2}} x} \right) \;dx$

$2I = \int\limits_0^{\frac{π}{2}} \frac{\cos^{\frac{3}{2}} x + \sin^{\frac{3}{2}} x}{\cos^{\frac{3}{2}} x + \sin^{\frac{3}{2}} x} \;dx$


Simplifying the integrand:

$2I = \int\limits_0^{\frac{π}{2}} 1 \;dx$


Evaluating the integral:

$2I = [x]_0^{\frac{π}{2}}$

$2I = \frac{π}{2} - 0$

$2I = \frac{π}{2}$


Solving for $I$:

$I = \frac{π}{2} \times \frac{1}{2}$

$I = \frac{π}{4}$


The value of the integral is $\frac{π}{4}$.

Question 4. $\int\limits_0^{\frac{π}{2}} \frac{\cos^5 x \;dx}{\sin^5 x + \cos^5 x} $

Answer:

Solution:


Let the given integral be $I$.

$I = \int\limits_0^{\frac{π}{2}} \frac{\cos^5 x}{\sin^5 x + \cos^5 x} \;dx$


Using the property of definite integrals $\int\limits_0^a f(x) dx = \int\limits_0^a f(a-x) dx$, with $a = \frac{π}{2}$, we replace $x$ with $\frac{π}{2} - x$.

We know that $\sin\left(\frac{π}{2} - x\right) = \cos x$ and $\cos\left(\frac{π}{2} - x\right) = \sin x$.

Applying this property, the integral becomes:

$I = \int\limits_0^{\frac{π}{2}} \frac{\cos^5(\frac{π}{2} - x)}{\sin^5(\frac{π}{2} - x) + \cos^5(\frac{π}{2} - x)} \;dx = \int\limits_0^{\frac{π}{2}} \frac{\sin^5 x}{\cos^5 x + \sin^5 x} \;dx$

... (i)


The original integral is:

$I = \int\limits_0^{\frac{π}{2}} \frac{\cos^5 x}{\sin^5 x + \cos^5 x} \;dx$

... (ii)


Adding equation (i) and equation (ii):

$I + I = \int\limits_0^{\frac{π}{2}} \frac{\sin^5 x}{\cos^5 x + \sin^5 x} \;dx + \int\limits_0^{\frac{π}{2}} \frac{\cos^5 x}{\sin^5 x + \cos^5 x} \;dx$

$2I = \int\limits_0^{\frac{π}{2}} \left( \frac{\sin^5 x}{\sin^5 x + \cos^5 x} + \frac{\cos^5 x}{\sin^5 x + \cos^5 x} \right) \;dx$

$2I = \int\limits_0^{\frac{π}{2}} \frac{\sin^5 x + \cos^5 x}{\sin^5 x + \cos^5 x} \;dx$


Simplifying the integrand:

$2I = \int\limits_0^{\frac{π}{2}} 1 \;dx$


Evaluating the integral:

$2I = [x]_0^{\frac{π}{2}}$

$2I = \frac{π}{2} - 0$

$2I = \frac{π}{2}$


Solving for $I$:

$I = \frac{π}{2} \times \frac{1}{2}$

$I = \frac{π}{4}$


The value of the integral is $\frac{π}{4}$.

Question 5. $\int\limits_{−5}^5 |x + 2| \;dx$

Answer:

Solution:


Let the given integral be $I$.

$I = \int\limits_{−5}^5 |x + 2| \;dx$


The integrand is $|x + 2|$. We need to analyze the absolute value function.

The expression $|x + 2|$ changes its definition at $x + 2 = 0$, which is $x = -2$.

The interval of integration is $[-5, 5]$. The point $x = -2$ lies within this interval.

The definition of $|x+2|$ is:

$|x+2| = x+2$ if $x+2 \geq 0$, i.e., $x \geq -2$.

$|x+2| = -(x+2) = -x-2$ if $x+2 < 0$, i.e., $x < -2$.


We can split the integral into two parts using the property $\int\limits_a^b f(x) dx = \int\limits_a^c f(x) dx + \int\limits_c^b f(x) dx$, where $a < c < b$. Here $a=-5$, $b=5$, and $c=-2$.

$I = \int\limits_{−5}^{-2} |x + 2| \;dx + \int\limits_{-2}^5 |x + 2| \;dx$


Substitute the appropriate definition of $|x+2|$ in each integral:

$I = \int\limits_{−5}^{-2} (-x - 2) \;dx + \int\limits_{-2}^5 (x + 2) \;dx$

... (i)


Evaluate the first integral:

$\int\limits_{−5}^{-2} (-x - 2) \;dx = \left[ -\frac{x^2}{2} - 2x \right]_{-5}^{-2}$

$= \left( -\frac{(-2)^2}{2} - 2(-2) \right) - \left( -\frac{(-5)^2}{2} - 2(-5) \right)$

$= \left( -\frac{4}{2} + 4 \right) - \left( -\frac{25}{2} + 10 \right)$

$= (-2 + 4) - \left( -\frac{25}{2} + \frac{20}{2} \right)$

$= 2 - \left( -\frac{5}{2} \right) = 2 + \frac{5}{2} = \frac{4}{2} + \frac{5}{2} = \frac{9}{2}$


Evaluate the second integral:

$\int\limits_{-2}^5 (x + 2) \;dx = \left[ \frac{x^2}{2} + 2x \right]_{-2}^5$

$= \left( \frac{5^2}{2} + 2(5) \right) - \left( \frac{(-2)^2}{2} + 2(-2) \right)$

$= \left( \frac{25}{2} + 10 \right) - \left( \frac{4}{2} - 4 \right)$

$= \left( \frac{25}{2} + \frac{20}{2} \right) - (2 - 4)$

$= \frac{45}{2} - (-2) = \frac{45}{2} + 2 = \frac{45}{2} + \frac{4}{2} = \frac{49}{2}$


Substitute these values back into equation (i):

$I = \frac{9}{2} + \frac{49}{2}$

$I = \frac{9 + 49}{2}$

$I = \frac{58}{2}$

$I = 29$


The value of the integral is $29$.

Question 6. $\int\limits_2^8 |x − 5| \;dx$

Answer:

Solution:


Let the given integral be $I$.

$I = \int\limits_2^8 |x - 5| \;dx$


The integrand is $|x - 5|$. The expression inside the absolute value, $x - 5$, is zero when $x = 5$. This point lies within the interval of integration $[2, 8]$.

The definition of the absolute value function $|x - 5|$ is:

$|x - 5| = x - 5$, if $x - 5 \geq 0$, i.e., $x \geq 5$.

$|x - 5| = -(x - 5) = -x + 5$, if $x - 5 < 0$, i.e., $x < 5$.


We split the integral into two parts at $x = 5$ using the property $\int\limits_a^b f(x) dx = \int\limits_a^c f(x) dx + \int\limits_c^b f(x) dx$, where $a=2$, $b=8$, and $c=5$.

$I = \int\limits_2^5 |x - 5| \;dx + \int\limits_5^8 |x - 5| \;dx$

... (i)


For the interval $[2, 5)$, $x < 5$, so $|x - 5| = -x + 5$.

For the interval $[5, 8]$, $x \geq 5$, so $|x - 5| = x - 5$.


Substitute these into equation (i):

$I = \int\limits_2^5 (-x + 5) \;dx + \int\limits_5^8 (x - 5) \;dx$


Evaluate the first integral:

$\int\limits_2^5 (-x + 5) \;dx = \left[ -\frac{x^2}{2} + 5x \right]_2^5$

$= \left( -\frac{5^2}{2} + 5(5) \right) - \left( -\frac{2^2}{2} + 5(2) \right)$

$= \left( -\frac{25}{2} + 25 \right) - \left( -\frac{4}{2} + 10 \right)$

$= \left( -\frac{25}{2} + \frac{50}{2} \right) - (-2 + 10)$

$= \frac{25}{2} - 8 = \frac{25}{2} - \frac{16}{2} = \frac{9}{2}$


Evaluate the second integral:

$\int\limits_5^8 (x - 5) \;dx = \left[ \frac{x^2}{2} - 5x \right]_5^8$

$= \left( \frac{8^2}{2} - 5(8) \right) - \left( \frac{5^2}{2} - 5(5) \right)$

$= \left( \frac{64}{2} - 40 \right) - \left( \frac{25}{2} - 25 \right)$

$= (32 - 40) - \left( \frac{25}{2} - \frac{50}{2} \right)$

$= -8 - \left( -\frac{25}{2} \right) = -8 + \frac{25}{2} = -\frac{16}{2} + \frac{25}{2} = \frac{9}{2}$


Add the results of the two integrals:

$I = \frac{9}{2} + \frac{9}{2}$

$I = \frac{18}{2}$

$I = 9$


The value of the integral is $9$.

Question 7. $\int\limits_0^1 x (1 − x)^n \;dx$

Answer:

Solution:


Let the given integral be $I$.

$I = \int\limits_0^1 x (1 - x)^n \;dx$


Using the property of definite integrals $\int\limits_0^a f(x) dx = \int\limits_0^a f(a-x) dx$, with $a = 1$ and $f(x) = x(1-x)^n$, we replace $x$ with $1 - x$.

$f(1-x) = (1-x)(1 - (1-x))^n = (1-x)(1 - 1 + x)^n = (1-x)x^n$

Applying this property, the integral becomes:

$I = \int\limits_0^1 (1 - x) x^n \;dx$


Expand the integrand:

$(1 - x) x^n = x^n - x \cdot x^n = x^n - x^{n+1}$


So, the integral is:

$I = \int\limits_0^1 (x^n - x^{n+1}) \;dx$


Evaluate the integral:

$I = \left[ \frac{x^{n+1}}{n+1} - \frac{x^{n+2}}{n+2} \right]_0^1$


Apply the limits of integration:

$I = \left( \frac{1^{n+1}}{n+1} - \frac{1^{n+2}}{n+2} \right) - \left( \frac{0^{n+1}}{n+1} - \frac{0^{n+2}}{n+2} \right)$

$I = \left( \frac{1}{n+1} - \frac{1}{n+2} \right) - (0 - 0)$

$I = \frac{1}{n+1} - \frac{1}{n+2}$


Combine the fractions:

$I = \frac{(n+2) - (n+1)}{(n+1)(n+2)}$

$I = \frac{n + 2 - n - 1}{(n+1)(n+2)}$

$I = \frac{1}{(n+1)(n+2)}$


The value of the integral is $\frac{1}{(n+1)(n+2)}$.

Question 8. $\int\limits_0^{\frac{π}{4}} \log (1 + \tan x) \;dx$

Answer:

Solution:


Let the given integral be $I$.

$I = \int\limits_0^{\frac{π}{4}} \log (1 + \tan x) \;dx$

... (i)


Using the property of definite integrals $\int\limits_0^a f(x) dx = \int\limits_0^a f(a-x) dx$, with $a = \frac{π}{4}$, we replace $x$ with $\frac{π}{4} - x$.

Applying this property to (i):

$I = \int\limits_0^{\frac{π}{4}} \log \left(1 + \tan\left(\frac{π}{4} - x\right)\right) \;dx$


Using the tangent subtraction formula $\tan(A - B) = \frac{\tan A - \tan B}{1 + \tan A \tan B}$ with $A = \frac{π}{4}$ and $B = x$:

$\tan\left(\frac{π}{4} - x\right) = \frac{\tan \frac{π}{4} - \tan x}{1 + \tan \frac{π}{4} \tan x} = \frac{1 - \tan x}{1 + 1 \cdot \tan x} = \frac{1 - \tan x}{1 + \tan x}$


Substitute this back into the expression for $I$:

$I = \int\limits_0^{\frac{π}{4}} \log \left(1 + \frac{1 - \tan x}{1 + \tan x}\right) \;dx$


Simplify the expression inside the logarithm:

$1 + \frac{1 - \tan x}{1 + \tan x} = \frac{(1 + \tan x) + (1 - \tan x)}{1 + \tan x} = \frac{1 + \tan x + 1 - \tan x}{1 + \tan x} = \frac{2}{1 + \tan x}$


So, the integral becomes:

$I = \int\limits_0^{\frac{π}{4}} \log \left(\frac{2}{1 + \tan x}\right) \;dx$


Using the logarithm property $\log \left(\frac{A}{B}\right) = \log A - \log B$:

$I = \int\limits_0^{\frac{π}{4}} (\log 2 - \log (1 + \tan x)) \;dx$


Split the integral:

$I = \int\limits_0^{\frac{π}{4}} \log 2 \;dx - \int\limits_0^{\frac{π}{4}} \log (1 + \tan x) \;dx$


Notice that the second integral on the right side is the original integral $I$ (from equation (i)).

$I = \int\limits_0^{\frac{π}{4}} \log 2 \;dx - I$


Add $I$ to both sides of the equation:

$I + I = \int\limits_0^{\frac{π}{4}} \log 2 \;dx$

$2I = \int\limits_0^{\frac{π}{4}} \log 2 \;dx$


Since $\log 2$ is a constant with respect to $x$, we can take it outside the integral:

$2I = \log 2 \int\limits_0^{\frac{π}{4}} 1 \;dx$


Evaluate the integral of $1$:

$2I = \log 2 [x]_0^{\frac{π}{4}}$

$2I = \log 2 \left(\frac{π}{4} - 0\right)$

$2I = \frac{π}{4} \log 2$


Solve for $I$:

$I = \frac{1}{2} \cdot \frac{π}{4} \log 2$

$I = \frac{π}{8} \log 2$


The value of the integral is $\frac{π}{8} \log 2$.

Question 9. $\int\limits_0^2 x \sqrt{2 − x} \;dx$

Answer:

Solution:


Let the given integral be $I$.

$I = \int\limits_0^2 x \sqrt{2 - x} \;dx$


Using the property of definite integrals $\int\limits_0^a f(x) dx = \int\limits_0^a f(a-x) dx$, with $a = 2$ and $f(x) = x \sqrt{2-x}$, we replace $x$ with $2 - x$.

$f(2-x) = (2-x) \sqrt{2 - (2-x)} = (2-x) \sqrt{2 - 2 + x} = (2-x) \sqrt{x}$


Applying this property, the integral becomes:

$I = \int\limits_0^2 (2 - x) \sqrt{x} \;dx$


Expand the integrand by multiplying $(2-x)$ by $\sqrt{x} = x^{1/2}$:

$(2 - x) \sqrt{x} = 2 \sqrt{x} - x \sqrt{x} = 2x^{1/2} - x^{1} \cdot x^{1/2} = 2x^{1/2} - x^{1 + \frac{1}{2}} = 2x^{1/2} - x^{3/2}$


So, the integral is:

$I = \int\limits_0^2 (2x^{1/2} - x^{3/2}) \;dx$


Evaluate the integral using the power rule for integration, $\int x^n dx = \frac{x^{n+1}}{n+1}$:

$I = \left[ 2 \cdot \frac{x^{\frac{1}{2} + 1}}{\frac{1}{2} + 1} - \frac{x^{\frac{3}{2} + 1}}{\frac{3}{2} + 1} \right]_0^2$

$I = \left[ 2 \cdot \frac{x^{\frac{3}{2}}}{\frac{3}{2}} - \frac{x^{\frac{5}{2}}}{\frac{5}{2}} \right]_0^2$

$I = \left[ 2 \cdot \frac{2}{3} x^{3/2} - \frac{2}{5} x^{5/2} \right]_0^2$

$I = \left[ \frac{4}{3} x^{3/2} - \frac{2}{5} x^{5/2} \right]_0^2$


Apply the limits of integration $[0, 2]$:

$I = \left( \frac{4}{3} (2)^{3/2} - \frac{2}{5} (2)^{5/2} \right) - \left( \frac{4}{3} (0)^{3/2} - \frac{2}{5} (0)^{5/2} \right)$

$I = \left( \frac{4}{3} (2\sqrt{2}) - \frac{2}{5} (4\sqrt{2}) \right) - (0 - 0)$

$I = \frac{8\sqrt{2}}{3} - \frac{8\sqrt{2}}{5}$


Combine the terms by finding a common denominator, which is 15:

$I = \frac{8\sqrt{2} \cdot 5}{3 \cdot 5} - \frac{8\sqrt{2} \cdot 3}{5 \cdot 3}$

$I = \frac{40\sqrt{2}}{15} - \frac{24\sqrt{2}}{15}$

$I = \frac{40\sqrt{2} - 24\sqrt{2}}{15}$

$I = \frac{16\sqrt{2}}{15}$


The value of the integral is $\frac{16\sqrt{2}}{15}$.


Alternate Solution using Substitution:

Let $u = 2 - x$. Then $du = -dx$. Also, $x = 2 - u$.

When $x = 0$, $u = 2 - 0 = 2$.

When $x = 2$, $u = 2 - 2 = 0$.

The integral becomes:

$I = \int\limits_2^0 (2-u) \sqrt{u} (-du)$

Using the property $\int\limits_a^b f(x) dx = - \int\limits_b^a f(x) dx$, we can reverse the limits and change the sign:

$I = - \int\limits_2^0 (2-u) \sqrt{u} du = \int\limits_0^2 (2-u) \sqrt{u} du$

This is the same integral form as obtained using the property $\int\limits_0^a f(x) dx = \int\limits_0^a f(a-x) dx$. The evaluation proceeds exactly as above, replacing $x$ with $u$.

$I = \int\limits_0^2 (2u^{1/2} - u^{3/2}) du = \left[ \frac{4}{3} u^{3/2} - \frac{2}{5} u^{5/2} \right]_0^2$

$I = \left( \frac{4}{3} (2)^{3/2} - \frac{2}{5} (2)^{5/2} \right) - \left( \frac{4}{3} (0)^{3/2} - \frac{2}{5} (0)^{5/2} \right)$

$I = \frac{16\sqrt{2}}{15}$


Both methods yield the same result, $\frac{16\sqrt{2}}{15}$.

Question 10. $\int\limits_0^{\frac{π}{2}} (2 \log \sin x − \log \sin 2x) \;dx$

Answer:

Solution:


Let the given integral be $I$.

$I = \int\limits_0^{\frac{π}{2}} (2 \log \sin x - \log \sin 2x) \;dx$

... (i)


Using logarithm properties, we can simplify the integrand:

$2 \log \sin x - \log \sin 2x = \log (\sin^2 x) - \log (\sin 2x)$

$= \log \left(\frac{\sin^2 x}{\sin 2x}\right)$


Using the double angle identity $\sin 2x = 2 \sin x \cos x$:

$\frac{\sin^2 x}{\sin 2x} = \frac{\sin^2 x}{2 \sin x \cos x} = \frac{\sin x}{2 \cos x} = \frac{1}{2} \tan x$


Substituting this back into the integral:

$I = \int\limits_0^{\frac{π}{2}} \log \left(\frac{1}{2} \tan x\right) \;dx$


Using the logarithm property $\log(AB) = \log A + \log B$ (or $\log(A/B) = \log A - \log B$):

$\log \left(\frac{1}{2} \tan x\right) = \log \left(\frac{1}{2}\right) + \log (\tan x) = -\log 2 + \log (\tan x)$


So the integral becomes:

$I = \int\limits_0^{\frac{π}{2}} (-\log 2 + \log (\tan x)) \;dx$

$I = \int\limits_0^{\frac{π}{2}} -\log 2 \;dx + \int\limits_0^{\frac{π}{2}} \log (\tan x) \;dx$


Let's evaluate the first part:

$\int\limits_0^{\frac{π}{2}} -\log 2 \;dx = -\log 2 \int\limits_0^{\frac{π}{2}} 1 \;dx$

$= -\log 2 [x]_0^{\frac{π}{2}}$

$= -\log 2 \left(\frac{π}{2} - 0\right)$

$= -\frac{π}{2} \log 2$


Now consider the second part, let $I_1 = \int\limits_0^{\frac{π}{2}} \log (\tan x) \;dx$.

$I_1 = \int\limits_0^{\frac{π}{2}} \log (\tan x) \;dx$

... (ii)


Using the property $\int\limits_0^a f(x) dx = \int\limits_0^a f(a-x) dx$, with $a = \frac{π}{2}$:

$I_1 = \int\limits_0^{\frac{π}{2}} \log \left(\tan\left(\frac{π}{2} - x\right)\right) \;dx$


Since $\tan\left(\frac{π}{2} - x\right) = \cot x$:

$I_1 = \int\limits_0^{\frac{π}{2}} \log (\cot x) \;dx$

... (iii)


Adding equations (ii) and (iii):

$I_1 + I_1 = \int\limits_0^{\frac{π}{2}} \log (\tan x) \;dx + \int\limits_0^{\frac{π}{2}} \log (\cot x) \;dx$

$2I_1 = \int\limits_0^{\frac{π}{2}} (\log (\tan x) + \log (\cot x)) \;dx$


Using the logarithm property $\log A + \log B = \log(AB)$ and $\cot x = \frac{1}{\tan x}$:

$\log (\tan x) + \log (\cot x) = \log (\tan x \cdot \cot x) = \log \left(\tan x \cdot \frac{1}{\tan x}\right) = \log 1$

Since $\log 1 = 0$:

$2I_1 = \int\limits_0^{\frac{π}{2}} 0 \;dx$

$2I_1 = 0$

$I_1 = 0$


Substitute the value of $I_1$ back into the expression for $I$:

$I = -\frac{π}{2} \log 2 + I_1$

$I = -\frac{π}{2} \log 2 + 0$

$I = -\frac{π}{2} \log 2$


The value of the integral is $-\frac{π}{2} \log 2$.

Question 11. $\int\limits_{\frac{−π}{2}}^{\frac{π}{2}} \sin^2 x \;dx$

Answer:

Solution:


Let the given integral be $I$. The limits of integration are from $-\frac{π}{2}$ to $\frac{π}{2}$, which are symmetric about $0$.

$I = \int\limits_{-\frac{π}{2}}^{\frac{π}{2}} \sin^2 x \;dx$


We check if the integrand $f(x) = \sin^2 x$ is an even or odd function.

Recall that a function $f(x)$ is even if $f(-x) = f(x)$ and odd if $f(-x) = -f(x)$.

Let's evaluate $f(-x)$:

$f(-x) = \sin^2 (-x) = (\sin(-x))^2$


Since $\sin(-x) = -\sin x$, we have:

$f(-x) = (-\sin x)^2 = \sin^2 x$


Thus, $f(-x) = f(x)$. This means that $\sin^2 x$ is an even function.


For an even function $f(x)$, we use the property of definite integrals: $\int\limits_{-a}^a f(x) dx = 2 \int\limits_0^a f(x) dx$.

Here, $a = \frac{π}{2}$ and $f(x) = \sin^2 x$. So, the integral becomes:

$I = 2 \int\limits_0^{\frac{π}{2}} \sin^2 x \;dx$

... (i)


To evaluate $\int\limits_0^{\frac{π}{2}} \sin^2 x \;dx$, we use the half-angle identity: $\sin^2 x = \frac{1 - \cos(2x)}{2}$.

$\int \sin^2 x \;dx = \int \frac{1 - \cos(2x)}{2} \;dx = \frac{1}{2} \int (1 - \cos(2x)) \;dx$

$= \frac{1}{2} \left( x - \frac{\sin(2x)}{2} \right) + C$


Now, substitute this result back into equation (i) and apply the limits $[0, \frac{π}{2}]$:

$I = 2 \left[ \frac{1}{2} \left( x - \frac{\sin(2x)}{2} \right) \right]_0^{\frac{π}{2}}$

$I = \left[ x - \frac{\sin(2x)}{2} \right]_0^{\frac{π}{2}}$


Evaluate the expression at the upper limit $\frac{π}{2}$ and the lower limit $0$:

$I = \left( \frac{π}{2} - \frac{\sin\left(2 \cdot \frac{π}{2}\right)}{2} \right) - \left( 0 - \frac{\sin(2 \cdot 0)}{2} \right)$

$I = \left( \frac{π}{2} - \frac{\sin(π)}{2} \right) - \left( 0 - \frac{\sin(0)}{2} \right)$


Since $\sin(π) = 0$ and $\sin(0) = 0$:

$I = \left( \frac{π}{2} - \frac{0}{2} \right) - \left( 0 - \frac{0}{2} \right)$

$I = \left( \frac{π}{2} - 0 \right) - (0 - 0)$

$I = \frac{π}{2} - 0$

$I = \frac{π}{2}$


The value of the integral is $\frac{π}{2}$.


Alternate Method:

We know from Question 1 that $\int\limits_0^{\frac{π}{2}} \cos^2 x \;dx = \frac{π}{4}$.

Also, we know that $\int\limits_0^{\frac{π}{2}} \sin^2 x \;dx + \int\limits_0^{\frac{π}{2}} \cos^2 x \;dx = \int\limits_0^{\frac{π}{2}} (\sin^2 x + \cos^2 x) \;dx = \int\limits_0^{\frac{π}{2}} 1 \;dx = [x]_0^{\frac{π}{2}} = \frac{π}{2}$.

So, $\int\limits_0^{\frac{π}{2}} \sin^2 x \;dx = \frac{π}{2} - \int\limits_0^{\frac{π}{2}} \cos^2 x \;dx = \frac{π}{2} - \frac{π}{4} = \frac{2π - π}{4} = \frac{π}{4}$.

Using the property for even functions: $I = 2 \int\limits_0^{\frac{π}{2}} \sin^2 x \;dx = 2 \times \frac{π}{4} = \frac{π}{2}$.


Both methods yield the same result, $\frac{π}{2}$.

Question 12. $\int\limits_0^π \frac{x \;dx}{1 + \sin x} $

Answer:

Solution:


Let the given integral be $I$.

$I = \int\limits_0^π \frac{x}{1 + \sin x} \;dx$

... (i)


Using the property of definite integrals $\int\limits_0^a f(x) dx = \int\limits_0^a f(a-x) dx$, with $a = π$, we replace $x$ with $π - x$ in the integral (i).

$I = \int\limits_0^π \frac{(π - x)}{1 + \sin(π - x)} \;dx$

... (ii)


Using the trigonometric identity $\sin(π - x) = \sin x$, equation (ii) becomes:

$I = \int\limits_0^π \frac{π - x}{1 + \sin x} \;dx$


Split the integrand into two terms:

$I = \int\limits_0^π \left(\frac{π}{1 + \sin x} - \frac{x}{1 + \sin x}\right) \;dx$


Separate the integral into two parts:

$I = \int\limits_0^π \frac{π}{1 + \sin x} \;dx - \int\limits_0^π \frac{x}{1 + \sin x} \;dx$


Notice that the second integral on the right side is the original integral $I$ (from equation (i)).

$I = π \int\limits_0^π \frac{1}{1 + \sin x} \;dx - I$


Add $I$ to both sides of the equation:

$2I = π \int\limits_0^π \frac{1}{1 + \sin x} \;dx$

... (iii)


Now, we need to evaluate the integral $J = \int\limits_0^π \frac{1}{1 + \sin x} \;dx$.

$J = \int\limits_0^π \frac{1}{1 + \sin x} \;dx$


We can use the substitution $t = \tan(x/2)$.

From this substitution, we have $dx = \frac{2 dt}{1+t^2}$ and $\sin x = \frac{2t}{1+t^2}$.

Change the limits of integration:

When $x = 0$, $t = \tan(0/2) = \tan 0 = 0$.

When $x = π$, $t = \tan(π/2)$, which approaches infinity. The integral is an improper integral at the upper limit.


The integral $J$ in terms of $t$ becomes:

$J = \int\limits_0^\infty \frac{1}{1 + \frac{2t}{1+t^2}} \cdot \frac{2 dt}{1+t^2}$


Simplify the integrand:

$\frac{1}{1 + \frac{2t}{1+t^2}} \cdot \frac{2}{1+t^2} = \frac{1+t^2}{1+t^2+2t} \cdot \frac{2}{1+t^2} = \frac{1}{(1+t)^2} \cdot 2 = \frac{2}{(1+t)^2}$


So, the integral $J$ is:

$J = \int\limits_0^\infty \frac{2}{(1+t)^2} \;dt$


Evaluate this improper integral:

$J = 2 \int\limits_0^\infty (1+t)^{-2} \;dt = 2 \left[ \frac{(1+t)^{-1}}{-1} \right]_0^\infty = 2 \left[ -\frac{1}{1+t} \right]_0^\infty$


Applying the limits of integration:

$J = 2 \left( \lim_{t \to \infty} \left(-\frac{1}{1+t}\right) - \left(-\frac{1}{1+0}\right) \right)$

$J = 2 \left( 0 - (-1) \right) = 2(1) = 2$


Now, substitute the value of $J = 2$ back into equation (iii):

$2I = π \cdot 2$


Divide by 2 to find $I$:

$I = π$


The value of the integral is $π$.

Question 13. $\int\limits_{\frac{−π}{2}}^{\frac{π}{2}} \sin^7 x \;dx$

Answer:

Solution:


Let the given integral be $I$. The limits of integration are from $-\frac{π}{2}$ to $\frac{π}{2}$. These limits are symmetric about $0$, i.e., of the form $[-a, a]$ where $a = \frac{π}{2}$.

$I = \int\limits_{-\frac{π}{2}}^{\frac{π}{2}} \sin^7 x \;dx$


We examine the integrand $f(x) = \sin^7 x$ to determine if it is an even or odd function.

A function $f(x)$ is even if $f(-x) = f(x)$, and odd if $f(-x) = -f(x)$.

Let's find $f(-x)$:

$f(-x) = \sin^7 (-x)$


Using the trigonometric identity $\sin(-x) = -\sin x$, we have:

$f(-x) = (-\sin x)^7$

$f(-x) = (-1)^7 (\sin x)^7$

$f(-x) = - \sin^7 x$


Since $f(-x) = -f(x)$, the function $f(x) = \sin^7 x$ is an odd function.


For an odd function $f(x)$ integrated over a symmetric interval $[-a, a]$, the property of definite integrals states that $\int\limits_{-a}^a f(x) dx = 0$.

Here, $a = \frac{π}{2}$ and the integrand $\sin^7 x$ is an odd function.


Applying this property:

$I = \int\limits_{-\frac{π}{2}}^{\frac{π}{2}} \sin^7 x \;dx = 0$


The value of the integral is $0$.

Question 14. $\int\limits_0^{2π} \cos^5 x \;dx$

Answer:

Solution:


Let the given integral be $I$.

$I = \int\limits_0^{2π} \cos^5 x \;dx$


We use the property of definite integrals: $\int\limits_0^{2a} f(x) dx = 2 \int\limits_0^a f(x) dx$ if $f(2a - x) = f(x)$, and $\int\limits_0^{2a} f(x) dx = 0$ if $f(2a - x) = -f(x)$.

In this integral, the upper limit is $2π$. So we can take $2a = 2π$, which means $a = π$.

Let $f(x) = \cos^5 x$. We need to check the value of $f(2π - x)$.

$f(2π - x) = \cos^5 (2π - x)$


Using the trigonometric identity $\cos(2π - x) = \cos x$, we have:

$f(2π - x) = (\cos x)^5 = \cos^5 x$


Since $f(2π - x) = f(x)$, according to the property $\int\limits_0^{2a} f(x) dx = 2 \int\limits_0^a f(x) dx$.

Applying this property to the given integral:

$I = 2 \int\limits_0^π \cos^5 x \;dx$

... (i)


Now consider the new integral $\int\limits_0^π \cos^5 x \;dx$. This integral is of the form $\int\limits_0^{2b} g(x) dx$, where $2b = π$, so $b = \frac{π}{2}$. Let $g(x) = \cos^5 x$.

We need to check the value of $g(π - x)$.

$g(π - x) = \cos^5 (π - x)$


Using the trigonometric identity $\cos(π - x) = -\cos x$, we have:

$g(π - x) = (-\cos x)^5 = (-1)^5 (\cos x)^5 = -\cos^5 x$


Since $g(π - x) = -g(x)$, according to the property $\int\limits_0^{2b} g(x) dx = 0$ if $g(2b - x) = -g(x)$.

Applying this property to the integral $\int\limits_0^π \cos^5 x \;dx$ (with $2b = π$):

$\int\limits_0^π \cos^5 x \;dx = 0$


Substitute this result back into equation (i):

$I = 2 \cdot 0$

$I = 0$


The value of the integral is $0$.

Question 15. $\int\limits_0^{\frac{π}{2}} \frac{\sin x − \cos x}{1 + \sin x \cos x} \;dx$

Answer:

Solution:


Let the given integral be $I$.

$I = \int\limits_0^{\frac{π}{2}} \frac{\sin x - \cos x}{1 + \sin x \cos x} \;dx$

... (i)


Using the property of definite integrals $\int\limits_0^a f(x) dx = \int\limits_0^a f(a-x) dx$, with $a = \frac{π}{2}$, we replace $x$ with $\frac{π}{2} - x$ in the integral (i).

Let $f(x) = \frac{\sin x - \cos x}{1 + \sin x \cos x}$.

We evaluate $f\left(\frac{π}{2} - x\right)$:

$f\left(\frac{π}{2} - x\right) = \frac{\sin\left(\frac{π}{2} - x\right) - \cos\left(\frac{π}{2} - x\right)}{1 + \sin\left(\frac{π}{2} - x\right) \cos\left(\frac{π}{2} - x\right)}$


Using the trigonometric identities $\sin\left(\frac{π}{2} - x\right) = \cos x$ and $\cos\left(\frac{π}{2} - x\right) = \sin x$:

$f\left(\frac{π}{2} - x\right) = \frac{\cos x - \sin x}{1 + \cos x \sin x}$

$= \frac{-(\sin x - \cos x)}{1 + \sin x \cos x}$

$= - \frac{\sin x - \cos x}{1 + \sin x \cos x}$

$= -f(x)$


Applying the property, the integral becomes:

$I = \int\limits_0^{\frac{π}{2}} f\left(\frac{π}{2} - x\right) \;dx = \int\limits_0^{\frac{π}{2}} -f(x) \;dx = \int\limits_0^{\frac{π}{2}} - \frac{\sin x - \cos x}{1 + \sin x \cos x} \;dx$

... (ii)


Adding equation (i) and equation (ii):

$I + I = \int\limits_0^{\frac{π}{2}} \frac{\sin x - \cos x}{1 + \sin x \cos x} \;dx + \int\limits_0^{\frac{π}{2}} - \frac{\sin x - \cos x}{1 + \sin x \cos x} \;dx$

$2I = \int\limits_0^{\frac{π}{2}} \left( \frac{\sin x - \cos x}{1 + \sin x \cos x} - \frac{\sin x - \cos x}{1 + \sin x \cos x} \right) \;dx$

$2I = \int\limits_0^{\frac{π}{2}} 0 \;dx$


Evaluating the integral of 0:

$2I = 0$


Solving for $I$:

$I = 0$


The value of the integral is $0$.

Question 16. $\int\limits_0^π \log (1 + \cos x) \;dx$

Answer:

Solution:


Let the given integral be $I$.

$I = \int\limits_0^π \log (1 + \cos x) \;dx$

... (i)


Using the property of definite integrals $\int\limits_0^a f(x) dx = \int\limits_0^a f(a-x) dx$, with $a = π$, we replace $x$ with $π - x$ in the integral (i).

We evaluate the integrand at $π-x$: $f(π - x) = \log(1 + \cos(π - x))$.


Using the trigonometric identity $\cos(π - x) = -\cos x$:

$f(π - x) = \log(1 - \cos x)$.


Applying the property, the integral becomes:

$I = \int\limits_0^π \log (1 - \cos x) \;dx$

... (ii)


Adding equation (i) and equation (ii):

$I + I = \int\limits_0^π \log (1 + \cos x) \;dx + \int\limits_0^π \log (1 - \cos x) \;dx$

$2I = \int\limits_0^π [\log (1 + \cos x) + \log (1 - \cos x)] \;dx$


Using the logarithm property $\log A + \log B = \log(AB)$:

$2I = \int\limits_0^π \log [(1 + \cos x)(1 - \cos x)] \;dx$

$2I = \int\limits_0^π \log (1 - \cos^2 x) \;dx$


Using the trigonometric identity $1 - \cos^2 x = \sin^2 x$:

$2I = \int\limits_0^π \log (\sin^2 x) \;dx$


Using the logarithm property $\log (A^n) = n \log A$:

$2I = \int\limits_0^π 2 \log (\sin x) \;dx$


Simplifying:

$I = \int\limits_0^π \log (\sin x) \;dx$

... (iii)


Now consider the integral $I = \int\limits_0^π \log (\sin x) \;dx$. This is of the form $\int\limits_0^{2a} f(x) dx$ with $2a = π$, so $a = π/2$. Let $f(x) = \log (\sin x)$.

We check $f(2a - x) = f(π - x) = \log(\sin(π - x))$.


Using the identity $\sin(π - x) = \sin x$:

$f(π - x) = \log(\sin x) = f(x)$.


Since $f(π - x) = f(x)$, we use the property $\int\limits_0^{2a} f(x) dx = 2 \int\limits_0^a f(x) dx$:

$I = 2 \int\limits_0^{\frac{π}{2}} \log (\sin x) \;dx$

... (iv)


Let $J = \int\limits_0^{\frac{π}{2}} \log (\sin x) \;dx$.

$J = \int\limits_0^{\frac{π}{2}} \log (\sin x) \;dx$

... (v)


Using the property $\int\limits_0^a f(x) dx = \int\limits_0^a f(a-x) dx$ on $J$, with $a = π/2$:

$J = \int\limits_0^{\frac{π}{2}} \log \left(\sin\left(\frac{π}{2} - x\right)\right) \;dx$


Using the identity $\sin\left(\frac{π}{2} - x\right) = \cos x$:

$J = \int\limits_0^{\frac{π}{2}} \log (\cos x) \;dx$

... (vi)


Adding equation (v) and equation (vi):

$J + J = \int\limits_0^{\frac{π}{2}} \log (\sin x) \;dx + \int\limits_0^{\frac{π}{2}} \log (\cos x) \;dx$

$2J = \int\limits_0^{\frac{π}{2}} [\log (\sin x) + \log (\cos x)] \;dx$


Using the logarithm property $\log A + \log B = \log(AB)$:

$2J = \int\limits_0^{\frac{π}{2}} \log (\sin x \cos x) \;dx$


Multiply and divide by 2 inside the logarithm: $\sin x \cos x = \frac{1}{2} (2 \sin x \cos x) = \frac{1}{2} \sin 2x$:

$2J = \int\limits_0^{\frac{π}{2}} \log \left(\frac{\sin 2x}{2}\right) \;dx$


Using the logarithm property $\log \left(\frac{A}{B}\right) = \log A - \log B$:

$2J = \int\limits_0^{\frac{π}{2}} (\log (\sin 2x) - \log 2) \;dx$


Splitting the integral:

$2J = \int\limits_0^{\frac{π}{2}} \log (\sin 2x) \;dx - \int\limits_0^{\frac{π}{2}} \log 2 \;dx$


Evaluate the second integral:

$\int\limits_0^{\frac{π}{2}} \log 2 \;dx = \log 2 [x]_0^{\frac{π}{2}} = \log 2 \left(\frac{π}{2} - 0\right) = \frac{π}{2} \log 2$


Consider the first integral $\int\limits_0^{\frac{π}{2}} \log (\sin 2x) \;dx$. Let $u = 2x$. Then $du = 2 dx$, so $dx = \frac{1}{2} du$.

Change the limits of integration:

When $x = 0$, $u = 2(0) = 0$.

When $x = \frac{π}{2}$, $u = 2\left(\frac{π}{2}\right) = π$.

The integral becomes:

$\int\limits_0^{\frac{π}{2}} \log (\sin 2x) \;dx = \int\limits_0^π \log (\sin u) \frac{1}{2} du = \frac{1}{2} \int\limits_0^π \log (\sin u) \;du$


From equation (iii), we have $I = \int\limits_0^π \log (\sin x) \;dx$. Replacing the dummy variable $x$ with $u$, we get $\int\limits_0^π \log (\sin u) \;du = I$.

So, $\int\limits_0^{\frac{π}{2}} \log (\sin 2x) \;dx = \frac{1}{2} I$.


Substitute these results back into the equation for $2J$:

$2J = \frac{1}{2} I - \frac{π}{2} \log 2$


From equation (iv), we have $I = 2J$. Substitute this into the equation for $2J$:

$I = \frac{1}{2} I - \frac{π}{2} \log 2$


Rearrange the terms to solve for $I$:

$I - \frac{1}{2} I = - \frac{π}{2} \log 2$

$\frac{1}{2} I = - \frac{π}{2} \log 2$


Multiply both sides by 2:

$I = -π \log 2$


The value of the integral is $-π \log 2$.

Question 17. $\int\limits_0^a \frac{\sqrt{x}}{\sqrt{x} + \sqrt{a − x}} \;dx$

Answer:

Solution:


Let the given integral be $I$.

$I = \int\limits_0^a \frac{\sqrt{x}}{\sqrt{x} + \sqrt{a − x}} \;dx$

... (i)


Using the property of definite integrals $\int\limits_0^a f(x) dx = \int\limits_0^a f(a-x) dx$, we replace $x$ with $a - x$ in the integral (i).

Let $f(x) = \frac{\sqrt{x}}{\sqrt{x} + \sqrt{a − x}}$.

We evaluate $f(a-x)$:

$f(a-x) = \frac{\sqrt{a-x}}{\sqrt{a-x} + \sqrt{a - (a-x)}} = \frac{\sqrt{a-x}}{\sqrt{a-x} + \sqrt{a - a + x}} = \frac{\sqrt{a-x}}{\sqrt{a-x} + \sqrt{x}}$


Applying the property, the integral becomes:

$I = \int\limits_0^a \frac{\sqrt{a-x}}{\sqrt{a-x} + \sqrt{x}} \;dx$

... (ii)


Adding equation (i) and equation (ii):

$I + I = \int\limits_0^a \frac{\sqrt{x}}{\sqrt{x} + \sqrt{a − x}} \;dx + \int\limits_0^a \frac{\sqrt{a-x}}{\sqrt{a-x} + \sqrt{x}} \;dx$

$2I = \int\limits_0^a \left( \frac{\sqrt{x}}{\sqrt{x} + \sqrt{a − x}} + \frac{\sqrt{a-x}}{\sqrt{a-x} + \sqrt{x}} \right) \;dx$


Since the denominators are the same, we combine the numerators:

$2I = \int\limits_0^a \frac{\sqrt{x} + \sqrt{a-x}}{\sqrt{x} + \sqrt{a-x}} \;dx$


The integrand simplifies to 1:

$2I = \int\limits_0^a 1 \;dx$


Evaluate the integral:

$2I = [x]_0^a$

$2I = a - 0$

$2I = a$


Solve for $I$:

$I = \frac{a}{2}$


The value of the integral is $\frac{a}{2}$.

Question 18. $\int\limits_0^4 |x − 1| \;dx$

Answer:

Solution:


Let the given integral be $I$.

$I = \int\limits_0^4 |x - 1| \;dx$


The integrand is $|x - 1|$. The expression inside the absolute value, $x - 1$, is zero when $x = 1$. This point lies within the interval of integration $[0, 4]$.

The definition of the absolute value function $|x - 1|$ is:

$|x - 1| = x - 1$, if $x - 1 \geq 0$, i.e., $x \geq 1$.

$|x - 1| = -(x - 1) = 1 - x$, if $x - 1 < 0$, i.e., $x < 1$.


We split the integral into two parts at $x = 1$ using the property $\int\limits_a^b f(x) dx = \int\limits_a^c f(x) dx + \int\limits_c^b f(x) dx$, where $a=0$, $b=4$, and $c=1$.

$I = \int\limits_0^1 |x - 1| \;dx + \int\limits_1^4 |x - 1| \;dx$

... (i)


For the interval $[0, 1)$, $x < 1$, so $|x - 1| = 1 - x$.

For the interval $[1, 4]$, $x \geq 1$, so $|x - 1| = x - 1$.


Substitute these into equation (i):

$I = \int\limits_0^1 (1 - x) \;dx + \int\limits_1^4 (x - 1) \;dx$


Evaluate the first integral:

$\int\limits_0^1 (1 - x) \;dx = \left[ x - \frac{x^2}{2} \right]_0^1$

$= \left( 1 - \frac{1^2}{2} \right) - \left( 0 - \frac{0^2}{2} \right)$

$= \left( 1 - \frac{1}{2} \right) - (0 - 0)$

$= \frac{1}{2} - 0 = \frac{1}{2}$


Evaluate the second integral:

$\int\limits_1^4 (x - 1) \;dx = \left[ \frac{x^2}{2} - x \right]_1^4$

$= \left( \frac{4^2}{2} - 4 \right) - \left( \frac{1^2}{2} - 1 \right)$

$= \left( \frac{16}{2} - 4 \right) - \left( \frac{1}{2} - 1 \right)$

$= (8 - 4) - \left( \frac{1}{2} - \frac{2}{2} \right)$

$= 4 - \left( -\frac{1}{2} \right) = 4 + \frac{1}{2} = \frac{8}{2} + \frac{1}{2} = \frac{9}{2}$


Add the results of the two integrals:

$I = \frac{1}{2} + \frac{9}{2}$

$I = \frac{1 + 9}{2}$

$I = \frac{10}{2}$

$I = 5$


The value of the integral is $5$.

Question 19. Show that $\int\limits_0^a f(x) g (x) \;dx = 2 \int\limits_0^a f(x) \;dx$, if f and g are defined as f(x) = f(a – x) and g(x) + g(a – x) = 4

Answer:

Solution:


Let the given integral be $I$.

$I = \int\limits_0^a f(x) g(x) \;dx$

... (i)


We are given the properties:

$f(x) = f(a - x)$

(Given)

$g(x) + g(a - x) = 4$

(Given)


From the second property, we can write:

$g(a - x) = 4 - g(x)$


Using the property of definite integrals $\int\limits_0^a h(x) dx = \int\limits_0^a h(a-x) dx$, we apply it to the integral $I$. Let $h(x) = f(x) g(x)$.

Then $h(a-x) = f(a-x) g(a-x)$.


Using the given properties $f(a-x) = f(x)$ and $g(a-x) = 4 - g(x)$, we substitute these into the expression for $h(a-x)$:

$h(a-x) = f(x) (4 - g(x)) = 4f(x) - f(x)g(x)$


Applying the integral property:

$I = \int\limits_0^a h(a - x) \;dx = \int\limits_0^a (4f(x) - f(x)g(x)) \;dx$

... (ii)


Split the integral on the right side of equation (ii):

$I = \int\limits_0^a 4f(x) \;dx - \int\limits_0^a f(x)g(x) \;dx$


Take the constant 4 out of the first integral:

$I = 4 \int\limits_0^a f(x) \;dx - \int\limits_0^a f(x)g(x) \;dx$


Notice that the second integral on the right side is the original integral $I$ (from equation (i)). Substitute $I$ back into the equation:

$I = 4 \int\limits_0^a f(x) \;dx - I$


Add $I$ to both sides of the equation:

$I + I = 4 \int\limits_0^a f(x) \;dx$

$2I = 4 \int\limits_0^a f(x) \;dx$


Divide both sides by 2:

$I = 2 \int\limits_0^a f(x) \;dx$


Substituting the original expression for $I$ back into the equation, we get:

$\int\limits_0^a f(x) g(x) \;dx = 2 \int\limits_0^a f(x) \;dx$


This is the required result that needs to be shown.

Choose the correct answer in Exercises 20 and 21.

Question 20. The value of $\int\limits_{\frac{−π}{2}}^{\frac{π}{2}} (x^3 + x \cos x + \tan^5 x + 1) \;dx$ is

(A) 0

(B) 2

(C) π

(D) 1

Answer:

Solution:


Let the given integral be $I$. The limits of integration are from $-\frac{π}{2}$ to $\frac{π}{2}$, which is an interval of the form $[-a, a]$ where $a = \frac{π}{2}$.

$I = \int\limits_{-\frac{π}{2}}^{\frac{π}{2}} (x^3 + x \cos x + \tan^5 x + 1) \;dx$


We can split the integral into the sum of integrals of each term:

$I = \int\limits_{-\frac{π}{2}}^{\frac{π}{2}} x^3 \;dx + \int\limits_{-\frac{π}{2}}^{\frac{π}{2}} x \cos x \;dx + \int\limits_{-\frac{π}{2}}^{\frac{π}{2}} \tan^5 x \;dx + \int\limits_{-\frac{π}{2}}^{\frac{π}{2}} 1 \;dx$


We use the property for integrals over a symmetric interval $[-a, a]$: $\int\limits_{-a}^a f(x) dx = 0$ if $f(x)$ is an odd function ($f(-x) = -f(x)$) and $\int\limits_{-a}^a f(x) dx = 2 \int\limits_0^a f(x) dx$ if $f(x)$ is an even function ($f(-x) = f(x)$).


Let's check the parity of each term in the integrand:

For $f_1(x) = x^3$: $f_1(-x) = (-x)^3 = -x^3 = -f_1(x)$. So, $x^3$ is an odd function.

$\int\limits_{-\frac{π}{2}}^{\frac{π}{2}} x^3 \;dx = 0$


For $f_2(x) = x \cos x$: $f_2(-x) = (-x) \cos(-x) = -x \cos x = -f_2(x)$ (since $\cos(-x) = \cos x$). So, $x \cos x$ is an odd function.

$\int\limits_{-\frac{π}{2}}^{\frac{π}{2}} x \cos x \;dx = 0$


For $f_3(x) = \tan^5 x$: $f_3(-x) = \tan^5 (-x) = (\tan(-x))^5 = (-\tan x)^5 = -\tan^5 x = -f_3(x)$ (since $\tan(-x) = -\tan x$). So, $\tan^5 x$ is an odd function.

$\int\limits_{-\frac{π}{2}}^{\frac{π}{2}} \tan^5 x \;dx = 0$


For $f_4(x) = 1$: $f_4(-x) = 1 = f_4(x)$. So, $1$ is an even function.

$\int\limits_{-\frac{π}{2}}^{\frac{π}{2}} 1 \;dx = 2 \int\limits_0^{\frac{π}{2}} 1 \;dx$


Substituting these results back into the expression for $I$:

$I = 0 + 0 + 0 + 2 \int\limits_0^{\frac{π}{2}} 1 \;dx$

$I = 2 \int\limits_0^{\frac{π}{2}} 1 \;dx$


Evaluate the remaining integral:

$I = 2 [x]_0^{\frac{π}{2}}$

$I = 2 \left(\frac{π}{2} - 0\right)$

$I = 2 \cdot \frac{π}{2}$

$I = π$


The value of the integral is $π$.


The correct answer is (C) π.

Question 21. The value of $\int\limits_0^{\frac{π}{2}} \log \left( \frac{4 + 3 \sin x}{4 + 3 \cos x} \right) \;dx$ is

(A) 2

(B) $\frac{3}{4}$

(C) 0

(D) –2

Answer:

Solution:


Let the given integral be $I$.

$I = \int\limits_0^{\frac{π}{2}} \log \left( \frac{4 + 3 \sin x}{4 + 3 \cos x} \right) \;dx$

... (i)


Using the property of definite integrals $\int\limits_0^a f(x) dx = \int\limits_0^a f(a-x) dx$, with $a = \frac{π}{2}$, we replace $x$ with $\frac{π}{2} - x$ in the integral (i).

Let $f(x) = \log \left( \frac{4 + 3 \sin x}{4 + 3 \cos x} \right)$.

We evaluate $f\left(\frac{π}{2} - x\right)$:

$f\left(\frac{π}{2} - x\right) = \log \left( \frac{4 + 3 \sin(\frac{π}{2} - x)}{4 + 3 \cos(\frac{π}{2} - x)} \right)$


Using the trigonometric identities $\sin\left(\frac{π}{2} - x\right) = \cos x$ and $\cos\left(\frac{π}{2} - x\right) = \sin x$:

$f\left(\frac{π}{2} - x\right) = \log \left( \frac{4 + 3 \cos x}{4 + 3 \sin x} \right)$


Applying the property $\int\limits_0^a f(x) dx = \int\limits_0^a f(a-x) dx$, the integral becomes:

$I = \int\limits_0^{\frac{π}{2}} \log \left( \frac{4 + 3 \cos x}{4 + 3 \sin x} \right) \;dx$

... (ii)


Adding equation (i) and equation (ii):

$I + I = \int\limits_0^{\frac{π}{2}} \log \left( \frac{4 + 3 \sin x}{4 + 3 \cos x} \right) \;dx + \int\limits_0^{\frac{π}{2}} \log \left( \frac{4 + 3 \cos x}{4 + 3 \sin x} \right) \;dx$

$2I = \int\limits_0^{\frac{π}{2}} \left[ \log \left( \frac{4 + 3 \sin x}{4 + 3 \cos x} \right) + \log \left( \frac{4 + 3 \cos x}{4 + 3 \sin x} \right) \right] \;dx$


Using the logarithm property $\log A + \log B = \log(AB)$:

$2I = \int\limits_0^{\frac{π}{2}} \log \left[ \left( \frac{4 + 3 \sin x}{4 + 3 \cos x} \right) \cdot \left( \frac{4 + 3 \cos x}{4 + 3 \sin x} \right) \right] \;dx$

$2I = \int\limits_0^{\frac{π}{2}} \log (1) \;dx$


Since $\log 1 = 0$:

$2I = \int\limits_0^{\frac{π}{2}} 0 \;dx$


Evaluating the integral of 0:

$2I = [C]_0^{\frac{π}{2}} = C - C = 0$ (where C is any constant)

$2I = 0$


Solving for $I$:

$I = 0$


The value of the integral is $0$.


The correct answer is (C) 0.



Example 35 to 42 - Miscellaneous Examples

Example 35: Find $\int \cos 6x \sqrt{1 + \sin 6x} \;dx$

Answer:

Solution:


Let the given integral be $I$.

$I = \int \cos 6x \sqrt{1 + \sin 6x} \;dx$


We use the method of substitution. Let $u$ be the expression inside the square root, plus the constant 1.

$u = 1 + \sin 6x$

... (1)


Now, we find the differential $du$ by differentiating both sides of equation (1) with respect to $x$.

$\frac{du}{dx} = \frac{d}{dx} (1 + \sin 6x)$

$\frac{du}{dx} = 0 + \cos(6x) \cdot \frac{d}{dx}(6x)$

(Using the chain rule for $\sin 6x$)

$\frac{du}{dx} = \cos(6x) \cdot 6$

$du = 6 \cos 6x \;dx$

... (2)


From equation (2), we can isolate $\cos 6x \;dx$:

$\cos 6x \;dx = \frac{1}{6} du$

... (3)


Now, substitute $u$ from (1) and $\cos 6x \;dx$ from (3) into the original integral $I$.

$I = \int \sqrt{u} \cdot \frac{1}{6} du$


Take the constant $\frac{1}{6}$ outside the integral:

$I = \frac{1}{6} \int \sqrt{u} \;du$


Rewrite the square root as a power:

$I = \frac{1}{6} \int u^{1/2} \;du$


Integrate $u^{1/2}$ using the power rule $\int u^n du = \frac{u^{n+1}}{n+1} + C$:

$I = \frac{1}{6} \left( \frac{u^{\frac{1}{2} + 1}}{\frac{1}{2} + 1} \right) + C$

$I = \frac{1}{6} \left( \frac{u^{3/2}}{3/2} \right) + C$

$I = \frac{1}{6} \left( \frac{2}{3} u^{3/2} \right) + C$


Simplify the constant factor:

$I = \frac{2}{18} u^{3/2} + C$

$I = \frac{1}{9} u^{3/2} + C$


Finally, substitute back the expression for $u$ from equation (1): $u = 1 + \sin 6x$.

$I = \frac{1}{9} (1 + \sin 6x)^{3/2} + C$


The value of the integral is $\frac{1}{9} (1 + \sin 6x)^{3/2} + C$, where $C$ is the constant of integration.

Example 36: Find $\int \frac{(x^4 − x)^{\frac{1}{4}}}{x^5} \;dx$

Answer:

Solution:


Let the given integral be $I$.

$I = \int \frac{(x^4 − x)^{\frac{1}{4}}}{x^5} \;dx$


We can rewrite the integrand by factoring out $x^4$ from the term inside the bracket $(x^4 - x)$.

$(x^4 - x)^{\frac{1}{4}} = [x^4(1 - \frac{x}{x^4})]^{\frac{1}{4}} = [x^4(1 - x^{-3})]^{\frac{1}{4}}$

Using the property $(ab)^n = a^n b^n$, we get:

$= (x^4)^{\frac{1}{4}} (1 - x^{-3})^{\frac{1}{4}} = x (1 - x^{-3})^{\frac{1}{4}}$


Now, substitute this back into the integral:

$I = \int \frac{x (1 - x^{-3})^{\frac{1}{4}}}{x^5} \;dx$

Cancel out one power of $x$ from the numerator and denominator:

$I = \int \frac{(1 - x^{-3})^{\frac{1}{4}}}{x^4} \;dx$


We can use the method of substitution. Let $u$ be the expression $(1 - x^{-3})$.

$u = 1 - x^{-3}$

... (1)


Now, we find the differential $du$ by differentiating both sides of equation (1) with respect to $x$.

$\frac{du}{dx} = \frac{d}{dx} (1 - x^{-3})$

$\frac{du}{dx} = 0 - (-3)x^{-3-1} = 3x^{-4}$

$\frac{du}{dx} = \frac{3}{x^4}$


Rearrange the terms to find the expression for $\frac{1}{x^4} dx$:

$du = \frac{3}{x^4} \;dx \implies \frac{1}{x^4} \;dx = \frac{1}{3} du$

... (2)


Substitute $u$ from (1) and $\frac{1}{x^4} dx$ from (2) into the simplified integral:

$I = \int (u)^{\frac{1}{4}} \cdot \frac{1}{3} du$


Take the constant $\frac{1}{3}$ outside the integral:

$I = \frac{1}{3} \int u^{\frac{1}{4}} \;du$


Integrate $u^{\frac{1}{4}}$ using the power rule for integration, $\int u^n du = \frac{u^{n+1}}{n+1} + C$:

$I = \frac{1}{3} \left( \frac{u^{\frac{1}{4} + 1}}{\frac{1}{4} + 1} \right) + C$

$I = \frac{1}{3} \left( \frac{u^{\frac{5}{4}}}{\frac{5}{4}} \right) + C$

$I = \frac{1}{3} \left( \frac{4}{5} u^{\frac{5}{4}} \right) + C$


Simplify the constant factor:

$I = \frac{4}{15} u^{\frac{5}{4}} + C$


Finally, substitute back the expression for $u$ from equation (1): $u = 1 - x^{-3}$.

$I = \frac{4}{15} (1 - x^{-3})^{\frac{5}{4}} + C$


We can also write $(1 - x^{-3})$ as $1 - \frac{1}{x^3} = \frac{x^3 - 1}{x^3}$.

So, the result can also be written as:

$I = \frac{4}{15} \left(\frac{x^3 - 1}{x^3}\right)^{\frac{5}{4}} + C$


The value of the integral is $\frac{4}{15} (1 - x^{-3})^{\frac{5}{4}} + C$ or $\frac{4}{15} \left(\frac{x^3 - 1}{x^3}\right)^{\frac{5}{4}} + C$, where $C$ is the constant of integration.

Example 37: Find $\int \frac{x^4 \;dx}{(x − 1 ) (x^2 + 1)}$

Answer:

To evaluate the integral $\int \frac{x^4 \;dx}{(x − 1 ) (x^2 + 1)}$, we first observe that the degree of the numerator ($4$) is greater than the degree of the denominator ($(x-1)(x^2+1) = x^3 - x^2 + x - 1$, degree $3$). Thus, this is an improper rational function, and we must perform polynomial long division.


Dividing $x^4$ by $x^3 - x^2 + x - 1$:

$\begin{array}{r} x+1\phantom{2x+3)} \\ x^3-x^2+x-1{\overline{\smash{\big)}\,x^4\phantom{+0x^3+0x^2+0x+0}}} \\ \underline{-~\phantom{(}(x^4-x^3+x^2-x)\phantom{-b)}} \\ 0+x^3-x^2+x\phantom{+0} \\ \underline{-~\phantom{()}(x^3-x^2+x-1)} \\ 0+0+0+1\phantom{)} \end{array}$

So, $\frac{x^4}{(x − 1 ) (x^2 + 1)} = x + 1 + \frac{1}{(x − 1 ) (x^2 + 1)}$.


Now we need to integrate this expression:

$\int \frac{x^4 \;dx}{(x − 1 ) (x^2 + 1)} = \int \left(x + 1 + \frac{1}{(x − 1 ) (x^2 + 1)}\right) \;dx$

$= \int (x + 1) \;dx + \int \frac{1}{(x − 1 ) (x^2 + 1)} \;dx$


We evaluate the first part:

$\int (x + 1) \;dx = \int x \;dx + \int 1 \;dx = \frac{x^2}{2} + x$


For the second part, we use partial fraction decomposition on $\frac{1}{(x − 1 ) (x^2 + 1)}$.

We set up the decomposition as:

$\frac{1}{(x − 1 ) (x^2 + 1)} = \frac{A}{x - 1} + \frac{Bx + C}{x^2 + 1}$

Multiplying by $(x - 1)(x^2 + 1)$, we get:

$1 = A(x^2 + 1) + (Bx + C)(x - 1)$

$1 = A x^2 + A + B x^2 - Bx + Cx - C$

$1 = (A + B)x^2 + (C - B)x + (A - C)$


Equating coefficients of corresponding powers of $x$:

Coefficient of $x^2$: $A + B = 0$

Coefficient of $x$: $C - B = 0 \implies C = B$

Constant term: $A - C = 1$


Substituting $C=B$ into $A - C = 1$, we get $A - B = 1$.

We now solve the system:

$A + B = 0$

$A - B = 1$

Adding the two equations: $(A + B) + (A - B) = 0 + 1 \implies 2A = 1 \implies A = \frac{1}{2}$.

Substituting $A = \frac{1}{2}$ into $A + B = 0$: $\frac{1}{2} + B = 0 \implies B = -\frac{1}{2}$.

Since $C = B$, $C = -\frac{1}{2}$.


So the partial fraction decomposition is:

$\frac{1}{(x − 1 ) (x^2 + 1)} = \frac{\frac{1}{2}}{x - 1} + \frac{-\frac{1}{2}x - \frac{1}{2}}{x^2 + 1} = \frac{1}{2(x - 1)} - \frac{x + 1}{2(x^2 + 1)}$

We can rewrite the second term as two separate fractions:

$\frac{1}{(x − 1 ) (x^2 + 1)} = \frac{1}{2(x - 1)} - \frac{x}{2(x^2 + 1)} - \frac{1}{2(x^2 + 1)}$


Now we integrate each term of the partial fraction decomposition:

$\int \frac{1}{2(x - 1)} \;dx = \frac{1}{2} \int \frac{1}{x - 1} \;dx = \frac{1}{2} \log_e |x - 1|$

$\int -\frac{x}{2(x^2 + 1)} \;dx = -\frac{1}{2} \int \frac{x}{x^2 + 1} \;dx$. Let $u = x^2 + 1$, $du = 2x \;dx$. So $x \;dx = \frac{1}{2} du$.

$-\frac{1}{2} \int \frac{1}{u} \left(\frac{1}{2} du\right) = -\frac{1}{4} \int \frac{1}{u} \;du = -\frac{1}{4} \log_e |u| = -\frac{1}{4} \log_e (x^2 + 1)$ (since $x^2 + 1 > 0$)

$\int -\frac{1}{2(x^2 + 1)} \;dx = -\frac{1}{2} \int \frac{1}{x^2 + 1} \;dx = -\frac{1}{2} \tan^{-1}(x)$


Combining all parts of the integral:

$\int \frac{x^4 \;dx}{(x − 1 ) (x^2 + 1)} = \left(\frac{x^2}{2} + x\right) + \left(\frac{1}{2} \log_e |x - 1| - \frac{1}{4} \log_e (x^2 + 1) - \frac{1}{2} \tan^{-1}(x)\right) + C$


Final Answer:

$\int \frac{x^4 \;dx}{(x − 1 ) (x^2 + 1)} = \frac{x^2}{2} + x + \frac{1}{2} \log_e |x - 1| - \frac{1}{4} \log_e (x^2 + 1) - \frac{1}{2} \tan^{-1}(x) + C$

Example 38: Find $\int \left[ \log (\log x) + \frac{1}{(\log x)^2} \right] \;dx$

Answer:

Solution:


Let the given integral be $I$.

$I = \int \left[ \log (\log x) + \frac{1}{(\log x)^2} \right] \;dx$


We can split the integral into two parts:

$I = \int \log (\log x) \;dx + \int \frac{1}{(\log x)^2} \;dx$


Let's evaluate the first integral $\int \log (\log x) \;dx$ using integration by parts. Recall the formula $\int u \;dv = uv - \int v \;du$.

Let $u = \log (\log x)$ and $dv = 1 \;dx$.


Then $du = \frac{d}{dx} (\log (\log x)) \;dx = \frac{1}{\log x} \cdot \frac{d}{dx} (\log x) \;dx = \frac{1}{\log x} \cdot \frac{1}{x} \;dx = \frac{1}{x \log x} \;dx$.

And $v = \int 1 \;dx = x$.


Applying the integration by parts formula to the first integral:

$\int \log (\log x) \;dx = (\log (\log x)) \cdot x - \int x \cdot \left( \frac{1}{x \log x} \right) \;dx$

$= x \log (\log x) - \int \frac{1}{\log x} \;dx$


Now substitute this result back into the original integral expression:

$I = \left( x \log (\log x) - \int \frac{1}{\log x} \;dx \right) + \int \frac{1}{(\log x)^2} \;dx$

$I = x \log (\log x) - \int \frac{1}{\log x} \;dx + \int \frac{1}{(\log x)^2} \;dx$


Now consider the integral $\int \frac{1}{\log x} \;dx$. Let's evaluate this using integration by parts as well, setting $u = \frac{1}{\log x}$ and $dv = 1 \;dx$.

Let $u = \frac{1}{\log x} = (\log x)^{-1}$ and $dv = 1 \;dx$.


Then $du = \frac{d}{dx} ((\log x)^{-1}) \;dx = -1 (\log x)^{-2} \cdot \frac{d}{dx} (\log x) \;dx = -(\log x)^{-2} \cdot \frac{1}{x} \;dx = -\frac{1}{x (\log x)^2} \;dx$.

And $v = \int 1 \;dx = x$.


Applying the integration by parts formula to $\int \frac{1}{\log x} \;dx$:

$\int \frac{1}{\log x} \;dx = \left( \frac{1}{\log x} \right) \cdot x - \int x \cdot \left( -\frac{1}{x (\log x)^2} \right) \;dx$

$= \frac{x}{\log x} - \int \left( -\frac{1}{(\log x)^2} \right) \;dx$

$= \frac{x}{\log x} + \int \frac{1}{(\log x)^2} \;dx$


Now substitute this result for $\int \frac{1}{\log x} \;dx$ back into the expression for $I$:

$I = x \log (\log x) - \left( \frac{x}{\log x} + \int \frac{1}{(\log x)^2} \;dx \right) + \int \frac{1}{(\log x)^2} \;dx$


Distribute the negative sign:

$I = x \log (\log x) - \frac{x}{\log x} - \int \frac{1}{(\log x)^2} \;dx + \int \frac{1}{(\log x)^2} \;dx$


The two integral terms cancel each other out:

$I = x \log (\log x) - \frac{x}{\log x} + C$


The value of the integral is $x \log (\log x) - \frac{x}{\log x} + C$, where $C$ is the constant of integration.

Example 39: Find $\int \; [\sqrt{\cot x} − \sqrt{\tan x}] \;dx$

Answer:

Solution:


Let the given integral be $I$.

$I = \int (\sqrt{\cot x} - \sqrt{\tan x}) \;dx$


Rewrite the integrand in terms of $\sin x$ and $\cos x$:

$\sqrt{\cot x} - \sqrt{\tan x} = \frac{\sqrt{\cos x}}{\sqrt{\sin x}} - \frac{\sqrt{\sin x}}{\sqrt{\cos x}}$

$= \frac{(\sqrt{\cos x})^2 - (\sqrt{\sin x})^2}{\sqrt{\sin x} \sqrt{\cos x}}$

$= \frac{\cos x - \sin x}{\sqrt{\sin x \cos x}}$


So, the integral becomes:

$I = \int \frac{\cos x - \sin x}{\sqrt{\sin x \cos x}} \;dx$


We use the method of substitution. Let $u = \sin x + \cos x$.

$u = \sin x + \cos x$

... (1)


Now, find the differential $du$ by differentiating both sides of equation (1) with respect to $x$.

$\frac{du}{dx} = \frac{d}{dx} (\sin x + \cos x)$

$\frac{du}{dx} = \cos x - \sin x$

$du = (\cos x - \sin x) \;dx$

... (2)


We also need to express the term $\sin x \cos x$ in the denominator in terms of $u$. Square both sides of equation (1):

$u^2 = (\sin x + \cos x)^2$

$u^2 = \sin^2 x + \cos^2 x + 2 \sin x \cos x$


Using the identity $\sin^2 x + \cos^2 x = 1$:

$u^2 = 1 + 2 \sin x \cos x$


Solve for $\sin x \cos x$:

$2 \sin x \cos x = u^2 - 1 \implies \sin x \cos x = \frac{u^2 - 1}{2}$

... (3)


Now substitute $u$, $du$, and $\sin x \cos x$ from (1), (2), and (3) into the integral $I = \int \frac{\cos x - \sin x}{\sqrt{\sin x \cos x}} \;dx$:

$I = \int \frac{du}{\sqrt{\frac{u^2 - 1}{2}}}$


Rewrite the denominator:

$\sqrt{\frac{u^2 - 1}{2}} = \frac{\sqrt{u^2 - 1}}{\sqrt{2}}$


So, the integral becomes:

$I = \int \frac{du}{\frac{\sqrt{u^2 - 1}}{\sqrt{2}}} = \int \frac{\sqrt{2} \;du}{\sqrt{u^2 - 1}}$


Take the constant $\sqrt{2}$ outside the integral:

$I = \sqrt{2} \int \frac{1}{\sqrt{u^2 - 1}} \;du$


This is a standard integral of the form $\int \frac{1}{\sqrt{x^2 - a^2}} dx = \log|x + \sqrt{x^2 - a^2}| + C$. Here, $a = 1$.

$I = \sqrt{2} \log|u + \sqrt{u^2 - 1}| + C$


Finally, substitute back the expression for $u$ from equation (1): $u = \sin x + \cos x$. Also substitute $u^2 - 1 = 2 \sin x \cos x$ from equation (3).

$I = \sqrt{2} \log|\sin x + \cos x + \sqrt{2 \sin x \cos x}| + C$


The value of the integral is $\sqrt{2} \log|\sin x + \cos x + \sqrt{2 \sin x \cos x}| + C$, where $C$ is the constant of integration.

Example 40: Find $\int \frac{\sin 2x \;\cos 2x \;dx}{\sqrt{9 − \cos^4 (2x)}}$

Answer:

Solution:


Let the given integral be $I$.

$I = \int \frac{\sin 2x \;\cos 2x \;dx}{\sqrt{9 − \cos^4 (2x)}}$


We use the method of substitution. Let $u = \cos^2 (2x)$.

$u = \cos^2 (2x)$

... (1)


Find the differential $du$ by differentiating both sides of equation (1) with respect to $x$ using the chain rule.

$\frac{du}{dx} = \frac{d}{dx} (\cos^2 (2x))$

$\frac{du}{dx} = 2 \cos(2x) \cdot \frac{d}{dx}(\cos(2x))$

$\frac{du}{dx} = 2 \cos(2x) \cdot (-\sin(2x)) \cdot \frac{d}{dx}(2x)$

$\frac{du}{dx} = 2 \cos(2x) (-\sin(2x)) (2) = -4 \sin(2x) \cos(2x)$


Rearranging the terms to find the expression for $\sin 2x \cos 2x \;dx$:

$du = -4 \sin(2x) \cos(2x) \;dx \implies \sin 2x \cos 2x \;dx = -\frac{1}{4} du$

... (2)


Express the term under the square root in terms of $u$. From equation (1), $u = \cos^2 (2x)$.

$\cos^4 (2x) = (\cos^2 (2x))^2 = u^2$

So the denominator is $\sqrt{9 - \cos^4 (2x)} = \sqrt{9 - u^2} = \sqrt{3^2 - u^2}$.


Substitute the expressions from (1) and (2) into the original integral:

$I = \int \frac{-\frac{1}{4} du}{\sqrt{3^2 - u^2}}$


Take the constant $-\frac{1}{4}$ outside the integral:

$I = -\frac{1}{4} \int \frac{1}{\sqrt{3^2 - u^2}} \;du$


This is a standard integral of the form $\int \frac{1}{\sqrt{a^2 - x^2}} dx = \sin^{-1}\left(\frac{x}{a}\right) + C$. Here, $a = 3$ and the variable is $u$.

$\int \frac{1}{\sqrt{3^2 - u^2}} \;du = \sin^{-1}\left(\frac{u}{3}\right) + C'$


Substitute this result back into the expression for $I$:

$I = -\frac{1}{4} \sin^{-1}\left(\frac{u}{3}\right) + C$


Finally, substitute back the expression for $u$ from equation (1): $u = \cos^2 (2x)$.

$I = -\frac{1}{4} \sin^{-1}\left(\frac{\cos^2 (2x)}{3}\right) + C$


The value of the integral is $-\frac{1}{4} \sin^{-1}\left(\frac{\cos^2 (2x)}{3}\right) + C$, where $C$ is the constant of integration.

Example 41: Evaluate $\int\limits_{−1}^{\frac{3}{2}} |x \sin (πx)| \;dx$

Answer:

Introduction:

The problem asks for the evaluation of the definite integral $\int\limits_{−1}^{\frac{3}{2}} |x \sin (πx)| \;dx$. To handle the absolute value, we need to determine the sign of the expression $x \sin(\pi x)$ within the interval of integration $[-1, 3/2]$.


Sign Analysis of $x \sin(\pi x)$ in $[-1, 3/2]$:

Let $f(x) = x \sin(\pi x)$. The sign of $f(x)$ changes when $x=0$ or when $\sin(\pi x) = 0$. $\sin(\pi x) = 0$ when $\pi x = n\pi$ for an integer $n$, which means $x = n$. The integers in the interval $[-1, 3/2]$ are $-1, 0, 1$. We examine the sign of $f(x)$ in the intervals determined by these points and the endpoints:

1. For $x \in [-1, 0]$: $x \le 0$ and $\pi x \in [-\pi, 0]$, so $\sin(\pi x) \le 0$. Thus, $x \sin(\pi x) = (\text{negative or zero}) \times (\text{negative or zero}) \ge 0$ in $[-1, 0]$.

2. For $x \in [0, 1]$: $x \ge 0$ and $\pi x \in [0, \pi]$, so $\sin(\pi x) \ge 0$. Thus, $x \sin(\pi x) = (\text{positive or zero}) \times (\text{positive or zero}) \ge 0$ in $[0, 1]$.

3. For $x \in [1, 3/2]$: $x > 0$ and $\pi x \in [\pi, 3\pi/2]$, so $\sin(\pi x) \le 0$. Thus, $x \sin(\pi x) = (\text{positive}) \times (\text{negative or zero}) \le 0$ in $[1, 3/2]$.

So, $|x \sin(\pi x)| = x \sin(\pi x)$ for $x \in [-1, 1]$ and $|x \sin(\pi x)| = -x \sin(\pi x)$ for $x \in [1, 3/2]$.


Splitting the Integral:

Based on the sign analysis, we split the integral as follows:

$\int\limits_{−1}^{\frac{3}{2}} |x \sin (πx)| \;dx = \int\limits_{−1}^{0} x \sin (πx) \;dx + \int\limits_{0}^{1} x \sin (πx) \;dx + \int\limits_{1}^{\frac{3}{2}} -x \sin (πx) \;dx$

Combining the first two integrals since the integrand is the same and the intervals are contiguous:

$\int\limits_{−1}^{\frac{3}{2}} |x \sin (πx)| \;dx = \int\limits_{−1}^{1} x \sin (πx) \;dx - \int\limits_{1}^{\frac{3}{2}} x \sin (πx) \;dx$


Finding the Indefinite Integral:

We find the indefinite integral $\int x \sin(\pi x) \;dx$ using integration by parts, $\int u \;dv = uv - \int v \;du$.

Let $u = x$ and $dv = \sin(\pi x) \;dx$.

Then $du = dx$ and $v = \int \sin(\pi x) \;dx = -\frac{1}{\pi} \cos(\pi x)$.

$\int x \sin(\pi x) \;dx = x \left(-\frac{1}{\pi} \cos(\pi x)\right) - \int \left(-\frac{1}{\pi} \cos(\pi x)\right) \;dx$

$= -\frac{x}{\pi} \cos(\pi x) + \frac{1}{\pi} \int \cos(\pi x) \;dx$

$= -\frac{x}{\pi} \cos(\pi x) + \frac{1}{\pi} \left(\frac{1}{\pi} \sin(\pi x)\right) + C$

$= -\frac{x}{\pi} \cos(\pi x) + \frac{1}{\pi^2} \sin(\pi x) + C$

Let $F(x) = -\frac{x}{\pi} \cos(\pi x) + \frac{1}{\pi^2} \sin(\pi x)$ be the antiderivative.


Evaluating the Definite Integrals:

First, we evaluate $\int\limits_{-1}^{1} x \sin (πx) \;dx = [F(x)]_{-1}^1 = F(1) - F(-1)$.

$F(1) = -\frac{1}{\pi} \cos(\pi) + \frac{1}{\pi^2} \sin(\pi) = -\frac{1}{\pi}(-1) + \frac{1}{\pi^2}(0) = \frac{1}{\pi}$

$F(-1) = -\frac{-1}{\pi} \cos(-\pi) + \frac{1}{\pi^2} \sin(-\pi) = \frac{1}{\pi}(-1) + \frac{1}{\pi^2}(0) = -\frac{1}{\pi}$

$\int\limits_{-1}^{1} x \sin (πx) \;dx = \frac{1}{\pi} - (-\frac{1}{\pi}) = \frac{1}{\pi} + \frac{1}{\pi} = \frac{2}{\pi}$

Next, we evaluate $\int\limits_{1}^{\frac{3}{2}} x \sin (πx) \;dx = [F(x)]_{1}^{3/2} = F(3/2) - F(1)$.

$F(3/2) = -\frac{3/2}{\pi} \cos(3\pi/2) + \frac{1}{\pi^2} \sin(3\pi/2) = -\frac{3}{2\pi}(0) + \frac{1}{\pi^2}(-1) = -\frac{1}{\pi^2}$

$F(1) = \frac{1}{\pi}$

$\int\limits_{1}^{\frac{3}{2}} x \sin (πx) \;dx = -\frac{1}{\pi^2} - \frac{1}{\pi}$


Final Calculation:

Substitute the results back into the split integral expression:

$\int\limits_{−1}^{\frac{3}{2}} |x \sin (πx)| \;dx = \int\limits_{−1}^{1} x \sin (πx) \;dx - \int\limits_{1}^{\frac{3}{2}} x \sin (πx) \;dx$

$= \frac{2}{\pi} - \left(-\frac{1}{\pi^2} - \frac{1}{\pi}\right)$

$= \frac{2}{\pi} + \frac{1}{\pi^2} + \frac{1}{\pi}$

$= \frac{3}{\pi} + \frac{1}{\pi^2}$

Thus, the value of the integral is $\frac{3}{\pi} + \frac{1}{\pi^2}$.

Example 42: Evaluate $\int\limits_0^π \frac{x \;dx}{a^2 \cos^2 x + b^2 \sin^2 x}$

Answer:

Given:

The definite integral to evaluate is $\int\limits_0^π \frac{x \;dx}{a^2 \cos^2 x + b^2 \sin^2 x}$.

Let the integral be $I$.

$I = \int\limits_0^π \frac{x \;dx}{a^2 \cos^2 x + b^2 \sin^2 x}$


Solution:

We use the property of definite integrals: $\int\limits_0^a f(x) \;dx = \int\limits_0^a f(a-x) \;dx$. Here $a = \pi$ and $f(x) = \frac{x}{a^2 \cos^2 x + b^2 \sin^2 x}$.

Applying the property, we get:

$I = \int\limits_0^π \frac{(\pi - x) \;dx}{a^2 \cos^2 (\pi - x) + b^2 \sin^2 (\pi - x)}$

Since $\cos(\pi - x) = -\cos x$ and $\sin(\pi - x) = \sin x$, we have $\cos^2 (\pi - x) = \cos^2 x$ and $\sin^2 (\pi - x) = \sin^2 x$.

So, the expression becomes:

$I = \int\limits_0^π \frac{(\pi - x) \;dx}{a^2 \cos^2 x + b^2 \sin^2 x}$

I = $\int\limits_0^π \frac{\pi - x}{a^2 \cos^2 x + b^2 \sin^2 x} \;dx$

... (i)

The original integral is:

I = $\int\limits_0^π \frac{x}{a^2 \cos^2 x + b^2 \sin^2 x} \;dx$

... (ii)

Adding (i) and (ii):

$I + I = \int\limits_0^π \frac{\pi - x}{a^2 \cos^2 x + b^2 \sin^2 x} \;dx + \int\limits_0^π \frac{x}{a^2 \cos^2 x + b^2 \sin^2 x} \;dx$

$2I = \int\limits_0^π \frac{(\pi - x) + x}{a^2 \cos^2 x + b^2 \sin^2 x} \;dx$

$2I = \int\limits_0^π \frac{\pi}{a^2 \cos^2 x + b^2 \sin^2 x} \;dx$

$2I = \pi \int\limits_0^π \frac{dx}{a^2 \cos^2 x + b^2 \sin^2 x}$

Now, we use another property: $\int\limits_0^{2a} f(x) \;dx = 2 \int\limits_0^a f(x) \;dx$ if $f(2a-x) = f(x)$. Here $2a = \pi$, so $a = \pi/2$. Let $g(x) = \frac{1}{a^2 \cos^2 x + b^2 \sin^2 x}$.

$g(\pi - x) = \frac{1}{a^2 \cos^2 (\pi - x) + b^2 \sin^2 (\pi - x)} = \frac{1}{a^2 \cos^2 x + b^2 \sin^2 x} = g(x)$.

So, $\int\limits_0^π \frac{dx}{a^2 \cos^2 x + b^2 \sin^2 x} = 2 \int\limits_0^{\pi/2} \frac{dx}{a^2 \cos^2 x + b^2 \sin^2 x}$.

$2I = \pi \left( 2 \int\limits_0^{\pi/2} \frac{dx}{a^2 \cos^2 x + b^2 \sin^2 x} \right)$

$I = \pi \int\limits_0^{\pi/2} \frac{dx}{a^2 \cos^2 x + b^2 \sin^2 x}$

To evaluate this integral, we divide the numerator and denominator by $\cos^2 x$ (assuming $\cos x \neq 0$ in the interval $(0, \pi/2)$).

$I = \pi \int\limits_0^{\pi/2} \frac{\frac{1}{\cos^2 x} \;dx}{\frac{a^2 \cos^2 x}{\cos^2 x} + \frac{b^2 \sin^2 x}{\cos^2 x}}$

$I = \pi \int\limits_0^{\pi/2} \frac{\sec^2 x \;dx}{a^2 + b^2 \tan^2 x}$

Let $t = \tan x$. Then $dt = \sec^2 x \;dx$. When $x=0$, $t = \tan 0 = 0$. When $x=\pi/2$, $t = \tan(\pi/2) \to \infty$.

$I = \pi \int\limits_0^{\infty} \frac{dt}{a^2 + b^2 t^2}$

$I = \pi \int\limits_0^{\infty} \frac{dt}{b^2 \left( \frac{a^2}{b^2} + t^2 \right)}$

$I = \frac{\pi}{b^2} \int\limits_0^{\infty} \frac{dt}{\left(\frac{a}{b}\right)^2 + t^2}$

This is a standard integral of the form $\int \frac{dx}{k^2 + x^2} = \frac{1}{k} \tan^{-1}\left(\frac{x}{k}\right)$. Here $k = a/b$. We assume $a, b \neq 0$ for the original integral to converge.

$I = \frac{\pi}{b^2} \left[ \frac{1}{a/b} \tan^{-1}\left(\frac{t}{a/b}\right) \right]_0^{\infty}$

$I = \frac{\pi}{b^2} \left[ \frac{b}{a} \tan^{-1}\left(\frac{bt}{a}\right) \right]_0^{\infty}$

$I = \frac{\pi}{ab} \left[ \tan^{-1}\left(\frac{bt}{a}\right) \right]_0^{\infty}$

Assuming $a, b$ are real and non-zero, the term $\frac{bt}{a}$ goes from $0$ to $\pm \infty$ as $t$ goes from $0$ to $\infty$. However, since the original integrand contains $a^2$ and $b^2$, the result must be independent of the signs of $a$ and $b$. We consider the magnitude.

The standard result for $\int\limits_0^{\pi/2} \frac{dx}{A^2 \cos^2 x + B^2 \sin^2 x}$ is $\frac{\pi}{2|AB|}$.

Comparing this with $I = \pi \int\limits_0^{\pi/2} \frac{dx}{a^2 \cos^2 x + b^2 \sin^2 x}$, where $A^2=a^2 \implies A = |a|$ and $B^2=b^2 \implies B = |b|$.

So, $\int\limits_0^{\pi/2} \frac{dx}{a^2 \cos^2 x + b^2 \sin^2 x} = \frac{\pi}{2|a||b|} = \frac{\pi}{2|ab|}$.

Substituting this back into the expression for $I$:

$I = \pi \left( \frac{\pi}{2|ab|} \right)$

$I = \frac{\pi^2}{2|ab|}$


Conclusion:

The value of the integral is $\frac{\pi^2}{2|ab|}$.



Miscellaneous Exercise on Chapter 7

Integrate the functions in Exercises 1 to 23.

Question 1. $\frac{1}{x − x^3}$

Answer:

Solution:


We are asked to integrate the function $\frac{1}{x - x^3}$.

First, we factor the denominator:

$x - x^3 = x(1 - x^2) = x(1 - x)(1 + x)$

So, the integral is $\int \frac{1}{x(1 - x)(1 + x)} dx$.

We can integrate this using the method of partial fraction decomposition.

We set up the decomposition as follows:

$\frac{1}{x(1 - x)(1 + x)} = \frac{A}{x} + \frac{B}{1 - x} + \frac{C}{1 + x}$

To find the constants A, B, and C, we multiply both sides by $x(1 - x)(1 + x)$:

$1 = A(1 - x)(1 + x) + Bx(1 + x) + Cx(1 - x)$

Now, we substitute specific values of $x$ to solve for A, B, and C.

Let $x = 0$:

$1 = A(1 - 0)(1 + 0) + B(0)(1 + 0) + C(0)(1 - 0)$

$1 = A(1)(1) + 0 + 0$

$1 = A$


Let $x = 1$:

$1 = A(1 - 1)(1 + 1) + B(1)(1 + 1) + C(1)(1 - 1)$

$1 = A(0)(2) + B(1)(2) + C(1)(0)$

$1 = 0 + 2B + 0$

$1 = 2B$

$B = \frac{1}{2}$


Let $x = -1$:

$1 = A(1 - (-1))(1 + (-1)) + B(-1)(1 + (-1)) + C(-1)(1 - (-1))$

$1 = A(2)(0) + B(-1)(0) + C(-1)(2)$

$1 = 0 + 0 - 2C$

$1 = -2C$

$C = -\frac{1}{2}$


Now we substitute the values of A, B, and C back into the partial fraction decomposition:

$\frac{1}{x - x^3} = \frac{1}{x} + \frac{1/2}{1 - x} + \frac{-1/2}{1 + x}$

Now we integrate each term:

$\int \frac{1}{x - x^3} dx = \int \left( \frac{1}{x} + \frac{1}{2(1 - x)} - \frac{1}{2(1 + x)} \right) dx$

$\int \frac{1}{x} dx = \log_e |x|$

$\int \frac{1}{2(1 - x)} dx = \frac{1}{2} \int \frac{1}{1 - x} dx$. Let $u = 1 - x$, $du = -dx$. So, $\int \frac{1}{2} \frac{-du}{u} = -\frac{1}{2} \log_e |u| = -\frac{1}{2} \log_e |1 - x|$.

$\int -\frac{1}{2(1 + x)} dx = -\frac{1}{2} \int \frac{1}{1 + x} dx$. Let $v = 1 + x$, $dv = dx$. So, $\int -\frac{1}{2} \frac{dv}{v} = -\frac{1}{2} \log_e |v| = -\frac{1}{2} \log_e |1 + x|$.

Combining the results and adding the constant of integration C:

$\int \frac{1}{x - x^3} dx = \log_e |x| - \frac{1}{2} \log_e |1 - x| - \frac{1}{2} \log_e |1 + x| + C$


We can simplify the expression using logarithm properties ($\log_e a - \log_e b = \log_e \frac{a}{b}$ and $n \log_e a = \log_e a^n$):

$\int \frac{1}{x - x^3} dx = \log_e |x| - \frac{1}{2} (\log_e |1 - x| + \log_e |1 + x|) + C$

$= \log_e |x| - \frac{1}{2} \log_e |(1 - x)(1 + x)| + C$

$= \log_e |x| - \frac{1}{2} \log_e |1 - x^2| + C$

$= \log_e |x| - \log_e |(1 - x^2)^{1/2}| + C$

$= \log_e \left|\frac{x}{(1 - x^2)^{1/2}}\right| + C$

$= \log_e \left|\frac{x}{\sqrt{1 - x^2}}\right| + C$


The final answer is $\mathbf{\log_e \left|\frac{x}{\sqrt{1 - x^2}}\right| + C}$.

Question 2. $\frac{1}{\sqrt{x + a}+ \sqrt{x + b}}$

Answer:

Solution:


We need to integrate the function $\frac{1}{\sqrt{x + a}+ \sqrt{x + b}}$.

To integrate this function, we can rationalize the denominator by multiplying the numerator and denominator by the conjugate of the denominator, which is $\sqrt{x + a}- \sqrt{x + b}$.

Assuming $a \neq b$, we have:

$\frac{1}{\sqrt{x + a}+ \sqrt{x + b}} = \frac{1}{\sqrt{x + a}+ \sqrt{x + b}} \times \frac{\sqrt{x + a}- \sqrt{x + b}}{\sqrt{x + a}- \sqrt{x + b}}$

Using the difference of squares formula $(p+q)(p-q) = p^2 - q^2$ in the denominator:

$= \frac{\sqrt{x + a}- \sqrt{x + b}}{(\sqrt{x + a})^2 - (\sqrt{x + b})^2}$

$= \frac{\sqrt{x + a}- \sqrt{x + b}}{(x + a) - (x + b)}$

$= \frac{\sqrt{x + a}- \sqrt{x + b}}{x + a - x - b}$

$= \frac{\sqrt{x + a}- \sqrt{x + b}}{a - b}$


Now we need to integrate this expression:

$\int \frac{1}{\sqrt{x + a}+ \sqrt{x + b}} dx = \int \frac{\sqrt{x + a}- \sqrt{x + b}}{a - b} dx$

Since $\frac{1}{a - b}$ is a constant (assuming $a \neq b$), we can take it out of the integral:

$= \frac{1}{a - b} \int (\sqrt{x + a}- \sqrt{x + b}) dx$

$= \frac{1}{a - b} \left( \int \sqrt{x + a} dx - \int \sqrt{x + b} dx \right)$


We know that $\int \sqrt{u} du = \int u^{1/2} du = \frac{u^{1/2 + 1}}{1/2 + 1} + C = \frac{u^{3/2}}{3/2} + C = \frac{2}{3} u^{3/2} + C$.

For the first integral, $\int \sqrt{x + a} dx$, let $u = x + a$. Then $du = dx$.

$\int \sqrt{x + a} dx = \int \sqrt{u} du = \frac{2}{3} u^{3/2} + C_1 = \frac{2}{3} (x + a)^{3/2} + C_1$

For the second integral, $\int \sqrt{x + b} dx$, let $v = x + b$. Then $dv = dx$.

$\int \sqrt{x + b} dx = \int \sqrt{v} dv = \frac{2}{3} v^{3/2} + C_2 = \frac{2}{3} (x + b)^{3/2} + C_2$


Substituting these results back into the main integral:

$\int \frac{1}{\sqrt{x + a}+ \sqrt{x + b}} dx = \frac{1}{a - b} \left( \frac{2}{3} (x + a)^{3/2} - \frac{2}{3} (x + b)^{3/2} \right) + C$

where $C = \frac{1}{a - b} (C_1 - C_2)$ is the constant of integration.

We can factor out the common term $\frac{2}{3}$:

$= \frac{2}{3(a - b)} \left( (x + a)^{3/2} - (x + b)^{3/2} \right) + C$


Note: If $a=b$, the original function becomes $\frac{1}{2\sqrt{x+a}} = \frac{1}{2}(x+a)^{-1/2}$.

$\int \frac{1}{2}(x+a)^{-1/2} dx = \frac{1}{2} \frac{(x+a)^{1/2}}{1/2} + C = \sqrt{x+a} + C$.

However, the standard question implies $a \neq b$ when rationalizing the denominator leads to a term like $(a-b)$ in the denominator.


The final answer is $\mathbf{\frac{2}{3(a - b)} \left( (x + a)^{3/2} - (x + b)^{3/2} \right) + C}$.

Question 3. $\frac{1}{x \sqrt{ax − x^2}}$

[Hint : Put x = $\frac{a}{t}$]

Answer:

Solution:


We need to integrate the function $\frac{1}{x \sqrt{ax − x^2}}$.

We use the given hint: substitute $x = \frac{a}{t}$.

From $x = \frac{a}{t}$, we can find $t = \frac{a}{x}$.

Now, we find the differential $dx$ in terms of $dt$.

$x = a t^{-1}$

$\frac{dx}{dt} = a(-1)t^{-2} = -\frac{a}{t^2}$

So, $dx = -\frac{a}{t^2} dt$.


Next, we substitute $x = \frac{a}{t}$ into the term under the square root:

$ax - x^2 = a\left(\frac{a}{t}\right) - \left(\frac{a}{t}\right)^2$

$= \frac{a^2}{t} - \frac{a^2}{t^2}$

$= a^2 \left(\frac{1}{t} - \frac{1}{t^2}\right)$

$= a^2 \left(\frac{t - 1}{t^2}\right)$


Now substitute this into the square root:

$\sqrt{ax - x^2} = \sqrt{a^2 \left(\frac{t - 1}{t^2}\right)}$

Assuming $a > 0$ and $t > 0$ (which implies $x > 0$ and $ax-x^2 \ge 0$, so $0 \le x \le a$), we have:

$= \sqrt{a^2} \sqrt{\frac{t - 1}{t^2}}$

$= a \frac{\sqrt{t - 1}}{\sqrt{t^2}}$

$= a \frac{\sqrt{t - 1}}{|t|}$

Assuming $t > 0$ (since $x>0$ and $a>0$):

$= a \frac{\sqrt{t - 1}}{t}$


Now, substitute $x$, $\sqrt{ax - x^2}$, and $dx$ into the integral $\int \frac{1}{x \sqrt{ax − x^2}} dx$:

$\int \frac{1}{\left(\frac{a}{t}\right) \left(a \frac{\sqrt{t - 1}}{t}\right)} \left(-\frac{a}{t^2}\right) dt$

$= \int \frac{1}{\frac{a^2}{t^2} \sqrt{t - 1}} \left(-\frac{a}{t^2}\right) dt$

$= \int \frac{t^2}{a^2 \sqrt{t - 1}} \left(-\frac{a}{t^2}\right) dt$

$= \int -\frac{a t^2}{a^2 t^2 \sqrt{t - 1}} dt$

$= \int -\frac{1}{a \sqrt{t - 1}} dt$

$= -\frac{1}{a} \int (t - 1)^{-1/2} dt$


Now we integrate with respect to $t$. Let $u = t - 1$, then $du = dt$.

$-\frac{1}{a} \int u^{-1/2} du = -\frac{1}{a} \frac{u^{-1/2 + 1}}{-1/2 + 1} + C$

$= -\frac{1}{a} \frac{u^{1/2}}{1/2} + C$

$= -\frac{1}{a} (2 \sqrt{u}) + C$

$= -\frac{2}{a} \sqrt{t - 1} + C$


Finally, substitute back $t = \frac{a}{x}$:

$= -\frac{2}{a} \sqrt{\frac{a}{x} - 1} + C$

$= -\frac{2}{a} \sqrt{\frac{a - x}{x}} + C$

We can also write this as:

$= -\frac{2}{a} \frac{\sqrt{a - x}}{\sqrt{x}} + C$

$= -\frac{2}{a} \frac{\sqrt{a - x}}{\sqrt{x}} \times \frac{\sqrt{x}}{\sqrt{x}} + C$

$= -\frac{2}{a} \frac{\sqrt{x(a - x)}}{x} + C$

$= -\frac{2}{a x} \sqrt{ax - x^2} + C$


The final answer is $\mathbf{-\frac{2}{a} \sqrt{\frac{a - x}{x}} + C}$ or $\mathbf{-\frac{2}{a x} \sqrt{ax - x^2} + C}$.

Question 4. $\frac{1}{x^2 (x^4 + 1)^{\frac{3}{4}}}$

Answer:

Solution:


We need to integrate the function $\frac{1}{x^2 (x^4 + 1)^{\frac{3}{4}}}$.

Let the integral be $I = \int \frac{1}{x^2 (x^4 + 1)^{\frac{3}{4}}} dx$.

We can rewrite the term $(x^4 + 1)^{\frac{3}{4}}$ by factoring out $x^4$ from the expression inside the parenthesis:

$(x^4 + 1)^{\frac{3}{4}} = \left[x^4 \left(1 + \frac{1}{x^4}\right)\right]^{\frac{3}{4}}$

$= (x^4)^{\frac{3}{4}} \left(1 + x^{-4}\right)^{\frac{3}{4}}$

$= x^3 \left(1 + x^{-4}\right)^{\frac{3}{4}}$


Substitute this back into the integrand:

$\frac{1}{x^2 (x^4 + 1)^{\frac{3}{4}}} = \frac{1}{x^2 \cdot x^3 (1 + x^{-4})^{\frac{3}{4}}}$

$= \frac{1}{x^5 (1 + x^{-4})^{\frac{3}{4}}}$


Now the integral becomes $I = \int \frac{1}{x^5 (1 + x^{-4})^{\frac{3}{4}}} dx$.

We can rewrite this as $I = \int (1 + x^{-4})^{-\frac{3}{4}} \cdot x^{-5} dx$.

Let's use the substitution $u = 1 + x^{-4}$.

Differentiate $u$ with respect to $x$:

$\frac{du}{dx} = \frac{d}{dx}(1 + x^{-4}) = 0 + (-4)x^{-5} = -4x^{-5}$

So, $du = -4x^{-5} dx$.

This implies $x^{-5} dx = -\frac{1}{4} du$.


Substitute $u = 1 + x^{-4}$ and $x^{-5} dx = -\frac{1}{4} du$ into the integral:

$I = \int u^{-\frac{3}{4}} \left(-\frac{1}{4} du\right)$

$I = -\frac{1}{4} \int u^{-\frac{3}{4}} du$


Now, integrate with respect to $u$ using the power rule for integration $\int u^n du = \frac{u^{n+1}}{n+1} + C$ (for $n \neq -1$). Here $n = -\frac{3}{4}$.

$I = -\frac{1}{4} \left( \frac{u^{-\frac{3}{4} + 1}}{-\frac{3}{4} + 1} \right) + C'$

$I = -\frac{1}{4} \left( \frac{u^{\frac{1}{4}}}{\frac{1}{4}} \right) + C'$

$I = -\frac{1}{4} \left( 4 u^{\frac{1}{4}} \right) + C'$

$I = -u^{\frac{1}{4}} + C'$


Finally, substitute back $u = 1 + x^{-4}$:

$I = -(1 + x^{-4})^{\frac{1}{4}} + C$

We can simplify $(1 + x^{-4})^{\frac{1}{4}}$:

$(1 + x^{-4})^{\frac{1}{4}} = \left(1 + \frac{1}{x^4}\right)^{\frac{1}{4}} = \left(\frac{x^4 + 1}{x^4}\right)^{\frac{1}{4}} = \frac{(x^4 + 1)^{\frac{1}{4}}}{(x^4)^{\frac{1}{4}}}$

Assuming $x$ is real, $(x^4)^{\frac{1}{4}} = |x|$.

So, $I = -\frac{(x^4 + 1)^{\frac{1}{4}}}{|x|} + C$.

If the domain of integration is restricted to $x > 0$, then $|x| = x$, and the result is $-\frac{(x^4 + 1)^{1/4}}{x} + C$. If the domain is restricted to $x < 0$, then $|x| = -x$, and the result is $-\frac{(x^4 + 1)^{1/4}}{-x} + C = \frac{(x^4 + 1)^{1/4}}{x} + C$. However, typically, when simplifying $(x^n)^{1/n}$ for even $n$, $|x|$ is kept unless the domain is specified. A common convention in such integration problems is to assume the domain where $|x|=x$ for the final form, or just leave it as $-(1+x^{-4})^{1/4}$. Let's use the form $\frac{(x^4 + 1)^{1/4}}{|x|}$.


The final answer is $\mathbf{-\frac{(x^4 + 1)^{\frac{1}{4}}}{|x|} + C}$.

Question 5. $\frac{1}{x^{\frac{1}{2}} + x^{\frac{1}{3}}}$

[Hint : $\frac{1}{x^{\frac{1}{2}} + x^{\frac{1}{3}}} = \frac{1}{x^{\frac{1}{3}} \left(1 + x^{\frac{1}{6}} \right)}$ , put x = t6 ]

Answer:

Solution:


We need to integrate the function $\frac{1}{x^{\frac{1}{2}} + x^{\frac{1}{3}}}$.

Let the integral be $I = \int \frac{1}{x^{\frac{1}{2}} + x^{\frac{1}{3}}} dx$.

We use the given hint and make the substitution $x = t^6$.

From $x = t^6$, we have:

$x^{1/2} = (t^6)^{1/2} = t^3$

$x^{1/3} = (t^6)^{1/3} = t^2$

Now, we find the differential $dx$ in terms of $dt$:

$dx = \frac{d}{dt}(t^6) dt = 6t^5 dt$


Substitute $x$, $x^{1/2}$, $x^{1/3}$, and $dx$ into the integral:

$I = \int \frac{1}{t^3 + t^2} (6t^5 dt)$

$I = \int \frac{6t^5}{t^2(t + 1)} dt$

$I = \int \frac{6t^3}{t + 1} dt$


Now, we perform polynomial division or algebraic manipulation to integrate $\frac{6t^3}{t + 1}$.

$\frac{6t^3}{t + 1} = \frac{6t^3 + 6 - 6}{t+1} = \frac{6(t^3 + 1) - 6}{t+1}$

Using the sum of cubes factorization $t^3 + 1 = (t + 1)(t^2 - t + 1)$:

$= \frac{6(t + 1)(t^2 - t + 1) - 6}{t+1}$

$= 6(t^2 - t + 1) - \frac{6}{t+1}$

$= 6t^2 - 6t + 6 - \frac{6}{t+1}$


Now, integrate this expression with respect to $t$:

$I = \int \left(6t^2 - 6t + 6 - \frac{6}{t+1}\right) dt$

$I = \int 6t^2 dt - \int 6t dt + \int 6 dt - \int \frac{6}{t+1} dt$

Using the power rule $\int t^n dt = \frac{t^{n+1}}{n+1}$ and $\int \frac{1}{u} du = \log_e |u|$:

$I = 6 \left(\frac{t^{2+1}}{2+1}\right) - 6 \left(\frac{t^{1+1}}{1+1}\right) + 6t - 6 \log_e |t+1| + C$

$I = 6 \left(\frac{t^3}{3}\right) - 6 \left(\frac{t^2}{2}\right) + 6t - 6 \log_e |t+1| + C$

$I = 2t^3 - 3t^2 + 6t - 6 \log_e |t+1| + C$


Finally, substitute back $t = x^{1/6}$:

$t^2 = (x^{1/6})^2 = x^{2/6} = x^{1/3}$

$t^3 = (x^{1/6})^3 = x^{3/6} = x^{1/2}$

$I = 2(x^{1/2}) - 3(x^{1/3}) + 6(x^{1/6}) - 6 \log_e |x^{1/6}+1| + C$

$I = 2\sqrt{x} - 3x^{1/3} + 6x^{1/6} - 6 \log_e (x^{1/6}+1) + C$ (since $x^{1/6} > 0$ for real $x>0$, $|x^{1/6}+1| = x^{1/6}+1$)


The final answer is $\mathbf{2\sqrt{x} - 3x^{1/3} + 6x^{1/6} - 6 \log_e (x^{1/6}+1) + C}$.

Question 6. $\frac{5x}{(x + 1) (x^2 + 9)}$

Answer:

Solution:


We need to integrate the function $\frac{5x}{(x + 1) (x^2 + 9)}$.

Let the integral be $I = \int \frac{5x}{(x + 1) (x^2 + 9)} dx$.

We will use the method of partial fraction decomposition since the integrand is a rational function.

The denominator has a linear factor $(x+1)$ and an irreducible quadratic factor $(x^2 + 9)$.

We decompose the integrand into the following form:

$\frac{5x}{(x + 1) (x^2 + 9)} = \frac{A}{x+1} + \frac{Bx+C}{x^2+9}$


To find the constants A, B, and C, we multiply both sides by $(x + 1) (x^2 + 9)$:

$5x = A(x^2 + 9) + (Bx+C)(x+1)$


We can find the constants by substituting values of $x$ or by comparing coefficients.

Using substitution:

Let $x = -1$:

$5(-1) = A((-1)^2 + 9) + (B(-1)+C)(-1+1)$

$-5 = A(1 + 9) + (-B+C)(0)$

$-5 = 10A$

$A = -\frac{5}{10} = -\frac{1}{2}$


Using comparison of coefficients (or substitute other values, e.g., $x=0$ and $x=1$):

Expand the right side: $5x = Ax^2 + 9A + Bx^2 + Bx + Cx + C$

$5x = (A+B)x^2 + (B+C)x + (9A+C)$

Comparing coefficients of $x^2$:

$0 = A+B$

Substituting $A = -\frac{1}{2}$: $0 = -\frac{1}{2} + B \implies B = \frac{1}{2}$

Comparing coefficients of $x$:

$5 = B+C$

Substituting $B = \frac{1}{2}$: $5 = \frac{1}{2} + C \implies C = 5 - \frac{1}{2} = \frac{10 - 1}{2} = \frac{9}{2}$

Comparing constant terms (check):

$0 = 9A + C$

$0 = 9\left(-\frac{1}{2}\right) + \frac{9}{2} = -\frac{9}{2} + \frac{9}{2} = 0$. The values are consistent.


So the partial fraction decomposition is:

$\frac{5x}{(x + 1) (x^2 + 9)} = \frac{-1/2}{x+1} + \frac{(1/2)x + 9/2}{x^2+9}$

$= -\frac{1}{2(x+1)} + \frac{x}{2(x^2+9)} + \frac{9}{2(x^2+9)}$


Now we integrate each term:

$I = \int \left(-\frac{1}{2(x+1)} + \frac{x}{2(x^2+9)} + \frac{9}{2(x^2+9)}\right) dx$

$I = -\frac{1}{2} \int \frac{1}{x+1} dx + \frac{1}{2} \int \frac{x}{x^2+9} dx + \frac{9}{2} \int \frac{1}{x^2+9} dx$


First integral: $\int \frac{1}{x+1} dx = \log_e |x+1|$


Second integral: $\int \frac{x}{x^2+9} dx$. Let $u = x^2+9$, then $du = 2x dx \implies x dx = \frac{1}{2} du$.

$\int \frac{x}{x^2+9} dx = \int \frac{1}{u} \cdot \frac{1}{2} du = \frac{1}{2} \int \frac{1}{u} du = \frac{1}{2} \log_e |u| = \frac{1}{2} \log_e (x^2+9)$ (since $x^2+9 > 0$).


Third integral: $\int \frac{1}{x^2+9} dx = \int \frac{1}{x^2+3^2} dx$. This is of the form $\int \frac{1}{x^2+a^2} dx = \frac{1}{a} \tan^{-1}\left(\frac{x}{a}\right)$. Here $a=3$.

$\int \frac{1}{x^2+9} dx = \frac{1}{3} \tan^{-1}\left(\frac{x}{3}\right)$


Combine the results and add the constant of integration C:

$I = -\frac{1}{2} \log_e |x+1| + \frac{1}{2} \left(\frac{1}{2} \log_e (x^2+9)\right) + \frac{9}{2} \left(\frac{1}{3} \tan^{-1}\left(\frac{x}{3}\right)\right) + C$

$I = -\frac{1}{2} \log_e |x+1| + \frac{1}{4} \log_e (x^2+9) + \frac{3}{2} \tan^{-1}\left(\frac{x}{3}\right) + C$


The final answer is $\mathbf{-\frac{1}{2} \log_e |x+1| + \frac{1}{4} \log_e (x^2+9) + \frac{3}{2} \tan^{-1}\left(\frac{x}{3}\right) + C}$.

Question 7. $\frac{\sin x}{\sin (x − a)}$

Answer:

Solution:


We need to integrate the function $\frac{\sin x}{\sin (x − a)}$.

Let the integral be $I = \int \frac{\sin x}{\sin (x − a)} dx$.

We can use a substitution to simplify the denominator. Let $u = x - a$.

Then $x = u + a$.

Differentiating with respect to $x$, we get $\frac{du}{dx} = 1$, so $du = dx$.


Now substitute $x = u + a$ and $dx = du$ into the integral:

$I = \int \frac{\sin (u + a)}{\sin u} du$

Use the sine addition formula: $\sin(u + a) = \sin u \cos a + \cos u \sin a$.

$I = \int \frac{\sin u \cos a + \cos u \sin a}{\sin u} du$

Split the fraction into two terms:

$I = \int \left(\frac{\sin u \cos a}{\sin u} + \frac{\cos u \sin a}{\sin u}\right) du$

$I = \int \left(\cos a + \sin a \frac{\cos u}{\sin u}\right) du$

$I = \int (\cos a + \sin a \cot u) du$


Now, integrate with respect to $u$. Note that $\cos a$ and $\sin a$ are constants with respect to $u$ because $a$ is a constant.

$I = \int \cos a du + \int \sin a \cot u du$

$I = \cos a \int du + \sin a \int \cot u du$

We know that $\int du = u$ and $\int \cot u du = \log_e |\sin u|$.

$I = (\cos a) u + (\sin a) \log_e |\sin u| + C$


Finally, substitute back $u = x - a$:

$I = (\cos a) (x - a) + (\sin a) \log_e |\sin (x - a)| + C$

We can rewrite $(\cos a)(x-a)$ as $x \cos a - a \cos a$. The term $-a \cos a$ is a constant and can be absorbed into the integration constant C. Let $C' = C - a \cos a$.

$I = x \cos a + \sin a \log_e |\sin (x - a)| + C'$


The final answer is $\mathbf{x \cos a + \sin a \log_e |\sin (x - a)| + C}$.

Question 8. $\frac{e^{5 \log x} − e^{4 \log x}}{e^{3 \log x} − e^{2 \log x}}$

Answer:

Solution:


We need to integrate the function $\frac{e^{5 \log x} − e^{4 \log x}}{e^{3 \log x} − e^{2 \log x}}$.

Recall the logarithm property: $b \log x = \log (x^b)$.

Also, recall the exponential property: $e^{\log u} = u$ (assuming $u > 0$).


Apply these properties to the terms in the expression:

$e^{5 \log x} = e^{\log (x^5)} = x^5$

$e^{4 \log x} = e^{\log (x^4)} = x^4$

$e^{3 \log x} = e^{\log (x^3)} = x^3$

$e^{2 \log x} = e^{\log (x^2)} = x^2$

Note: For $e^{\log u}$ to be defined, $u$ must be positive. Thus, this simplification is valid for $x > 0$.


Substitute these back into the function:

$\frac{e^{5 \log x} − e^{4 \log x}}{e^{3 \log x} − e^{2 \log x}} = \frac{x^5 − x^4}{x^3 − x^2}$

Now, factor the numerator and the denominator:

Numerator: $x^5 - x^4 = x^4(x - 1)$

Denominator: $x^3 - x^2 = x^2(x - 1)$

So the expression becomes:

$\frac{x^4(x - 1)}{x^2(x - 1)}$

Assuming $x \neq 0$ and $x \neq 1$, we can cancel the $(x-1)$ terms and simplify the powers of $x$:

$\frac{x^4}{x^2} = x^{4-2} = x^2$

So, for $x > 0$ and $x \neq 1$, the integrand simplifies to $x^2$.


Now we need to integrate $x^2$ with respect to $x$:

$\int x^2 dx$

Using the power rule for integration $\int x^n dx = \frac{x^{n+1}}{n+1} + C$ (for $n \neq -1$):

$\int x^2 dx = \frac{x^{2+1}}{2+1} + C$

$= \frac{x^3}{3} + C$


The final answer is $\mathbf{\frac{x^3}{3} + C}$.

Question 9. $\frac{\cos x}{\sqrt{4 − \sin^2 x}}$

Answer:

Solution:


We need to integrate the function $\frac{\cos x}{\sqrt{4 − \sin^2 x}}$.

Let the integral be $I = \int \frac{\cos x}{\sqrt{4 − \sin^2 x}} dx$.

We can use a substitution to simplify the integrand. Let $u = \sin x$.

Differentiate $u$ with respect to $x$:

$\frac{du}{dx} = \frac{d}{dx}(\sin x) = \cos x$

So, $du = \cos x dx$.


Now, substitute $u = \sin x$ and $\cos x dx = du$ into the integral:

$I = \int \frac{du}{\sqrt{4 − u^2}}$

The integral is now in the form $\int \frac{du}{\sqrt{a^2 - u^2}}$, where $a^2 = 4$, so $a = 2$.

Recall the standard integral formula: $\int \frac{du}{\sqrt{a^2 - u^2}} = \sin^{-1}\left(\frac{u}{a}\right) + C$.

Using $a = 2$:

$I = \sin^{-1}\left(\frac{u}{2}\right) + C$


Finally, substitute back $u = \sin x$:

$I = \sin^{-1}\left(\frac{\sin x}{2}\right) + C$


The domain of the arcsin function is $[-1, 1]$. This implies that $-1 \le \frac{\sin x}{2} \le 1$, which means $-2 \le \sin x \le 2$. This condition is always satisfied since the range of $\sin x$ is $[-1, 1]$. Also, for the square root $\sqrt{4 - \sin^2 x}$ to be real, $4 - \sin^2 x \ge 0$, which means $\sin^2 x \le 4$, also always true. If $4 - \sin^2 x = 0$, the integrand is undefined.


The final answer is $\mathbf{\sin^{-1}\left(\frac{\sin x}{2}\right) + C}$.

Question 10. $\frac{\sin^8 x − \cos^8 x}{1 − 2\sin^2 x \;\cos^2 x}$

Answer:

Solution:


We need to integrate the function $\frac{\sin^8 x − \cos^8 x}{1 − 2\sin^2 x \;\cos^2 x}$.

Let the integral be $I = \int \frac{\sin^8 x − \cos^8 x}{1 − 2\sin^2 x \;\cos^2 x} dx$.

First, simplify the numerator using the difference of squares formula repeatedly: $a^2 - b^2 = (a-b)(a+b)$.

$\sin^8 x − \cos^8 x = (\sin^4 x)^2 − (\cos^4 x)^2$

$= (\sin^4 x − \cos^4 x)(\sin^4 x + \cos^4 x)$

Now apply difference of squares to the first factor:

$\sin^4 x − \cos^4 x = (\sin^2 x)^2 − (\cos^2 x)^2 = (\sin^2 x − \cos^2 x)(\sin^2 x + \cos^2 x)$

Using the identity $\sin^2 x + \cos^2 x = 1$:

$= (\sin^2 x − \cos^2 x)(1) = \sin^2 x − \cos^2 x$

Now consider the second factor of the numerator: $\sin^4 x + \cos^4 x$.

We know that $(\sin^2 x + \cos^2 x)^2 = \sin^4 x + 2\sin^2 x \cos^2 x + \cos^4 x$.

So, $\sin^4 x + \cos^4 x = (\sin^2 x + \cos^2 x)^2 - 2\sin^2 x \cos^2 x$

Using the identity $\sin^2 x + \cos^2 x = 1$:

$= (1)^2 - 2\sin^2 x \cos^2 x = 1 - 2\sin^2 x \cos^2 x$


Substitute these simplified factors back into the numerator $\sin^8 x − \cos^8 x$:

$\sin^8 x − \cos^8 x = (\sin^2 x − \cos^2 x)(1 - 2\sin^2 x \cos^2 x)$


Now, substitute this back into the integrand:

$\frac{\sin^8 x − \cos^8 x}{1 − 2\sin^2 x \;\cos^2 x} = \frac{(\sin^2 x − \cos^2 x)(1 - 2\sin^2 x \cos^2 x)}{1 − 2\sin^2 x \;\cos^2 x}$

Assuming $1 - 2\sin^2 x \cos^2 x \neq 0$, we can cancel the term $(1 - 2\sin^2 x \cos^2 x)$:

$= \sin^2 x − \cos^2 x$

Recall the double angle identity for cosine: $\cos(2x) = \cos^2 x - \sin^2 x = -(\sin^2 x - \cos^2 x)$.

So, $\sin^2 x − \cos^2 x = -\cos(2x)$.


Now we need to integrate $-\cos(2x)$ with respect to $x$:

$I = \int -\cos(2x) dx$

$I = - \int \cos(2x) dx$

Let $u = 2x$, then $du = 2 dx \implies dx = \frac{1}{2} du$.

$I = - \int \cos u \left(\frac{1}{2} du\right)$

$I = -\frac{1}{2} \int \cos u du$

We know that $\int \cos u du = \sin u$.

$I = -\frac{1}{2} \sin u + C$


Finally, substitute back $u = 2x$:

$I = -\frac{1}{2} \sin(2x) + C$


The final answer is $\mathbf{-\frac{1}{2} \sin(2x) + C}$.

Question 11. $\frac{1}{\cos(x + a) \;\cos(x + b)}$

Answer:

Solution:


We need to integrate the function $\frac{1}{\cos(x + a) \;\cos(x + b)}$.

Let the integral be $I = \int \frac{1}{\cos(x + a) \;\cos(x + b)} dx$.

This type of integral can be solved by multiplying the numerator and denominator by $\sin(a - b)$ (assuming $a \neq b$).

Multiply the numerator and denominator by $\sin(a-b)$:

$I = \int \frac{\sin(a - b)}{\sin(a - b) \cos(x + a) \cos(x + b)} dx$

$I = \frac{1}{\sin(a - b)} \int \frac{\sin(a - b)}{\cos(x + a) \cos(x + b)} dx$


Rewrite the term $\sin(a - b)$ in the numerator as $\sin((x + a) - (x + b))$:

$I = \frac{1}{\sin(a - b)} \int \frac{\sin((x + a) - (x + b))}{\cos(x + a) \cos(x + b)} dx$

Use the sine difference formula: $\sin(P - Q) = \sin P \cos Q - \cos P \sin Q$. Let $P = x + a$ and $Q = x + b$.

$I = \frac{1}{\sin(a - b)} \int \frac{\sin(x + a) \cos(x + b) - \cos(x + a) \sin(x + b)}{\cos(x + a) \cos(x + b)} dx$


Split the fraction into two terms:

$I = \frac{1}{\sin(a - b)} \int \left(\frac{\sin(x + a) \cos(x + b)}{\cos(x + a) \cos(x + b)} - \frac{\cos(x + a) \sin(x + b)}{\cos(x + a) \cos(x + b)}\right) dx$

$I = \frac{1}{\sin(a - b)} \int \left(\frac{\sin(x + a)}{\cos(x + a)} - \frac{\sin(x + b)}{\cos(x + b)}\right) dx$

$I = \frac{1}{\sin(a - b)} \int (\tan(x + a) - \tan(x + b)) dx$


Now, integrate each term:

$I = \frac{1}{\sin(a - b)} \left( \int \tan(x + a) dx - \int \tan(x + b) dx \right)$

We know that $\int \tan u du = \log_e |\sec u| + C_1$ or $-\log_e |\cos u| + C_2$. Let's use the $-\log_e |\cos u|$ form.

$\int \tan(x + a) dx = -\log_e |\cos(x + a)|$

$\int \tan(x + b) dx = -\log_e |\cos(x + b)|$


Substitute these results back into the integral expression:

$I = \frac{1}{\sin(a - b)} \left( -\log_e |\cos(x + a)| - (-\log_e |\cos(x + b)|) \right) + C$

$I = \frac{1}{\sin(a - b)} \left( -\log_e |\cos(x + a)| + \log_e |\cos(x + b)| \right) + C$

Using the logarithm property $\log_e M - \log_e N = \log_e \frac{M}{N}$:

$I = \frac{1}{\sin(a - b)} \log_e \left| \frac{\cos(x + b)}{\cos(x + a)} \right| + C$


Note: This solution is valid when $\sin(a - b) \neq 0$, i.e., $a - b \neq n\pi$ for any integer $n$. If $a - b = n\pi$, then $a = b + n\pi$. In this case, $\cos(x+a) = \cos(x+b+n\pi) = \cos(x+b)\cos(n\pi) - \sin(x+b)\sin(n\pi) = \cos(x+b)(\pm 1) - 0 = \pm \cos(x+b)$. The original integral becomes $\int \frac{1}{\pm \cos^2(x+b)} dx = \pm \int \sec^2(x+b) dx = \pm \tan(x+b) + C$. The formula derived above would involve division by zero $\sin(a-b)=0$. So the formula is for the case $a-b \neq n\pi$.


The final answer is $\mathbf{\frac{1}{\sin(a - b)} \log_e \left| \frac{\cos(x + b)}{\cos(x + a)} \right| + C}$.

Question 12. $\frac{x^3}{\sqrt{1 − x^8}}$

Answer:

Solution:


We need to integrate the function $\frac{x^3}{\sqrt{1 − x^8}}$.

Let the integral be $I = \int \frac{x^3}{\sqrt{1 − x^8}} dx$.

We notice that the derivative of $x^4$ is $4x^3$. This suggests a substitution involving $x^4$.

Let $u = x^4$.

Differentiate $u$ with respect to $x$:

$\frac{du}{dx} = \frac{d}{dx}(x^4) = 4x^3$

So, $du = 4x^3 dx$, which means $x^3 dx = \frac{1}{4} du$.


Also, $x^8 = (x^4)^2 = u^2$.

Now, substitute $u = x^4$, $x^8 = u^2$, and $x^3 dx = \frac{1}{4} du$ into the integral:

$I = \int \frac{1}{\sqrt{1 − u^2}} \left(\frac{1}{4} du\right)$

$I = \frac{1}{4} \int \frac{1}{\sqrt{1 − u^2}} du$

This integral is in the standard form $\int \frac{1}{\sqrt{a^2 - u^2}} du$, where $a^2 = 1$, so $a = 1$.

Recall the standard integral formula: $\int \frac{du}{\sqrt{a^2 - u^2}} = \sin^{-1}\left(\frac{u}{a}\right) + C'$.

Using $a = 1$:

$I = \frac{1}{4} \sin^{-1}\left(\frac{u}{1}\right) + C'$

$I = \frac{1}{4} \sin^{-1}(u) + C'$


Finally, substitute back $u = x^4$:

$I = \frac{1}{4} \sin^{-1}(x^4) + C$


For the integral to be real, we require $1 - x^8 > 0$, which means $x^8 < 1$, or $-1 < x < 1$. Also, for the arcsin function, the argument must be in $[-1, 1]$. Since $u = x^4$, $u \ge 0$. So we need $0 \le u \le 1$, which means $0 \le x^4 \le 1$. This gives $-1 \le x \le 1$. The domain of the function $\frac{x^3}{\sqrt{1 − x^8}}$ requires $1-x^8 > 0$, so $-1 < x < 1$. The result is valid for this domain.


The final answer is $\mathbf{\frac{1}{4} \sin^{-1}(x^4) + C}$.

Question 13. $\frac{e^x}{(1 + e^x) (2 + e^x)}$

Answer:

Solution:


We need to integrate the function $\frac{e^x}{(1 + e^x) (2 + e^x)}$.

Let the integral be $I = \int \frac{e^x}{(1 + e^x) (2 + e^x)} dx$.

We can use a substitution to simplify the integrand. Let $u = e^x$.

Differentiate $u$ with respect to $x$:

$\frac{du}{dx} = \frac{d}{dx}(e^x) = e^x$

So, $du = e^x dx$.


Substitute $u = e^x$ and $e^x dx = du$ into the integral:

$I = \int \frac{du}{(1 + u) (2 + u)}$

Now we can use the method of partial fraction decomposition for the rational function $\frac{1}{(1 + u) (2 + u)}$.

Set up the decomposition:

$\frac{1}{(1 + u) (2 + u)} = \frac{A}{1 + u} + \frac{B}{2 + u}$


Multiply both sides by $(1 + u) (2 + u)$:

$1 = A(2 + u) + B(1 + u)$


Substitute values of $u$ to find A and B.

Let $u = -1$:

$1 = A(2 - 1) + B(1 - 1)$

$1 = A(1) + B(0)$

$A = 1$


Let $u = -2$:

$1 = A(2 - 2) + B(1 - 2)$

$1 = A(0) + B(-1)$

$1 = -B$

$B = -1$


So the partial fraction decomposition is:

$\frac{1}{(1 + u) (2 + u)} = \frac{1}{1 + u} + \frac{-1}{2 + u} = \frac{1}{1 + u} - \frac{1}{2 + u}$


Now, integrate this expression with respect to $u$:

$I = \int \left(\frac{1}{1 + u} - \frac{1}{2 + u}\right) du$

$I = \int \frac{1}{1 + u} du - \int \frac{1}{2 + u} du$

Using the integral $\int \frac{1}{w} dw = \log_e |w|$:

$\int \frac{1}{1 + u} du = \log_e |1 + u|$

$\int \frac{1}{2 + u} du = \log_e |2 + u|$


Combine the results and add the constant of integration C':

$I = \log_e |1 + u| - \log_e |2 + u| + C'$

Using the logarithm property $\log_e M - \log_e N = \log_e \frac{M}{N}$:

$I = \log_e \left|\frac{1 + u}{2 + u}\right| + C'$


Finally, substitute back $u = e^x$. Since $e^x > 0$ for all real $x$, $1 + e^x$ and $2 + e^x$ are always positive. So we can remove the absolute value signs.

$I = \log_e \left(\frac{1 + e^x}{2 + e^x}\right) + C$


The final answer is $\mathbf{\log_e \left(\frac{1 + e^x}{2 + e^x}\right) + C}$.

Question 14. $\frac{1}{(x^2 + 1) (x^2 + 4)}$

Answer:

Solution:


We need to integrate the function $\frac{1}{(x^2 + 1) (x^2 + 4)}$.

Let the integral be $I = \int \frac{1}{(x^2 + 1) (x^2 + 4)} dx$.

We use the method of partial fraction decomposition. Although the factors in the denominator are quadratic, we can treat $x^2$ as a variable temporarily for decomposition purposes.

Set up the decomposition:

$\frac{1}{(x^2 + 1) (x^2 + 4)} = \frac{A}{x^2 + 1} + \frac{B}{x^2 + 4}$


Multiply both sides by $(x^2 + 1) (x^2 + 4)$:

$1 = A(x^2 + 4) + B(x^2 + 1)$

$1 = Ax^2 + 4A + Bx^2 + B$

$1 = (A+B)x^2 + (4A+B)$


Equating the coefficients of $x^2$ and the constant terms:

Coefficient of $x^2$: $0 = A + B$ ... (1)

Constant term: $1 = 4A + B$ ... (2)


From equation (1), $B = -A$. Substitute this into equation (2):

$1 = 4A + (-A)$

$1 = 3A$

$A = \frac{1}{3}$

Now find B using $B = -A$:

$B = -\frac{1}{3}$


So the partial fraction decomposition is:

$\frac{1}{(x^2 + 1) (x^2 + 4)} = \frac{1/3}{x^2 + 1} + \frac{-1/3}{x^2 + 4}$

$= \frac{1}{3(x^2 + 1)} - \frac{1}{3(x^2 + 4)}$


Now, integrate each term:

$I = \int \left(\frac{1}{3(x^2 + 1)} - \frac{1}{3(x^2 + 4)}\right) dx$

$I = \frac{1}{3} \int \frac{1}{x^2 + 1} dx - \frac{1}{3} \int \frac{1}{x^2 + 4} dx$


We use the standard integral formula $\int \frac{1}{x^2 + a^2} dx = \frac{1}{a} \tan^{-1}\left(\frac{x}{a}\right)$.

For the first integral, $a^2 = 1$, so $a = 1$:

$\int \frac{1}{x^2 + 1} dx = \frac{1}{1} \tan^{-1}\left(\frac{x}{1}\right) = \tan^{-1}(x)$

For the second integral, $a^2 = 4$, so $a = 2$:

$\int \frac{1}{x^2 + 4} dx = \int \frac{1}{x^2 + 2^2} dx = \frac{1}{2} \tan^{-1}\left(\frac{x}{2}\right)$


Combine the results and add the constant of integration C:

$I = \frac{1}{3} \tan^{-1}(x) - \frac{1}{3} \left(\frac{1}{2} \tan^{-1}\left(\frac{x}{2}\right)\right) + C$

$I = \frac{1}{3} \tan^{-1}(x) - \frac{1}{6} \tan^{-1}\left(\frac{x}{2}\right) + C$


The final answer is $\mathbf{\frac{1}{3} \tan^{-1}(x) - \frac{1}{6} \tan^{-1}\left(\frac{x}{2}\right) + C}$.

Question 15. cos3 x elog sin x

Answer:

Solution:


We need to integrate the function $\cos^3 x e^{\log \sin x}$.

Let the integral be $I = \int \cos^3 x e^{\log \sin x} dx$.

Recall the property $e^{\log u} = u$, provided $u > 0$.

In this case, $u = \sin x$. For $\log \sin x$ to be defined and real, $\sin x > 0$. This means $x$ must be in intervals like $(2n\pi, (2n+1)\pi)$ for integer $n$. For such intervals, $e^{\log \sin x} = \sin x$.


Substitute this into the integrand:

$\cos^3 x e^{\log \sin x} = \cos^3 x \sin x$


Now we need to integrate $\int \cos^3 x \sin x dx$.

We can use a substitution here. Let $u = \cos x$.

Differentiate $u$ with respect to $x$:

$\frac{du}{dx} = \frac{d}{dx}(\cos x) = -\sin x$

So, $du = -\sin x dx$, which means $\sin x dx = -du$.


Substitute $u = \cos x$ and $\sin x dx = -du$ into the integral:

$I = \int u^3 (-du)$

$I = -\int u^3 du$


Integrate with respect to $u$ using the power rule $\int u^n du = \frac{u^{n+1}}{n+1} + C'$.

$I = -\left(\frac{u^{3+1}}{3+1}\right) + C'$

$I = -\frac{u^4}{4} + C'$


Finally, substitute back $u = \cos x$:

$I = -\frac{(\cos x)^4}{4} + C$

$I = -\frac{1}{4} \cos^4 x + C$


The final answer is $\mathbf{-\frac{1}{4} \cos^4 x + C}$.

Question 16. e3 log x (x4 + 1)–1

Answer:

Solution:


We need to integrate the function $e^{3 \log x} (x^4 + 1)^{-1}$.

Let the integral be $I = \int e^{3 \log x} (x^4 + 1)^{-1} dx$.

Rewrite the terms using logarithm and exponential properties:

$e^{3 \log x} = e^{\log (x^3)}$

Using the property $e^{\log u} = u$ (for $u > 0$), we have $e^{\log (x^3)} = x^3$. This is valid for $x^3 > 0$, which implies $x > 0$.

$(x^4 + 1)^{-1} = \frac{1}{x^4 + 1}$


Substitute these back into the integrand:

$e^{3 \log x} (x^4 + 1)^{-1} = x^3 \cdot \frac{1}{x^4 + 1} = \frac{x^3}{x^4 + 1}$


Now we need to integrate $\int \frac{x^3}{x^4 + 1} dx$.

We can use a substitution here. Notice that the derivative of the denominator is related to the numerator.

Let $u = x^4 + 1$.

Differentiate $u$ with respect to $x$:

$\frac{du}{dx} = \frac{d}{dx}(x^4 + 1) = 4x^3$

So, $du = 4x^3 dx$, which means $x^3 dx = \frac{1}{4} du$.


Substitute $u = x^4 + 1$ and $x^3 dx = \frac{1}{4} du$ into the integral:

$I = \int \frac{1}{u} \left(\frac{1}{4} du\right)$

$I = \frac{1}{4} \int \frac{1}{u} du$


Integrate with respect to $u$ using the formula $\int \frac{1}{u} du = \log_e |u|$.

$I = \frac{1}{4} \log_e |u| + C'$


Finally, substitute back $u = x^4 + 1$. Since $x^4 \ge 0$, $x^4 + 1 \ge 1 > 0$, so $|x^4 + 1| = x^4 + 1$.

$I = \frac{1}{4} \log_e (x^4 + 1) + C$


The final answer is $\mathbf{\frac{1}{4} \log_e (x^4 + 1) + C}$.

Question 17. f ′ (ax + b) [f (ax + b)]n

Answer:

Solution:


We need to integrate the function $f'(ax + b) [f (ax + b)]^n$.

Let the integral be $I = \int f'(ax + b) [f (ax + b)]^n dx$.

This integral can be solved using a substitution.

Let $u = f(ax + b)$.

To find $du$, we differentiate $u$ with respect to $x$ using the chain rule.

$\frac{du}{dx} = \frac{d}{dx}[f(ax + b)]$

Let $y = ax + b$. Then $u = f(y)$. By the chain rule, $\frac{du}{dx} = \frac{du}{dy} \cdot \frac{dy}{dx}$.

$\frac{du}{dy} = f'(y) = f'(ax + b)$

$\frac{dy}{dx} = \frac{d}{dx}(ax + b) = a$

So, $\frac{du}{dx} = f'(ax + b) \cdot a = a f'(ax + b)$.

This means $du = a f'(ax + b) dx$.

Rearranging for $f'(ax + b) dx$, we get $f'(ax + b) dx = \frac{1}{a} du$ (assuming $a \neq 0$).


Substitute $u = f(ax + b)$ and $f'(ax + b) dx = \frac{1}{a} du$ into the integral:

$I = \int u^n \left(\frac{1}{a} du\right)$

$I = \frac{1}{a} \int u^n du$


Now, integrate with respect to $u$. The integration depends on whether $n = -1$ or $n \neq -1$.

Case 1: $n \neq -1$

Using the power rule $\int u^n du = \frac{u^{n+1}}{n+1} + C'$:

$I = \frac{1}{a} \left(\frac{u^{n+1}}{n+1}\right) + C'$

$I = \frac{u^{n+1}}{a(n+1)} + C'$

Substitute back $u = f(ax + b)$:

$I = \frac{[f(ax + b)]^{n+1}}{a(n+1)} + C$


Case 2: $n = -1$

In this case, the integral is $I = \frac{1}{a} \int u^{-1} du = \frac{1}{a} \int \frac{1}{u} du$.

Using the integral $\int \frac{1}{u} du = \log_e |u| + C'$:

$I = \frac{1}{a} \log_e |u| + C'$

Substitute back $u = f(ax + b)$:

$I = \frac{1}{a} \log_e |f(ax + b)| + C$


Summary of results:

If $n \neq -1$ and $a \neq 0$, $I = \frac{[f(ax + b)]^{n+1}}{a(n+1)} + C$.

If $n = -1$ and $a \neq 0$, $I = \frac{1}{a} \log_e |f(ax + b)| + C$.

If $a = 0$, the original function becomes $f'(b) [f(b)]^n$, which is a constant. The integral of a constant $k$ is $kx + C$. So, if $a=0$, $I = f'(b) [f(b)]^n x + C$. However, the structure of the problem strongly suggests $a \neq 0$.


Assuming $n \neq -1$ and $a \neq 0$, the final answer is $\mathbf{\frac{[f(ax + b)]^{n+1}}{a(n+1)} + C}$.

Assuming $n = -1$ and $a \neq 0$, the final answer is $\mathbf{\frac{1}{a} \log_e |f(ax + b)| + C}$.

The provided question does not specify the value of $n$, so both cases should ideally be mentioned or the case $n \neq -1$ is the intended one if a single formula is expected.

Let's provide the general case for $n \neq -1$ as it aligns with the power function notation.


The final answer (for $n \neq -1, a \neq 0$) is $\mathbf{\frac{[f(ax + b)]^{n+1}}{a(n+1)} + C}$.

Question 18. $\frac{1}{\sqrt{\sin^3 x \sin (x + \alpha)}}$

Answer:

Solution:


We need to integrate the function $\frac{1}{\sqrt{\sin^3 x \sin (x + \alpha)}}$.

Let the integral be $I = \int \frac{1}{\sqrt{\sin^3 x \sin (x + \alpha)}} dx$.

First, expand $\sin (x + \alpha)$ using the identity $\sin(A+B) = \sin A \cos B + \cos A \sin B$:

$\sin(x + \alpha) = \sin x \cos \alpha + \cos x \sin \alpha$

Substitute this into the integrand:

$\frac{1}{\sqrt{\sin^3 x (\sin x \cos \alpha + \cos x \sin \alpha)}}$

$= \frac{1}{\sqrt{\sin^4 x \cos \alpha + \sin^3 x \cos x \sin \alpha}}$

Factor out $\sin^4 x$ from the expression under the square root:

$= \frac{1}{\sqrt{\sin^4 x (\cos \alpha + \frac{\cos x \sin \alpha}{\sin x})}}$

$= \frac{1}{\sqrt{\sin^4 x (\cos \alpha + \cot x \sin \alpha)}}$

Assuming we are on an interval where $\sin x > 0$, $\sqrt{\sin^4 x} = |\sin^2 x| = \sin^2 x$.

$= \frac{1}{\sin^2 x \sqrt{\cos \alpha + \cot x \sin \alpha}}$

Recall that $\frac{1}{\sin^2 x} = \text{cosec}^2 x$.

So the integrand becomes $\frac{\text{cosec}^2 x}{\sqrt{\cos \alpha + \cot x \sin \alpha}}$.


Now, let's use the substitution method. Let $u = \cos \alpha + \cot x \sin \alpha$.

Differentiate $u$ with respect to $x$. Note that $\cos \alpha$ and $\sin \alpha$ are constants.

$\frac{du}{dx} = \frac{d}{dx}(\cos \alpha) + \frac{d}{dx}(\cot x \sin \alpha)$

$= 0 + \sin \alpha \frac{d}{dx}(\cot x)$

We know that $\frac{d}{dx}(\cot x) = -\text{cosec}^2 x$.

$\frac{du}{dx} = \sin \alpha (-\text{cosec}^2 x) = -\sin \alpha \text{cosec}^2 x$

So, $du = -\sin \alpha \text{cosec}^2 x dx$ (assuming $\sin \alpha \neq 0$).

This implies $\text{cosec}^2 x dx = -\frac{1}{\sin \alpha} du$.


Substitute $u = \cos \alpha + \cot x \sin \alpha$ and $\text{cosec}^2 x dx = -\frac{1}{\sin \alpha} du$ into the integral:

$I = \int \frac{1}{\sqrt{u}} \left(-\frac{1}{\sin \alpha} du\right)$

$I = -\frac{1}{\sin \alpha} \int u^{-1/2} du$


Integrate $u^{-1/2}$ with respect to $u$ using the power rule $\int u^n du = \frac{u^{n+1}}{n+1} + C'$:

$I = -\frac{1}{\sin \alpha} \left(\frac{u^{-1/2 + 1}}{-1/2 + 1}\right) + C'$

$I = -\frac{1}{\sin \alpha} \left(\frac{u^{1/2}}{1/2}\right) + C'$

$I = -\frac{1}{\sin \alpha} (2 u^{1/2}) + C'$

$I = -\frac{2}{\sin \alpha} \sqrt{u} + C'$


Finally, substitute back $u = \cos \alpha + \cot x \sin \alpha$:

$I = -\frac{2}{\sin \alpha} \sqrt{\cos \alpha + \cot x \sin \alpha} + C$

We can rewrite the expression under the square root:

$\cos \alpha + \cot x \sin \alpha = \cos \alpha + \frac{\cos x}{\sin x} \sin \alpha = \frac{\sin x \cos \alpha + \cos x \sin \alpha}{\sin x} = \frac{\sin(x + \alpha)}{\sin x}$.

So, $I = -\frac{2}{\sin \alpha} \sqrt{\frac{\sin(x + \alpha)}{\sin x}} + C$.


This solution is valid assuming $\sin x > 0$ and $\sin \alpha \neq 0$. The term under the square root $\sin^3 x \sin(x+\alpha)$ must also be non-negative for a real result, which along with $\sin x > 0$ implies $\sin(x+\alpha) \ge 0$.


The final answer is $\mathbf{-\frac{2}{\sin \alpha} \sqrt{\frac{\sin(x + \alpha)}{\sin x}} + C}$.

Question 19. $\sqrt{\frac{1 − \sqrt{x}}{1 + \sqrt{x}}}$

Answer:

Solution:


We need to integrate the function $\sqrt{\frac{1 − \sqrt{x}}{1 + \sqrt{x}}}$.

Let the integral be $I = \int \sqrt{\frac{1 − \sqrt{x}}{1 + \sqrt{x}}} dx$.

We can rationalize the numerator inside the square root:

$\sqrt{\frac{1 − \sqrt{x}}{1 + \sqrt{x}}} = \sqrt{\frac{1 − \sqrt{x}}{1 + \sqrt{x}} \times \frac{1 - \sqrt{x}}{1 - \sqrt{x}}}$

$= \sqrt{\frac{(1 - \sqrt{x})^2}{1 - (\sqrt{x})^2}}$

$= \sqrt{\frac{(1 - \sqrt{x})^2}{1 - x}}$

Assuming $1 - \sqrt{x} \ge 0$ (i.e., $0 \le x \le 1$), we have $\sqrt{(1 - \sqrt{x})^2} = |1 - \sqrt{x}| = 1 - \sqrt{x}$.

$= \frac{1 - \sqrt{x}}{\sqrt{1 - x}}$

So the integral becomes $I = \int \frac{1 - \sqrt{x}}{\sqrt{1 - x}} dx = \int \left(\frac{1}{\sqrt{1 - x}} - \frac{\sqrt{x}}{\sqrt{1 - x}}\right) dx$.

This might still be complicated. Let's try a different approach using trigonometric substitution for the term $\sqrt{1 - x}$.


Consider the original integral: $I = \int \sqrt{\frac{1 − \sqrt{x}}{1 + \sqrt{x}}} dx$.

Let's use the substitution $\sqrt{x} = \cos \theta$. This implies $x = \cos^2 \theta$.

For the square root $\sqrt{x}$ to be defined, $x \ge 0$. For $\sqrt{1-x}$ to be defined, $1-x \ge 0 \implies x \le 1$. So we consider the interval $0 \le x \le 1$. For $0 \le x \le 1$, $0 \le \sqrt{x} \le 1$. We can let $\theta$ be in the interval $[0, \frac{\pi}{2}]$.

If $\sqrt{x} = \cos \theta$, then $x = \cos^2 \theta$.

Differentiate $x$ with respect to $\theta$:

$\frac{dx}{d\theta} = \frac{d}{d\theta}(\cos^2 \theta) = 2 \cos \theta (-\sin \theta) = -2 \sin \theta \cos \theta$

So, $dx = -2 \sin \theta \cos \theta d\theta$.


Substitute $\sqrt{x} = \cos \theta$ and $dx = -2 \sin \theta \cos \theta d\theta$ into the integral:

$I = \int \sqrt{\frac{1 − \cos \theta}{1 + \cos \theta}} (-2 \sin \theta \cos \theta) d\theta$

Use half-angle identities: $1 - \cos \theta = 2 \sin^2 \frac{\theta}{2}$ and $1 + \cos \theta = 2 \cos^2 \frac{\theta}{2}$.

$\sqrt{\frac{1 − \cos \theta}{1 + \cos \theta}} = \sqrt{\frac{2 \sin^2 \frac{\theta}{2}}{2 \cos^2 \frac{\theta}{2}}} = \sqrt{\tan^2 \frac{\theta}{2}}$

For $\theta \in [0, \frac{\pi}{2}]$, $\frac{\theta}{2} \in [0, \frac{\pi}{4}]$, so $\tan \frac{\theta}{2} \ge 0$. Thus, $\sqrt{\tan^2 \frac{\theta}{2}} = |\tan \frac{\theta}{2}| = \tan \frac{\theta}{2}$.

The integral becomes:

$I = \int \tan \frac{\theta}{2} (-2 \sin \theta \cos \theta) d\theta$

Use double-angle identity for sine: $\sin \theta = 2 \sin \frac{\theta}{2} \cos \frac{\theta}{2}$.

$I = \int \frac{\sin \frac{\theta}{2}}{\cos \frac{\theta}{2}} (-2 (2 \sin \frac{\theta}{2} \cos \frac{\theta}{2}) \cos \theta) d\theta$

$I = \int \sin \frac{\theta}{2} (-4 \sin \frac{\theta}{2} \cos \theta) d\theta$ (The $\cos \frac{\theta}{2}$ term cancels with the denominator of $\tan \frac{\theta}{2}$)

$I = -4 \int \sin^2 \frac{\theta}{2} \cos \theta d\theta$

Use half-angle identity for $\sin^2 \frac{\theta}{2}$: $\sin^2 \frac{\theta}{2} = \frac{1 - \cos \theta}{2}$.

$I = -4 \int \frac{1 - \cos \theta}{2} \cos \theta d\theta$

$I = -2 \int (1 - \cos \theta) \cos \theta d\theta$

$I = -2 \int (\cos \theta - \cos^2 \theta) d\theta$

$I = -2 \int \cos \theta d\theta + 2 \int \cos^2 \theta d\theta$


Integrate the first term: $\int \cos \theta d\theta = \sin \theta$.

Integrate the second term using the power reduction formula $\cos^2 \theta = \frac{1 + \cos(2\theta)}{2}$:

$\int \cos^2 \theta d\theta = \int \frac{1 + \cos(2\theta)}{2} d\theta = \frac{1}{2} \int (1 + \cos(2\theta)) d\theta$

$= \frac{1}{2} \left(\int 1 d\theta + \int \cos(2\theta) d\theta\right)$

$= \frac{1}{2} \left(\theta + \frac{1}{2} \sin(2\theta)\right) = \frac{1}{2} \theta + \frac{1}{4} \sin(2\theta)$


Combine the results:

$I = -2 (\sin \theta) + 2 \left(\frac{1}{2} \theta + \frac{1}{4} \sin(2\theta)\right) + C'$

$I = -2 \sin \theta + \theta + \frac{1}{2} \sin(2\theta) + C'$

Use the identity $\sin(2\theta) = 2 \sin \theta \cos \theta$:

$I = -2 \sin \theta + \theta + \frac{1}{2} (2 \sin \theta \cos \theta) + C'$

$I = -2 \sin \theta + \theta + \sin \theta \cos \theta + C'$


Now, substitute back in terms of $x$. We have $\sqrt{x} = \cos \theta$, so $\theta = \cos^{-1}(\sqrt{x})$.

$\cos \theta = \sqrt{x}$.

Since $\cos \theta = \sqrt{x}$ and $\theta \in [0, \frac{\pi}{2}]$, $\sin \theta = \sqrt{1 - \cos^2 \theta} = \sqrt{1 - (\sqrt{x})^2} = \sqrt{1 - x}$.

Substitute these into the expression for I:

$I = -2 \sqrt{1 - x} + \cos^{-1}(\sqrt{x}) + \sqrt{1 - x} \cdot \sqrt{x} + C'$

$I = \cos^{-1}(\sqrt{x}) - 2 \sqrt{1 - x} + \sqrt{x(1 - x)} + C$

$I = \cos^{-1}(\sqrt{x}) - 2 \sqrt{1 - x} + \sqrt{x - x^2} + C$


The final answer is $\mathbf{\cos^{-1}(\sqrt{x}) - 2 \sqrt{1 - x} + \sqrt{x - x^2} + C}$.

Question 20. $\frac{2 + \sin 2x}{1 + \cos 2x} e^x$

Answer:

Solution:


We need to integrate the function $\frac{2 + \sin 2x}{1 + \cos 2x} e^x$.

Let the integral be $I = \int \frac{2 + \sin 2x}{1 + \cos 2x} e^x dx$.

This integral is of the form $\int e^x [f(x) + f'(x)] dx$, whose integral is $e^x f(x) + C$. We need to manipulate the expression $\frac{2 + \sin 2x}{1 + \cos 2x}$ to fit this form.

Use trigonometric identities:

$\sin 2x = 2 \sin x \cos x$

$1 + \cos 2x = 2 \cos^2 x$


Substitute these identities into the fraction:

$\frac{2 + \sin 2x}{1 + \cos 2x} = \frac{2 + 2 \sin x \cos x}{2 \cos^2 x}$

Split the fraction into two terms:

$= \frac{2}{2 \cos^2 x} + \frac{2 \sin x \cos x}{2 \cos^2 x}$

$= \frac{1}{\cos^2 x} + \frac{\sin x}{\cos x}$

$= \sec^2 x + \tan x$


So the integrand becomes $(\tan x + \sec^2 x) e^x$.

Now the integral is $I = \int (\tan x + \sec^2 x) e^x dx$.

This is in the form $\int e^x [f(x) + f'(x)] dx$, where $f(x) = \tan x$.

The derivative of $f(x) = \tan x$ is $f'(x) = \frac{d}{dx}(\tan x) = \sec^2 x$.

Thus, the integrand is exactly $e^x [f(x) + f'(x)]$.


Using the standard integral formula $\int e^x [f(x) + f'(x)] dx = e^x f(x) + C$:

$I = e^x \tan x + C$


The final answer is $\mathbf{e^x \tan x + C}$.

Question 21. $\frac{x^2 + x + 1}{(x + 1)^2 \;(x + 2)}$

Answer:

Solution:


We need to integrate the function $\frac{x^2 + x + 1}{(x + 1)^2 \;(x + 2)}$.

Let the integral be $I = \int \frac{x^2 + x + 1}{(x + 1)^2 \;(x + 2)} dx$.

We use the method of partial fraction decomposition. The denominator has a repeated linear factor $(x+1)^2$ and a distinct linear factor $(x+2)$.

Set up the decomposition:

$\frac{x^2 + x + 1}{(x + 1)^2 (x + 2)} = \frac{A}{x + 1} + \frac{B}{(x + 1)^2} + \frac{C}{x + 2}$


Multiply both sides by $(x + 1)^2 (x + 2)$:

$x^2 + x + 1 = A(x + 1)(x + 2) + B(x + 2) + C(x + 1)^2$

$x^2 + x + 1 = A(x^2 + 3x + 2) + B(x + 2) + C(x^2 + 2x + 1)$

$x^2 + x + 1 = Ax^2 + 3Ax + 2A + Bx + 2B + Cx^2 + 2Cx + C$

$x^2 + x + 1 = (A + C)x^2 + (3A + B + 2C)x + (2A + 2B + C)$


Equating the coefficients of the powers of $x$:

Coefficient of $x^2$: $1 = A + C$ ... (1)

Coefficient of $x$: $1 = 3A + B + 2C$ ... (2)

Constant term: $1 = 2A + 2B + C$ ... (3)


Alternatively, we can substitute specific values of $x$ to solve for A, B, and C.

Let $x = -1$:

$(-1)^2 + (-1) + 1 = A(-1 + 1)(-1 + 2) + B(-1 + 2) + C(-1 + 1)^2$

$1 - 1 + 1 = A(0)(1) + B(1) + C(0)^2$

$1 = 0 + B + 0$

$B = 1$


Let $x = -2$:

$(-2)^2 + (-2) + 1 = A(-2 + 1)(-2 + 2) + B(-2 + 2) + C(-2 + 1)^2$

$4 - 2 + 1 = A(-1)(0) + B(0) + C(-1)^2$

$3 = 0 + 0 + C(1)$

$C = 3$


Now use equation (1) to find A:

$1 = A + C$

$1 = A + 3$

$A = 1 - 3 = -2$


Check the values using equation (2) or (3).

Using equation (2): $1 = 3A + B + 2C = 3(-2) + 1 + 2(3) = -6 + 1 + 6 = 1$. This is consistent.


So the partial fraction decomposition is:

$\frac{x^2 + x + 1}{(x + 1)^2 (x + 2)} = \frac{-2}{x + 1} + \frac{1}{(x + 1)^2} + \frac{3}{x + 2}$


Now we integrate each term:

$I = \int \left(\frac{-2}{x + 1} + \frac{1}{(x + 1)^2} + \frac{3}{x + 2}\right) dx$

$I = -2 \int \frac{1}{x + 1} dx + \int (x + 1)^{-2} dx + 3 \int \frac{1}{x + 2} dx$


First integral: $\int \frac{1}{x + 1} dx = \log_e |x + 1|$

Second integral: $\int (x + 1)^{-2} dx$. Let $u = x + 1$, $du = dx$. $\int u^{-2} du = \frac{u^{-1}}{-1} = -\frac{1}{u}$. So, $\int (x + 1)^{-2} dx = -\frac{1}{x + 1}$.

Third integral: $\int \frac{1}{x + 2} dx = \log_e |x + 2|$


Combine the results and add the constant of integration C':

$I = -2 \log_e |x + 1| - \frac{1}{x + 1} + 3 \log_e |x + 2| + C'$


Using logarithm properties, we can rewrite $-2 \log_e |x + 1| + 3 \log_e |x + 2|$ as $\log_e |(x+1)^{-2}| + \log_e |(x+2)^3| = \log_e \left|\frac{(x+2)^3}{(x+1)^2}\right|$.

$I = \log_e \left|\frac{(x+2)^3}{(x+1)^2}\right| - \frac{1}{x + 1} + C$


The final answer is $\mathbf{3 \log_e |x + 2| - 2 \log_e |x + 1| - \frac{1}{x + 1} + C}$.

Question 22. $\tan^{-1} \sqrt{\frac{1 − x}{1 + x}}$

Answer:

Solution:


We need to integrate the function $\tan^{-1} \sqrt{\frac{1 − x}{1 + x}}$.

Let the integral be $I = \int \tan^{-1} \sqrt{\frac{1 − x}{1 + x}} dx$.

First, simplify the expression inside the inverse tangent function using a trigonometric substitution for $x$. The form $\sqrt{\frac{1-x}{1+x}}$ suggests the substitution $x = \cos \theta$.

Let $x = \cos \theta$. For the term $\sqrt{1-x^2}$ etc., the standard substitution is $x=\sin\theta$ or $x=\cos\theta$. Here we have $\sqrt{\frac{1-x}{1+x}}$. Let's try $x = \cos(2\theta)$ for simplicity with half-angle formulas.

Let $x = \cos(2\theta)$. This implies $2\theta = \cos^{-1} x$, so $\theta = \frac{1}{2} \cos^{-1} x$.

For $\tan^{-1}$ to be defined, the term inside must be real. For $\sqrt{\frac{1-x}{1+x}}$ to be real, $\frac{1-x}{1+x} \ge 0$ and $1+x \neq 0$. This implies $-1 < x \le 1$. If we take $x = \cos(2\theta)$ with $0 \le 2\theta < \pi$, then $0 \le \theta < \frac{\pi}{2}$. This range for $\theta$ corresponds to $1 > x \ge -1$.


Substitute $x = \cos(2\theta)$ into the expression inside the inverse tangent:

$\sqrt{\frac{1 − \cos(2\theta)}{1 + \cos(2\theta)}}$

Use the double-angle identities: $1 - \cos(2\theta) = 2 \sin^2 \theta$ and $1 + \cos(2\theta) = 2 \cos^2 \theta$.

$= \sqrt{\frac{2 \sin^2 \theta}{2 \cos^2 \theta}} = \sqrt{\tan^2 \theta}$

For $0 \le \theta < \frac{\pi}{2}$, $\tan \theta \ge 0$. So, $\sqrt{\tan^2 \theta} = |\tan \theta| = \tan \theta$.

Thus, $\tan^{-1} \sqrt{\frac{1 − x}{1 + x}} = \tan^{-1}(\tan \theta)$.

For $0 \le \theta < \frac{\pi}{2}$, $\tan^{-1}(\tan \theta) = \theta$.

So, the integrand simplifies to $\theta$, where $\theta = \frac{1}{2} \cos^{-1} x$.


Now, differentiate $x = \cos(2\theta)$ to find $dx$ in terms of $d\theta$:

$dx = \frac{d}{d\theta}(\cos(2\theta)) d\theta = -2 \sin(2\theta) d\theta$


Substitute $\tan^{-1} \sqrt{\frac{1 − x}{1 + x}} = \theta$ and $dx = -2 \sin(2\theta) d\theta$ into the integral:

$I = \int \theta (-2 \sin(2\theta)) d\theta$

$I = -2 \int \theta \sin(2\theta) d\theta$


We need to integrate this by parts. Recall the integration by parts formula: $\int u dv = uv - \int v du$.

Let $u = \theta$ and $dv = \sin(2\theta) d\theta$.

Then $du = d\theta$.

To find $v$, integrate $dv$: $v = \int \sin(2\theta) d\theta$. Let $w = 2\theta$, $dw = 2 d\theta$. $\int \sin w \frac{1}{2} dw = \frac{1}{2} (-\cos w) = -\frac{1}{2} \cos(2\theta)$.

$v = -\frac{1}{2} \cos(2\theta)$.


Apply the integration by parts formula:

$I = -2 \left[ \theta \left(-\frac{1}{2} \cos(2\theta)\right) - \int \left(-\frac{1}{2} \cos(2\theta)\right) d\theta \right]$

$I = -2 \left[ -\frac{1}{2} \theta \cos(2\theta) + \frac{1}{2} \int \cos(2\theta) d\theta \right]$

$I = \theta \cos(2\theta) - \int \cos(2\theta) d\theta$


Integrate $\int \cos(2\theta) d\theta$. Let $w = 2\theta$, $dw = 2 d\theta$. $\int \cos w \frac{1}{2} dw = \frac{1}{2} \sin w = \frac{1}{2} \sin(2\theta)$.

$I = \theta \cos(2\theta) - \frac{1}{2} \sin(2\theta) + C'$


Finally, substitute back in terms of $x$.

We have $x = \cos(2\theta)$, so $\cos(2\theta) = x$.

$\theta = \frac{1}{2} \cos^{-1} x$.

Use the identity $\sin(2\theta) = \sqrt{1 - \cos^2(2\theta)}$ (since for $0 \le 2\theta < \pi$, $\sin(2\theta) \ge 0$).

$\sin(2\theta) = \sqrt{1 - x^2}$.

Substitute these back into the expression for I:

$I = \left(\frac{1}{2} \cos^{-1} x\right) (x) - \frac{1}{2} \sqrt{1 - x^2} + C'$

$I = \frac{x}{2} \cos^{-1} x - \frac{1}{2} \sqrt{1 - x^2} + C$


The final answer is $\mathbf{\frac{x}{2} \cos^{-1} x - \frac{1}{2} \sqrt{1 - x^2} + C}$.

Question 23. $\frac{\sqrt{x^2 + 1} \;[\log (x^2 + 1 ) − 2 \log x]}{x^4}$

Answer:

Solution:


We need to integrate the function $\frac{\sqrt{x^2 + 1} \;[\log (x^2 + 1 ) − 2 \log x]}{x^4}$.

Let the integral be $I = \int \frac{\sqrt{x^2 + 1} \;[\log (x^2 + 1 ) − 2 \log x]}{x^4} dx$.

First, simplify the term inside the square brackets using logarithm properties: $\log a - \log b^n = \log \frac{a}{b^n}$.

$\log (x^2 + 1 ) − 2 \log x = \log (x^2 + 1 ) − \log (x^2)$

Assuming $x > 0$ for $\log x$ to be real, we have:

$= \log \left(\frac{x^2 + 1}{x^2}\right) = \log \left(1 + \frac{1}{x^2}\right) = \log (1 + x^{-2})$


Rewrite the term $\sqrt{x^2 + 1}$ by factoring out $x^2$:

$\sqrt{x^2 + 1} = \sqrt{x^2 \left(1 + \frac{1}{x^2}\right)}$

Assuming $x > 0$, $\sqrt{x^2} = |x| = x$.

$= x \sqrt{1 + \frac{1}{x^2}} = x \sqrt{1 + x^{-2}}$


Substitute the simplified terms back into the integrand:

$\frac{x \sqrt{1 + x^{-2}} \cdot \log (1 + x^{-2})}{x^4}$

$= \frac{\sqrt{1 + x^{-2}} \cdot \log (1 + x^{-2})}{x^3}$

$= (1 + x^{-2})^{1/2} \cdot \log (1 + x^{-2}) \cdot x^{-3}$


The integral is $I = \int (1 + x^{-2})^{1/2} \cdot \log (1 + x^{-2}) \cdot x^{-3} dx$.

This form suggests a substitution involving $1 + x^{-2}$.

Let $u = 1 + x^{-2}$.

Differentiate $u$ with respect to $x$:

$\frac{du}{dx} = \frac{d}{dx}(1 + x^{-2}) = 0 + (-2)x^{-3} = -2x^{-3}$

So, $du = -2x^{-3} dx$, which means $x^{-3} dx = -\frac{1}{2} du$.


Substitute $u = 1 + x^{-2}$ and $x^{-3} dx = -\frac{1}{2} du$ into the integral:

$I = \int u^{1/2} \cdot \log u \cdot \left(-\frac{1}{2} du\right)$

$I = -\frac{1}{2} \int u^{1/2} \log u du$


Now we need to integrate $\int u^{1/2} \log u du$ by parts. Recall the integration by parts formula: $\int p dq = pq - \int q dp$.

Let $p = \log u$ and $dq = u^{1/2} du$.

Then $dp = \frac{1}{u} du$.

To find $q$, integrate $dq$: $q = \int u^{1/2} du = \frac{u^{3/2}}{3/2} = \frac{2}{3} u^{3/2}$.


Apply the integration by parts formula to $\int u^{1/2} \log u du$:

$\int u^{1/2} \log u du = (\log u) \left(\frac{2}{3} u^{3/2}\right) - \int \left(\frac{2}{3} u^{3/2}\right) \left(\frac{1}{u}\right) du$

$= \frac{2}{3} u^{3/2} \log u - \int \frac{2}{3} u^{3/2 - 1} du$

$= \frac{2}{3} u^{3/2} \log u - \int \frac{2}{3} u^{1/2} du$


Integrate $\int \frac{2}{3} u^{1/2} du = \frac{2}{3} \int u^{1/2} du = \frac{2}{3} \left(\frac{u^{3/2}}{3/2}\right) = \frac{2}{3} \cdot \frac{2}{3} u^{3/2} = \frac{4}{9} u^{3/2}$.

So, $\int u^{1/2} \log u du = \frac{2}{3} u^{3/2} \log u - \frac{4}{9} u^{3/2} + K$, where K is the integration constant for this step.


Now substitute this result back into the expression for $I = -\frac{1}{2} \int u^{1/2} \log u du$:

$I = -\frac{1}{2} \left( \frac{2}{3} u^{3/2} \log u - \frac{4}{9} u^{3/2} \right) + C'$

$I = -\frac{1}{3} u^{3/2} \log u + \frac{2}{9} u^{3/2} + C'$


Factor out $u^{3/2}$:

$I = u^{3/2} \left( \frac{2}{9} - \frac{1}{3} \log u \right) + C'$

$I = \frac{1}{9} u^{3/2} \left( 2 - 3 \log u \right) + C'$


Finally, substitute back $u = 1 + x^{-2} = 1 + \frac{1}{x^2} = \frac{x^2 + 1}{x^2}$.

$u^{3/2} = \left(\frac{x^2 + 1}{x^2}\right)^{3/2} = \frac{(x^2 + 1)^{3/2}}{(x^2)^{3/2}} = \frac{(x^2 + 1)^{3/2}}{|x|^3}$

Assuming $x > 0$, $|x|^3 = x^3$.

$u^{3/2} = \frac{(x^2 + 1)^{3/2}}{x^3}$

$\log u = \log \left(\frac{x^2 + 1}{x^2}\right) = \log(x^2 + 1) - \log(x^2) = \log(x^2 + 1) - 2 \log x$.


Substitute these back into the expression for I:

$I = \frac{1}{9} \frac{(x^2 + 1)^{3/2}}{x^3} \left( 2 - 3 [\log(x^2 + 1) - 2 \log x] \right) + C$

$I = \frac{(x^2 + 1)^{3/2}}{9x^3} \left( 2 - 3 \log(x^2 + 1) + 6 \log x \right) + C$


The final answer is $\mathbf{\frac{(x^2 + 1)^{3/2}}{9x^3} \left( 2 + 6 \log x - 3 \log(x^2 + 1) \right) + C}$.

Evaluate the definite integrals in Exercises 24 to 31.

Question 24. $\int\limits_{\frac{π}{2}}^π e^x \left( \frac{1 − \sin x}{1 − \cos x} \right) \;dx$

Answer:

Solution:


We need to evaluate the definite integral $\int\limits_{\frac{\pi}{2}}^\pi e^x \left( \frac{1 − \sin x}{1 − \cos x} \right) \;dx$.

Let the integral be $I = \int\limits_{\frac{\pi}{2}}^\pi e^x \left( \frac{1 − \sin x}{1 − \cos x} \right) \;dx$.

We first simplify the term inside the parenthesis using trigonometric identities:

$1 - \cos x = 2 \sin^2 \frac{x}{2}$

$\sin x = 2 \sin \frac{x}{2} \cos \frac{x}{2}$


Substitute these into the fraction:

$\frac{1 − \sin x}{1 − \cos x} = \frac{1 - 2 \sin \frac{x}{2} \cos \frac{x}{2}}{2 \sin^2 \frac{x}{2}}$

Split the fraction into two terms:

$= \frac{1}{2 \sin^2 \frac{x}{2}} - \frac{2 \sin \frac{x}{2} \cos \frac{x}{2}}{2 \sin^2 \frac{x}{2}}$

$= \frac{1}{2} \text{cosec}^2 \frac{x}{2} - \frac{\cos \frac{x}{2}}{\sin \frac{x}{2}}$

$= \frac{1}{2} \text{cosec}^2 \frac{x}{2} - \cot \frac{x}{2}$

$= -\cot \frac{x}{2} + \frac{1}{2} \text{cosec}^2 \frac{x}{2}$


So the integrand is $e^x \left(-\cot \frac{x}{2} + \frac{1}{2} \text{cosec}^2 \frac{x}{2}\right)$.

This is in the form $e^x [f(x) + f'(x)]$, where $f(x) = -\cot \frac{x}{2}$.

Let's find the derivative of $f(x) = -\cot \frac{x}{2}$:

$f'(x) = \frac{d}{dx}\left(-\cot \frac{x}{2}\right)$

Using the chain rule, $\frac{d}{dx}(\cot u) = -\text{cosec}^2 u \cdot \frac{du}{dx}$. Here $u = \frac{x}{2}$, so $\frac{du}{dx} = \frac{1}{2}$.

$f'(x) = - (-\text{cosec}^2 \frac{x}{2}) \cdot \frac{1}{2}$

$f'(x) = \frac{1}{2} \text{cosec}^2 \frac{x}{2}$

Indeed, the integrand is $e^x [f(x) + f'(x)]$.


The indefinite integral is $\int e^x [f(x) + f'(x)] dx = e^x f(x) + C = e^x \left(-\cot \frac{x}{2}\right) + C = -e^x \cot \frac{x}{2} + C$.


Now, we evaluate the definite integral using the limits of integration $\frac{\pi}{2}$ to $\pi$:

$I = \left[ -e^x \cot \frac{x}{2} \right]_{\frac{\pi}{2}}^\pi$

$I = \left(-e^\pi \cot \frac{\pi}{2}\right) - \left(-e^{\frac{\pi}{2}} \cot \frac{\frac{\pi}{2}}{2}\right)$

$I = \left(-e^\pi \cot \frac{\pi}{2}\right) - \left(-e^{\frac{\pi}{2}} \cot \frac{\pi}{4}\right)$


Evaluate the cotangent terms:

$\cot \frac{\pi}{2} = \frac{\cos \frac{\pi}{2}}{\sin \frac{\pi}{2}} = \frac{0}{1} = 0$

$\cot \frac{\pi}{4} = \frac{\cos \frac{\pi}{4}}{\sin \frac{\pi}{4}} = \frac{1/\sqrt{2}}{1/\sqrt{2}} = 1$


Substitute these values back into the expression for I:

$I = (-e^\pi \cdot 0) - (-e^{\frac{\pi}{2}} \cdot 1)$

$I = 0 - (-e^{\frac{\pi}{2}})$

$I = e^{\frac{\pi}{2}}$


The final answer is $\mathbf{e^{\frac{\pi}{2}}}$.

Question 25. $\int\limits_0^{\frac{π}{4}} \frac{\sin x \;\cos x}{\cos^4 x + \sin^4 x} \;dx$

Answer:

Solution:


We need to evaluate the definite integral $\int\limits_0^{\frac{\pi}{4}} \frac{\sin x \;\cos x}{\cos^4 x + \sin^4 x} \;dx$.

Let the integral be $I = \int\limits_0^{\frac{\pi}{4}} \frac{\sin x \;\cos x}{\cos^4 x + \sin^4 x} \;dx$.

Divide the numerator and denominator by $\cos^4 x$. Assuming $\cos x \neq 0$ on the interval $(0, \frac{\pi}{4})$.

Numerator: $\frac{\sin x \cos x}{\cos^4 x} = \frac{\sin x}{\cos^3 x} = \tan x \sec^2 x$

Denominator: $\frac{\cos^4 x + \sin^4 x}{\cos^4 x} = \frac{\cos^4 x}{\cos^4 x} + \frac{\sin^4 x}{\cos^4 x} = 1 + \tan^4 x$

So the integrand becomes $\frac{\tan x \sec^2 x}{1 + \tan^4 x}$.


The integral is $I = \int\limits_0^{\frac{\pi}{4}} \frac{\tan x \sec^2 x}{1 + \tan^4 x} \;dx$.

This suggests the substitution $u = \tan^2 x$.

Differentiate $u$ with respect to $x$ using the chain rule:

$\frac{du}{dx} = \frac{d}{dx}(\tan^2 x) = 2 \tan x \cdot \frac{d}{dx}(\tan x) = 2 \tan x \sec^2 x$

So, $du = 2 \tan x \sec^2 x dx$, which means $\tan x \sec^2 x dx = \frac{1}{2} du$.


Now, change the limits of integration based on the substitution $u = \tan^2 x$.

Lower limit: When $x = 0$, $u = \tan^2(0) = 0^2 = 0$.

Upper limit: When $x = \frac{\pi}{4}$, $u = \tan^2\left(\frac{\pi}{4}\right) = 1^2 = 1$.


Substitute $u = \tan^2 x$, $\tan x \sec^2 x dx = \frac{1}{2} du$, and the new limits into the integral:

$I = \int\limits_0^1 \frac{1}{1 + u^2} \cdot \frac{1}{2} du$

$I = \frac{1}{2} \int\limits_0^1 \frac{1}{1 + u^2} du$


The integral $\int \frac{1}{1 + u^2} du$ is a standard integral equal to $\tan^{-1} u$.

$I = \frac{1}{2} \left[ \tan^{-1} u \right]_0^1$


Evaluate the definite integral using the limits:

$I = \frac{1}{2} (\tan^{-1}(1) - \tan^{-1}(0))$

We know that $\tan^{-1}(1) = \frac{\pi}{4}$ and $\tan^{-1}(0) = 0$.

$I = \frac{1}{2} \left(\frac{\pi}{4} - 0\right)$

$I = \frac{1}{2} \cdot \frac{\pi}{4} = \frac{\pi}{8}$


The final answer is $\mathbf{\frac{\pi}{8}}$.

Question 26. $\int\limits_0^{\frac{π}{2}} \frac{\cos^2 x \;dx}{\cos^2 x + 4\sin^2 x}$

Answer:

Solution:


We need to evaluate the definite integral $\int\limits_0^{\frac{\pi}{2}} \frac{\cos^2 x \;dx}{\cos^2 x + 4\sin^2 x}$.

Let the integral be $I = \int\limits_0^{\frac{\pi}{2}} \frac{\cos^2 x}{\cos^2 x + 4\sin^2 x} \;dx$.

We can simplify the denominator using $\sin^2 x = 1 - \cos^2 x$:

$\cos^2 x + 4\sin^2 x = \cos^2 x + 4(1 - \cos^2 x)$

$= \cos^2 x + 4 - 4\cos^2 x$

$= 4 - 3\cos^2 x$

The integrand becomes $\frac{\cos^2 x}{4 - 3\cos^2 x}$. This does not look simpler to integrate directly.


Let's try dividing the numerator and the denominator by $\cos^2 x$ (assuming $\cos x \neq 0$ on the interval, which is true for $(0, \frac{\pi}{2})$ but $\cos \frac{\pi}{2}=0$, so we need to be careful if a substitution involving tan or sec is used). However, the integrand is well-defined at $x=\frac{\pi}{2}$.

Divide numerator and denominator by $\cos^2 x$:

$\frac{\cos^2 x / \cos^2 x}{(\cos^2 x + 4\sin^2 x) / \cos^2 x} = \frac{1}{1 + 4 \frac{\sin^2 x}{\cos^2 x}} = \frac{1}{1 + 4 \tan^2 x}$

The integral becomes $I = \int\limits_0^{\frac{\pi}{2}} \frac{1}{1 + 4 \tan^2 x} \;dx$. This form is suitable for the substitution $u = \tan x$.

Let $u = \tan x$.

Then $du = \sec^2 x \;dx = (1 + \tan^2 x) \;dx = (1 + u^2) \;dx$.

So, $dx = \frac{du}{1 + u^2}$.


Change the limits of integration based on $u = \tan x$:

Lower limit: When $x = 0$, $u = \tan(0) = 0$.

Upper limit: When $x = \frac{\pi}{2}$, $u = \tan\left(\frac{\pi}{2}\right)$, which approaches infinity. This is an improper integral.


Let's rewrite the denominator $1 + 4 \tan^2 x$ in terms of $u$: $1 + 4u^2$.

Substitute $u = \tan x$ and $dx = \frac{du}{1 + u^2}$ into the integral:

$I = \int\limits_0^\infty \frac{1}{1 + 4 u^2} \cdot \frac{du}{1 + u^2}$

This requires partial fraction decomposition for $\frac{1}{(1 + 4 u^2)(1 + u^2)}$.

$\frac{1}{(1 + 4 u^2)(1 + u^2)} = \frac{Au + B}{1 + 4 u^2} + \frac{Cu + D}{1 + u^2}$

$1 = (Au + B)(1 + u^2) + (Cu + D)(1 + 4u^2)$

$1 = Au^3 + Bu^2 + Au + B + 4Cu^3 + Du^2 + 4Cu + D$

$1 = (A + 4C)u^3 + (B + D)u^2 + (A + 4C)u + (B + D)$

Comparing coefficients:

$u^3$: $0 = A + 4C$

$u^2$: $0 = B + D$

$u$: $0 = A + 4C$ (redundant)

Constant: $1 = B + D$ (redundant)

Something is wrong with comparing coefficients this way when the quadratic terms are irreducible. Let's treat $u^2$ as a variable, say $y=u^2$.

$\frac{1}{(1 + 4 y)(1 + y)} = \frac{A}{1 + 4 y} + \frac{B}{1 + y}$

$1 = A(1 + y) + B(1 + 4y)$

If $y = -1$, $1 = A(0) + B(1 - 4) = -3B \implies B = -\frac{1}{3}$.

If $y = -\frac{1}{4}$, $1 = A(1 - \frac{1}{4}) + B(0) = A(\frac{3}{4}) \implies A = \frac{4}{3}$.

So, $\frac{1}{(1 + 4 u^2)(1 + u^2)} = \frac{4/3}{1 + 4 u^2} + \frac{-1/3}{1 + u^2} = \frac{4}{3(1 + 4 u^2)} - \frac{1}{3(1 + u^2)}$.


Now integrate with respect to $u$ from 0 to $\infty$:

$I = \int\limits_0^\infty \left(\frac{4}{3(1 + 4 u^2)} - \frac{1}{3(1 + u^2)}\right) du$

$I = \frac{4}{3} \int\limits_0^\infty \frac{1}{1 + (2u)^2} du - \frac{1}{3} \int\limits_0^\infty \frac{1}{1 + u^2} du$


For the first integral, let $v = 2u$, $dv = 2 du \implies du = \frac{1}{2} dv$. Limits: if $u=0, v=0$; if $u=\infty, v=\infty$.

$\frac{4}{3} \int\limits_0^\infty \frac{1}{1 + v^2} \frac{1}{2} dv = \frac{2}{3} \int\limits_0^\infty \frac{1}{1 + v^2} dv = \frac{2}{3} [\tan^{-1} v]_0^\infty$

$= \frac{2}{3} (\lim_{v \to \infty} \tan^{-1} v - \tan^{-1} 0) = \frac{2}{3} (\frac{\pi}{2} - 0) = \frac{\pi}{3}$.


For the second integral:

$\frac{1}{3} \int\limits_0^\infty \frac{1}{1 + u^2} du = \frac{1}{3} [\tan^{-1} u]_0^\infty = \frac{1}{3} (\lim_{u \to \infty} \tan^{-1} u - \tan^{-1} 0)$

$= \frac{1}{3} (\frac{\pi}{2} - 0) = \frac{\pi}{6}$.


Combine the results:

$I = \frac{\pi}{3} - \frac{\pi}{6} = \frac{2\pi}{6} - \frac{\pi}{6} = \frac{\pi}{6}$.


The final answer is $\mathbf{\frac{\pi}{6}}$.

Question 27. $\int\limits_{\frac{π}{6}}^{\frac{π}{3}} \frac{\sin x + \cos x}{\sqrt{\sin 2x}} \;dx$

Answer:

Solution:


We need to evaluate the definite integral $\int\limits_{\frac{\pi}{6}}^{\frac{\pi}{3}} \frac{\sin x + \cos x}{\sqrt{\sin 2x}} \;dx$.

Let the integral be $I = \int\limits_{\frac{\pi}{6}}^{\frac{\pi}{3}} \frac{\sin x + \cos x}{\sqrt{\sin 2x}} \;dx$.

Consider the numerator $\sin x + \cos x$. The derivative of $\sin x - \cos x$ is $\cos x - (-\sin x) = \cos x + \sin x$.

Let's try a substitution involving $\sin x - \cos x$.

Let $t = \sin x - \cos x$.

Then $dt = (\cos x - (-\sin x)) dx = (\cos x + \sin x) dx$. The numerator is $dt$.

Now, express the denominator $\sqrt{\sin 2x}$ in terms of $t$.

$t^2 = (\sin x - \cos x)^2 = \sin^2 x - 2 \sin x \cos x + \cos^2 x$

$t^2 = (\sin^2 x + \cos^2 x) - 2 \sin x \cos x$

Using the identity $\sin^2 x + \cos^2 x = 1$ and $\sin 2x = 2 \sin x \cos x$:

$t^2 = 1 - \sin 2x$

So, $\sin 2x = 1 - t^2$.

The denominator is $\sqrt{1 - t^2}$.


Now, change the limits of integration based on the substitution $t = \sin x - \cos x$.

Lower limit: When $x = \frac{\pi}{6}$, $t = \sin \frac{\pi}{6} - \cos \frac{\pi}{6} = \frac{1}{2} - \frac{\sqrt{3}}{2} = \frac{1 - \sqrt{3}}{2}$.

Upper limit: When $x = \frac{\pi}{3}$, $t = \sin \frac{\pi}{3} - \cos \frac{\pi}{3} = \frac{\sqrt{3}}{2} - \frac{1}{2} = \frac{\sqrt{3} - 1}{2}$.


Substitute $t = \sin x - \cos x$, $(\sin x + \cos x) dx = dt$, $\sqrt{\sin 2x} = \sqrt{1 - t^2}$, and the new limits into the integral:

$I = \int\limits_{\frac{1 - \sqrt{3}}{2}}^{\frac{\sqrt{3} - 1}{2}} \frac{dt}{\sqrt{1 - t^2}}$

The integral $\int \frac{dt}{\sqrt{1 - t^2}}$ is a standard integral equal to $\sin^{-1} t$.

$I = \left[ \sin^{-1} t \right]_{\frac{1 - \sqrt{3}}{2}}^{\frac{\sqrt{3} - 1}{2}}$


Evaluate the definite integral using the limits:

$I = \sin^{-1}\left(\frac{\sqrt{3} - 1}{2}\right) - \sin^{-1}\left(\frac{1 - \sqrt{3}}{2}\right)$

Recall that $\sin^{-1}(-y) = -\sin^{-1}(y)$. Let $y = \frac{\sqrt{3} - 1}{2}$. Then $\frac{1 - \sqrt{3}}{2} = -y$.

$\sin^{-1}\left(\frac{1 - \sqrt{3}}{2}\right) = \sin^{-1}\left(-\frac{\sqrt{3} - 1}{2}\right) = -\sin^{-1}\left(\frac{\sqrt{3} - 1}{2}\right)$

So, $I = \sin^{-1}\left(\frac{\sqrt{3} - 1}{2}\right) - \left(-\sin^{-1}\left(\frac{\sqrt{3} - 1}{2}\right)\right)$

$I = \sin^{-1}\left(\frac{\sqrt{3} - 1}{2}\right) + \sin^{-1}\left(\frac{\sqrt{3} - 1}{2}\right)$

$I = 2 \sin^{-1}\left(\frac{\sqrt{3} - 1}{2}\right)$


We know that $\sin\left(\frac{\pi}{12}\right) = \sin(15^\circ) = \sin(45^\circ - 30^\circ) = \sin 45^\circ \cos 30^\circ - \cos 45^\circ \sin 30^\circ = \frac{\sqrt{2}}{2} \frac{\sqrt{3}}{2} - \frac{\sqrt{2}}{2} \frac{1}{2} = \frac{\sqrt{6} - \sqrt{2}}{4}$. This is not the value we have.

Let's check the limits again. $\frac{1 - \sqrt{3}}{2} \approx \frac{1 - 1.732}{2} = \frac{-0.732}{2} = -0.366$. $\frac{\sqrt{3} - 1}{2} \approx \frac{1.732 - 1}{2} = \frac{0.732}{2} = 0.366$. These values are between -1 and 1, so arcsin is defined.

The value $\frac{\sqrt{3} - 1}{2}$ is related to $\sin(\frac{\pi}{12})$? No, that's $\frac{\sqrt{6} - \sqrt{2}}{4}$.

Let's re-check the substitution and the limits.

The limits are correct. The substitution is correct.

The integral $\int \frac{dt}{\sqrt{1-t^2}} = \sin^{-1} t + C$ is correct.

The evaluation of the definite integral is $\sin^{-1}\left(\frac{\sqrt{3} - 1}{2}\right) - \sin^{-1}\left(\frac{1 - \sqrt{3}}{2}\right) = 2 \sin^{-1}\left(\frac{\sqrt{3} - 1}{2}\right)$ is correct.

Is $\frac{\sqrt{3} - 1}{2}$ a known value of sine?

Let's try a different approach just to verify.


Alternate Approach:

$I = \int\limits_{\frac{\pi}{6}}^{\frac{\pi}{3}} \frac{\sin x + \cos x}{\sqrt{1 - (\sin x - \cos x)^2}} dx$

Let $t = \sin x - \cos x$. $dt = (\cos x + \sin x) dx$. Limits are $\frac{1-\sqrt{3}}{2}$ and $\frac{\sqrt{3}-1}{2}$.

$I = \int\limits_{\frac{1 - \sqrt{3}}{2}}^{\frac{\sqrt{3} - 1}{2}} \frac{dt}{\sqrt{1 - t^2}} = [\sin^{-1} t]_{\frac{1 - \sqrt{3}}{2}}^{\frac{\sqrt{3} - 1}{2}}$

$= \sin^{-1}\left(\frac{\sqrt{3} - 1}{2}\right) - \sin^{-1}\left(\frac{1 - \sqrt{3}}{2}\right) = 2 \sin^{-1}\left(\frac{\sqrt{3} - 1}{2}\right)$.

This confirms the result from the first method.

Let's consider the interval of integration $\left[\frac{\pi}{6}, \frac{\pi}{3}\right]$. In this interval, $\sin x$ increases from $\frac{1}{2}$ to $\frac{\sqrt{3}}{2}$ and $\cos x$ decreases from $\frac{\sqrt{3}}{2}$ to $\frac{1}{2}$. $\sin x - \cos x$ goes from $\frac{1 - \sqrt{3}}{2}$ to $\frac{\sqrt{3} - 1}{2}$.

Also, $\sin 2x$ goes from $\sin(\frac{\pi}{3}) = \frac{\sqrt{3}}{2}$ to $\sin(\frac{2\pi}{3}) = \frac{\sqrt{3}}{2}$, reaching a maximum of $\sin(\frac{\pi}{2}) = 1$ at $x = \frac{\pi}{4}$. $\sin 2x > 0$ on this interval, so $\sqrt{\sin 2x}$ is well-defined and real.

The value $\frac{\sqrt{3}-1}{2}$ is indeed $\sin(\frac{\pi}{12})$? Let's check again. $\sin(\frac{\pi}{12}) = \frac{\sqrt{6}-\sqrt{2}}{4}$.

Let's check if $\frac{\sqrt{3}-1}{2}$ is $\sin(\frac{\pi}{12})$. No, it's not.

Is it possible that there's a simplification I'm missing?

Consider $\sin(\frac{\pi}{12}) = \sin(15^\circ) = \sin(45^\circ - 30^\circ) = \frac{\sqrt{6}-\sqrt{2}}{4}$.

Consider $\cos(\frac{\pi}{12}) = \cos(15^\circ) = \cos(45^\circ - 30^\circ) = \cos 45^\circ \cos 30^\circ + \sin 45^\circ \sin 30^\circ = \frac{\sqrt{2}}{2} \frac{\sqrt{3}}{2} + \frac{\sqrt{2}}{2} \frac{1}{2} = \frac{\sqrt{6} + \sqrt{2}}{4}$.

The term $\frac{\sqrt{3} - 1}{2}$ seems correct based on the substitution and limits. Unless there is a standard angle whose sine is $\frac{\sqrt{3} - 1}{2}$, the answer will involve $\sin^{-1}$.

Let's re-check the formula for $\sin(\frac{\pi}{12})$ and similar values. Yes, $\sin(15^\circ) = \frac{\sqrt{6}-\sqrt{2}}{4}$.

It seems the answer cannot be simplified further in terms of elementary trigonometric values. The result $2 \sin^{-1}\left(\frac{\sqrt{3} - 1}{2}\right)$ is the correct form.


The final answer is $\mathbf{2 \sin^{-1}\left(\frac{\sqrt{3} - 1}{2}\right)}$.

Question 28. $\int\limits_0^1 \frac{dx}{\sqrt{1 + x} − \sqrt{x}}$

Answer:

Solution:


We need to evaluate the definite integral $\int\limits_0^1 \frac{dx}{\sqrt{1 + x} − \sqrt{x}}$.

Let the integral be $I = \int\limits_0^1 \frac{dx}{\sqrt{1 + x} − \sqrt{x}}$.

First, rationalize the denominator by multiplying the numerator and denominator by the conjugate of the denominator, which is $\sqrt{1 + x} + \sqrt{x}$.

$\frac{1}{\sqrt{1 + x} − \sqrt{x}} = \frac{1}{\sqrt{1 + x} − \sqrt{x}} \times \frac{\sqrt{1 + x} + \sqrt{x}}{\sqrt{1 + x} + \sqrt{x}}$

Using the difference of squares formula $(p-q)(p+q) = p^2 - q^2$ in the denominator:

$= \frac{\sqrt{1 + x} + \sqrt{x}}{(\sqrt{1 + x})^2 - (\sqrt{x})^2}$

$= \frac{\sqrt{1 + x} + \sqrt{x}}{(1 + x) - x}$

$= \frac{\sqrt{1 + x} + \sqrt{x}}{1 + x - x} = \frac{\sqrt{1 + x} + \sqrt{x}}{1}$

$= \sqrt{1 + x} + \sqrt{x}$


So the integral becomes $I = \int\limits_0^1 (\sqrt{1 + x} + \sqrt{x}) dx$.

$I = \int\limits_0^1 (1 + x)^{1/2} dx + \int\limits_0^1 x^{1/2} dx$


Integrate the first term: $\int (1 + x)^{1/2} dx$. Let $u = 1 + x$, $du = dx$.

$\int u^{1/2} du = \frac{u^{3/2}}{3/2} = \frac{2}{3} u^{3/2}$.

So, $\int (1 + x)^{1/2} dx = \frac{2}{3} (1 + x)^{3/2}$.


Integrate the second term: $\int x^{1/2} dx$.

$\int x^{1/2} dx = \frac{x^{3/2}}{3/2} = \frac{2}{3} x^{3/2}$.


Combine the indefinite integrals: $\frac{2}{3} (1 + x)^{3/2} + \frac{2}{3} x^{3/2}$.

Now, evaluate the definite integral using the limits 0 to 1:

$I = \left[ \frac{2}{3} (1 + x)^{3/2} + \frac{2}{3} x^{3/2} \right]_0^1$

$I = \left(\frac{2}{3} (1 + 1)^{3/2} + \frac{2}{3} (1)^{3/2}\right) - \left(\frac{2}{3} (1 + 0)^{3/2} + \frac{2}{3} (0)^{3/2}\right)$

$I = \left(\frac{2}{3} (2)^{3/2} + \frac{2}{3} (1)\right) - \left(\frac{2}{3} (1)^{3/2} + \frac{2}{3} (0)\right)$

$I = \left(\frac{2}{3} (2\sqrt{2}) + \frac{2}{3}\right) - \left(\frac{2}{3} (1) + 0\right)$

$I = \frac{4\sqrt{2}}{3} + \frac{2}{3} - \frac{2}{3}$

$I = \frac{4\sqrt{2}}{3}$


The final answer is $\mathbf{\frac{4\sqrt{2}}{3}}$.

Question 29. $\int\limits_0^{\frac{π}{4}} \frac{\sin x + \cos x}{9 + 16 \sin 2x} \;dx$

Answer:

Solution:


We need to evaluate the definite integral $\int\limits_0^{\frac{\pi}{4}} \frac{\sin x + \cos x}{9 + 16 \sin 2x} \;dx$.

Let the integral be $I = \int\limits_0^{\frac{\pi}{4}} \frac{\sin x + \cos x}{9 + 16 \sin 2x} \;dx$.

Consider the numerator $\sin x + \cos x$. This is the derivative of $\sin x - \cos x$.

Let's use the substitution $t = \sin x - \cos x$.

Then $dt = (\cos x - (-\sin x)) dx = (\cos x + \sin x) dx$. The numerator is $dt$.

Now, express the denominator $9 + 16 \sin 2x$ in terms of $t$.

We know that $t^2 = (\sin x - \cos x)^2 = 1 - \sin 2x$.

So, $\sin 2x = 1 - t^2$.

Substitute this into the denominator:

$9 + 16 \sin 2x = 9 + 16(1 - t^2)$

$= 9 + 16 - 16t^2 = 25 - 16t^2$


Now, change the limits of integration based on the substitution $t = \sin x - \cos x$.

Lower limit: When $x = 0$, $t = \sin(0) - \cos(0) = 0 - 1 = -1$.

Upper limit: When $x = \frac{\pi}{4}$, $t = \sin\left(\frac{\pi}{4}\right) - \cos\left(\frac{\pi}{4}\right) = \frac{1}{\sqrt{2}} - \frac{1}{\sqrt{2}} = 0$.


Substitute $t = \sin x - \cos x$, $(\sin x + \cos x) dx = dt$, $9 + 16 \sin 2x = 25 - 16t^2$, and the new limits into the integral:

$I = \int\limits_{-1}^0 \frac{dt}{25 - 16t^2}$

$I = \int\limits_{-1}^0 \frac{dt}{5^2 - (4t)^2}$

This integral is of the form $\int \frac{du}{a^2 - u^2} = \frac{1}{2a} \log_e \left|\frac{a + u}{a - u}\right|$.

Let $v = 4t$, then $dv = 4 dt \implies dt = \frac{1}{4} dv$. Limits: if $t=-1, v=-4$; if $t=0, v=0$.

$I = \int\limits_{-4}^0 \frac{\frac{1}{4} dv}{5^2 - v^2} = \frac{1}{4} \int\limits_{-4}^0 \frac{dv}{5^2 - v^2}$

Here $a = 5$ and $u = v$.

$I = \frac{1}{4} \left[ \frac{1}{2 \cdot 5} \log_e \left|\frac{5 + v}{5 - v}\right| \right]_{-4}^0$

$I = \frac{1}{40} \left[ \log_e \left|\frac{5 + v}{5 - v}\right| \right]_{-4}^0$


Evaluate the definite integral using the limits:

$I = \frac{1}{40} \left( \log_e \left|\frac{5 + 0}{5 - 0}\right| - \log_e \left|\frac{5 + (-4)}{5 - (-4)}\right| \right)$

$I = \frac{1}{40} \left( \log_e \left|\frac{5}{5}\right| - \log_e \left|\frac{1}{9}\right| \right)$

$I = \frac{1}{40} \left( \log_e (1) - \log_e \left(\frac{1}{9}\right) \right)$

We know that $\log_e (1) = 0$ and $\log_e \left(\frac{1}{9}\right) = \log_e (9^{-1}) = -\log_e (9)$.

$I = \frac{1}{40} (0 - (-\log_e (9)))$

$I = \frac{1}{40} \log_e (9)$

We can write $\log_e (9) = \log_e (3^2) = 2 \log_e 3$.

$I = \frac{1}{40} (2 \log_e 3) = \frac{2 \log_e 3}{40} = \frac{\log_e 3}{20}$


The final answer is $\mathbf{\frac{\log_e 3}{20}}$.

Question 30. $\int\limits_0^{\frac{π}{2}} \sin 2x \tan^{−1} (\sin x) \;dx$

Answer:

Solution:


We need to evaluate the definite integral $\int\limits_0^{\frac{\pi}{2}} \sin 2x \tan^{-1} (\sin x) \;dx$.

Let the integral be $I = \int\limits_0^{\frac{\pi}{2}} \sin 2x \tan^{-1} (\sin x) \;dx$.

Use the identity $\sin 2x = 2 \sin x \cos x$.

$I = \int\limits_0^{\frac{\pi}{2}} 2 \sin x \cos x \tan^{-1} (\sin x) \;dx$

Let's use a substitution. Let $u = \sin x$.

Differentiate $u$ with respect to $x$: $du = \cos x dx$.


Change the limits of integration based on the substitution $u = \sin x$.

Lower limit: When $x = 0$, $u = \sin(0) = 0$.

Upper limit: When $x = \frac{\pi}{2}$, $u = \sin\left(\frac{\pi}{2}\right) = 1$.


Substitute $u = \sin x$, $\cos x dx = du$ into the integral. The term $2 \sin x$ becomes $2u$, and $\tan^{-1}(\sin x)$ becomes $\tan^{-1} u$.

$I = \int\limits_0^1 2u \tan^{-1} u \;du$

$I = 2 \int\limits_0^1 u \tan^{-1} u \;du$


Now we need to integrate $\int u \tan^{-1} u \;du$ by parts. Recall the integration by parts formula: $\int p dq = pq - \int q dp$.

Let $p = \tan^{-1} u$ and $dq = u \;du$.

Then $dp = \frac{1}{1 + u^2} du$.

To find $q$, integrate $dq$: $q = \int u \;du = \frac{u^2}{2}$.


Apply the integration by parts formula to $\int u \tan^{-1} u \;du$:

$\int u \tan^{-1} u \;du = (\tan^{-1} u) \left(\frac{u^2}{2}\right) - \int \left(\frac{u^2}{2}\right) \left(\frac{1}{1 + u^2}\right) du$

$= \frac{u^2}{2} \tan^{-1} u - \frac{1}{2} \int \frac{u^2}{1 + u^2} du$


Evaluate the remaining integral $\int \frac{u^2}{1 + u^2} du$. We can add and subtract 1 in the numerator:

$\int \frac{u^2}{1 + u^2} du = \int \frac{1 + u^2 - 1}{1 + u^2} du = \int \left(\frac{1 + u^2}{1 + u^2} - \frac{1}{1 + u^2}\right) du$

$= \int \left(1 - \frac{1}{1 + u^2}\right) du = \int 1 du - \int \frac{1}{1 + u^2} du$

$= u - \tan^{-1} u$


Substitute this back into the integration by parts result:

$\int u \tan^{-1} u \;du = \frac{u^2}{2} \tan^{-1} u - \frac{1}{2} (u - \tan^{-1} u)$

$= \frac{u^2}{2} \tan^{-1} u - \frac{u}{2} + \frac{1}{2} \tan^{-1} u$

Now, evaluate the definite integral $2 \int\limits_0^1 u \tan^{-1} u \;du$ using the limits 0 to 1:

$I = 2 \left[ \frac{u^2}{2} \tan^{-1} u - \frac{u}{2} + \frac{1}{2} \tan^{-1} u \right]_0^1$

$I = 2 \left[ \left(\frac{1^2}{2} \tan^{-1} 1 - \frac{1}{2} + \frac{1}{2} \tan^{-1} 1\right) - \left(\frac{0^2}{2} \tan^{-1} 0 - \frac{0}{2} + \frac{1}{2} \tan^{-1} 0\right) \right]$

$I = 2 \left[ \left(\frac{1}{2} \cdot \frac{\pi}{4} - \frac{1}{2} + \frac{1}{2} \cdot \frac{\pi}{4}\right) - (0 - 0 + 0) \right]$

$I = 2 \left[ \frac{\pi}{8} - \frac{1}{2} + \frac{\pi}{8} \right]$

$I = 2 \left[ \frac{2\pi}{8} - \frac{1}{2} \right] = 2 \left[ \frac{\pi}{4} - \frac{1}{2} \right]$

$I = 2 \cdot \frac{\pi}{4} - 2 \cdot \frac{1}{2} = \frac{\pi}{2} - 1$


The final answer is $\mathbf{\frac{\pi}{2} - 1}$.

Question 31. $\int\limits_1^4 \; [|x − 1| + |x − 2| + |x − 3|] \;dx$

Answer:

Solution:


We need to evaluate the definite integral $\int\limits_1^4 \; [|x − 1| + |x − 2| + |x − 3|] \;dx$.

Let the integral be $I = \int\limits_1^4 \; [|x − 1| + |x − 2| + |x − 3|] \;dx$.

We need to split the integral based on where the expressions inside the absolute value change sign. The critical points are $x=1$, $x=2$, and $x=3$. The interval of integration is $[1, 4]$.

For $x \in [1, 4]$, we analyze the sign of each term:

$|x - 1| = \begin{cases} x - 1 & \text{if } x \ge 1 \\ -(x - 1) & \text{if } x < 1 \end{cases}$

On $[1, 4]$, $|x - 1| = x - 1$.


$|x - 2| = \begin{cases} x - 2 & \text{if } x \ge 2 \\ -(x - 2) & \text{if } x < 2 \end{cases}$

$|x - 3| = \begin{cases} x - 3 & \text{if } x \ge 3 \\ -(x - 3) & \text{if } x < 3 \end{cases}$


We split the integral into subintervals based on the critical points within $[1, 4]$:

Interval 1: $[1, 2]$

On $[1, 2]$, we have $x \ge 1$, $x < 2$, $x < 3$.

$|x - 1| = x - 1$

$|x - 2| = -(x - 2) = 2 - x$

$|x - 3| = -(x - 3) = 3 - x$

Sum: $(x - 1) + (2 - x) + (3 - x) = x - 1 + 2 - x + 3 - x = 4 - x$


Interval 2: $[2, 3]$

On $[2, 3]$, we have $x \ge 1$, $x \ge 2$, $x < 3$.

$|x - 1| = x - 1$

$|x - 2| = x - 2$

$|x - 3| = -(x - 3) = 3 - x$

Sum: $(x - 1) + (x - 2) + (3 - x) = x - 1 + x - 2 + 3 - x = x$


Interval 3: $[3, 4]$

On $[3, 4]$, we have $x \ge 1$, $x \ge 2$, $x \ge 3$.

$|x - 1| = x - 1$

$|x - 2| = x - 2$

$|x - 3| = x - 3$

Sum: $(x - 1) + (x - 2) + (x - 3) = x - 1 + x - 2 + x - 3 = 3x - 6$


Now split the integral into these subintervals:

$I = \int\limits_1^2 (4 - x) \;dx + \int\limits_2^3 x \;dx + \int\limits_3^4 (3x - 6) \;dx$


Evaluate the first integral: $\int\limits_1^2 (4 - x) \;dx$

$= \left[ 4x - \frac{x^2}{2} \right]_1^2 = \left(4(2) - \frac{2^2}{2}\right) - \left(4(1) - \frac{1^2}{2}\right)$

$= \left(8 - \frac{4}{2}\right) - \left(4 - \frac{1}{2}\right) = (8 - 2) - \left(\frac{8 - 1}{2}\right) = 6 - \frac{7}{2} = \frac{12 - 7}{2} = \frac{5}{2}$.


Evaluate the second integral: $\int\limits_2^3 x \;dx$

$= \left[ \frac{x^2}{2} \right]_2^3 = \frac{3^2}{2} - \frac{2^2}{2} = \frac{9}{2} - \frac{4}{2} = \frac{5}{2}$.


Evaluate the third integral: $\int\limits_3^4 (3x - 6) \;dx$

$= \left[ \frac{3x^2}{2} - 6x \right]_3^4 = \left(\frac{3(4)^2}{2} - 6(4)\right) - \left(\frac{3(3)^2}{2} - 6(3)\right)$

$= \left(\frac{3(16)}{2} - 24\right) - \left(\frac{3(9)}{2} - 18\right) = \left(3(8) - 24\right) - \left(\frac{27}{2} - 18\right)$

$= (24 - 24) - \left(\frac{27 - 36}{2}\right) = 0 - \left(-\frac{9}{2}\right) = \frac{9}{2}$.


Add the results from the three integrals:

$I = \frac{5}{2} + \frac{5}{2} + \frac{9}{2} = \frac{5 + 5 + 9}{2} = \frac{19}{2}$.


The final answer is $\mathbf{\frac{19}{2}}$.

Prove the following (Exercises 32 to 37)

Question 32. $\int\limits_1^3 \frac{dx}{x^2 (x + 1)} = \frac{2}{3} + \log \frac{2}{3}$

Answer:

To Prove:

$\int\limits_1^3 \frac{dx}{x^2 (x + 1)} = \frac{2}{3} + \log \frac{2}{3}$


Solution:

Let the Left Hand Side (LHS) of the equation be $I = \int\limits_1^3 \frac{1}{x^2 (x + 1)} dx$.

We will evaluate this definite integral using the method of partial fraction decomposition for the integrand $\frac{1}{x^2 (x + 1)}$.

The denominator has a repeated linear factor $x^2$ and a distinct linear factor $(x+1)$. We set up the decomposition as:

$\frac{1}{x^2 (x + 1)} = \frac{A}{x} + \frac{B}{x^2} + \frac{C}{x + 1}$


Multiply both sides by $x^2 (x + 1)$ to clear the denominators:

$1 = A x (x + 1) + B (x + 1) + C x^2$

Expand the right side:

$1 = A (x^2 + x) + (Bx + B) + Cx^2$

$1 = Ax^2 + Ax + Bx + B + Cx^2$

Group terms by powers of $x$:

$1 = (A + C)x^2 + (A + B)x + B$


Equating the coefficients of the powers of $x$ on both sides:

Coefficient of $x^2$: $0 = A + C$ ... (1)

Coefficient of $x$: $0 = A + B$ ... (2)

Constant term: $1 = B$ ... (3)


From equation (3), we find $B = 1$.

Substitute the value of $B$ into equation (2): $0 = A + 1$, which implies $A = -1$.

Substitute the value of $A$ into equation (1): $0 = -1 + C$, which implies $C = 1$.


So the partial fraction decomposition of the integrand is:

$\frac{1}{x^2 (x + 1)} = \frac{-1}{x} + \frac{1}{x^2} + \frac{1}{x + 1}$


Now, we integrate this expression from the lower limit 1 to the upper limit 3:

$I = \int\limits_1^3 \left(-\frac{1}{x} + \frac{1}{x^2} + \frac{1}{x + 1}\right) dx$

$I = \int\limits_1^3 -\frac{1}{x} dx + \int\limits_1^3 x^{-2} dx + \int\limits_1^3 \frac{1}{x + 1} dx$


Evaluate each definite integral:

For $\int\limits_1^3 -\frac{1}{x} dx$: The antiderivative of $-\frac{1}{x}$ is $-\log_e |x|$. On the interval $[1, 3]$, $x$ is positive, so $|x|=x$.

$\int\limits_1^3 -\frac{1}{x} dx = [-\log_e x]_1^3 = (-\log_e 3) - (-\log_e 1) = -\log_e 3 - 0 = -\log_e 3$.

For $\int\limits_1^3 x^{-2} dx$: The antiderivative of $x^{-2}$ is $\frac{x^{-2+1}}{-2+1} = \frac{x^{-1}}{-1} = -\frac{1}{x}$.

$\int\limits_1^3 x^{-2} dx = \left[-\frac{1}{x}\right]_1^3 = \left(-\frac{1}{3}\right) - \left(-\frac{1}{1}\right) = -\frac{1}{3} + 1 = \frac{2}{3}$.

For $\int\limits_1^3 \frac{1}{x + 1} dx$: The antiderivative of $\frac{1}{x+1}$ is $\log_e |x+1|$. On the interval $[1, 3]$, $x+1$ is positive, so $|x+1|=x+1$.

$\int\limits_1^3 \frac{1}{x + 1} dx = [\log_e (x + 1)]_1^3 = \log_e (3 + 1) - \log_e (1 + 1) = \log_e 4 - \log_e 2$.

Using the logarithm property $\log_e M - \log_e N = \log_e \frac{M}{N}$, we have $\log_e 4 - \log_e 2 = \log_e \frac{4}{2} = \log_e 2$.


Sum the results of the three definite integrals to find the value of $I$:

$I = (-\log_e 3) + \frac{2}{3} + (\log_e 2)$

$I = \frac{2}{3} + \log_e 2 - \log_e 3$

Using the logarithm property $\log_e M - \log_e N = \log_e \frac{M}{N}$ again:

$I = \frac{2}{3} + \log_e \left(\frac{2}{3}\right)$

Assuming $\log$ denotes the natural logarithm (which is commonly denoted by $\log_e $ in calculus), the result is $\frac{2}{3} + \log \frac{2}{3}$.


The value of the definite integral is $\frac{2}{3} + \log \frac{2}{3}$.

This is equal to the Right Hand Side (RHS) of the given equation.

Therefore, we have proved that $\int\limits_1^3 \frac{dx}{x^2 (x + 1)} = \frac{2}{3} + \log \frac{2}{3}$.

Question 33. $\int\limits_0^1 x e^x \;dx = 1$

Answer:

To Prove:

$\int\limits_0^1 x e^x \;dx = 1$


Solution:

Let the Left Hand Side (LHS) of the equation be $I = \int\limits_0^1 x e^x \;dx$.

We will evaluate this definite integral using integration by parts. The formula for definite integration by parts is $\int\limits_a^b u \;dv = [uv]_a^b - \int\limits_a^b v \;du$.

Choose $u$ and $dv$ based on the LIATE rule (Logarithmic, Inverse trigonometric, Algebraic, Trigonometric, Exponential).

Let $u = x$ (Algebraic) and $dv = e^x dx$ (Exponential).

Then, differentiate $u$ to find $du$: $du = dx$.

Integrate $dv$ to find $v$: $v = \int e^x dx = e^x$.


Apply the integration by parts formula with the limits of integration $a=0$ and $b=1$:

$I = \left[ x e^x \right]_0^1 - \int\limits_0^1 e^x dx$


Evaluate the first term $[xe^x]_0^1$:

$[xe^x]_0^1 = (1 \cdot e^1) - (0 \cdot e^0) = e - 0 = e$


Evaluate the second integral $\int\limits_0^1 e^x dx$:

$\int\limits_0^1 e^x dx = [e^x]_0^1 = e^1 - e^0 = e - 1$


Substitute the results back into the expression for I:

$I = e - (e - 1)$

$I = e - e + 1$

$I = 1$


The value of the definite integral is 1.

This is equal to the Right Hand Side (RHS) of the given equation.

Therefore, we have proved that $\int\limits_0^1 x e^x \;dx = 1$.

Question 34. $\int\limits_{−1}^1 x^{17} \cos^4 x \;dx = 0$

Answer:

To Prove:

$\int\limits_{−1}^1 x^{17} \cos^4 x \;dx = 0$


Solution:

Let the integral be $I = \int\limits_{−1}^1 x^{17} \cos^4 x \;dx$.

The limits of integration are from -a to a, where $a=1$. This suggests checking if the integrand is an even or odd function.

Let $f(x) = x^{17} \cos^4 x$.

To determine if the function is even or odd, we evaluate $f(-x)$.

$f(-x) = (-x)^{17} (\cos(-x))^4$


Properties of exponents: $(-x)^{17} = (-1)^{17} x^{17} = -x^{17}$ (since 17 is an odd power).

Properties of cosine: $\cos(-x) = \cos x$.

So, $(\cos(-x))^4 = (\cos x)^4 = \cos^4 x$.


Substitute these back into the expression for $f(-x)$:

$f(-x) = (-x^{17}) (\cos^4 x) = -x^{17} \cos^4 x$

Comparing $f(-x)$ with $f(x)$: $f(-x) = -f(x)$.

This means that $f(x) = x^{17} \cos^4 x$ is an odd function.


Recall the property of definite integrals for odd functions over a symmetric interval $[-a, a]$:

$\int\limits_{-a}^a f(x) dx = 0$ if $f(x)$ is an odd function.


In this case, $a = 1$ and the integrand $f(x) = x^{17} \cos^4 x$ is an odd function over the interval $[-1, 1]$.

Therefore, the integral is equal to 0.

$I = \int\limits_{−1}^1 x^{17} \cos^4 x \;dx = 0$


This is equal to the Right Hand Side (RHS) of the given equation.

Thus, we have proved that $\int\limits_{−1}^1 x^{17} \cos^4 x \;dx = 0$.

Question 35. $\int\limits_0^{\frac{π}{2}} \sin^3 x \;dx = \frac{2}{3}$

Answer:

To Prove:

$\int\limits_0^{\frac{\pi}{2}} \sin^3 x \;dx = \frac{2}{3}$


Solution:

Let the Left Hand Side (LHS) of the equation be $I = \int\limits_0^{\frac{\pi}{2}} \sin^3 x \;dx$.

We can rewrite $\sin^3 x$ using the identity $\sin^2 x = 1 - \cos^2 x$:

$\sin^3 x = \sin^2 x \cdot \sin x = (1 - \cos^2 x) \sin x$


So the integral becomes $I = \int\limits_0^{\frac{\pi}{2}} (1 - \cos^2 x) \sin x \;dx$.

We use the substitution $u = \cos x$.

Differentiate $u$ with respect to $x$: $du = -\sin x dx$, which means $\sin x dx = -du$.


Change the limits of integration based on the substitution $u = \cos x$.

Lower limit: When $x = 0$, $u = \cos(0) = 1$.

Upper limit: When $x = \frac{\pi}{2}$, $u = \cos\left(\frac{\pi}{2}\right) = 0$.


Substitute $u = \cos x$ and $\sin x dx = -du$ into the integral with the new limits:

$I = \int\limits_1^0 (1 - u^2) (-du)$

$I = \int\limits_1^0 -(1 - u^2) du = \int\limits_1^0 (u^2 - 1) du$

Using the property $\int\limits_a^b f(x) dx = -\int\limits_b^a f(x) dx$, we can swap the limits and change the sign:

$I = -\int\limits_0^1 (u^2 - 1) du = \int\limits_0^1 (1 - u^2) du$


Integrate $(1 - u^2)$ with respect to $u$ and evaluate from 0 to 1:

$I = \left[ u - \frac{u^3}{3} \right]_0^1$

$I = \left(1 - \frac{1^3}{3}\right) - \left(0 - \frac{0^3}{3}\right)$

$I = \left(1 - \frac{1}{3}\right) - (0)$

$I = \frac{3 - 1}{3} = \frac{2}{3}$


The value of the definite integral is $\frac{2}{3}$.

This is equal to the Right Hand Side (RHS) of the given equation.

Therefore, we have proved that $\int\limits_0^{\frac{\pi}{2}} \sin^3 x \;dx = \frac{2}{3}$.


Alternate Solution using Reduction Formula:

The reduction formula for $\int\limits_0^{\frac{\pi}{2}} \sin^n x \;dx$ is $\frac{n-1}{n} \int\limits_0^{\frac{\pi}{2}} \sin^{n-2} x \;dx$ for $n \ge 2$.

Here $n = 3$.

$I = \int\limits_0^{\frac{\pi}{2}} \sin^3 x \;dx = \frac{3-1}{3} \int\limits_0^{\frac{\pi}{2}} \sin^{3-2} x \;dx$

$I = \frac{2}{3} \int\limits_0^{\frac{\pi}{2}} \sin^1 x \;dx = \frac{2}{3} \int\limits_0^{\frac{\pi}{2}} \sin x \;dx$

Evaluate the integral of $\sin x$:

$\int\limits_0^{\frac{\pi}{2}} \sin x \;dx = [-\cos x]_0^{\frac{\pi}{2}} = (-\cos \frac{\pi}{2}) - (-\cos 0)$

$= -(0) - (-1) = 1$.

Substitute this back into the expression for I:

$I = \frac{2}{3} \cdot 1 = \frac{2}{3}$.

This matches the RHS.

Question 36. $\int\limits_0^{\frac{π}{4}} 2\tan^3 x \;dx = 1 − \log 2$

Answer:

To Prove:

$\int\limits_0^{\frac{\pi}{4}} 2\tan^3 x \;dx = 1 − \log 2$


Solution:

Let the Left Hand Side (LHS) of the equation be $I = \int\limits_0^{\frac{\pi}{4}} 2\tan^3 x \;dx$.

$I = 2 \int\limits_0^{\frac{\pi}{4}} \tan^3 x \;dx$

Rewrite $\tan^3 x$ as $\tan^2 x \cdot \tan x$ and use the identity $\tan^2 x = \sec^2 x - 1$:

$\tan^3 x = (\sec^2 x - 1) \tan x = \tan x \sec^2 x - \tan x$


So the integral becomes $I = 2 \int\limits_0^{\frac{\pi}{4}} (\tan x \sec^2 x - \tan x) \;dx$.

$I = 2 \left( \int\limits_0^{\frac{\pi}{4}} \tan x \sec^2 x \;dx - \int\limits_0^{\frac{\pi}{4}} \tan x \;dx \right)$


Evaluate the first integral: $\int\limits_0^{\frac{\pi}{4}} \tan x \sec^2 x \;dx$.

Use the substitution $u = \tan x$. Then $du = \sec^2 x dx$.

Limits: if $x=0$, $u=\tan 0 = 0$; if $x=\frac{\pi}{4}$, $u=\tan \frac{\pi}{4} = 1$.

$\int\limits_0^1 u \;du = \left[\frac{u^2}{2}\right]_0^1 = \frac{1^2}{2} - \frac{0^2}{2} = \frac{1}{2}$.


Evaluate the second integral: $\int\limits_0^{\frac{\pi}{4}} \tan x \;dx$.

We know that $\int \tan x dx = \log_e |\sec x|$.

$\int\limits_0^{\frac{\pi}{4}} \tan x \;dx = [\log_e |\sec x|]_0^{\frac{\pi}{4}} = \log_e |\sec \frac{\pi}{4}| - \log_e |\sec 0|$.

$\sec \frac{\pi}{4} = \frac{1}{\cos \frac{\pi}{4}} = \frac{1}{1/\sqrt{2}} = \sqrt{2}$.

$\sec 0 = \frac{1}{\cos 0} = \frac{1}{1} = 1$.

So, $[\log_e |\sec x|]_0^{\frac{\pi}{4}} = \log_e (\sqrt{2}) - \log_e (1) = \log_e (\sqrt{2}) - 0 = \log_e (\sqrt{2})$.

Alternatively, $\int \tan x dx = -\log_e |\cos x|$.

$[-\log_e |\cos x|]_0^{\frac{\pi}{4}} = (-\log_e |\cos \frac{\pi}{4}|) - (-\log_e |\cos 0|) = (-\log_e (\frac{1}{\sqrt{2}})) - (-\log_e (1)) = -\log_e (2^{-1/2}) + \log_e (1) = -(-\frac{1}{2}\log_e 2) + 0 = \frac{1}{2}\log_e 2$.

And $\log_e (\sqrt{2}) = \log_e (2^{1/2}) = \frac{1}{2} \log_e 2$. Both methods give the same result.


Substitute the results of the two integrals back into the expression for I:

$I = 2 \left( \frac{1}{2} - \log_e (\sqrt{2}) \right)$

$I = 2 \cdot \frac{1}{2} - 2 \log_e (\sqrt{2})$

$I = 1 - 2 \log_e (2^{1/2}) = 1 - 2 \left(\frac{1}{2} \log_e 2\right)$

$I = 1 - \log_e 2$

Assuming $\log$ denotes the natural logarithm, this is $1 - \log 2$.


The value of the definite integral is $1 - \log 2$.

This is equal to the Right Hand Side (RHS) of the given equation.

Therefore, we have proved that $\int\limits_0^{\frac{\pi}{4}} 2\tan^3 x \;dx = 1 − \log 2$.

Question 37. $\int\limits_0^1 \sin^{−1} x \;dx = \frac{π}{2} − 1$

Answer:

To Prove:

$\int\limits_0^1 \sin^{-1} x \;dx = \frac{\pi}{2} − 1$


Solution:

Let the Left Hand Side (LHS) of the equation be $I = \int\limits_0^1 \sin^{-1} x \;dx$.

We evaluate this definite integral using integration by parts. The formula for definite integration by parts is $\int\limits_a^b u \;dv = [uv]_a^b - \int\limits_a^b v \;du$.

Choose $u$ and $dv$ based on the LIATE rule. Let $u = \sin^{-1} x$ (Inverse trigonometric) and $dv = dx$ (Algebraic/Differential).

Then, differentiate $u$ to find $du$: $du = \frac{1}{\sqrt{1 - x^2}} dx$.

Integrate $dv$ to find $v$: $v = \int dx = x$.


Apply the integration by parts formula with the limits of integration $a=0$ and $b=1$:

$I = \left[ x \sin^{-1} x \right]_0^1 - \int\limits_0^1 x \cdot \frac{1}{\sqrt{1 - x^2}} dx$

$I = \left[ x \sin^{-1} x \right]_0^1 - \int\limits_0^1 \frac{x}{\sqrt{1 - x^2}} dx$


Evaluate the first term $[x \sin^{-1} x]_0^1$:

$[x \sin^{-1} x]_0^1 = (1 \cdot \sin^{-1} 1) - (0 \cdot \sin^{-1} 0)$

We know that $\sin^{-1} 1 = \frac{\pi}{2}$ (since $\sin \frac{\pi}{2} = 1$) and $\sin^{-1} 0 = 0$ (since $\sin 0 = 0$).

$= (1 \cdot \frac{\pi}{2}) - (0 \cdot 0) = \frac{\pi}{2} - 0 = \frac{\pi}{2}$.


Evaluate the second integral $\int\limits_0^1 \frac{x}{\sqrt{1 - x^2}} dx$.

Use the substitution $w = 1 - x^2$.

Differentiate $w$ with respect to $x$: $dw = -2x dx$, which means $x dx = -\frac{1}{2} dw$.

Change the limits of integration based on the substitution $w = 1 - x^2$.

Lower limit: When $x = 0$, $w = 1 - 0^2 = 1$.

Upper limit: When $x = 1$, $w = 1 - 1^2 = 0$.

The integral becomes:

$\int\limits_1^0 \frac{-\frac{1}{2} dw}{\sqrt{w}} = -\frac{1}{2} \int\limits_1^0 w^{-1/2} dw$

Use the property $\int\limits_a^b f(x) dx = -\int\limits_b^a f(x) dx$ to swap the limits:

$= -\frac{1}{2} \left(-\int\limits_0^1 w^{-1/2} dw\right) = \frac{1}{2} \int\limits_0^1 w^{-1/2} dw$

Integrate $w^{-1/2}$: $\int w^{-1/2} dw = \frac{w^{-1/2 + 1}}{-1/2 + 1} = \frac{w^{1/2}}{1/2} = 2\sqrt{w}$.

Evaluate the definite integral:

$\frac{1}{2} [2\sqrt{w}]_0^1 = \frac{1}{2} (2\sqrt{1} - 2\sqrt{0}) = \frac{1}{2} (2 - 0) = \frac{1}{2} \cdot 2 = 1$.


Substitute the results back into the expression for I:

$I = \frac{\pi}{2} - 1$


The value of the definite integral is $\frac{\pi}{2} - 1$.

This is equal to the Right Hand Side (RHS) of the given equation.

Therefore, we have proved that $\int\limits_0^1 \sin^{-1} x \;dx = \frac{\pi}{2} − 1$.


Alternate Solution using Integration by Parts with a different choice:

We can also integrate by parts by letting $u = \sin^{-1} x$ and $dv = dx$ (as before) but evaluating the indefinite integral first.

$\int \sin^{-1} x \;dx = x \sin^{-1} x - \int x \frac{1}{\sqrt{1-x^2}} dx$.

Let $w = 1 - x^2$, $dw = -2x dx$. $\int \frac{x}{\sqrt{1-x^2}} dx = \int \frac{-\frac{1}{2} dw}{\sqrt{w}} = -\frac{1}{2} \int w^{-1/2} dw = -\frac{1}{2} (2\sqrt{w}) + K' = -\sqrt{1-x^2} + K'$.

So, the indefinite integral is $\int \sin^{-1} x \;dx = x \sin^{-1} x - (-\sqrt{1-x^2}) + C = x \sin^{-1} x + \sqrt{1-x^2} + C$.

Now, evaluate the definite integral from 0 to 1:

$I = \left[ x \sin^{-1} x + \sqrt{1-x^2} \right]_0^1$

$I = \left(1 \cdot \sin^{-1} 1 + \sqrt{1-1^2}\right) - \left(0 \cdot \sin^{-1} 0 + \sqrt{1-0^2}\right)$

$I = \left(1 \cdot \frac{\pi}{2} + \sqrt{0}\right) - \left(0 + \sqrt{1}\right)$

$I = \left(\frac{\pi}{2} + 0\right) - (0 + 1)$

$I = \frac{\pi}{2} - 1$.

This confirms the result.

Choose the correct answers in Exercises 38 to 40.

Question 38. $\int \frac{dx}{e^x + e^{−x}}$ is equal to

(A) tan–1 (ex) + C

(B) tan–1 (e–x) + C

(C) log (ex – e–x) + C

(D) log (ex + e–x) + C

Answer:

Solution:


We need to evaluate the integral $\int \frac{dx}{e^x + e^{−x}}$.

Let the integral be $I = \int \frac{dx}{e^x + e^{−x}}$.

Rewrite $e^{-x}$ as $\frac{1}{e^x}$:

$e^x + e^{-x} = e^x + \frac{1}{e^x} = \frac{(e^x)^2 + 1}{e^x} = \frac{e^{2x} + 1}{e^x}$


Substitute this back into the integral:

$I = \int \frac{dx}{\frac{e^{2x} + 1}{e^x}} = \int \frac{e^x}{e^{2x} + 1} dx$

Now, we can use a substitution. Let $u = e^x$.

Differentiate $u$ with respect to $x$: $du = e^x dx$.


Substitute $u = e^x$ and $e^x dx = du$ into the integral:

$I = \int \frac{du}{u^2 + 1}$

This is a standard integral formula: $\int \frac{du}{u^2 + a^2} = \frac{1}{a} \tan^{-1}\left(\frac{u}{a}\right) + C'$. Here $a=1$.

$I = \frac{1}{1} \tan^{-1}\left(\frac{u}{1}\right) + C' = \tan^{-1}(u) + C'$


Finally, substitute back $u = e^x$:

$I = \tan^{-1}(e^x) + C$


This is equivalent to $\tan^{-1}(e^x) + C$.

Compare this result with the given options:

(A) tan–1 (ex) + C

(B) tan–1 (e–x) + C

(C) log (ex – e–x) + C

(D) log (ex + e–x) + C

The result matches option (A).


The final answer is $\mathbf{(A)}$.

Question 39. $\int \frac{\cos 2x}{(\sin x + \cos x)^2} \;dx$ is equal to

(A) $\frac{−1}{\sin x + \cos x} + C$

(B) log |sin x + cos x| + C

(C) log |sin x - cos x| + C

(D) $\frac{1}{(\sin x + \cos x)^2}$

Answer:

To evaluate the integral $\int \frac{\cos 2x}{(\sin x + \cos x)^2} \;dx$.


We first simplify the integrand using trigonometric identities.

We know that $\cos 2x = \cos^2 x - \sin^2 x$.

The denominator is $(\sin x + \cos x)^2 = (\cos x + \sin x)^2$.

So the integrand is $\frac{\cos^2 x - \sin^2 x}{(\cos x + \sin x)^2}$.


The numerator $\cos^2 x - \sin^2 x$ is a difference of squares, which can be factored as $(\cos x - \sin x)(\cos x + \sin x)$.

Substituting this factorization into the integrand:

$\frac{(\cos x - \sin x)(\cos x + \sin x)}{(\cos x + \sin x)^2}$

Assuming $\cos x + \sin x \neq 0$, we can cancel one factor of $(\cos x + \sin x)$ from the numerator and the denominator:

= $\frac{\cos x - \sin x}{\cos x + \sin x}$


Now the integral becomes $\int \frac{\cos x - \sin x}{\cos x + \sin x} \;dx$.

We can evaluate this integral using the method of substitution.


Let the denominator be $u$.

$u = \cos x + \sin x$


Now, differentiate $u$ with respect to $x$ to find $du$:

$\frac{du}{dx} = \frac{d}{dx}(\cos x + \sin x)$

$\frac{du}{dx} = -\sin x + \cos x$

$\frac{du}{dx} = \cos x - \sin x$

From this, we get $du = (\cos x - \sin x) \;dx$.


Notice that the numerator of the simplified integrand, $\cos x - \sin x$, is exactly the derivative of the denominator, $\cos x + \sin x$.

Substitute $u = \cos x + \sin x$ and $du = (\cos x - \sin x) \;dx$ into the integral $\int \frac{\cos x - \sin x}{\cos x + \sin x} \;dx$:

$\int \frac{du}{u}$


The integral of $\frac{1}{u}$ with respect to $u$ is $\log |u|$.

$\int \frac{du}{u} = \log |u| + C$

Where $C$ is the constant of integration.


Finally, substitute back $u = \cos x + \sin x$ to get the result in terms of $x$:

The integral is equal to:

$\log |\cos x + \sin x| + C$


Thus, $\int \frac{\cos 2x}{(\sin x + \cos x)^2} \;dx = \mathbf{\log |\sin x + \cos x| + C}$.


Comparing this result with the given options:

(A) $\frac{−1}{\sin x + \cos x} + C$

(B) log |sin x + cos x| + C

(C) log |sin x - cos x| + C

(D) $\frac{1}{(\sin x + \cos x)^2}$

The result matches option (B).


The correct option is (B) log |sin x + cos x| + C.

Question 40. If (a + b - x) = f(x), then $\int\limits_a^b x \;f(x) \;dx$ is equal to

(A) $\frac{a + b}{2} \int\limits_a^b f (b − x) \;dx$

(B) $\frac{a + b}{2} \int\limits_a^b f (b + x) \;dx$

(C) $\frac{b − a}{2} \int\limits_a^b f(x) \;dx$

(D) $\frac{a + b}{2} \int\limits_a^b f(x) \;dx$

Answer:

Solution:


The phrasing of the question "If (a + b - x) = f(x)" seems to contain a typographical error based on the standard properties of definite integrals and the given options. It is highly probable that the intended condition is that the function $f(x)$ satisfies the property $f(a + b - x) = f(x)$ for all $x$ in the interval $[a, b]$.

Assuming the intended question is: If $f(a + b - x) = f(x)$, then $\int\limits_a^b x \;f(x) \;dx$ is equal to ...


Let the integral be $I = \int\limits_a^b x \;f(x) \;dx$.

We use the property of definite integrals: $\int\limits_a^b h(x) \;dx = \int\limits_a^b h(a + b - x) \;dx$.

Let $h(x) = x f(x)$. Then $h(a + b - x) = (a + b - x) f(a + b - x)$.


Applying the property:

$\int\limits_a^b x \;f(x) \;dx = \int\limits_a^b (a + b - x) f(a + b - x) \;dx$

... (1)


We are given the condition $f(a + b - x) = f(x)$ for all $x \in [a, b]$. Substitute this into the right side of equation (1):

$\int\limits_a^b x \;f(x) \;dx = \int\limits_a^b (a + b - x) f(x) \;dx$

[Using $f(a+b-x)=f(x)$]


Split the integral on the right side:

$\int\limits_a^b x \;f(x) \;dx = \int\limits_a^b (a + b) f(x) \;dx - \int\limits_a^b x f(x) \;dx$

... (2)


The term $(a+b)$ is a constant, so it can be taken out of the integral:

$\int\limits_a^b x \;f(x) \;dx = (a + b) \int\limits_a^b f(x) \;dx - \int\limits_a^b x f(x) \;dx$

... (3)


Notice that the integral on the left side is the same as the second integral on the right side. Let $I = \int\limits_a^b x \;f(x) \;dx$. Equation (3) becomes:

$I = (a + b) \int\limits_a^b f(x) \;dx - I$

... (4)


Now, solve for $I$ by adding $I$ to both sides of equation (4):

$I + I = (a + b) \int\limits_a^b f(x) \;dx$

[Adding I to both sides]

$2I = (a + b) \int\limits_a^b f(x) \;dx$

[Simplify]


Finally, divide by 2 to find $I$:

$I = \frac{a + b}{2} \int\limits_a^b f(x) \;dx$

[Divide by 2]


Compare this result with the given options:

(A) $\frac{a + b}{2} \int\limits_a^b f (b − x) \;dx$

(B) $\frac{a + b}{2} \int\limits_a^b f (b + x) \;dx$

(C) $\frac{b − a}{2} \int\limits_a^b f(x) \;dx$

(D) $\frac{a + b}{2} \int\limits_a^b f(x) \;dx$

The result matches option (D).


Based on the most likely intended meaning of the question, the correct answer is (D).


The final answer is $\mathbf{(D)}$.